You are on page 1of 276

Contents

UNIT-I ............................................................................................................................................ 4
BIRD SIZE VIEW OF INDIA’S ECONOMY AND DEVELOPMENT ....................................................... 4
INDIAN ECONOMY BEFORE INDEPENDENCE ................................................................................ 4
PRE-LIBERALIZATION ERA ............................................................................................................. 6
LIBERALISATION, PRIVATIZATION AND GLOBALIZATION.............................................................. 7
COMPARATIVE ECONOMIC DEVELOPMENT ............................................................................... 10
UNIT-II ......................................................................................................................................... 10
BASIC CONECPTS OF ECONOMICS .............................................................................................. 10
MICRO ECONOMICS –BASIC CONCEPTS ..................................................................................... 11
MACRO ECONOMICS -BASIC CONCEPTS ..................................................................................... 11
UNIT-III ........................................................................................................................................ 24
NATIONAL INCOME CALCULATION ............................................................................................. 24
NATIONAL INCOME..................................................................................................................... 24
UNIT-IV ........................................................................................................................................ 29
MONEY , BANKING AND INFLATION ........................................................................................... 29
MONEY AND BANKING ............................................................................................................... 29
MONEY .............................................................................................................................................................................. 29
MONETARY POLICY ........................................................................................................................................................... 31
BANKS , NBFCS AND OTHER FINANCIAL INSTITUTIONS.............................................................. 36
BANKS ............................................................................................................................................................................... 36
QUANTITATIVE EASING ..................................................................................................................................................... 44
FINANCIAL INTERMEDIARIES AND INSTRUMENTS ........................................................................................................... 44
NPAs .................................................................................................................................................................................. 45
FINANCIAL INCLUSION ...................................................................................................................................................... 47
INFLATION ................................................................................................................................... 48
CAUSES AND CONSEQUENCES .......................................................................................................................................... 48
MEASUREMENT INDICES .................................................................................................................................................. 53
UNIT-V ......................................................................................................................................... 56
THE GOVERNMENT-BUDGET&FISCAL POLICY ............................................................................ 56
BUDGET ....................................................................................................................................... 56
POLICIES AND PROCEDURES ............................................................................................................................................. 56
TYPES OF BUDGET ............................................................................................................................................................. 56
RECEIPTS ........................................................................................................................................................................... 57
EXPENDITURE .................................................................................................................................................................... 58
DEFICIT .............................................................................................................................................................................. 59
FISCAL POLICY ............................................................................................................................. 62
TAXES ................................................................................................................................................................................ 62
SUBSIDIES .......................................................................................................................................................................... 74
PUBLIC FINANCE ............................................................................................................................................................... 74
DISINVESTMENT.......................................................................................................................... 74
MISC ........................................................................................................................................... 76
UNIT-VI ........................................................................................................................................ 81
AGRICULTURE AND FOOD MANAGEMENT ................................................................................. 81
AGRICULTURE AND ALLIED ACTIVITIES ....................................................................................... 81
MSP\ ........................................................................................................................................... 94
AGRICULTURE CREDIT ................................................................................................................. 98
EXTENSION .................................................................................................................................. 98
MARKETING AND APMC ACT .................................................................................................... 100
IMPORTS AND EXPORTS ........................................................................................................... 103
FOOD –DISTRIBUTION, SAFETY AND STANDARDS .................................................................... 103
INTERNATIONAL EFFORTS......................................................................................................... 108
UNIT-VII ..................................................................................................................................... 109
INFRASTRUCTURE, INDUSTRY AND SERVICES........................................................................... 109
INFRASTRUCTURE ..................................................................................................................... 109
LAND ACQUISITION AND REAL ESTATE ........................................................................................................................... 110
ROADS ............................................................................................................................................................................. 110
RAILWAYS........................................................................................................................................................................ 111
PORTS&WATERWAYS ..................................................................................................................................................... 114
CIVIL AVIATION ............................................................................................................................................................... 115
DIGITAL INFRASTRUCTURE ............................................................................................................................................. 116
URBAN INFRASTRUCTURE............................................................................................................................................... 117
ENERGY ........................................................................................................................................................................... 117
MINES AND MINERALS.................................................................................................................................................... 121
INDUSTRY .................................................................................................................................. 122
GENERAL ......................................................................................................................................................................... 122
STEEL ............................................................................................................................................................................... 129
PSUs ................................................................................................................................................................................ 129
TEXTILES INDUSTRY......................................................................................................................................................... 131
COOPERATIVE SECTOR INDUSTRIES................................................................................................................................ 132
STARTUPs ........................................................................................................................................................................ 132
MSMEs ............................................................................................................................................................................ 133
MISCELLANEOUS ............................................................................................................................................................. 135
SERVICES SECTOR...................................................................................................................... 137
CORPORATE GOVERNANCE ...................................................................................................... 137
INTELLECTUAL PROPERTY RIGHTS ............................................................................................ 137
CONCEPTS ....................................................................................................................................................................... 137
POLICIES .......................................................................................................................................................................... 137
INSTITUTIONS.................................................................................................................................................................. 140
UNIT-VIII .................................................................................................................................... 140
INVESTMENT , MONEY AND CAPITAL MARKETS ...................................................................... 140
INVESTMENT-THEORY .............................................................................................................. 140
INVESTMENT -GOVERNMENT INITIATIVES ............................................................................... 140
MONEY AND CAPITAL MARKETS............................................................................................... 142
UNIT-IX ...................................................................................................................................... 155
EXTERNAL SECTOR .................................................................................................................... 155
INTERNATIONAL TRADE ............................................................................................................ 155
CONCEPTS ....................................................................................................................................................................... 155
WTO ................................................................................................................................................................................ 163
INDIA’S FOREIGN TRADE POLICY&SCHEMES .................................................................................................................. 168
INDIA’s TRADE ................................................................................................................................................................. 172
TRADE BLOCs .................................................................................................................................................................. 179
THE BALANCE OF PAYMENTS.................................................................................................... 182
THE FOREIGN EXCHANGE MARKET ........................................................................................... 183
BASIC CONCEPTS ............................................................................................................................................................. 183
FOREX RESERVES ............................................................................................................................................................. 184
ARBITRAGE ...................................................................................................................................................................... 184
FDI&FII............................................................................................................................................................................. 184
EXTERNAL DEBT ........................................................................................................................ 185
UNIT-X ....................................................................................................................................... 185
CHALLENGES: POVERTY, SOCIAL INFRASTRUCTURE, SKILLING AND EMPLOYMENT ................ 185
LABOUR ..................................................................................................................................... 185
LABOUR REFORMS .................................................................................................................... 187
POVERTY ................................................................................................................................... 189
DEMOGRAPHIC DIVIDEND ........................................................................................................ 193
SOCIAL INFRASTRUCTURE......................................................................................................... 195
HEALTH............................................................................................................................................................................ 197
EDUCATION ..................................................................................................................................................................... 203
SKILLING .................................................................................................................................... 209
EMPLOYMENT ........................................................................................................................... 211
UNIT-XI ...................................................................................................................................... 214
DEVELOPMENT SCHEMES ......................................................................................................... 214
SCHEMES ................................................................................................................................... 214
UNIT-XII ..................................................................................................................................... 226
ENVIRONMENT ECONOMICS .................................................................................................... 226
ENVIRONMENT ......................................................................................................................... 226
WATER ...................................................................................................................................... 226
UNIT-XIII .................................................................................................................................... 227
ECONOMISTS AND THEIR CONTRIBUTIONS.............................................................................. 227
ECONOMISTS AND THEIR CONTRIBUTIONS.............................................................................. 227
UNIT-XIV.................................................................................................................................... 227
INSTITUTIONS –ROLE AND FUNCTIONS .................................................................................... 227
NATIONAL INSTITUTIONS ......................................................................................................... 227
INTERNATIONAL INSTITUTIONS ................................................................................................ 245
UNIT-XV..................................................................................................................................... 259
REPORTS AND INDICES ............................................................................................................. 259
REPORTS AND INDICES ............................................................................................................. 259
UNIT-XVI.................................................................................................................................... 273
ECONOMIC SURVEY 2017-18 .................................................................................................... 273
ECONOMIC SURVEY 2017-18 .................................................................................................... 273

UNIT-I
BIRD SIZE VIEW OF INDIA’S ECONOMY AND DEVELOPMENT
 BEFORE INDEPENDENCE
 PRE-LIBERALIZATION ERA
 LIBERALIZATION, PRIVATIZATION AND GLOBALIZATION [LPG] REFORMS
 COMPARATIVE DEVELOPMENT

INDIAN ECONOMY BEFORE INDEPENDENCE


1. In the middle of the 19th Century, which of these factors pushed cotton production in India that later gave a fillip to
the textile Industry
1. Organization of the Cotton Supply Association in India by the British
2. American Civil War that broke out in 1861
3. Construction of the first railways line in India in 1870s
Select the correct answer using the codes below.
a) 1 and 2 only
b) 2 and 3 only
c) 2 only
d) 1 and 3 only

Solution: c)
Justification:
Statement 1: Before the 1860s, three-fourths of raw cotton imports into Britain came from America. British cotton
manufacturers had for long been worried about this dependence on American supplies. 1857 the Cotton Supply
Association was founded in Britain, and in 1859 the Manchester Cotton Company was formed. Their objective was
“to encourage cotton production in every part of the world suited for its growth”.

Statement 2: When the American Civil War broke out in 1861, a wave of panic spread through cotton circles in
Britain. Raw cotton imports from America fell to less than three per cent of the normal: from over 2,000,000 bales
(of 400 lbs. each) in 1861 to 55,000 bales in 1862. Frantic messages were sent to India and elsewhere to increase
cotton exports to Britain. In Bombay, cotton merchants visited the cotton districts to assess supplies and encourage
cultivation.
India was seen as a country that could supply cotton to Lancashire if the American supply dried up. It possessed
suitable soil, a climate favourable to cotton cultivation, and cheap labour.

Statement 3: It was first constructed in the 1850s and only later became a medium for India to export cotton to
Britain and import manufactured textiles.

2. The consequences of the Great Depression (1929) for India include


1. Mass production of defence goods in India
2. Collapse of prices of many food grains
3. India became a net importer of precious metals like gold.
Select the correct answer using the codes below.
a) 1 and 2 only
b) 2 only
c) 2 and 3 only
d) 1 and 3 only

Solution: b)
Justification: First World War (FWW) led to mass production in the US and lowered goods prices. With rising wages
in US, consumption rocketed as people could afford more goods. Demand for consumer goods was fuelled by a
boom in housing construction and real estate business. This created the base for prosperity in the US leading to
greater investments and incomes.US started exporting capital to the World and became the largest overseas lender.
But since 1929 the world along with US economy slumped into a deep recession.

Implications for Colonial India


India was connected to global economy by way of imports and exports. Both halved between 1928 and
1934.

Negatives
Rural economy suffered more than urban economy. As international food grain prices collapsed, they declined in
India too. For e.g. wheat prices declined by 50% in the same period. Peasants who had borrowed in hope of good
prices in future, suffered from indebtedness because of falling demand and lower prices. Even as farm incomes
lowered, the British government refused to reduce revenue demand. Peasant indebtedness spiked.

Positives
India became a net exporter of previous metals like gold. Keynes thought that India's gold export will help global
economic recovery. Since urban incomes (salaries) were fixed, they were better off as prices had plunged. Effective
demand for goods increased. Industrial investment also grew as the government extended tariff protection to
industries, under the pressure of nationalist opinion.

PRE-LIBERALIZATION ERA

3. In post-independent India, the Mahalanobis model of economic growth adopted laid stress on
a) Focus on consumer goods instead of capital goods
b) Development of agriculture
c) Reliance on heavy industries and import substitution
d) Promoting Foreign Direct Investment

Solution: c)
Justification: At the time of formulation of the 2nd five year plan, Mahalanobis showed that to achieve a rapid long-
term rate of growth it would be essential to devote a major part of the investment outlay to building of basic heavy
industries. The pillars of Nehru - Mahalanobis strategy were - (a) high savings rate, (b) heavy industry bias, (c)
protectionist policies and public sector, (d) import substitution, and (e) socialistic pattern of society. However, in
critique of the Mahalanobis heavy industry development strategy, Professors Vakil and Brahmanand of Bombay
University put forward a wage-goods (consumer goods) model of development and suggested a development
strategy which accorded a top priority to agriculture and other wage-goods industries in sharp contrast to the
Mahalanobis heavy industry biased strategy of development.

4. The "commanding heights" of the economy, as known in post-independent India goes along with which of these
economic philosophies?
a) Capitalist
b) Socialist
c) Communist
d) Anarchic

Solution: b)
Justification: The Economic Model of India assigned a pivotal role to its public sector, which came to control
"commanding heights" of the economy after the Indian parliament adopted a resolution in 1956 to achieve a
"socialistic pattern of society" as a national goal. In pursuance to this goal, a large number of public sector units
(PSUs) were set up in different sectors of the economy. The expansion of the public sector did not take place only in
a traditional direction i.e. (correct provision of public goods and infrastructure), but also to non-traditional
command over the economy, in particular:

 Nationalization to occupy the "commanding heights" of the economy and regulations to protect them from
internal and foreign competition
 Widespread subsidies, explicit and implicit
 Administered prices and related regulations

5. Which of these sectors was chosen as the 'prime moving force' (PMF) of the Indian economy post- independence in
the initial days of planning and development?
a) Agriculture
b) Industry
c) Services sector
d) Foreign Investment

Solution: b)
Justification: Industry, infrastructure, science and technology and higher education, to name a few, received
greater emphasis in the early years after independence. This is because all these areas of development required
heavy capital investment as they had been severely avoided by the colonial rulers for over 150 years or so.
Increasing the growth of the economy and that too with a faster pace was the urgent need of the economy. Looking
at the pros and cons of the available options, India decided the industrial sector to be the 'prime moving force'
(PMF) of the economy—the logical choice for a faster growth (a fully established idea at that time, all over the
world).

6. The political leaders and the industrialists were very much aware and conscious about India’s economic inheritance
right after India became independent. Which of these were part of the broad consensus that emerged from the
policymakers?
1. State should provide only goods and the direct responsibility for development should be given to the private sector.
2. Foreign investments should be discouraged.
3. Economic development of India needs to be planned.

Select the correct answer using the codes below.


a) 1 and 2 only
b) 2 and 3 only
c) 3 only
d) 1 only

Solution: b)
Justification: Statement 1: Private sector was at an infant stage and could not tackle the core developmental issues.
So, state must take complete responsibility for development.
Statement 2: This is because of the colonial drain theory as popularized by Dadabhai Naoroji.
Statement 3: This was because the core sectors (e.g. steel, iron, cement etc.) were underdeveloped and their
expansion was of utmost importance for economic development. Planned development was agreed upon.

LIBERALISATION, PRIVATIZATION AND GLOBALIZATION

7. Structural reforms, in the context of the historical ‘shock therapy’, were aimed at
a) Nationalization of the economy
b) Shedding taxation to boost individual and corporate income
c) Greater push to free markets
d) Promoting communism and socialism

Solution: c)
Justification: The collapse of communism was followed in most of these countries by a painful process of transition
from an authoritarian socialist system to a democratic capitalist system. The model of transition in Russia, Central
Asia and east Europe that was influenced by the World Bank and the IMF came to be known as ‘shock therapy’. Part
of it could be seen in how the economy was restructured post-breakdown, that involved more freedom to markets
and enterprise and shunning the red tape. Shock therapy varied in intensity and speed amongst the former second
world countries, but its direction and features were quite similar.

8. The New Economic Policy (NEP) 1991 was an outcome of


a) East Asian Financial Crisis
b) Stagflation in Indian economy for over a decade
c) Withdrawal of large chunks of liquidity by the Reserve Bank of India
d) Balance of Payments (BoP) crisis

Solution: d)
Justification: A BoP crisis is a situation when the foreign exchange that a nation has is not enough to finance the
external account deficit. Foreign exchange reserves in the late 1980s declined to a level that was not adequate to
finance imports for more than two weeks. There was also not sufficient foreign exchange to pay the interest that
needs to be paid to international lenders. Therefore, a BoP crisis was looming to avert which India resorted to the
help of WB and IMF. So, (d) is correct.

Hyperinflation is a situation of exceedingly high inflation (likes of 300-5000%). India was not suffering with
hyperinflation, however, inflation was high. Asian financial crisis happened in 1997 from which India was largely
insulated. Other objectives of the policy were to tackle high inflation, mismanagement of the economy, revive
growth prospects etc. However, the most appropriate reason is to avert a BoP crisis.

The NEP aimed towards creating a more competitive environment in the economy and removing the
barriers to entry and growth of firms. It can broadly be classified into two groups: the stabilization measures and the
structural reform measures. Stabilization measures are short-term measures, intended to correct some of the
weaknesses that have developed in the balance of payments and to bring inflation under control. On the other
hand, structural reform policies are long-term measures, aimed at improving the efficiency of the economy and
increasing its international competitiveness by removing the rigidities in various segments of the Indian economy

9. Consider the following statements.

Assertion (A): Before 1991, majority of female workers, in India, were from rural areas, but post-1991 their majority
is found in urban areas.
Reason (R): Industrial and services sector have grown faster than agriculture after 1991 post the introduction of
New Economic Policy.

In the context of the above, which of these is correct?


a) A is correct, and R is an appropriate explanation of A.
b) A is correct, but R is not an appropriate explanation of A.
c) A is correct, but R is incorrect.
d) A is incorrect, but R is correct.

Solution: d)
Justification: Of the total 402 million workers, 275 million are males and 127 million females. This would mean that
51.7 percent of the total males and 25.6 percent of the total females are workers. Majority of female workers (87.3
percent) are from rural areas. This is also twice that of male workers, which may be due to their being employed
predominantly in activities like cultivation and agricultural labour. The trend has not changed much even after 1991.
So, A is wrong. In the urban areas, majority of female workers are engaged in Households industry and other work.
Interestingly, among marginal workers females outnumber the males. In three of the four categories, viz.
cultivators, agricultural labourers and household industries, female marginal workers outnumber male workers.

10. Which of these were significant elements of the New Industrial Policy, 1991?
1. De-reservation and de-licensing of certain industries
2. Revival of the MRTP Limit (Monopolistic and Restrictive Trade Practices)
3. Promotion to Foreign Investment
4. Commitment to a Foreign Exchange Management Act allowing improved current account convertibility
Select the correct answer using the codes below.
a) 1 and 2 only
b) 1, 3 and 4 only
c) 2 and 4 only
d) 1 and 3 only

Solution: b)
Justification: Statement 1: This policy abolished the Industrial licensing for all industries except for a short list of 18
industries. This list of 18 industries was further pruned in 1999 whereby the number reduced to six industries viz.
drugs and pharmaceuticals, hazardous chemicals, explosives such as gun powder and detonating fuses, tobacco
products, alcoholic drinks, and electronic, aerospace and defence equipment.
Statement 2: The limit was abolished and companies were allowed to expand freely with only certain limitations.
Statement 3: This was the first Industrial policy in which foreign companies were allowed to have majority stake in
India. In 47 high priority industries, up to 51% FDI was allowed. For export trading houses, FDI up to 74% was
allowed. Today, there are numerous sectors in the economy where government allows 100% FDI.
Statement 4: Restrictions on exchange of currency for the purposes of current account (such as trade or
remittances) were revoked.

11. Agriculture was chosen as the Prime Moving force of the economy since 2002 on the advice of the Planning
Commission. As a major policy shift, it was aimed at solving which of the following issues affecting the Indian
Economy?
1. Boosting aggregate demand in the economy
2. Ending the production monopoly of the PSUs in the agricultural sector
3. Increasing rural employment

Select the correct answer using the codes below.


a) 3 only
b) 1 and 3 only
c) 1, 2 and 3
d) 1 and 2 only

Solution: b)
Justification: Statement 1: Agriculture is a key feed to other industries, for e.g. food processing sector. An increase
in agricultural growth directly translates to a growth in other sectors of the economy.
Statement 2: The movement was intended not to cut down competition in any particular sector of the economy
rather to facilitate growth in all the sectors.
Statement 3: Investment in agriculture also involves creation of capital goods such as warehousing, irrigation
infrastructure which both directly and indirectly generates employment.
The other major challenges to be solved were:
 Food Security, which in a sense means ensuring adequate infrastructure in rural areas in agriculture. It is
because food security is not just about productivity; it is also about accessibility of food.
 Market Failure: It also had to solve the case of India being a market failure. It means that goods and services are
available in the economy, but there is little demand.
This is because majority of population lives on agriculture which does not provide them with good purchasing
power. With this policy shift their incomes were supposed to be improved and market demand be boosted. This
would also involve generating more rural employment.

12. India agreed to the conditionalities of World Bank and IMF and announced the New Economic Policy (NEP) 1991 to
avert
a) A Balance of Payments (BoP) crisis
b) A situation of hyperinflation in the economy
c) A situation of monetary depression in the Indian economy
d) The ill spread effects of the Asian Financial Crisis

Solution: a)
Justification: Foreign exchange reserves in the late 1980s declined to a level that was not adequate to finance
imports for more than two weeks. There was also not sufficient foreign exchange to pay the interest that needs to
be paid to international lenders. Therefore, a BoP crisis was looming to avert which India resorted to the help of WB
and IMF.
Other objectives of the policy were to tackle high inflation, mismanagement of the economy, revive growth
prospects etc. However, the most appropriate reason is to avert a BoP crisis.
The NEP aimed towards creating a more competitive environment in the economy and removing the barriers to
entry and growth of firms. It can broadly be classified into two groups: the stabilization measures and the structural
reform measures.

13. Which of the following has been peculiar or noteworthy to the Indian economic growth story?
1. Services sector has always grown at a slower rate than any other sector of the economy.
2. Agriculture sector has always employed the largest number of workers in the economy.
3. Services sector contributes more to the GDP than agriculture and manufacturing even though India is not an
industrially developed country.
Select the correct answer using the codes below.
a) 1 and 2 only
b) 2 and 3 only
c) 3 only
d) 1 and 3 only

Solution: b)
Justification: As a country develops, it undergoes ‘structural change’. Usually, with development, the share of
agriculture declines and the share of industry become dominant. At higher levels of development, the service sector
contributes more to the GDP than the other two sectors. In the case of India, the structural change is ‘peculiar’ with
regard to its sectoral composition especially services sector. Services sector has grown much faster compared to
other sectors of the economy post-liberalization. Traditionally services sector growth is propelled by the
manufacturing sector but in India services has overtaken manufacturing, in terms of GDP, even though
manufacturing has not been fully developed. Hence, the service sector contributes more to the GDP (over 60%) than
agriculture (14%) and manufacturing even though India is not a developed country

COMPARATIVE ECONOMIC DEVELOPMENT


UNIT-II
BASIC CONECPTS OF ECONOMICS
 MICRO ECONOMICS
 MACRO ECONOMICS

14. What are ‘Wage Goods’?


a) Goods consumed by the salaried class
b) Heavy capital goods used for income generation in a nation
c) Goods produced by the organized sector
d) Export based goods

Solution: a)
Justification: Wage goods, in contrast to capital goods, are consumed by the workers of the country. In the initial
phase of Indian economic development, it was alleged that wage goods were given a lesser priority as compared to
the capital goods that require a specialized workforce and heavy investment to pick up. This industrialization-led
strategy of development neglected the constraint of wage goods availability on two things: Employment generation
as demand for workers and thus wage goods increases post-economic growth, and its potential influence on
inflation that is caused by the shortage of wage goods as a nation’s GDP starts growing. However, with the renewed
focus on catching up with National GDP, wage goods are no longer neglected
MICRO ECONOMICS –BASIC CONCEPTS

15. The concept of Engel’s elasticity, in Economics, is used to analyze


a) Inward foreign remittances
b) Consumer Behavior
c) Impact of Monetary Policy
d) Government Budgeting

Solution: b)
Justification: The Economic Survey mentions this concept, which is one of the most basic concepts of
Microeconomics. Since UPSC keeps asking micro related Qs, this one is in order. One of the ways to analyze the
consumer’s behavior is through Engel’s elasticity. Table 1 below gives the Engel’s elasticity for some selected group
of commodities. It may be mentioned that Engel elasticity calculated here refer to the responsiveness of PFCE of a
particular commodity group to the aggregate PFCE, at constant 2011-12 prices.

As expected, the Engel elasticity of food items is significantly less than one, confirming the hypothesis that as the
income levels rise, the expenditure on food increases less than proportionately.
Within food, the elasticity of products like eggs, meat and fish, vegetables, etc. is higher than that on items like
bread, cereals and pulses, etc. While the elasticity of expenditure on health is significantly higher than unity,
surprisingly that of education is a little less than unity

MACRO ECONOMICS -BASIC CONCEPTS

16. According to Ricardian ideas, a landowner should have a claim only to

a) Half of the produce of an agricultural land


b) ‘Average rent’ that prevailed at a given time
c) Taxable part of the produce
d) The produce voluntarily given by the cultivators

Solution: b)
Justification: As per David Ricardo, an economist of the classical period, when the land yielded more than this
“average rent”, the landowner had a surplus that the state needed to tax. If tax was not levied, cultivators were
likely to turn into rentiers, and their surplus income was unlikely to be productively invested in the improvement of
the land. Many British officials in India thought that the history of Bengal confirmed Ricardo’s theory. There the
zamindars seemed to have turned into rentiers, leasing out land and living on the rental incomes. It was therefore
necessary, the British officials now felt, to have a different system which led to devising of various land revenue
arrangements by the British.

17. What is Paradox of thrift?

a) When people try to save more, they actually end up saving less.
b) When a nation tries to produce more, it ends up producing less.
c) When a bank lends, the borrowers do not receive an amount less than that was lent to them
d) When a nation exports more, its total savings decline.

Solution: a)
Justification: Paradox of thrift was popularized by the renowned economist John Maynard Keynes. It states that
individuals try to save more during an economic recession, which essentially leads to a fall in aggregate demand and
hence in economic growth. Such a situation is harmful for everybody as investments give lower returns than normal.
Keynes further said that such a mass increase in savings eventually hurts the economy as a whole. This theory was
heavily criticized by non-Keynesian economists on the ground that an increase in savings allows banks to lend
more. This will make interest rates go down and lead to an increase in lending and, therefore, spending.

18. Characteristics that are central to capitalism include


1. A Market determined price system
2. Competitive markets
3. Ownership of factors of production by the State
Select the correct answer using the codes below.
a) 1 only
b) 1 and 2 only
c) 2 and 3 only
d) 1, 2 and 3

Solution: b)
Justification: Statement 1 and 2: Private property, capital accumulation, wage labour, voluntary exchange, a price
system and competitive markets are some of the major features of capitalism. In a capitalist market economy,
decision-making and investment are determined by every owner of wealth, property or production ability in
financial and capital markets, whereas prices and the distribution of goods and services are mainly determined by
competition in goods and services markets.

Statement 3: This is a feature of either communism or extreme socialism. Capitalism is an economic system based
upon private ownership of the means of production and their operation for profit. Different forms of capitalism
feature varying degrees of free markets, public ownership, obstacles to free competition and state-sanctioned
social policies. The degree of competition in markets, the role of intervention and regulation and the scope of state
ownership vary across different models of capitalism

19. In the situation of an Economic Boom, which of the following are witnessed?
1. The economy heats up and a demand-supply lag is visible.
2. There can be a shortage of investible capital and savings.
3. A buyer’s market may be created.

Select the correct answer using the codes below.


a) 1 and 2 only
b) 2 and 3 only
c) 1 and 3 only
d) 1 only

Solution: a)
Justification: Statement 2: Due to a mismatch between supply and demand, and higher consumption (and lower
savings), there can be a shortage of investible capital.
Due to this the economy heats up and supply-side bottlenecks become clearly visible. So, statement 1 is also
associated here.
Overheating of an economy occurs when its productive capacity is unable to keep pace with growing aggregate
demand leading to inflation.
Statement 3: A seller’s market is an economic situation in which goods or shares are scarce and sellers can keep
prices high. During boom as demand outstrips supply, the situation can often be seen across.
The situation is reversed in a buyer’s market.

20. Pareto optimality” is a situation where


a) Resources are allocated in a way in which it is impossible to make any one individual better off without making at
least one individual worse off
b) Resources are allocated in the most socially desirable way
c) Resource are allocated as per democratic means based on the principles of equity and fairness
d) All of the above apply

Solution: a)
Justification: The situation in option (a) may not be socially desirable.
For e.g. we may take away Rs. 10,000 from a billionaire and give it to a BPL family; it would hardly make a difference
to the welfare of the rich but would matter a lot for the BPL family.
So, while this transfer of Rs. 10,000 is socially desirable, it would not be considered pareto optimum, nor is it done
based on democratic means.
The concept is purely based on the idea that greater economic efficiency leads to more economic welfare, which as
we just verified, may not be correct.

21. An economy heats up due to


1. Demand deficit
2. Excess supply
3. High liquidity infusion
Select the correct answer using the codes below.
a) 1 and 2 only
b) 2 only
c) 3 only
d) 2 and 3 only

Solution: c)
Justification: Overheating of an economy occurs when its productive capacity is unable to keep pace with growing
aggregate demand leading to inflation.
When there is excess demand and low supply, high consumption and low savings can lead to a situation where
there is a shortage of investible capital.
The economy is unable to supply demanded goods and prices start increases.

22. A seller’s market is an economic situation in which


a) Goods are scarce and sellers can keep prices high
b) There is only one buyer and a large number of seller leading to monopsony.
c) Goods are plenty and sellers are forced to keep prices low
d) There is only one seller and a large number of buyer leading to monopoly.

Solution: a)
Justification: this is a market situation wherein sellers are outnumbered by buyers by a big margin and the quantity
of goods available for sale becomes far less than the quantity demanded. During boom as demand outstrips supply,
the situation can often be seen across. For e.g. in summer, there is a huge demand for mangoes. In a certain
location where there are just two mango sellers and the demand is extremely high for the same, then that would be
a seller's market. Since the demand is high, the sellers will be able to charge a high price for mangoes and get better
sale conditions.

23. Indicative planning is most likely to be followed by which of these economies?


a) Capitalist economies
b) Mixed economies
c) Communist economies
d) Primary economies

Solution: b)
Justification: Option A: Purely Capitalist economies would generally not follow any planning and let the market
decide how much and when to produce.
Option B and C: In communist economies the planning is imperative not indicative in nature. So, mixed economies
are fit for indicative planning where a plan making body (from the State) sets broad numerical targets to be
achieved with the help of market forces.

24. The Phillips Curve suggests that


a) Lower the inflation, lower is the unemployment
b) Higher the environmental degradation, higher is the economic inequality
Which of the above is/are correct?
a) 1 only
b) 2 only
c) Both 1 and 2
d) None

Solution: d)
Justification: Statement 2: It does not discuss about inequality or environmental degradation.
Statement 1: As per the curve there is a ‘trade off’ between inflation and unemployment i.e. an inverse relationship
between them.
The curve suggests that lower the inflation, higher the unemployment and higher the inflation, lower the
unemployment.
During 1960s, this idea was among the most important theories of the modern economists. However, economists
later contested this curve and suggested some modifications to it.

25. A shortage of investible capital in an economy can result from


1. Very high and erratic inflation
2. Low savings
3. High borrowing costs
Select the correct answer using the codes below.
a) 1 and 2 only
b) 2 and 3 only
c) 3 only
d) 1, 2 and 3

Solution: d)
Justification: Statement 1: High and erratic inflation deters investors from making investment decisions because
real returns on investments are uncertain.
Statement 2 and 3: Savings forms the base of the money that is borrowed for infrastructure building in the
economy. Low savings and high demand of capital lead to high borrowing costs and deter investors.

26. The main difference between secondary activities and tertiary activities is that
1. The expertise provided by tertiary services relies more on the specialised skills and knowledge of the workers than
on the production techniques or the machinery as in secondary sector.
2. Tertiary activities involve the commercial output of services rather than the production of tangible goods as in the
secondary sector.
Which of the above is/are correct?
a) 1 only
b) 2 only
c) Both 1 and 2
d) None

Solution: c)
Justification: Tertiary activities are an important part of the services sector as distinct from the secondary (industrial
and manufacturing sector). Tertiary activities include both production and exchange. The production involves the
‘provision’ of services that are ‘consumed’. The output is indirectly measured in terms of wages and salaries. They
are not directly involved in the processing of physical raw materials. Common examples are the work of a plumber,
electrician or a doctor.

27. As per Classical Economic thought


Assertion (A): Markets, even if left free, always require government intervention.
Reason (R): Buyers and sellers acting in their self-interest fail to coordinate with each other.
In the context of the above, which of these is correct?
a) A is correct, and R is an appropriate explanation of A.
b) A is correct, but R is not an appropriate explanation of A.
c) A is correct, but R is incorrect.
d) Both A and R are incorrect.
Solution: d)
Justification: Classical thought is also known as the non-interventionist school. As per them, markets function best
without government interference. Classical economists observe that markets generally regulate themselves, when
free of coercion. Adam Smith referred to this as a metaphorical "invisible hand," which moves markets toward their
natural equilibrium. In contrast to classical economics, Keynesian economics supports policies such as government
intervention, deficit spending, control of the money supply, and a progressive income tax to counter recession and
income inequality. Most classical economists reject these ideas. They assert that state intervention makes situations
of economic slowdown worse.

28. Information based and R&D based services would tend to fall under which of the following sectors?
a) Tertiary activities
b) Trade and commerce services
c) Quaternary activities
d) Telecommunication services

Solution: c)
Justification: This is a table from NCERT that would help you remember the finer distinctions between various
service types. The quaternary sector of the economy is a way to describe a knowledge-based part of the economy,
which typically includes services such as information technology, information-generation and - sharing, media, and
research and development, as well as knowledge-based services like consultation, education etc.
29. Rolling Back of the state’ is a popular catchphrase in economics. It is least likely to be used in the context of a
a) Communist economy
b) Socialist Economy
c) Economy following indicative planning
d) Capitalist economy

Solution: a)
Justification: Rolling back means that private sector has been given more space to operate and the state has
reduced its role from that of a regulator to that of a facilitator.
This will occur most likely in a capitalist or socialist economy and least in a communist economy where the state is
the chief architect of the economy.

30. In a perfectly competitive market, the equilibrium market price of goods is determined by
a) Total capital stock of the economy
b) Marginal cost and Marginal revenue
c) Average cost of production
d) Employment-output ratio

Solution: b)
Justification: There are a very large number of buyers and sellers in a perfectly competitive market and since there
is no monopoly of a single firm, all firms jointly take the prices.
Marginal cost means the cost of producing one additional unit of product. Marginal revenue is the price that every
additional unit will fetch in the market. This is based on the demand of the buyers.
The equilibrium market price of goods is a point where marginal cost becomes equal to the marginal revenue.
This is one the most fundamental concepts of economic theory and used by many industrial firms in deciding the
amount of output and prices. There have been instance when questions on the ‘marginal’ concept have been asked
by UPSC.

31. Public goods are those goods that are


a) Subsidized by the government to reduce the cost to the consumer
b) To be ideally provided by the government since private enterprises may not find it economically viable
c) Produced by the state owned enterprises in strategic interest
d) Consumed by more than one individual

Solution: b)
Justification: A public good is a product that one individual can consume without reducing its availability to another
individual and from which no one is excluded. For e.g. clean air, national defence, law and order, flood control
system, lighthouses etc.
Even LPG is subsidized, but it is not a public good. So, (a) is incorrect.
PSEs produce steel, defence equipment and weapons too, but none of them are public goods. So, (c) is also
incorrect.
Many public goods may at times be subject to excessive use resulting in negative externalities affecting all users; for
example air pollution and traffic congestion. Public goods problems are often closely related to the "free-rider"
problem, in which people not paying for the good may continue to access it.

These may also be called social goods as they look essential from the social justice and well-being perspective for
the people. Education, healthcare, sanitation, drinking water, nutrition, caring for the handicapped and old etc.,
come under this category.

32. The “trickle down” strategy of poverty reduction relies on


a) International Aid
b) Ecological Conservation
c) Subsidized public services
d) Economic growth

Solution: d)
Justification: It was felt that rapid industrial development and transformation of agriculture through green
revolution in select regions would benefit the underdeveloped regions and the more backward sections of the
community.

The argument is that promoting economic growth, increases total income in society, creating more jobs and income,
which could be redistributed.
This was the major focus of planning in the 1950s and early 1960s.

However, growth failed to “trickle down” and it widened relative poverty because it benefitted only the highly
skilled and wealthy classes more than those at the bottom. This led to the approach of providing subsidized public
services and generating gainful employment for the poor.

33. In Market socialism


a) Shift to imperative planning occurs in a capitalistic economy
b) Complete control of market resources by the State is a norm.
c) Production is not centrally planned but mediated through the market.
d) Factors of production are owned entirely by workers in each company

Solution: c)
Justification: Market Socialism is an economic system in which the means of production are owned either by the
state or by the workers in each company (meaning in general that "profits" in each company are distributed
between them: profit sharing) and the production is not centrally planned but mediated through the market.

Its central idea is that the market is not a mechanism exclusive to capitalism and that it is fully compatible with
collective worker ownership over the means of production — which is one of the fundamental principles of
socialism. Proponents of market socialism argue that it combines the advantages of a market economy with those
of socialist economics. The theory is fundamentally contradictory to orthodox Marxism.

34. The main concern(s) of Socialism is/are


1. Minimize existing inequality and distribute resources justly
2. Prevent market based distribution of goods and services
3. Cut down government regulation and planning to the least
Select the correct answer using the codes below.
a) 1 only
b) 1 and 2 only
c) 1 and 3 only
d) 2 and 3 only

Solution: a)
Justification: Here we talk about the Indian version of socialism and why is it the way it is.
Although advocates of socialism are not entirely opposed to the market, they favour some kind of government
regulation, planning and control over certain key areas such as education and health care. In India, the eminent
socialist thinker Rammanohar Lohia, identified five kinds of inequalities that need to be fought against
simultaneously: inequality between man and woman, inequality based on skin colour, caste-based inequality,
colonial rule of some countries over others, and, of course, economic inequality. This might appear a self-evident
idea today. But during Lohia’s time it was common for the socialists to argue that class inequality was the only
form of inequality worth struggling against. Other inequalities did not matter or would end automatically if
economic inequality could be ended.

35. With reference to the term Consumer surplus, consider the following statements.
1. Consumer surplus explains the situation of a buyers’ market where supply exceeds the demand.
2. Economies with a consumer surplus are developed economies.
Which of the above is/are correct?
a) 1 only
b) 2 only
c) Both 1 and 2
d) None

Solution: d)
Justification: Both statements are wrong. The benefits enjoyed by consumers as a result of being able to purchase a
product for a price that is less than the most that they would be willing to pay is what is termed as consumer
surplus. It is one of basic concepts to measure consumer wellness or welfare in economics. Total social surplus is
composed of consumer surplus and producer surplus. It is a measure of consumer satisfaction in terms of utility.

36. US economist Richard Thaler won the 2017 Nobel Economics Prize for his contributions in the field of behavioural
economics. The study of behavioural economics includes how market decisions are made and the mechanisms that
drive public choice. In this context, consider the following:
1. As per Heuristics, most decisions made by humans are done using rules of thumb.
2. Mis-pricings and non-rational decision are part of market behaviour which are considered as market inefficiencies.
Which of the above is/are correct?
a) 1 only
b) 2 only
c) Both 1 and 2
d) None

Solution: c)
Justification: Statement 1: Research says that Humans make nearly 95% of their decisions using mental shortcuts or
rules of thumb. This means that decisions do not involve elaborate reasoning and often based on simplistic rules
that are applied to a wide range of choices, even if those rules don’t really fit there.
Statement 2: Such mis-pricing can be due to lack of competition in the market, e.g. monopoly or due to irrational
decision-making for e.g. selling stocks in hoards in the event of a rumour.
These cause market turbulence and inefficiencies.
By exploring the consequences of limited rationality, social preferences, and lack of self- control, the Nobel laureate
has shown how these human traits systematically affect individual decisions as well as market outcomes. In total,
Richard Thaler’s contributions have built a bridge between the economic and psychological analyses of individual
decision-making.

37. If an economy predominantly keeps producing consumption goods at the expense of capital goods it will lead to
1. Short term as well as long term inflation
2. A roadblock in the long-term growth
Which of the above is/are correct?
a) 1 only
b) 2 only
c) Both 1 and 2
d) None

Solution: c)
Justification: Capital goods are tools, implements and machines etc. that make production of consumption goods
like furniture possible. Since resources are limited in an economy, they can either be channelized for immediate
consumption or immediate investment to feed long-term consumption. If they are channelized for immediate
consumption (i.e. say we are producing more aerated drinks but not drink making machines), a time would come
when demand would overshoot supply and economic stagnation would follow. If however, a balance is maintained
by sacrificing some consumption goods to produce more capital goods, we can have more aerated drinks in the
future as there will be more machines to fulfil demand.

Statement 1: As production expands it leads to an increase in the income of the workers and thus more demand for
goods and services. But this increased demand can be fulfilled only when there are capital goods to produce the
additionally required consumption goods. Otherwise this leads to inflation.

Statement 2: If this trend keeps going there will be very few capital goods left in the economy as the left overs will
start depreciating. It will become increasingly difficult to produce consumption goods leading to a halt in economic
growth. This is why the RBI Governor has prescribed moving from consumption spending to investment spending
for the Indian economy.

38. A country’s money multiplier depends on which of these factors?


1. Fraction of cash held by individuals and businesses
2. Fraction of cash banks hold as reserves
Which of the above is/are correct?
a) 1 only
b) 2 only
c) Both 1 and 2
d) None

Solution: c)
Justification:
Consider this date from Bloomberg: Every one rupee of central bank money in India is able to generate around 6
rupees of money supply in the economy. This ratio is called money multiplier. A higher money multiplier indicates
that the banking system generates a higher money supply out of money given by central bank. In India, the recent
push to financial inclusion has led to people holding less cash in hand (relative to deposits) leading to an increase in
the money multiplier. The more individuals hold cash in hand, the less the banking system will be able to create
money and hence a lower value for the multiplier. In other words, cash in hand acts as a leakage for the banking
system. Similarly, reserves that banks hold with the central bank also amount to a leakage, which again reduces the
money multiplier. It should be noted that central banks generally tell the banks to maintain a part of their deposits
as reserves, called the cash reserve ratio. So in essence, it is only the excess reserves (that banks maintain over and
above the central bank’s requirement) that constitute leakages. Indians have a tendency to hold more cash (as a
percentage of banking deposits) compared with people in the US or Europe, which depresses India’s multiplier
value.
39. A high savings rate in the economy can lead to
1. Lower interest rate for investments
2. Lower consumption demand leading to a moderation of inflation
3. Reduced revenue deficit in the public budget
Select the correct answer using the codes below.
a) 1 and 2 only
b) 2 and 3 only
c) 3 only
d) 1, 2 and 3

Solution: a)
Justification: Statement 1: Investments in the economy are financed either through private savings, public savings
or government investments; all of which require savings by some economic agent. More the saving, more are the
funds in the market, and thus based on the supply-demand principle, lower will be the interest rate.
Statement 2: Higher savings mean lower consumption and thus lower inflation.
Statement 3: This has nothing to do with the revenue deficit. However, this might have a relation with the capital
side of the budget where government borrowing cost might be reduced due to higher savings.

40. Aggregate Demand in an economy usually increases with an increase in


1. National Income
2. Investment
3. Government spending
Select the correct answer using the codes below.
a) 1 and 2 only
b) 2 and 3 only
c) 1 and 3 only
d) 1, 2 and 3

Solution: d)
Justification: Under the effective demand principle, the equilibrium output of the final goods is equal to ex ante
aggregate demand.
Thus, a basic equation used for determining aggregate demand is
AD = Y = C I G, where C is usually an increasing function of national income. Y is total output. Based on the above, if
any of the factors are increased, it increases the AD.

41. Which of the following are the major economic challenges for a State Economy?
1. Deciding what goods to produce
2. Allocating supply chain networks to private sector
3. Ensuring a balance between supply and demand of goods
4. To ascertain the level of investment required for an optimal productive capacity in the economy
Select the correct answer using the codes below.
a) 1, 2 and 3 only
b) 3 and 4 only
c) 1, 3 and 4 only
d) 2 and 4 only

Solution: c)
Justification: Every economy meets with certain challenge. One, to ascertain the availability of the goods and
services required by the population and second, the presence of the supply network. Every economy has to, at first,
guarantee the required level of goods and services out of its production process. For this, proper level of production
capacity should be built which requires a particular level of capital formation or investment. From where the
investible funds will be managed is altogether a separate question.
Whether the investment will come from the government, the domestic private sector or the foreigners?
Once these details are cleared and selected as per the socio-economic condition of the economy, a proper
distribution network for goods and services produced is assured.
Statement 3: But, in a State Economy the private sector does not play any role and is nationalized by the
government.

42. Which of these is/are examples of Fixed capital formation?


1) Accumulation of foreign exchange reserve
2) Road and bridge construction
3) Energy infrastructure
4) Office equipment, such as computers

Select the correct answer using the codes below.

a) 2, 3 and 4 only
b) 1, 2 and 4 only
c) 1, 3 and 4 only
d) 2 and 3 only

Solution: a)
Justification: Statement 1: Currency is not considered as fixed capital, it is liquid capital. Fixed capital are the assets
used in the productive process. Others examples include Building or expanding existing factory, Purchase of
transport equipment and all other machineries used in the productive process. Generally, the higher the capital
formation of an economy, the faster an economy can grow its aggregate income. Increasing an economy's capital
stock also increases its capacity for production, which means an economy can produce more. Producing more goods
and services can lead to an increase in national income levels.

43. Often public goods cannot be provided through market mechanism. This is because

1) It is difficult to separate individual users of the service.


2) The good does not decrease in value even after being consumed by additional users.

Which of the above is/are correct?


a) 1 only
b) 2 only
c) Both 1 and 2
d) None

Solution: c)
Justification: Statement 1: A good can be priced only when you know who is using and how much is being used,
for e.g. a soap is used by one person and it is quite easy to make them pay for it. But, there are some goods where
doing this is not possible. For e.g. if providing clean air is a public good, then it is not possible to separate users who
consumer a fixed quantity of clean air, and it is also not possible to exclude someone from using them. In such a
case, a central authority or body must provide the good at either a constant price to all, or make it free of cost for
users. However, it should be noted that, in either of these cases, the body cannot use the market principles of
pricing.

44. Consider the following about imperative planning.

1) This planning process is followed by socialist economies.


2) Economic planning decisions are made through a central planning authority instead of a market system.
3) This process assumes that state and non-state actors have equitable control over economic resources.

Select the correct answer using the codes below.


a) 1 and 2 only
b) 3 only
c) 2 and 3 only
d) 1, 2 and 3

Solution: a)
Justification: In this system, allocation of resources, the mix of output and the distribution of output among the
people (i.e., ‘What, How and For Whom’ problems) are determined centrally in accordance with the predetermined
plans and targets. Administrative control and regulation from the central planning authority flows in all directions.
Because of control over the available resources of the country by the state, resources are allocated in such a way
that production becomes maximum, people get goods and services in fixed quantities at fixed prices, and welfare of
the nation gets maximized. Thus, communist China was the purest example of such planning.

45. The idea of ‘non-interference’ as proposed by Adam Smith is essentially the philosophy of
a) Political liberalism
b) Capitalism
c) Social Marxism
d) Leninism

Solution: b)
Justification: Option A: Liberalism is a political philosophy or worldview founded on ideas such as freedom of
speech, freedom of the press, freedom of religion, free markets, civil rights, democratic societies, secular
governments, gender equality, and international cooperation.
Option B: This involves free market economies where each individual tries to maximize his or her own interest.
Option C: This is usually understood to be against the system of capitalism
Option D: This is essentially Marxism as applied by Lenin. It stands for the achievement of a dictatorship of the
proletariat (workforce), as a political prelude to the establishment of socialism (a system that is more or less
opposite of capitalism).

46. Fiscal consolidation can be achieved by


1. Reducing inefficient subsidies
2. Borrowing from domestic market rather than relying on foreign grants
3. Improve tax base
4. Enhancing the resource base of state governments to reduce their financial dependency on the Centre
Select the correct answer using the codes below.
a) 1, 3 and 4 only
b) 2 and 3 only
c) 1, 2, 3 and 4
d) 1 and 4 only
Solution: b)
Justification: Statement 1: It reduces wasteful government expenditure and size of fiscal deficit.
Statement 2: Borrowing, if replaced with grants, will further inflate fiscal deficit because it comes with an interest
payment obligation.
Statement 3: A better tax base yields more revenue and thus less revenue deficit.
Statement 4: A major part of the Union receipts go to states to fulfil their plan expenditure. Market borrowing will
reduce such transfers and make states self-reliant

47. The term ‘State-market mix’ can be applied to which of the following hypothetical economic situations?

1) The government sub-contracting the construction of a national highway to a private developer


2) An economy where all goods and services are produced by state-owned firms but bought by private parties
3) Opening bids for the management of a state-owned railway station to private companies

Select the correct answer using the codes below.


a) 1 only
b) 1 and 3 only
c) 2 and 3 only
d) 1 and 2 only

Solution: b)
Justification: The question essentially refers to a mixed economy where both state and the market undertake
economic activities.
Statement 1: In this case, the act of the state sub-letting an activity under its economic domain shows a state-
market mix approach.
Statement 2 and 3: if all factors of production are owned and used by the state, it is a socialist or communist
economy.

48. Which of the following characterize Fiscal decentralization?


1. Shifting general revenues from taxes collected by the central government to local governments
2. Spending and tax decisions by the government in the economy reflect local preferences.
Which of the above is/are correct?
a) 1 only
b) 2 only
c) Both 1 and 2
d) None

Solution: c)
Justification: Fiscal decentralization (FD) means devolution of power and responsibilities of national (central),
government towards sub-national (local), governments.
Fiscal decentralization can take many forms including a) self-financing or cost recovery through user charges, b) co-
financing or co-production arrangements through which the users participate in providing services and
infrastructure through monetary or labor contributions; c) expansion of local revenues through property or sales
taxes, or indirect charges. It also includes d) intergovernmental transfers that shift general revenues from taxes
collected by the central government to local governments for general or specific uses; and e) authorization of
municipal borrowing and the mobilization of either national or local government resources through loan
guarantees. This is captured in the idea that spending and tax decisions must reflect local preferences as far as
possible. That is, the share of own revenues (compared to devolved sources) in total revenues at lower levels of
government should also reflect local preferences.

49. Economically optimal or efficient allocation of goods and services is always reached in an economy when
1) Every resource in the economy isemployed.
2) Resources go to those who need them the most.

Which of the above is/are correct?

a) 1 only
b) 2 only
c) Both 1 and 2
d) None
Solution: d)
Justification: Optimal allocation happens when goods and services are distributed according to consumer
preferences and needs of corporations.
Statement 1: It is possible that by employing all resources, we may reach a stage of disguised unemployment, for
e.g. as common in rural areas.
Statement 2: This is distribution based on equity and welfare concerns, which may not be efficient from an
economic point of view.

50. A Communist economy advocates


1. State ownership of all properties including labour in the economy
2. Absolute power to state in running and managing the economy

Which of the above is/are correct?

a) 1 only
b) 2 only
c) Both 1 and 2
d) None

Solution: c)
Justification: Communism is an economic system where the collective owns the factors of production. The four
factors of production are labor, entrepreneurship, capital goods and natural resources.
The following form major features of the communist economies:
 Abolition of property in land and application of all rents of land to public purposes.
 A heavy progressive or graduated income tax.
 Abolition of all right of inheritance.

UNIT-III
NATIONAL INCOME CALCULATION

NATIONAL INCOME

51. The national income estimates of GDP are released quarterly by the
a) Chief Economic Advisor, Ministry of Finance
b) Reserve Bank of India (RBI)
c) Central Statistics Office (CSO)
d) Department of Commerce and Industry

Solution: c)
Justification: The Central Statistics Office coordinates the statistical activities in the country and evolves statistical
standards. It is headed by a Director General assisted by 5 Additional Director Generals.
CSO has five Divisions of which the NAD (National Accounts Division) is responsible for the preparation of national
accounts, which includes Gross Domestic Product, Government and Private Final Consumption Expenditure, Fixed
Capital Formation and other macro-economic aggregates.
For e.g. for the second quarter of 2017-18, as reported by the CSO, GDP has grown at 6.3% year-on- year compared
to 5.7% in the first quarter but still lower than 7.5% a year ago.
The manufacturing sector in the second quarter of 2017-18 grew by 7 per cent against 1.2% in the previous quarter.
This is really a turnaround. In the corresponding quarter in the previous year, the growth rate was 7.7%.
Agriculture, forestry and fishing sectors are estimated to have grown by 1.7 per cent.

52. Why is Gross National Product (GNP) a better measure of national income than Gross Domestic Product (GDP)?
1. GNP reflects an economy’s interconnection and interdependence on world economy, which is not captured by GDP.
2. GNP includes the idea of depreciation of national capital that is not accounted by GDP.
3. GNP accounts for the population headcount in the nation that GDP does not include.
Select the correct answer using the codes below.
a) 1 and 3 only
b) 1 only
c) 1 and 2 only
d) 2 and 3 only

Solution: b)
Justification: The normal formula is GNP = GDP Income from Abroad. But it becomes GNP = GDP (—Income from
Abroad) = GDP — Income from Abroad, in the case of India.
IFA usually includes remittances, trade balance and interest on external loans.
This is because usually the IFA component of India is negative. So, India's GNP is generally lower than its GDP.
The different uses of the concept GNP are as given below:
This is the 'national income' according to which the IMF ranks the nations of the world in terms of the volumes—at
Purchasing Power Parity (at PPP).
India is ranked as the 4th largest economy of the world (after the USA, China and Japan), while as per the nominal/
prevailing exchange rate of rupee, India is the 11th largest economy.
 It is the more exhaustive concept of national income than the GDP as it indicates towards the 'quantitative 'as
well as the 'qualitative 'aspects of the economy, i.e., the 'internal' as well as the 'external' strength of the
economy.
 It enables us to learn several facts about the production behaviour and pattern of an economy, such as, how
much the outside world is dependent on its product and how much it depends on the world for the same
(numerically shown by the size and net flow of its 'balance of trade') etc.
Statement 2: It is the Net National Income (NNI) or NNP or NDP that includes depreciation.

53. If the Gross Domestic Product (GDP) of a country is growing, it implies that

1. The nation must be experiencing an expansion of the share of manufacturing vis-à-vis other sectors in the GDP.
2. The nation must be experiencing a general rise in the prices of goods and services on an annual basis.

Which of the above is/are correct?

a) 1 only
b) 2 only
c) Both 1 and 2
d) None

Solution: d)
Justification: GDP calculates total value of goods and services produced within a year at market prices. It considers
production by all three sectors – primary, secondary and tertiary.
Statement 1: Manufacturing activity is considered a part of the secondary sector. Even if the GDP is expanding, it
does not mean that manufacturing must expand. The increase can come from services or agricultural sectors as
well.
Statement 2: If the demand is not growing more than the production of goods and services (owing to an increased
GDP), inflation will be under control.

54. Consider the following statements. If we take into account the economic growth data of last few years

1. India has consistently grown faster than US and all other OECD developed economies in Europe.
2. India’s average growth rate has been higher than that of China

Which of the above is/are correct?

a) 1 only
b) 2 only
c) Both 1 and 2
d) None

Solution: c)
Justification:

55. As per the presently followed system, Gross National Income (GDP) at Constant and Current Prices in India will be
equal in which of the following financial years?
a) 1951-52
b) 1991-92
c) 2004-05
d) 2011-12

Solution: d)
Justification: Constant prices are calculated at 2011-12 prices as per the latest series, and Current prices can only be
equal to constant prices in the year series which it starts from, i.e. 2011-12

56. Largest share in Gross World Product (GWP) is constituted by


a) Agriculture sector
b) Services sector
c) Industrial Sector
d) Tourism sector

Solution: b)
Justification: The gross world product (GWP) is the combined gross national product of all the countries in the
world. Because imports and exports balance exactly when considering the whole world, this also equals the total
global gross domestic product (GDP).In 2014, according to the CIA's World Fact book, the GWP totalled
approximately US$107.5 trillion in terms of purchasing power parity (PPP), and around US$78.28 trillion in nominal
terms .Share of services is close to 66%, followed by industrial sector and agriculture.

57. Gross Domestic Product (GDP) per capita of a country will necessarily increase if

1. Gross capital formation has increased


2. Export goods have increased
3. Index of Industrial production has increased
4. Higher remittances have been received in the previous year

Select the correct answer using the codes below.

a) 1 and 2 only
b) 1, 2 and 3 only
c) 2, 3 and 4 only
d) None of the above

Solution: d)
Justification: All these factors will be neutralized if population growth rate has surpassed economic growth rate.
Statement 1: If capital is formed, there can be more production and thus growth. Per capita income may or may not
increase. Overall income may increase.
Statement 2: Exports boost domestic production.
Statement 3: IIP increase signals rise in manufacturing and allied sectors.
Statement 4: Higher remittances lead to higher domestic spending and thus higher demand for goods and services.

58. What is/are the major criticisms of the Gross National Product (GDP) metric used to measure the status of an
economy?

1. It cannot be used to measure consumer welfare.


2. It suffers with the problem of double counting environmental degradation and then environmental restoration.
3. It includes the value of natural resources of the economy in the estimation.

Select the correct answer using the codes below.

a) 1 and 2 only
b) 2 only
c) 3 only
d) 1, 2 and 3

Solution: b)
Justification: Statement 1: It is used to measure consumer welfare, but it is not a good indicator of general
economic welfare. For e.g. in economic theory, consumer welfare usually increases with increased production and a
reduction in the amount if price that a consumer has to pay.

Statement 2: Critics suggest that GNP often includes the environment on the wrong side of the balance sheet
because if someone first pollutes and then another person cleans the pollution, both activities add to GNP making
environmental degradation frequently look good for the economy.

Critics of mainstream economics complain that GNP compiles spending that makes us worse off, spending that
allows us to stay in the same place, and spending that makes us better off all in a single measure, giving a nation no
clue if they are making progress or not.

Statement 3: GDP includes the value of only produced goods and services, not the natural resources as such.

59. During the last three financial years, growth rate of Gross Value Addition (GVA) at basic price at Constant (2011-12)
prices has been highest on an average for which of the following sectors?
a) Agriculture
b) Services
c) Mining & quarrying
d) Manufacturing

Solution: b)
Justification:
60. Which of these concepts is qualified as a “qualitative” rather than a “quantitative” progress?
a) Economic growth
b) Economic Development
c) Gross Domestic Product
d) Per capita income

Solution: b)
Justification: GDP or per capita income are indicators of economic growth, and do not cover qualitative aspects of
growth such as equality, education, health. Economic development covers all aspects of social and economic
development, for e.g. R&D, gender and caste equity in growth etc.

61. It is said that Net Domestic Product (NDP) is a more realistic assessment of an economy’s resources than Gross
Domestic Product (GDP). This is because

1. NDP takes into account taxes and subsidies paid which GDP does not.
2. NDP accounts for economic depreciation which GDP does not.

Which of the above is/are correct?

a) 1 only
b) 2 only
c) Both 1 and 2
d) None

Solution: b)
Justification: NDP is basically the GDP minus the total value of the ‘wear and tear’ (depreciation) that happened in
the assets while the goods and services were being produced.
So, if the depreciation levels are low, it may show the achievements of the economy in the area of research and
development which have tried cutting the levels of depreciation in a historical time period.
The governments of the economies decide and announce the rates by which assets depreciate (done in India by
the Ministry of Commerce and Industry) and a list is published.

62. Annual Economic growth can be most realistically determined by comparing which of these figures relating to two
successive years?
a) Gross Domestic Product (GDP) at market cost
b) Gross National Product (GNP) at factor cost
c) Net National Product (NNP) at market cost
d) Net National Product (NNP) at factor cost

Solution: d)
Justification: To calculate economic growth, you need to estimate the total actual production of the last year and
compare it to that of next year.
Option A: Market cost calculations include inflation, and thus a mere increase in prices will indicate an increase in
GDP even if there has not been an actual production increase. So, A is wrong.
Option B: GNP at factor cost is more realistic because it also takes into account the NFIA as well as factor cost (which
is the manufacturing cost). But, then it does not take into account the depreciation of goods in the economy which
effectively reduces the total effective production of last year.
Option C and D: By these means, option D is the best choice, as option C is calculated at market cost.

UNIT-IV
MONEY, BANKING AND INFLATION

MONEY AND BANKING

 MONEY SUPPLY AND MONETARY POLICY


 BANKS,NBFCS AND OTHER FINANCIAL INSTITUTIONS
 FINANCIAL INCLUSION

INFLATION

 CAUSES AND CONSEQUENCES


 MEASUREMENT-CPI,WPI, GDP Deflator
 INFLATION CONTROL

MONEY AND BANKING

MONEY

63. A nation’s currency will depreciate in value if

1. The nation stops exports completely


2. Citizens of the nation stop increase sending remittances
3. There is continued political and economic instability in the country

Select the correct answer using the codes below.

a) 1 and 2 only
b) 1 and 3 only
c) 2 and 3 only
d) 1, 2 and 3

Solution: b)
Justification: Value of a currency is determined by the forces of supply and demand in the international market. If
the demand for a currency is high, and supply low, its value tends to be higher. Any factor that increases supply or
reduces demand will instead lead to currency depreciation. Currency depreciation can occur due to any number of
reasons – economic fundamentals, interest rate differentials, political instability, risk aversion among investors and
so on. Countries with weak economic fundamentals such as chronic current account deficits and high rates of
inflation generally have depreciating currencies. Currency depreciation, if orderly and gradual, improves a nation’s
export competitiveness and may improve its trade deficit over time. But abrupt and sizeable currency depreciation
may scare foreign investors who fear the currency may fall further, and lead to them pulling portfolio investments
out of the country, putting further downward pressure on the currency.

64. A Hard currency is the one which


a) does not change its value relative to movements in the international market
b) is considered as a strong currency with high levels of liquidity
c) is abundant in the foreign exchange market
d) is not convertible to Special Drawing Rights (SDR)

Solution: b)
Justification: It is the international currency in which the highest faith is shown and is needed by every economy.
The strongest currency of the world is one which has a high level of liquidity, i.e. people are easily willing to sell or
buy it due to the high confidence shown in it. Basically, the economy with the highest as well as highly diversified
exports that are compulsive imports for other countries (as of high-level technology, defence products, life-saving
medicines and petroleum products) will also create high demand for its currency in the world and become the hard
currency. It is always scarce. Up to the Second World War, the best hard currency was the Pound Sterling (E) of the
UK, but soon it was replaced by the US Dollar. Some of the hard currencies are Yen, Euro and Sterling Pound

65. Notes and currency held with the public will be considered part of which of the following measures of money
supply?
a) M0
b) M1
c) M3
d) All of the above

Solution: d)
Justification:M0 and M1, for example, are also called narrow money and include coins and notes that are in
circulation and other money equivalents that can be converted easily to cash such as demand deposits held in
banks.M2 includes M1 and, in addition, short-term time deposits in banks and certain money market funds, such as
post office savings. M3 includes M2 in addition to long-term deposits, such as time deposits. M4 includes M3 plus
other deposits. The term broad money is used to describe M2, M3 or M4, depending on the local practice.

66. What is/are the benefits of the Cheque Truncation System (CTS) introduced by the Reserve Bank of India?
1. A physical cheque is not required to be issued by the account holder for payment purposes leading to improved
logistical management.
2. The system reduces the scope of loss of cheque instruments in transit from bank to bank resulting in better service
to customers.
Which of the above is/are correct?
a) 1 only
b) 2 only
c) Both 1 and 2
d) None

Solution: b)
Justification: Truncation is the process of stopping the flow of the physical cheque issued by a drawer at some point
by the presenting bank en-route to the paying bank branch. In its place an electronic image of the cheque is
transmitted to the paying branch through the clearing house, along with relevant information like data on the MICR
band, date of presentation, presenting bank, etc. Cheque truncation thus obviates the need to move the physical
instruments across bank branches, other than in exceptional circumstances for clearing purposes. This effectively
eliminates the associated cost of movement of the physical cheques, reduces the time required for their collection
and brings elegance to the entire activity of cheque processing. Thus, it also reduces the scope of loss of
instruments in transit and removes reconciliation-related and logistics-related problems, thus benefitting the
system as a whole. CTS has been implemented in New Delhi, Chennai and Mumbai with effect from February 1,
2008, September 24, 2011 and April 27, 2013 respectively. After migration of the entire cheque volume from MICR
system to CTS, the traditional MICR-based cheque processing has been discontinued across the country.

MONETARY POLICY

67. What is/are the difference(s) between repo and reverse repo rate?
1. Repo rate applies only to borrowings of banking institutions, whereas reverse repo is used for the financial deposits
and borrowings of all financial institutions in Indian registered with RBI.
2. An increase in repo rate has the opposite effect on primary liquidity in the economy as an increase in reverse repo
rate.
Which of the above is/are correct?
a) 1 only
b) 2 only
c) Both 1 and 2
d) None

Solution: d)
Justification: Statement 1: Repo Rate is the rate at which RBI lends money to commercial banks against the pledge
of government securities whenever the banks are in need of funds to meet their day-to-day obligations.
Reverse repo rate is the rate of interest offered by RBI, when banks deposit their surplus funds with the RBI for
short periods. When banks have surplus funds but have no lending (or) investment options, they deposit such funds
with RBI. Banks earn interest on such funds. Both repo and reverse repo apply to banking institutions.
Statement 2: So, higher the repo rate higher the cost of short-term money and vice versa and thus lower the
borrowing. Hence, liquidity is lower in the economy. Similarly, if the reverse repo rate increases banks park more
funds with the RBI and lesser liquidity is available with the economy. So, an increase in both would have similar
effects on primary liquidity (we will discuss about primary and secondary liquidity later).

68. Monetary transmission refers to the process by which a central bank’s monetary policy decisions are passed on to
the financial markets. Monetary transmission remains weak in India due to

1. High volume of government borrowing through the SLR route


2. High level of NPAs of banks
3. A number of Interest rate subvention schemes

Select the correct answer using the codes below.

a) 1 only
b) 2 only
c) 2 and 3 only
d) 1, 2 and 3

Solution: d)
Justification: It is essentially the process through which the policy action of the central bank is transmitted to the
ultimate objective of stable inflation and growth. The policy action consists typically of changing the interest rate at
which it borrows or lends “reserves” (in our case, Rupees) on an overnight basis with commercial banks. The
transmission mechanism hinges crucially on how monetary policy changes influence households’ and firms’
behavior. This change can take place through several channels. Studying these channels is a vast subject in finance
and economics literature. Changes in the central bank’s policy rate impact the economy with lags through a variety
of channels, the primary ones being (i) interest rate channel, (ii) credit channel, (iii) exchange rate channel, and (iv)
asset price channel. How these channels function in a given economy depends on the stage of development of the
economy and its underlying financial structure.

Statement 1: A large part of bank’s deposits are lent to the government through the SLR route at a certain interest
rate, which is not responsive to the general interest policy in the economy.
Statement 2: The implicit assumption here is that bank balance sheets are strong and in a position to step-up quickly
the supply of credit in response to lower funding cost and higher demand for credit – the bank lending or the credit
channel of transmission. Cross-country evidence indicates that monetary transmission is greatly hindered if bank
balance sheets are weak in that they do not have much loss-absorption capacity to deal squarely with their
problem.
Statement 3: If major schemes keep sub venting interest rates, even if the banks change the interest rate, it will not
elicit a response from the public because they are anyways borrowing funds at a lower interest rate (due to the
interest subvention).
Also, monetary transmission also remains weak in India due to following reasons:
Practice of yearly resetting of administered interest rates on small savings (including public provident fund) linked to
G-sec yields
Sticky bank rates

69. If RBI changes the Repo rate, it automatically leads to a change in which of the following monetary policy tools?

1. Reverse Repo rate


2. Statutory Liquidity Ratio (SLR)
3. Cash Reserve ratio (CRR)

Select the correct answer using the codes below

a) 2 and 3 only
b) 1 only
c) 1 and 3 only
d) None of the above

Solution: d)
Justification: All are independently changed based on market condition. However the repo and reverse repo are
usually changed with certain gap in base points but it is not a rule to change both.

70. Currency Deposit Ratio (CDR) in India is likely to increase in times of


a) Festive seasons
b) Subdued consumption
c) High monetary policy rates like Bank rate
d) None

Solution: a)
Justification: The currency deposit ratio (cdr) is the ratio of money held by the public in currency to that they hold in
bank deposits.
Cdr = CU/DD.
If a person gets Re 1 she will put Rs 1/ (1 cdr) in her bank account and keep Rs cdr/ (1 cdr) in cash.
It reflects people’s preference for liquidity. It is a purely behavioural parameter which depends, among other things,
on the seasonal pattern of expenditure
For example, cdr increases during the festive season as people convert deposits to cash balance for meeting extra
expenditure during such periods.
Option (b) can’t be the answer as subdued consumption means people would like to keep money in banks and hold
less cash.
Option (c) can’t be the answer as high policy rates promote savings, not consumption.
71. When CRR is reduced, banks have more cash to lend to customers. But, the amount of credit lent to consumers is
often more than the cash released by lowering of CRR. This is because of
a) Multiplication of banking operations owing to multiple subsidiaries
b) High Capital adequacy ratio (CAR) maintained by banks irrespective of CRR changes
c) Credit creation by banks
d) CRR reduction not being applicable to priority sector lending
Solution: c)
Justification: When CRR is reduced, banks have more cash to lend to customers. This increases the liquidity in the
market.
Primary liquidity is the amount that is injected in the market without any credit creation by banks. When banks lend
the same money (obtained after relaxing CRR) repeatedly, extra credit is created. This extra credit is called
secondary liquidity.
Option A and B are irrelevant to the analysis and option D is wrong in principle.

72. Inflation targeting is practiced in India through a Monetary Policy Framework Agreement between the Government
of India and Reserve Bank of India. If RBI fails to meet the inflation target, the agreement says that
1. The Chairman of the Monetary Policy Committee (MPC) RBI should resign
2. RBI should communicate the causes of such failure to the government and give the steps to tame inflation
3. RBI should immediately increase the CRR and Repo rates to cut down the inflation
4. The Liquidity Adjustment Facility (LAF) and Marginal Standing Facility (MSF) should be closed down
Select the correct answer using the codes below.
a) 1, 2 and 3 only
b) 2, 3 and 4 only
c) 2 only
d) 1, 3 and 4 only

Solution: c)
Justification: Inflation beyond a certain level is detrimental to the economy and monetary policy needs to be tied to
short-term and long-term economic goals. Both of these necessitate such monetary agreements between the
central bank and the government.
Statement 1: No such provision, but successive failures in reining the inflation may voluntarily result in the
resignation of the RBI Governor.
Statement 2: This is to introduce transparency and predictability in monetary policy.
Statement 3: There is no such provision, and merely increasing CRR or repo rates may not solve the problem
especially if it is rooted in infrastructural bottlenecks and deeper economic issues.
Statement 4: Marginal Standing Facility is a new Liquidity Adjustment Facility (LAF) window created by Reserve Bank
of India in its credit policy of May 2011. MSF is the rate at which the banks are able to borrow overnight funds from
RBI against the approved government securities.

73. Real interest rate will NOT be lower than the nominal interest rate in which of the following cases?
a) GNP is higher than GDP
b) A general decline in prices observed across the economy
c) Sudden increase in receipts of foreign remittances
d) Banks step up interest rates in the event of a macroeconomic shock

Solution: b)
Justification: The nominal interest rate is the simplest type of interest rate. It is the stated interest rate of a given
bond or loan. The nominal interest rate is in the actual monetary price that borrowers pay to lenders to use their
money. If the nominal rate on a loan is 5%, then borrowers can expect to pay $5 of interest for every $100 loaned to
them. But nominal interest rate doesn’t take inflation into account.

A real interest rate is the interest rate that does take inflation into account. As opposed to the nominal interest rate,
the real interest rate adjusts for the inflation and gives the real rate of a bond or a loan.
Now imagine that the inflation rate was 5%. A 5% inflation rate means that an average basket of goods you
purchased this year is 5% more expensive when compared to last year.
Continuing with our previous example, the lender would make nothing if he loaned it out at 5% when the rate of
inflation was 5%.
The approximate formula is Real Interest Rate (R) = Nominal Interest Rate (r) – Rate of Inflation (i)
Justification: Option B: When there is a general decline in prices, the component ‘I’ will be negative leading to a
higher real interest rate than nominal interest rate.
Option A: The inflation or deflation is not clear from this situation.
Option C: This might lead to more foreign exchange and thus higher prices, leading to a lower real interest rate than
nominal interest rate

74. What is the role of monetary policy in the economy?

1) During a situation of low demand, it can revive demand by increasing liquidity.


2) It affects government borrowing significantly thereby affecting public spending.

Which of the above is/are correct?

a) 1 only
b) 2 only
c) Both 1 and 2
d) None

Solution: c)
Justification: Statement 1: The RBI sells or buys bonds from the public affecting total liquidity in the market. Also,
the CRR and other bank related short-term rates affect the interest rates banks offer to clients and therefore
influences the total liquidity in the economy.
Statement 2: The SLR that banks are entitled to maintain ultimately goes to government spending. SLR is an
important part of monetary policy.

75. In India, monetary policy is designed with an objective to

1) Control inflation
2) Increase foreign exchange reserves
3) Reduce the fiscal deficit
4) Generate surplus factors of production

Select the correct answer using the codes below.

a) 1 and 2 only
b) 2, 3 and 4 only
c) 1 only
d) 1 and 4 only

Solution: c)
Justification: Monetary policy aims at stability of prices with a moderate rate of inflation. Some inflation is
necessary for growth.
This is ensured through instruments like Bank rate, CRR, SLR, Open Market operations etc.
Statement 2: Forex reserves accrue or deplete based on the foreign exchange received by India through trade, FDI,
remittances etc. It has no direct relation with the design of the monetary policy, even though monetary policy has a
bearing on such variables, which will be explained in later tests.
Statement 3: Monetary policy refers to the credit control measures adopted by the central bank of a country, not by
the government of the day.

76. With reference to Land development banks, consider the following statements.

1) They provide long-term funds for various agriculture related projects besides development of land.
2) They do not accept deposits or grant business loans.
3) These banks are mandated to give collateral free funding to eligible borrowers.

Select the correct answer using the codes below.

a) 1 only
b) 2 and 3 only
c) 1 and 3 only
d) 1 and 2 only

Solution: a)
Justification: Indian farmers need three types of credit, viz., short-term, medium-term and long- term. Their short-
term and medium-term credit requirements are fulfilled by the co-operative banking institutions like PACs, CCBs
and SCBs.
Statement 1 and 2: A land development bank is a special kind of bank in India, and is of quasi- commercial type that
provides services such as accepting deposits, making business loans, and offering basic investment products.
The main objective of the LDB is to promote the development of land, agriculture and increase the agricultural
production. The LDB provides long-term finance to members directly through its branches. The borrowing capacity
of a member is generally determined according to the number of shares he holds in the bank. The loan granted by
land development bank is repayable within 20 to 30 years. Normally, loans are granted up to 50% of the value of the
land or up to 30 times the revenue. The Land Development Banks have no uniform pattern. In some states, they are
unitary and in some others, they are federal in nature.

77. Consider the following about Marginal Standing Facility (MSF) Rate.

1) It is rate at which scheduled banks can borrow funds for their long-term needs.
2) Funds are borrowed at a rate higher than that prevailing under Liquidity Adjustment Facility (LAF).

Which of the above is/are correct?

a) 1 only
b) 2 only
c) Both 1 and 2
d) None

Solution: b)
Justification: Statement 1: Marginal standing facility (MSF) is a window for banks to borrow from the Reserve Bank
of India in an emergency situation when inter-bank liquidity dries up completely.
Funds are borrowed overnight from RBI against government securities. It is very short-term borrowing scheme for
scheduled banks.
Statement 2: Banks borrow from the central bank by pledging government securities at a rate higher than the repo
rate under liquidity adjustment facility or LAF in short.
The MSF rate is pegged 100 basis points or a percentage point above the repo rate.
Under MSF, banks can borrow funds up to one percentage of their net demand and time liabilities (NDTL).

78. An internal Study Group constituted by the Reserve Bank of India (RBI) has recommended that banks should set
interest rates based on an external benchmark and not as per internal benchmarks as is the practice now. What
would qualify as external benchmarks in this context?

1) Cost of funds for the banks


2) RBI’s policy repo rate
3) Global economic downturns

Select the correct answer using the codes below.


a) 2 only
b) 1 and 3 only
c) 1 and 2 only
d) 2 and 3 only

Solution: a)
Justification: Bank’s interest rates are decided on the marginal cost of fund based lending rate (MCLR). This is
calculated based on banks’ internal factors such as cost of funds.
But, there is a problem with this method. This method of calculating interest rates (for lending) is insensitive to
changes in the policy interest rate or repo rate.
Also, banks deviate in an ad hoc manner from the specified methodologies for calculating the MCLR to either inflate
the base rate or prevent the base rate from falling in line with the cost of funds.
The Study Group is of the view that the T-Bill rate, the CD rate and the RBI’s policy repo rate are better suited than
other interest rates to serve the role of an external benchmark.
This will make the bank’s interest rate responsive to the policy rates and RBI can easily affect the interest rates by
the policy rates.

79. The Monetary Policy Committee (MPC) of the Reserve Bank of India (RBI) has reduced short-term lending rate, or
repo rate, by 25 basis points to 6%. What may be the impact of this move on the banking system?

1) Draining of foreign exchange reserves


2) Pickup in short-term borrowing due to lowering of interest rates
3) Sudden increase in international trade and commerce of India

Select the correct answer using the codes below.

a) 1 and 2 only
b) 3 only
c) 2 and 3 only
d) 2 only

Solution: d)
Justification: Statement 1: Domestic borrowing and lending do not directly affect the forex reserves.
Statement 2: Repo rate represents the short-term borrowing cost of banks. If it is reduced, banks pass on the
lowered borrowing cost to consumers.
Statement 3: There may be an increase in trade and commerce as lowered interest rates lead to higher economic
activity and greater exports. However, this will not be a sudden impact, it will take time.

BANKS, NBFCS AND OTHER FINANCIAL INSTITUTIONS

BANKS

80. With reference to Chit funds, consider the following statements.

1) Any savings scheme run by a NBFC can be termed as chit funds.


2) Some chit funds are run by state governments.
3) A chit fund is regulated by Central as well as state laws.

Select the correct answer using the codes below.


a) 1 and 2 only
b) 2 and 3 only
c) 1 and 3 only
d) 3 only
Solution: b)
Justification: Statement 1: Many NBFCs run contractual savings schemes. Nidhi Companies (as notified by the
Central government) offer saving schemes and make credits available to those whose credit needs remain unmet by
his commercial banks. Chit Fund Company is a company which collects subscriptions from specified number of
subscribers periodically and in turn distributes the same as prizes amongst them. The chit fund companies enter
into an agreement with the subscribers that every one of them shall subscribe a certain amount in installments over
a definite period and that every one of such subscriber shall in his turn, as determined by lot or by auction or by
tender, be entitled to a prize amount.
Statement 2: For e.g. the Kerala State Financial Enterprise (KSFE) and Mysore Sales international limited (MSIL) are
PSUs that run chit fund business in a clean and transparent manner. To understand chit funds clearly, go through
this comprehensive article

81. Among various categories of ‘priority sectors’ in the Priority Sector Lending scheme, the one allocated the highest
target share is
a) Agriculture
b) Weaker sections
c) Micro enterprises
d) Information Technology

Solution: a)
Justification:

82. The Banking Ombudsman Scheme is an expeditious and inexpensive forum for bank customers for resolution of
complaints relating to certain services rendered by banks. The Banking Ombudsman can receive and consider any
complaint relating to which of the following deficiency in banking services?
1. Non-adherence to prescribed working hours of banks
2. Refusal to open deposit accounts without any valid reason for refusal
3. Levying of charges without adequate prior notice to the customer
4. Forced closure of deposit accounts without due notice or without sufficient reason
Select the correct answer using the codes below.
a) 2, 3 and 4 only
b) 2 and 4 only
c) 1, 2, 3 and 4
d) 1 and 3 only

Solution: c)
Justification: The Banking Ombudsman Scheme is introduced under Section 35 A of the Banking Regulation Act,
1949 by RBI with effect from 1995.
Presently the Banking Ombudsman Scheme 2006 (as amended in 2017) is in operation.
The Banking Ombudsman is a senior official appointed by the Reserve Bank of India to redress customer complaints
against deficiency in certain banking services covered under the grounds of complaint specified under Clause 8 of
the Banking Ombudsman Scheme 2006 (as amended in 2017).
All Scheduled Commercial Banks, Regional Rural Banks and Scheduled Primary Co-operative Banks are covered
under the Scheme.
Some other issues that can be reported are:
 non-payment or inordinate delay in the payment or collection of cheques, drafts, bills etc.;
 non-acceptance, without sufficient cause, of small denomination notes tendered for any purpose, and for charging
of commission in respect thereof;
 failure to provide or delay in providing a banking facility (other than loans and advances) promised in writing by a
bank or its direct selling agents;
 Non-adherence to the instructions of Reserve Bank with regard to Mobile Banking / Electronic Banking service in
India by the bank
 Non-disbursement or delay in disbursement of pension etc.

83. With reference to Regional Rural Banks (RRBs), consider the following statements.
1. They are not commercial banks.
2. By the mandate, their operations must be restricted to rural areas.
3. These banks cannot provide the services of credit or debit cards.
4. In some districts, they are nodal agencies for disbursement of MGNREGA wages.
Select the correct answer using the codes below.
a) 1 and 2 only
b) 2 and 4 only
c) 4 only
d) 1, 2 and 3 only

Solution: c)
Justification: Statement 1: These are scheduled commercial banks operating at regional level in different States of
India.
Statement 2: They have been created with a view to serve primarily the rural areas of India with basic banking and
financial services. However, RRBs may have branches set up for urban operations and their area of operation may
include urban areas too.
Statement 3 and 4: The area of operation of RRBs is limited to the area as notified by Government of India covering
one or more districts in the State. RRBs also perform a variety of different functions. RRBs perform various functions
in following heads:
Providing banking facilities to rural and semi-urban areas.
Carrying out government operations like disbursement of wages of MGNREGA workers, distribution of pensions etc.
Providing Para-Banking facilities like locker facilities, debit and credit cards.
Small man’s bank

84. A SWIFT code is required for a/an


a) Precursor of QR code reading
b) International banking transaction
c) Quick toll plaza electronic payment
d) Check-in to an international airport in a foreign country

Solution: b)
Justification: The SWIFT - Society for Worldwide Interbank Financial Telecommunication - is a secure financial
message carrier — it transports messages from one bank to its intended bank recipient. Its core role is to provide a
secure transmission channel so that Bank A knows that its message to Bank B goes to Bank B and no one else. Bank
B, in turn, knows that Bank A, and no one other than Bank A, sent, read or altered the message en route. Banks, of
course, need to have checks in place before actually sending messages. The SWIFT is a global member-owned
cooperative that is headquartered in Brussels, Belgium. The ₹11,500 crore fraud in the Punjab National Bank where
fund transfer through an inter-bank messaging system was not reported to the core banking solution, followed by
the cyber-attack on the City Union Bank, has put the spotlight once again on SWIFT or the Society for Worldwide
Interbank Financial Telecommunication.

85. What are the services/facilities NOT available at White Label ATMs (WLAs)?
1) Cash Deposit
2) Regular Bills Payment that is available through ATMS
3) Account Information

Select the correct answer using the codes below.


a) 1 and 2 only
b) 2 only
c) 1 and 3 only
d) 1 only

Solution: a)
Justification: The rationale of allowing non-bank entity to set up White Label ATMs has been to increase the
geographical spread of ATM for increased / enhanced customer service. In addition to cash dispensing, ATMs/WLAs
may offer many other services/facilities to bank customers. Some of these services include:
Account Information
Cash Deposit (Acceptance of deposits are not permitted at WLAs)
Regular Bills Payment (not permitted at WLAs)
Purchase of Re-load Vouchers for Mobiles (not permitted at WLAs)
Mini/Short Statement
PIN change
Request for Cheque Book

86. A company is registered as a NBFC by RBI if its principal business is conducting financial activity. What does
conducting financial activity as “principal business” mean?
a) That the company’s financial assets constitute more than fifty per cent of the total assets
b) That the company’s income from financial assets constitute more than fifty per cent of the gross income
c) That the company deals with financial products made for customers other than retail customers
d) Both (a) and (b)

Solution: d)
Justification: Financial activity as principal business is when a company’s financial assets constitute more than 50
per cent of the total assets and income from financial assets constitute more than 50 per cent of the gross income.
A company which fulfils both these criteria will be registered as NBFC by RBI. The term 'principal business' is not
defined by the Reserve Bank of India Act. The Reserve Bank has defined it so as to ensure that only companies
predominantly engaged in financial activity get registered with it and are regulated and supervised by it.

Hence if there are companies engaged in agricultural operations, industrial activity, purchase and sale of goods,
providing services or purchase, sale or construction of immovable property as their principal business and are doing
some financial business in a small way, they will not be regulated by the Reserve Bank.
Interestingly, this test is popularly known as 50-50 test and is applied to determine whether or not a company is into
financial business.
Learning: A Non-Banking Financial Company (NBFC) is a company registered under the Companies Act, 1956
engaged in the business of loans and advances, acquisition of shares/stocks/bonds/debentures/securities issued by
Government or local authority or other marketable securities.
However, this excludes institutions whose principal business is that of agriculture activity, industrial activity,
purchase or sale of any goods (other than securities) or providing any services and sale/purchase/construction of
immovable property.

87. NBFCs perform functions akin to that of banks, however there are a few differences between them that include
1) An NBFC cannot accept demand deposits.
2) Deposit insurance facility is not available for NBFC depositors.
3) NBFCs need not be registered with the RBI.
Select the correct answer using the codes below.

a) 1 and 2 only
b) 2 and 3 only
c) 1 and 3 only
d) 2 only

Solution: a)
Justification: Statement 1: It can’t accept deposits that are payable on demand like the savings and current
accounts. But, it can accept other kinds of deposits.
This is because it is not a part of the payment and settlement system and as such cannot issue cheques to its
customers.
Statement 2: Deposit insurance facility is not available for NBFC depositors unlike in the case of banks.
It means the public deposits with them are 'unsecured'. In case an NBFC defaults in repayment of deposit, the
depositor can approach Company Law Board or Consumer Forum or file a civil suit to recover the deposits.
Statement 3: Under the RBI Act, 1934, the NBFCs have to get registered with the RBI.
However, to obviate dual regulation, certain category of NBFCs which are regulated by other regulators are
exempted from the requirement of registration with RBI such as: venture capital fund, merchant banking companies
etc.

88. The Main functions of the Reserve Bank of India (RBI) include
1) Regulator and supervisor of the financial system
2) Protect depositors' interest and provide cost-effective banking services to the public
3) Maintaining banking accounts of all scheduled banks
4) To facilitate external trade and payment and promote orderly development of foreign exchange market
Select the correct answer using the codes below.

a) 1, 2 and 3 only
b) 1, 2 and 4 only
c) 3 and 4 only
d) 1, 2, 3 and 4

Solution: d)
Justification: It includes: Monetary Authority:
Formulates, implements and monitors the monetary policy
Objective: maintaining price stability while keeping in mind the objective of growth.

Regulator and supervisor of the financial system:


Prescribes broad parameters of banking operations within which the country's banking and financial system
functions
Objective: maintain public confidence in the system, protect depositors' interest and provide cost-effective banking
services to the public.

Manager of Foreign Exchange


Manages the Foreign Exchange Management Act, 1999
Objective: to facilitate external trade and payment and promote orderly development and maintenance of foreign
exchange market in India.

Issuer of currency:
Issues and exchanges or destroys currency and coins not fit for circulation.
Objective: to give the public adequate quantity of supplies of currency notes and coins and in good quality

Developmental role
Performs a wide range of promotional functions to support national objectives.
Related Functions
Banker to the Government: performs merchant banking function for the central and the state governments; also
acts as their banker.
Banker to banks: maintains banking accounts of all scheduled banks

89. With reference to the Ombudsman Scheme for Non-Banking Financial Companies, 2018, consider the following
statements.
1) The NBFC Ombudsman is appointed by the Reserve Bank of India.
2) The scheme does not cover NBFCs that are authorized to accept deposits.
Which of the above is/are correct?

a) 1 only
b) 2 only
c) Both 1 and 2
d) None

Solution: a)
Justification: Statement 1: The Reserve Bank of India has introduced an Ombudsman Scheme for customers of
NBFCs. It is an expeditious and cost free apex level mechanism for resolution of complaints of customers of NBFCs,
relating to certain services rendered by NBFCs.
The NBFC Ombudsman is a senior official appointed by the Reserve Bank of India to redress customer complaints
against NBFCs for deficiency in certain services covered under the grounds of complaint specified under the
Scheme.
Statement 2: The Scheme initially covers NBFCs authorized to accept deposits, and would be gradually extended to
cover other identified NBFCs.
NBFCs, as defined in Section 45-I (f) of the Reserve Bank of India Act, 1934 and registered with the RBI under Section
45-IA of the Reserve Bank of India Act, 1934, which (a) are authorized to accept deposits; or (b) have customer
interface, with assets size of one billion rupees or above, as on the date of the audited balance sheet of the previous
financial year, or of any such asset size as the RBI may prescribe, are covered under the Scheme.

90. With reference to Peer to Peer (P2P) lending, consider the following statements.
1) It enables individuals to borrow and lend money without any financial institution as an intermediary.
2) A hallmark of P2P lending is that it does not involve any collateral.
3) All P2P platforms are considered banking financial companies and regulated by the RBI.
Which of the above is/are correct?

a) 1 only
b) 2 and 3 only
c) 1 and 3 only
d) None of the above

Solution: a)
Justification: Statement 1: Peer-to-peer lending is a form of crowd-funding used to raise loans for people who need
to borrow, from people who want to invest.
It enables individuals to borrow and lend money without any financial institution as an intermediary, and extends
credit to borrowers who are unable to get it through traditional financial institutions.
The main idea is savers getting higher interest by lending out their money instead of saving it, and borrowers getting
funds at comparatively low interest rates.
Statement 2: It typically uses an online platform where the borrowers and lenders register themselves. Due
diligence is carried out before allowing the parties to participate in any lending or borrowing activity.
All P2P platforms are considered non-banking financial companies and regulated by the RBI.

91. What is one example of a Priority Sector with regards to the Priority Sector Lending (PSL) targets set by the RBI?

a) Industries attracting the most FDI


b) Ventures in satellite areas of metropolitans
c) Information Technology and Banking
d) Micro and small enterprises

Solution: d)
Justification: Priority Sector Lending is an important role given by the Reserve Bank of India (RBI) to the banks for
providing a specified portion of the bank lending to few specific sectors.
This is essentially meant for an all-round development of the economy as opposed to focusing only on the financial
sector.
Typically, these are small value loans to farmers for agriculture and allied activities, micro and small enterprises,
poor people for housing, students for education and other low income groups and weaker sections.

92. Peer to Peer (P2P) lending is a


1) Form of crowd-funding
2) Does not carry any interest rate
3) Platform that can be used online
Select the correct answer using the codes below.

a) 1 and 3 only
b) 2 and 3 only
c) 1 and 3 only
d) 1, 2 and 3

Solution: c)
Justification: P2P can be defined as the use of an online platform that matches lenders with borrowers in order to
provide unsecured loans.
It is used to raise loans which are paid back with interest.
The borrower can either be an individual or a legal person requiring a loan.
The interest rate may be set by the platform or mutual agreement between the borrower and the lender.
Fees are paid to the platform by both the lender as well as the borrower. Borrowers pay an origination fee — either
a flat rate fee or as a percentage of the loan amount raised — according to their risk category.

93. Consider the following about Indian Post Payments Bank (IPPB).

1) It will be a public-private venture partially owned by the Government of India via the Department of Posts (DoP).
2) It will focus on low-cost, low-risk, technology led solutions to extend access to formal banking.
3) All post offices across the country will function as customer access points for IPPB.

Select the correct answer using the codes below.

a) 1 and 2 only
b) 2 and 3 only
c) 3 only
d) 1 only

Solution: b)
Justification: Statement 1: It will be 100% owned by the Government of India via DoP, and will have an independent
board of directors with representation from DoP and other stakeholders from within the Government of India to
ensure strategic alignment with the overall objectives of the DoP and the Government of India.
Statement 3: IPPB is slated to have 650 branches at district headquarters and each branch would act as an access
point.
Statement 3: A Payments Bank is a “differentiated bank” to further financial inclusion for the underserved
population by providing (i) current and savings accounts and (ii) payments or remittance services to migrant labour
workforce, low income households, small businesses, unorganized sector entities and other users.
This is to be done by enabling high volume-low value transactions in deposits and payments or remittance services
in a secure technology-driven environment.

94. Consider the following statements.


Assertion (A): If all the account-holders of all commercial banks want their deposits back at the same time, the
banks will not be satisfy all these obligations.
Reason (R): The total amount of deposits held by all commercial banks in the country is much larger than the total
size of their reserves.
In the context of the above, which of these is correct?

a) A is correct, and R is an appropriate explanation of A.


b) A is correct, but R is not an appropriate explanation of A.
c) A is correct, but R is incorrect.
d) Both A and R are incorrect.

Solution: a)
Justification: Broadly when a bank receives cash deposits from the public, it keeps a fraction of deposits as cash
reserve (LRR) and uses the remaining amount for giving loans. In the process of lending money, banks are able to
create credit through secondary deposits many times more than initial deposits (primary deposits). This process has
been explained well in your NCERT book. When so happens, and when all depositors demand their money at the
same time, banks fail. The Reserve Bank of India plays a crucial role here. In case of a crisis like the above it stands
by the commercial banks as a guarantor and extends loans to ensure the solvency of the latter. This system of
guarantee assures individual account-holders that their banks will be able to pay their money back in case of a crisis
and there is no need to panic thus avoiding bank runs. This role of the monetary authority is known as the lender of
last resort.

95. What are Systematically Important Banks (D- SIBs) as recognized by the Reserve Bank of India?
1) These banks are exempt from CRR and SLR regulations.
2) These are perceived as perceived as ‘Too Big to Fail (TBTF) banks.
3) The RBI determines a cut-off score beyond which banks are considered as D-SIBs.
Select the correct answer using the codes below.
a) 1 only
b) 2 and 3 only
c) 1 and 3 only
d) 1, 2 and 3

Solution: b)
Justification: A few banks assume systemic importance due to their size, cross-jurisdictional activities, complexity,
lack of substitutability and interconnectedness. The disorderly failure of these banks has the propensity to cause
significant disruption to the essential services provided by the banking system, and in turn, to the overall economic
activity. These banks are considered Systemically Important Banks (SIBs) as their continued functioning is critical for
the uninterrupted availability of essential banking services to the real economy. After adding the State Bank of India
and private lender ICICI, the Reserve Bank of India has listed HDFC to systemically important banks list.
Statement 1: Since 2015, RBI has been identifying banks whose failure would impact the whole financial system.
These banks are subject to more rigorous regulation and capital requirement. This perception of TBTF creates an
expectation of government support for these banks at the time of distress. Due to this perception, these banks
enjoy certain advantages in the funding markets.
The indicators the RBI uses for identifying Domestic-SIBs are: size, interconnectedness, substitutability and
complexity.

QUANTITATIVE EASING

FINANCIAL INTERMEDIARIES AND INSTRUMENTS

96. Capital Adequacy Ratio (CAR) restrains the bank from

a) Investing too much in risky assets


b) Lending to the government
c) Borrowing from the RBI
d) Keeping idle deposits with the RBI

Solution: a)
Justification: A bank's failure has the potential of creating chaos in an economy. This is why governments of the
world pay special attention to the regulatory aspects of the banks.
Every regulatory provision for banks tries to achieve a simple equation, i.e. balancing returns with risks.
CAR, a measure of a bank's capital, is expressed as a percentage of a bank's risk weighted credit exposures.
Also known as 'Capital to Risk Weighted Assets Ratio (CRAR)' this ratio is used to protect depositors and promote
the stability and efficiency of financial systems around the world.
Basel norms provide for a CAR, and in India RBI also mandates CAR to banks.

97. Liquidity Adjustment Facility (LAF) is aimed at

a) Long-term borrowing needs of the banks


b) Daily lending and borrowing by RBI to banks
c) Monthly fulfilment of CRR and SLR needs of banks
d) Any of the above as it is a flexible system

Solution: b)
Justification: The LAF is the key element in the monetary policy operating framework of the RBI (introduced in
2000).
LAF is a facility extended by the Reserve Bank of India to the scheduled commercial banks (excluding RRBs) and
primary dealers to avail of liquidity in case of requirement or park excess funds with the RBI in case of excess
liquidity on an overnight basis against the collateral of Government securities including State Government
securities. Basically LAF enables liquidity management on a day to day basis. Liquidity adjustment facility (LAF) is a
monetary policy tool which allows banks to borrow money through repurchase agreements or repos. Liquidity of a
more durable nature are managed with other instruments like, cash reserve ratio (CRR) or Market Stabilization
Scheme (MSS)

98. The repo rate is the rate at which

a) Other banks borrow from Reserve Bank of India


b) Reserve Bank of India borrow from other banks
c) Banks in India lend to other financial institutions
d) Other financial institutions lend to scheduled banks

Solution: a)
Justification: Repo rate is the rate at which the central bank of a country (Reserve Bank of India in case of India)
lends money to commercial banks in the event of any shortfall of funds. Repo rate is used by monetary authorities
to control inflation. In the event of inflation, central banks increase repo rate as this acts as a disincentive for banks
to borrow from the central bank. This ultimately reduces the money supply in the economy and thus helps in
arresting inflation. The central bank takes the contrary position in the event of a fall in inflationary pressures. Repo
and reverse repo rates form a part of the liquidity adjustment facility.

99. Consider the following with reference to the Open Market Operations (OMOs)?
1) They are conducted by the Union Ministry of Finance.
2) Government securities are bought or sold to alter liquidity in the market.
3) It can lead to monetization of government debt.
Select the correct answer using the codes below.

a) 1 and 2 only
b) 2 and 3 only
c) 1 only
d) 3 only

Solution: b)
Justification: Statement 1 and 2: Open market operations are conducted by the RBI by way of sale or purchase of
government securities (g-secs) to adjust money supply conditions. The central bank sells g-secs to suck out liquidity
from the system and buys back g-secs to infuse liquidity into the system. These operations are often conducted on a
day-to-day basis in a manner that balances inflation while helping banks continue to lend. The RBI uses OMO along
with other monetary policy tools such as repo rate, cash reserve ratio and statutory liquidity ratio to adjust the
quantum and price of money in the system.

Statement 3: Large open market purchases by the RBI can give the government a helping hand in its borrowing
programme and are frowned upon for this reason. In April 2006, the RBI was barred from subscribing to primary
bond issues of the government. This was done to put an end to the monetization of debt by the Reserve Bank.
However, that didn’t stop the process. With rising fiscal deficit, the RBI has been criticized for accommodating larger
government debt by way of OMO.

NPAs
100. The Insolvency and Bankruptcy Code, 2016 (IBC)
1) Outlines separate insolvency resolution processes for individuals, companies and partnership firms
2) Establishes the Insolvency and Bankruptcy Board of India, to oversee the insolvency proceedings in the country
Which of the above is/are correct?

a) 1 only
b) 2 only
c) Both 1 and 2
d) None

Solution: c)
Justification: It is the bankruptcy law of India which seeks to consolidate the existing framework by creating a single
law for insolvency and bankruptcy. The Code outlines separate insolvency resolution processes for individuals,
companies and partnership firms. The process may be initiated by either the debtor or the creditors. A maximum
time limit, for completion of the insolvency resolution process, has been set for corporates and individuals. The
Code establishes the Insolvency and Bankruptcy Board of India, to oversee the insolvency proceedings in the
country and regulate the entities registered under it. The Board will have 10 members, including representatives
from the Ministries of Finance and Law, and the Reserve Bank of India. The Code was amended in 2017 to prohibit
certain persons from submitting a resolution plan in case of defaults, and to prohibit the sale of property of a
defaulter to such persons during liquidation.

101. Twin Balance Sheet Problem (TBS), frequently seen in news, deals with which of these major problems?
1) Poor performance of the Public Sector Units (PSUs)
2) Non-performing assets in the Indian Banks
3) Debt accumulation on Indian companies
4) Accretion of foreign currency debt on India

Select the correct answer using the codes below.

a) 1 and 3 only
b) 2 and 3 only
c) 1 and 4 only
d) 2 and 4 only

Solution: b)
Justification: TBS is two two-fold problem for Indian economy which deals with:
Overleveraged companies – Debt accumulation on companies is very high and thus they are unable to pay interest
payments on loans. Bad-loan-encumbered-banks – Non Performing Assets (NPA) of the banks is 9% for the total
banking system of India. It is as high as 12.1% for Public Sector Banks. As companies fail to pay back principal or
interest, banks are also in trouble. Higher cost, lower revenues, greater financial costs-all squeezed corporate cash
flow leading to NPAs in the banking sector.

102. In 2015, under the Indradhanush plan, the government had announced capital infusion of over seventy
thousand crores in public sector banks for four years. However, credit rating agencies had pointed out that the sum
was insufficient because banks needed additional funds to
1) Meet Basel-III norms
2) Tackle the menace of rising bad loans
Which of the above is/are correct?

a) 1 only
b) 2 only
c) Both 1 and 2
d) None
Solution: c)
Justification: "Basel III" is a comprehensive set of reform measures, developed by the Basel Committee on Banking
Supervision, to strengthen the regulation, supervision and risk management of the banking sector. It is an
improvement over Base I and Basel II norms released much earlier. These norms may include reducing bank
investment in risky assets, increasing their capital base, strengthening management etc.
Statement 1: Rating agencies pointed that apart from PSBs face lack of competition and if business as usual
continues, they will face dearth of funds for expansion because:

 Private sector will give them a tough competition in coming years


 Basel III norms required more funds to absorb greater risks for banks
Statement 2: Stressed assets (NPAs or bad debt) in the banking system have reached unacceptably high levels and
hence, urgent measures were required for their speedy resolution. One of the way is by capital infusion, which will
increase the liquid cash in banks.

103. The Banking Regulation (Amendment) Bill, 2017 seeks to address which of these major issues in the banking
sector?

a) Non-performing assets
b) Lack of competition in public sector banks
c) Increasing participation of women in top tier management of banks
d) Financial inclusion in rural areas

Solution: a)
Justification: Stressed assets (NPAs or bad debt) in the banking system have reached unacceptably high levels and
hence, urgent measures were required for their speedy resolution. Therefore, the government considered it
necessary to make provisions in the Banking Regulation Act, 1949 for authorizing the Reserve Bank of India to issue
directions to any banking company or banking companies to effectively use the provisions of the Insolvency and
Bankruptcy Code, 2016 for timely resolution of stressed assets. Learning: The 2017 bill gives powers to the Reserve
Bank of India (RBI) to ask any bank to initiate insolvency proceedings and give directions for resolution of stressed
assets. The RBI would also be empowered to issue other directions for resolution, appoint or approve for
appointment, authorities or committees to advise the banking companies for stressed asset resolution.

104. RBI has announced the creation of a panel to consider creation of a Public Credit Registry (PCR) operated by the
regulator. What is/are the advantages of PCR?
1) It can help banks in credit assessment and pricing of credit.
2) It will allow mandatory registration of all participatory notes (PN) FII entries in India.
Which of the above is/are correct?

a) 1 only
b) 2 only
c) Both 1 and 2
d) None

Solution: a)
Justification: Statement 1: The PCR will be an extensive database of credit information for India that is accessible to
all stakeholders. The idea is to capture all relevant information in one large database on the borrower and, in
particular, the borrower’s entire set of borrowing contracts and outcomes. Further, it can help supervisors,
regulators and banks in early intervention and effective restructuring of stressed bank credits.

FINANCIAL INCLUSION

105. Lack of financial inclusion is costly to society and the individual. What are the advantages of financial inclusion?
1) Increases the amount of available savings in the economy
2) Increases efficiency of financial intermediation
3) Allows for tapping new business opportunities in remote and rural areas
4) Promotes social equity and inclusive growth
Select the correct answer using the codes below.
a) 1 and 2 only
b) 3 and 4 only
c) 1, 3 and 4 only
d) 1, 2, 3 and 4

Solution: d)
Justification: Statement 1: Savings refer to the total amount of deposits with the banks and post offices. When
people have access to formal financial channels, their money moves into these channels and turns into savings.
Statement 2 and 3: Lack of financial inclusion forces the unbanked into informal banking sectors where interest
rates are higher and the amount of available funds much smaller. Because the informal banking structure is outside
any legislative framework, any dispute between lenders and borrowers cannot be settled legally. This is addressed if
formal banking channels are available. Both efficient and outreach is increased.
Statement 4: Scope of the financial inclusion is not limited to only banking services but it extends to other financial
services as well like insurance, equity products & pension products etc. Thus, financial inclusion is not just about
opening a simple bank account with a branch in an unbanked area.
Micro Finance Institutions (MFIs) are the classic examples for providing easy & affordable credit to poor people and
have got written innumerable success stories. When financial exploitation is reduced, it is natural that equity and
inclusion will be promoted.

106. Financial inclusion is a major priority of policymakers in India. What can be the direct and indirect benefits of
financial inclusion?

1) Cutting down investment in non-productive assets such as Gold


2) Sustainable economic growth
3) Reduction in the influence of informal and exploitative lending sources
4) Increase in domestic savings base

Select the correct answer using the codes below.

a) 2 and 3 only
b) 1, 2, 3 and 4
c) 1, 3 and 4 only
d) 2 and 4 only

Solution: b)
Justification: Statement 1: Many invest in Gold due to lack of alternative investment instruments. Post-financial
inclusion, an investor has a greater repertoire to choose from and thus investment in Gold reduces.
Statement 3: As people get access to institutional finance such as banks etc., demand for exploitative money lending
usually reduces as people resort more towards formal sources.
Statement 2 and 4: Increase in availability of finance leads to increase in institutional deposit and access to finance
for small businesses, agriculture, retail investors etc.

INFLATION

CAUSES AND CONSEQUENCES

107. High Inflation reflects a

1) Reduction in the real purchasing power per unit of money


2) Sustained GDP growth
3) Growing international trade in the currency
4) Near zero nominal interest rate situation

Select the correct answer using the codes below.


a) 1 only
b) 1 and 4 only
c) 1, 2 and 3 only
d) 2, 3 and 4 only

Solution: a)
Justification: Statement 1: If the prices of goods increase and you hold the same Rs. 1000 that you had a year back,
you naturally can’t buy as much as you could earlier. So, the purchasing power of your money has reduced.
Statement 2: It is not necessary. However, it is usually seen that high GDP growth induces some sort of inflation due
to mismatch between supply and demand as a part of the growth process.
Statement 3: it can work both ways, high international trade may flood local markets with goods and actually
reduce inflation by bridging the supply gap. Otherwise, it can even increase inflation, if the nation starts earning too
much by exports and the income gained from exports is being spent domestically.
Statement 4: It is real interest rates that are affected by high inflation rate and often reach near zero. This is
because inflation erodes the nominal interest rate gains.
We will cover this concept in detail later. You don’t have to worry if you don’t understand it well now.

108. How do you differentiate between Headline and Core Inflation?

1) Headline inflation is published by RBI whereas Core inflation is published by Central Statistical Organization (CSO).
2) Headline inflation is calculated from a base year, whereas Core inflation takes the current financial year as the base
year.
3) Core inflation excludes food and energy from the calculations, Headline inflation includes these figures.

Select the correct answer using the codes below.

a) 1 only
b) 2 and 3 only
c) 3 only
d) 1 and 3 only

Solution: c)
Justification: Headline inflation contains all the of CPI basket including items like food and oil, whose prices are
highly volatile. Core inflation excludes volatile items from the CPI basket. It thus gives a more real data, a clearer
picture of the production trends in the economy for the policymakers, while headline inflation is more of concern to
the common man, who gets affected with the prices of articles of daily consumption.
109. If the ‘Base Effect’ is dominating

1) Even a small increase in price levels would cause a high level of inflation.
2) There would be no Cost push inflation.

Which of the above is/are correct?

a) 1 only
b) 2 only
c) Both 1 and 2
d) None

Solution: d)
Justification: The base effect refers to the impact of the rise in price level (i.e. last year’s inflation) in the previous
year over the corresponding rise in price levels in the current year (i.e., current inflation): if the price index had risen
at a high rate in the corresponding period of the previous year leading to a high inflation rate, some of the potential
rise is already factored in, therefore a similar absolute increase in the Price index in the current year will lead to a
relatively lower inflation rates. On the other hand, if the inflation rate was too low in the corresponding period of
the previous year, even a relatively smaller rise in the Price Index will arithmetically give a high rate of current
inflation.

110. Which of the following measures will increase inflation rather than taming it?

1) Taking measures to curb hoarding of essential items


2) Tight monetary policy
3) Import of goods which are in short-supply
4) Loose fiscal policy

Select the correct answer using the codes below.

a) 1, 3 and 4 only
b) 1, 2 and 3 only
c) 4 only
d) 2 and 3 only

Solution: c)
Justification: Justification: Statement 1 and 3: It is common in the case of onion, pulses etc. where government
often cracks down on illegal hoarding to increase market supply, and at times imports pulses to increase their
domestic supply so that prices can be reduced.
Statement 2: Tight monetary policy is basically intended to cut down the money supply in the economy by siphoning
out the extra money from the economy, so that effective demand is reduced.
This is a short-term measure. In the long-run, the best way is to increase production with the help of the best
production practices.
Statement 4: Loose fiscal policy only increases the liquidity and pushes demand thereby raising inflation.

111. GDP deflator is primarily an indicator of

a) Net Income from abroad


b) Competitiveness of the economy
c) Employment Generation
d) Price inflation

Solution: d)
Justification: It is the ratio of the value of goods and services an economy produces in a particular year at current
prices to that at prices prevailing during any other reference (base) year. This ratio basically shows to what extent
an increase in GDP or gross value added (GVA) in an economy has happened on account of higher prices, rather
than increased output. Since the deflator covers the entire range of goods and services produced in the economy —
as against the limited commodity baskets for the wholesale or consumer price indices — it is seen as a more
comprehensive measure of inflation.

112. Consider the following statements about Headline inflation.

1) It is presently measured in terms of Wholesale Price Index (WPI) in India.


2) It excludes the prices of food and fuel that constitute the volatile component of inflation.

Which of the above is/are correct?

a) 1 only
b) 2 only
c) Both 1 and 2
d) None

Solution: d)
Justification: Statement 1: RBI Governor had adopted the new Consumer Price Index (CPI) (combined) as the key
measure of inflation in 2014 based on the Urijeet Patel committee recommendation. Earlier, RBI had given more
weightage to Wholesale Price Index (WPI) than CPI as the key measure of inflation for all policy purposes.
Statement 2: Headline inflation is a measure of the total inflation within an economy, including commodities such as
food and energy prices (e.g., oil and gas). Core inflation calculations excludes food and energy which tend to be
much more volatile and prone to inflationary spikes.

113. Consider the following statements.


Assertion (A): Inflation redistributes wealth from creditors to debtors.
Reason (R): Higher inflation lowers the effective cost of credit for the debtor.

In the context of the above, which of these is correct?

a) A is correct, and R is an appropriate explanation of A.


b) A is correct, but R is not an appropriate explanation of A.
c) A is correct, but R is incorrect.
d) Both A and R are incorrect.

Solution: a)
Justification: Suppose a bank lent you Rs. 1 lakh at 10% p.a. interest rate. When the inflation rate is 6%, the bank is
getting a net interest rate of 4% (ten minus six). This net interest rate is the bank’s profit margin from lending this
sum of money. When inflation increases, to say 8%, bank’s net earning reduces as net interest rate (or real interest
rate) has come down to 2%. This benefits the debtor because the real value of Rs. 1 Lakh has reduced due to
inflation, and in effect he is paying less than what he borrowed.

114. Consider the following statements.


Assertion (A): High inflation deters private investment, both domestic and foreign.
Reason (R): Inflation reduces effective returns on investments and financial planning becomes uncertain.

In the context of the above, which of these is correct?

a) A is correct, and R is an appropriate explanation of A.


b) A is correct, but R is not an appropriate explanation of A.
c) A is correct, but R is incorrect.
d) Both A and R are incorrect.

Solution: a)
Justification: The reason is clear enough.
If an investor has invested 100 crores and expects to get a return of 20% with over 120 crores as revenue; he will
receive less return if inflation is over 10% due to which effective return reduces to nearly 10%. The same 120 crore
of revenue will now mean only 108 crores (10% deduction due to inflation) and thus means a loss for the company.
Erratic inflation rates makes these cost-benefit calculations even more difficult and thus companies hesitate to
invest.

115. What is ‘Inflation tax’?

a) Higher tax collection due to an increase in prices and thus effective gross tax
b) Fiscal stimulus given by the government owing to high inflation
c) Inflation caused by deficit financing, which erodes peoples’ income, thus effectively acting as a tax
d) Adjustment in official tax rates in a response to the Inflation in the economy

Solution: c)
Justification: Inflation tax is not an actual legal tax paid to a government; instead "inflation tax" refers to the penalty
for holding cash at a time of high inflation. When the government prints more money or reduces interest rates, it
floods the market with cash, which raises inflation in the long run. If an investor is holding securities, real estate or
other assets, the effect of inflation may be negligible. If a person is holding cash, though, this cash is worth less after
inflation has risen. The degree of decrease in the value of cash is termed the inflation tax for the way it punishes
people who hold assets in cash, which tend to be lower- and middle-class wage earners. It is noteworthy that even
deficit financing (sometimes leading to high inflation) can act as a tax on people which actually was intended to
boost demand in the economy

116. Which of the following may be indicated by ‘deflation’ in the economy?

a) GDP of the country has been growing at a negative growth rate.


b) Prices of all goods must have declined
c) The Central bank is following an easy monetary policy.
d) The demand in the economy is weak or supply is excess.

Solution: d)
Justification: In economics, deflation is a decrease in the general price level of goods and services. Deflation occurs
when the inflation rate falls below 0% (a negative inflation rate). This should not be confused with disinflation, a
slow-down in the inflation rate (i.e., when inflation declines to lower levels).
So, in a situation of deflation GDP may be growing, reducing or constant. Option (a) is wrong.
Deflation shows a general decline in prices of goods. It is not necessary that prices of all goods must have declined.
So, (b) is not appropriate.
If Central bank follows an easy monetary policy, demand will be strong, and inflation is more likely than deflation.
So, (c) is wrong.
Most appropriate option is (d) since prices of goods may decline if demand decreases given constant supply of
goods. If demand is too weak, prices may reduce heavily.

117. Consider the following statements.


Assertion (A): Hyperinflation can lead to a loss of confidence in the currency.
Reason (R): Inflation erodes the value of currency.

In the context of the above, which of these is correct?

a) A is correct, and R is an appropriate explanation of A.


b) A is correct, but R is not an appropriate explanation of A.
c) A is correct, but R is incorrect.
d) Both A and R are incorrect.

Solution: a)
Justification: For e.g. if inflation in India is 5000%, then the very next year the value of Rs. 5000 will be near Rs. 100.
It also leads to depreciation in the exchange rate of the currency.
As a result, people lose confidence in the currency and holding it becomes a risky proposition. So, people switch to
other forms of wealth like Gold, Foreign Currency (also known as “inflation proof” assets) etc.
It is one of the important reasons for the recent Gold rush in India. Introducing inflation-indexed bonds, Gold bonds
is one way of reducing the demand of unproductive physical assets.

118. Deflation’ in an economy would necessarily suggest decline in the

1) Prices of all the goods


2) GDP of the country

Which of the above is/are correct?

a) 1 only
b) 2 only
c) Both 1 and 2
d) None

Solution: d)
Justification: Statement 1: Deflation shows a general decline in prices of a selected basket of goods. It is not
necessary that prices of all goods must have declined.
Statement 2: In economics, deflation is a decrease in the general price level of goods and services.
Deflation occurs when the inflation rate falls below 0% (a negative inflation rate). This should not be confused with
disinflation, a slow-down in the inflation rate (i.e., when inflation declines to lower levels).
So, in a situation of deflation GDP may be growing, reducing or constant.

MEASUREMENT INDICES

119. If the Index Numbers of Wholesale Price in India (Base: 2011-12=100) were last published officially on 15th
January 2018, the next publication is most likely to be on

a) 15th April, 2018


b) 15th July, 2018
c) 15th February, 2018
d) 15th March, 2018

Solution: c)
Justification: It is published by Office of Economic Adviser, Ministry of Commerce & Industry, New Delhi on a
monthly frequency.
Weight of items are:
Primary articles (weight 22.62%)
Fuel & power (weight 13.15%)
Manufactured products (weight 64.23%)
WPI food index (weight 24.38%)

120. Consider the following with reference to the Wholesale Price Index (WPI).
1) The present WPI base is 2004-05.
2) It excludes consumer durables and precious metals.
3) Highest weightage has been accorded to manufactured products in the index.

Select the correct answer using the codes below.

a) 1 and 2 only
b) 3 only
c) 2 only
d) None of the above

Solution: b)
Justification: There are three major groups in WPI – Primary Articles, Fuel & Power and Manufacture Products.
Highest weightage has been accorded to the manufactured products.
Consumer durables are a part of the manufactured products category.
The WPI series base year, article composition and weightage has been revised to 2011-12 recently. The Office of the
Economic Adviser takes the work relating to revision of the existing series of WPI.

121. Consumer Price Index (CPI) Inflation in India is measured on a

a) Weekly basis
b) Quarterly basis
c) Monthly basis
d) Yearly basis

Solution: c)
Justification: Reference dates for calculating inflation are from the preceding and succeeding month of the same
year.
You can see the data here https://data.gov.in/catalog/all-india-consumer-price-index-ruralurban
Until recently, India had several sectoral consumer price indices (CPI) and a national level wholesale price index
(WPI). A national level consumer price index (CPI-combined) was released in 2011 and the inflation target for
monetary policy is prescribed in term of all-India CPI-combined index.

122. An inflationary gap signals that the economy is in which part of the trade cycle?

a) Boom
b) Recession
c) Depression
d) Recovery

Solution: a)
Justification: It is a situation where resources are being used over their capacity, factories are operating with
increasing average costs; wage rates increase because labour is used beyond normal hours at overtime pay rates.
The inflationary gap is so named because the relative increase in real GDP causes an economy to increase its
consumption, which causes prices to rise in the long run.
The main cause of the gap is considered to be expansionary monetary policies carried out by the government.

123. Consider the following statements with reference to the Consumer Price Index (CPI) data before the unification
of the CPI.

1) When the Pay Commissions recommended pay revisions, the base was the CPI Industrial Workers (IW).
2) CPI for Agricultural Labourers (AL) was used for revising minimum wages for agricultural labourers in different
states.

Which of the above is/are correct?

a) 1 only
b) 2 only
c) Both 1 and 2
d) None

Solution: c)
Justification: Statement 1: CPI-IW specifies the government employees (other than banks’ and embassies’
personnel). The wages/salaries of the central government employees are revised on the basis of the changes
occurring in this index, the dearness allowance (DA) is announced twice a year.
CPI for the Urban Non-Manual Employees (CPI-UNME) is basically used for determining dearness allowances (DAs)
of employees of some foreign companies operating in India (i.e. airlines, communications, banking, insurance,
embassies, and other financial services).
Statement 2: The governments at the centre and states remain vigilant regarding the changes in CPI-AL as it shows
the price impact on the most vulnerable segment of the society, this segment spends almost 75 per cent of its total
income on the purchase of food articles.

124. The Wholesale Price Index (WPI) is released by

a) Office of the Governor, RBI


b) Ministry of Finance
c) Central Statistical Organization (CSO), Ministry of Statistics and Programme Implementation
d) Office of the Economic Adviser, DIPP

Solution: d)
Justification: WPI is an important statistical indicator, as various policy decisions of the Government, like inflation
management, monitoring of prices of essential commodities etc., are based on it.
It is one of the key variables for monetary policy changes by the Reserve Bank of India. In addition to its role as a
policy variable, WPI is also used by various departments for arriving at the escalation costs of various contracts.

125. Which of these constitutes the highest weightage in Consumer Price Index (Combined)?

a) Housing
b) Clothing and associated items
c) Fuel and light
d) Food and associated items

Solution: d)
Justification: CPI is disaggregated at the rural and urban levels. The new overall all India CPI is a weighted average of
the two. Picture below: Left column CPI Rural, Middle Column CPI Urban, and Right column CPI Combined weights
of various categories.

126. In India, how is the Producer Price Index (PPI) different from the new series of Wholesale Price Index (WPI)?
1) WPI captures the price changes at the point of bulk transactions whereas PPI measures the average change in
prices at the producer level.
2) PPI excludes indirect taxes whereas WPI includes all applicable direct and indirect taxes.
3) PPI covers services whereas WPI does not include services.
4) WPI removes the multiple counting biases inherent in PPI.

Select the correct answer using the codes below.

a) 1 and 3 only
b) 1, 3 and 4 only
c) 1, 2, 3 and 4
d) 2 and 4 only

Solution: a)

Justification: PPI is different from WPI on following grounds:

Statement 1 and 2: WPI captures the price changes at the point of bulk transactions and may include some taxes
levied and distribution costs up to the stage of wholesale transactions. PPI measures the average change in prices
received by the producer and excludes indirect taxes. A significant change in the new series of WPI (2011-12) has
been the exclusion of indirect taxes while compiling indices of manufactured products. The Working Group for
revision of WPI had recommended that taxes should not figure in this measure so that price signals emerging from
production side of the economy are not influenced by the fiscal policy.

Statement 3 and 4: WPI does not cover services and whereas PPI includes services. Weights of items in WPI are
based on net traded value whereas in PPI weights are derived from Supply Use Table. PPI removes the multiple
counting biases inherent in WPI. Multiple counting occurs when the price for a specific commodity and the inputs
used for its production are included in an aggregate index. PPIs significantly reduce the distortion arising from
multiple counting by deriving weights from Supply Use Table compiled by the CSO. The benefits of migrating from
WPI to PPI are to cover bulk transactions of all goods and services, do away with the bias of double counting
inherent in WPI and to compile indices that are conceptually consistent with the National Accounts Statistics (NAS)
for use as deflators.

UNIT-V
THE GOVERNMENT-BUDGET&FISCAL POLICY

BUDGET

POLICIES AND PROCEDURES

127. Mundell’s ‘impossible trinity’ does NOT include

a) Free capital flows


b) Fixed exchange rate
c) Balanced Budget
d) Independent monetary policy

Solution: c)
Justification: This old trilemma asserts that a country cannot maintain, simultaneously, all three policy goals of – (a)
free capital flows, (b) a fixed exchange rate, and (c) an independent monetary policy.
Option 1: If we maintain free capital flows and a fixed exchange rate, the money supply in the economy will
increase/decrease with an increase/decrease in capital flows. There is no way we can manage inflation in the
economy (due to external influences) as the option of sterilization and exchange rate management in not available.
Option 2: If we maintain free capital flows and have an independent monetary policy, then exchange rate will
change. It cannot be stable.
Option 3: The same applies for the other option, we cannot have free capital flows, if exchange rate is fixed and
monetary policy acts independently. It will block free flow of capital.

TYPES OF BUDGET

128. Consider the following about Gender Budgeting in India.


1) Gender Budget Statements have been made part of Annual Financial Statement by the Government of India.
2) All Ministries or Departments have been instructed to establish a ‘Gender Budgeting Cell (GBC)’.

Which of the above is/are correct?

a) 1 only
b) 2 only
c) Both 1 and 2
d) None
Solution: c)
Justification: Gender Budget Statement was introduced as a part of the Union Budget in 2005-06 in order to
mainstream gender across sectors and all levels of governance. To facilitate integration of gender analysis in
policies, programs and schemes, the Ministry of Finance in consultation with the Ministry of Women and Child
Development had issued a Gender Budget Charter on 8th March, 2007 outlining the composition and functions of
the Gender Budgeting Cells (GBCs). The most important milestone in this regard has been the institutionalization of
the progress through formation of GBCs in various Ministries and Departments. As of now, 57 Central Ministries
/Departments have set up GBCs. Another important progress made in the Gender Budgeting system is inclusion of a
column on gender impact in the Expenditure Finance Committee (EFC) document with effect from 1st April, 2014 for
inclusion of women’s concerns at the planning stage and inclusion of a gender perspective in the Outcome Budget
Process.

129. Zero-base budgeting (ZBB) involves

1) Planning in view that the nation is industrially backward


2) The assumption that there are little or no natural resources to be leveraged
3) Budgeting keeping in mind the GDP at constant prices at factor cost

Select the correct answer using the codes below.

a) 1 only
b) 2 and 3 only
c) 2 only
d) None of the above

Solution: d)

Justification: Zero-base budgeting is the allocation of resources to agencies based on periodic re- evaluation by
those agencies of the need for all the programs for which they are responsible, justifying the continuance or
termination of each program in the agency budget proposal. In other words, an agency reassesses what it is doing
from top to bottom from a hypothetical zero base. There are three special features of this budgeting which
distinguishes it from the traditional budgeting.

Should we spend?

How much should we spend?

Where should we spend?

In India, it is believed to be in practice since 1997–99. We cannot say that India is a success in ZBB, but many of the
profit-fetching PSUs have been able to use it successfully and optimize their profits

RECEIPTS

130. Non-tax revenue receipts of the Government of India do NOT include

a) Profits and dividends which the government gets from its public sector undertakings (PSUs)
b) Loans recovered by the Government of India out of all loans forwarded by it
c) External grants received by the Government
d) None of the above is a part of non-tax revenue receipts.
Solution: b)
Justification: These are the regular accruals to the government based on the revenue that does not come from tax.
In India they are:
Profits and dividends which the government gets from its public sector undertakings (PSUs)
Interest, received by the government out of all loans forwarded by it, be it inside the country (i.e., internal lending)
or outside the country (i.e., external lending).
It means this income might be in both domestic and foreign currencies. Pascal services also generate incomes for
the government. i.e., currency printing, stamp printing, coinage, medals, minting, etc. General Services also earn
money for the government as the power distribution, irrigation, banking, insurance, community services, etc.
Fees, Penalties and Fines received by the government.
Grants which the governments receives— it is always external in the case of the Central Government and internal in
the case of state governments.

131. Capital receipts of the government do NOT include

a) Disinvestment proceeds from PSUs


b) Custom duties on imports and exports
c) Loans received from international organizations
d) Market borrowings of the government from the public

Solution: b)
Justification: Option B: It forms part of the revenue receipt. The main items of capital receipts are loans raised by
the government from the public which are called market borrowings, borrowing by the government from the
Reserve Bank and commercial banks and other financial institutions through the sale of treasury bills, loans received
from foreign governments and international organizations, and recoveries of loans granted by the central
government. Other items include small savings (Post-Office Savings Accounts, National Savings Certificates,
etc.), provident funds and net receipts obtained from the sale of shares in Public Sector Undertakings (PSUs).

132. What is/are the features of the Revenue Receipts of the Government?

1) The receipts recur or flow annually to government corpus.


2) They cannot be reclaimed from the government.

Which of the above is/are correct?

a) 1 only
b) 2 only
c) Both 1 and 2
d) None

Solution: c)
Justification: Statement 1: These are receipts of the government which are non-redeemable, that is, they cannot
be reclaimed from the government. They are divided into tax and non-tax revenues.

Statement 2: Both tax and non-tax revenue occur annually to the government, i.e. they are a continuous source of
income for the government. If there is a source of income that does not recur annually, it is usually classified as
capital receipt, because it is generally a one-time receipt.

EXPENDITURE

133. Which of the following is INCORRECT regarding revenue expenditure of the Government of India?
a) These expenditures usually do not directly involve creation of capital assets.
b) Salaries paid to the Government employees come under revenue expenditure.
c) Expenditures on social services are excluded from this category because they augment the human capital of the
country.
d) Grants given by the government to Indian states and foreign countries come under revenue expenditure.

Solution: c)
Justification: The basic identity of such expenditures is that they are of consumptive kind and do not involve
creation of productive assets. They are either used in running of a productive process or running a government. A
broad category of things that fall under such expenditures in India are:
Interest payment by the government on the internal and external loans;
Salaries, Pension and Provident Fund paid by the government to government employees;
Subsidies forwarded to all sectors by the government;
Defense expenditures by the government;
Postal Deficits of the government;
Law and order expenditures (i.e. police & paramilitary);
Expenditures on social services (includes all social sector expenditures as education, health care, social security,
poverty alleviation, etc.) and general services (tax collection, etc.);
Grants given by the government to Indian states and foreign countries.

134. Revenue Expenditure does NOT include

a) Grants given to state governments


b) Interest payments on debt incurred by the government
c) Advances made from the Foreign Exchange Reserve of India
d) Subsidies offered for various public goods

Solution: c)
Justification: This is expenditure incurred for purposes other than the creation of physical or financial assets of the
central government. It relates to those expenses incurred for the normal functioning of the government
departments and various services, interest payments on debt incurred by the government, and grants given to state
governments and other parties (even though some of the grants may be meant for creation of assets). Apart from
providing implicit subsidies through underpricing of public goods and services like education and health, the
government also extends subsidies explicitly on items such as exports, interest on loans, food and fertilizers.

DEFICIT

135. India’s total external debt is dominated by

a) Debt assumed by the Government to finance social security projects


b) Non-government debt
c) Sovereign debt
d) Grants given by the Government of India

Solution: b)
Justification: India’s external debt stock stood at US$ 471.9 billion at end-March 2017, decreasing by US$ 13.1
billion (2.7 per cent) over the level at end-March 2016. The decline in external debt was due to the decrease in long-
term debt particularly NRI deposits and commercial borrowings. The dominance of non-government debt in total
external debt is evident from the fact that such debt accounted for 65.6 per cent of total debt during the 2000s
decade, against 45.3 per cent in the 1990s. Non-government debt accounted for over 70 per cent of total debt in
the last five years and stood at 80.6 per cent at end-September 2014.
136. External Aids is the best means to finance a government’s fiscal deficit because

1) It brings in foreign currency that is also useful to bridge the Balance of payments (BoP) apart from its utility in
developmental expenditures.
2) It does not cause crowding out effect in the domestic market and is favourable to the domestic borrowers.

Which of the above is/are correct?

a) 1 only
b) 2 only
c) Both 1 and 2
d) None

Solution: c)
Justification: Statement 1: If they are a grant or coming without interest, no better way to finance the deficit, if we
ignore their inflationary effects.
Statement 2: When the domestic market has limited amount of funds, and if the government desires to borrow a
large share of it to finance the fiscal deficit, it tends to raise the demand for funds in the market. This shoots the
market interest rate for the funds and causes problems to the domestic investors who now have to pay a higher
interest rate to avail the same loan.
If the same money is borrowed from abroad, the crowing out effect doesn’t occur.

137. In India, deficit financing is usually resorted in order to

1) Finance the revenue deficit component


2) Undertake developmental expenditure
3) Bridge the short-term Current Account Deficit (CAD)

Select the correct answer using the codes below.

a) 1 and 2 only
b) 2 and 3 only
c) 2 only
d) 1 and 3 only

Solution: a)
Justification: Statement 1: In India, revenue deficit is one of the major reasons for a large fiscal deficit. This means
that the government cannot finance its revenue operations by the resources it generates.
Statement 2: This is done because the internal resources of the government are not adequate to undertake
development expenditure. It must borrow money from the market or abroad.
Statement 3: CAD is financed by external flows. If government borrows from outside it would increase our external
capital deficit, but not affect the short-term CAD.

138. Revenue deficit is problematic in Government budget because

a) The government cannot borrow further for spending on developmental programmes.


b) It is not a sustainable financial position.
c) This indicates rampant tax evasion and tax avoidance.
d) It leads to hyperinflation in both the short-term and long-term.
Solution: b)
Justification: Option A: The government can still borrow leading to further expansion of the fiscal deficit. This has
been the GOI’s budget position since decades.
Option B: This is because the government is spending more than it is earning. It is not maintainable in the long-run,
the extra spending will somehow has to be financed else it will lead to a debt spiral.
Option C: It is not necessary because even a better managed economy in terms of taxation can have poor revenue
yield if the revenue resources are not significant. But, if tax evasion is rampant, it will surely deplete the revenue
resources and add to the revenue deficit.
Option D: This is only a possibility and that too when the revenue deficit is very high and the government ignores
creation of supply chain and production capacity in the economy. This is because due to high revenue expenditure,
demand will increase but supply will not increase in the same proportion.

139. In an open economy without government intervention, trade deficit can be financed by

a) Foreign institutional investment


b) Domestic consumption
c) Monetary expansion
d) All of the above

Solution: a)
Justification: Total balance of payments consists of current account (includes trade, invisibles, remittances etc.) as
well as capital account.

Option A: Capital inflows like FDI, FII help bridge the trade deficit and neutralize BoP.

Option B: High consumption expenditure will further inflate the import bill and cause trade deficit.

Option C: And, so will monetary expansion – pushes up demand and thus imports in the short-term aggravating the
BoP.

140. The situation of fiscal deficit necessarily implies that


1) The Gross Domestic Product (GDP) is higher than Gross National Product (GNP).
2) The government is borrowing more from abroad than it is borrowing from domestic sources.

Which of the above is/are correct?

a) 1 only
b) 2 only
c) Both 1 and 2
d) None

Solution: d)
Justification: Statement 1 and 2: These both are external situations, for e.g. in case of higher GDP than GNP, more
income is being generated abroad by Indian nationals than being generated domestically by foreigners working or
(investing) in India. Moreover, a fiscal deficit situation may not require external borrowing to bridge the deficit at
all, so statement 2 is also incorrect. What this indicates is that the government is spending beyond its means.
Basically, receipts are all forms of money accruing to the government, be it income or borrowings). Fiscal deficit may
be shown in the quantitative form (i.e., the total currency value of the deficit) or in the percentage form of the GDP
for that particular year (percentage of GDP). India has been a country of not only regular but higher fiscal deficits.
Moreover, the composition of its fiscal deficit has been more prone to criticism because it always contains a
revenue deficit component.

141. ‘Twin deficit’ identity in India is usually used to refer to


1) Current account deficit
2) Fiscal deficit
3) Remittance Deficit
4) Savings deficit
5) Balance Sheet Deficit

Select the correct answer using the codes below.

a) 1 and 4 only
b) 2 and 3 only
c) 1 and 2 only
d) 4 and 5 only

Solution: c)
Justification: Twin deficit identity is used to refer to a nation’s current account deficits and a simultaneous fiscal
deficit. The term became widely used in the 1980s until the 1990s because the United States experienced the “twin”
deficits during this timeframe. However, there is no reason why current account deficits and government budget
deficits occurs at the same time.

FISCAL POLICY

142. Which of these are the functions/responsibilities of the Financial Stability and Development Council
(FSDC) that is chaired by the Union Minister of Finance?
1) Inter-regulatory coordination amongst various financial agencies
2) Focus on financial literacy and financial inclusion
3) Assess the functioning of large financial conglomerates
4) Set inflationary targets for the Reserve Bank of India

Select the correct answer using the codes below.


a) 1 and 4 only
b) 2 and 3 only
c) 1, 2 and 3 only
d) 1,2,3 and 4

Solution: c)
Justification: FSDC is super regulatory body for regulating financial sector which is a vital for bringing healthy and
efficient financial system in the economy. The FSDC envisages to strengthen and institutionalize mechanism of (i)
maintaining financial stability, (ii) Financial sector development, (iii) inter-regulatory coordination along with
monitoring macro-prudential regulation of economy. Heads of the financial sector regulatory authorities (i.e., RBI,
SEBI, IRDA, PFRDA), Finance Secretary and others are its members.

TAXES

143. Profits or gains arising from transfer of a capital asset are called “Capital Gains” and are charged to tax under
the head “Capital Gains Tax”. A gain in the price of which of the following would be considered as Capital gains?
1) Archaeological collections
2) Jewellery
3) A work of art
4) Stocks

Select the correct answer using the codes below.

a) 1 and 2 only
b) 4 only
c) 2 and 4 only
d) 1,2,3 and 4

Solution: d)
Justification: Capital asset is defined to include: Any kind of property held by an assesse, whether or not connected
with business or profession of the assesse, stocks, movable property, jewellery; archaeological collections;
drawings; paintings; sculptures; or any work of art.
For e.g. if you purchased a residential house in 2015 for Rs. 10,00,000 and sold it in 2016 for Rs. 20,00,000, a capital
asset gain of Rs. 10,00,000 arising on account of sale of residential house will be charged to tax under the head
“Capital Gains”.

144. What was the reason for introducing the Advance Pricing Agreement (APA) Scheme in India?

1) Improving ease of doing business in India


2) To provide certainty to taxpayers in domain of transfer pricing
3) Monitoring Global price fluctuations in advance to take immediate corrective action
4) Coordinate Quantitative Easing (QE) policies across Central Banks of G-20 countries

Select the correct answer using the codes below.

a) 1 only
b) 1 and 2 only
c) 1 and 3 only
d) 2 and 4 only

Solution: b)
Justification: The APA scheme launched by Government endeavours to provide certainty to taxpayers in domain of
transfer pricing by specifying methods of pricing and setting prices of international transactions in advance. Its
provision was introduced in Income-tax Act, 1961 in 2012 and Rollback provisions to it were introduced in 2014. The
scheme aims to strengthen Government’s resolve of fostering non-adversarial tax regime. It has significantly
contributed towards improving ease of doing business in India and has been appreciated nationally and
internationally for being able to address complex transfer pricing issues in a fair and transparent manner. APA gives
certainty to MNCs that agree on certain principles in valuation of their cross- border transactions. They also provide
assessees with alternate dispute resolution mechanism with respect to transfer pricing. It helps in determining
arm’s length price of international transactions in advance for maximum period of five future years.

145. The term ‘Green Tax’ refers to

1) Excise taxes on environmental pollutants


2) Taxes on goods whose use produces such pollutants

Which of the above is/are correct?


a) 1 only
b) 2 only
c) Both 1 and 2
d) None

Solution: c)
Justification: Ecotax (short for Ecological taxation or Green taxation) refers to taxes intended to promote
environmentally friendly activities via economic incentives. Recently, in a bid to curb health hazards originating from
bursting of firecrackers, the Pune Bench of the National Green Tribunal has directed civic bodies to levy Rs. 3,000 as
‘green tax’ from sellers. The corpus collected from the tax will be used to clean solid waste generated from
firecrackers at public places. Part of the money will be used exclusively for environmental activities such as planting
trees and constructing toilets for women.

146. Consider the following statements about the Securities Transaction Tax (STT).

1) It is a type of an indirect tax.


2) It is levied on purchase or sale of securities that are listed on the Indian stock exchanges.
3) Off-market share transactions are also covered under it.

Select the correct answer using the codes below.

a) 1 and 2 only
b) 2 and 3 only
c) 2 only
d) 1 and 3 only

Solution: c)
Justification: STT is a type of direct tax payable on the value of taxable securities transaction done through a
recognized stock exchange in the country. The securities on which STT is applicable are shares, bonds, debentures,
derivatives, units issued by any collective investment scheme, equity based government rights or interests in
securities and equity mutual funds. Off-market share transactions are not covered under STT. For equity
transactions that are delivery-based, STT for purchase and sale is 0.1% of turnover and for intra-day transactions,
STT for purchase is nil and sale is 0.025% of the turnover.

147. Goods and Services Tax (GST) is a ‘Destination-Based’ Consumption Tax. This implies that

a) All State GST (SGST) collected will ordinarily accrue to the State where the consumer of the goods or services sold
resides.
b) The burden of GST collected within a state will be borne by the consumer of the goods or services within the state.
c) The burden of GST collected within the state will be borne by the producer of the goods or services within the state.
d) All State GST (SGST) collected within the state will accrue to the State where the goods or services are produced.

Solution: a)
Justification: Indirect taxes can be either origin based or destination based. Origin based tax (also known as
production tax) is levied where goods or services are produced. Destination based tax (consumption tax) are levied
where goods and services are consumed. The Goods and Services Tax (GST) is significantly different from the early
tax system as it is a destination-based tax. In destination-based taxation, exports are allowed with zero taxes
whereas imports are taxed on par with the domestic production. So as per current tax rules, the SGST collected will
generally accrue to the State where the consumer of the goods or services sold resides and not to the State where
the goods are produced. One important point to note is that there would be dual GST i.e. both the Centre and the
States would concurrently levy GST across the entire goods and services supply chain on a common base. The
Centre would levy Central GST (CGST) and States would levy State GST (SGST) on every supply of goods and services
within a State. Integrated GST (IGST) would be levied on all inter-state supplies by the Centre and then transferred
to the Destination State.

148. How does the VAT system of taxation reduce the ‘cascading effect’ of taxes?

1) It avoids imposition of tax upon tax.


2) The system insists on single point tax collection.

Which of the above is/are correct?

a) 1 only
b) 2 only
c) Both 1 and 2
d) None

Solution: a)
Justification: Production of goods or services is nothing our stages of value additions where production of goods is
done by the industrialists or manufacturers. From production to the level of sale, there are many points where value
is added in all goods. VAT method of tax collection is different from the non-VAT method in the sense that it is
imposed and collected at different points of value addition chain, i.e., multi-point tax collection. That is why there is
no chance of imposing tax upon tax which takes place in the non-VAT method—single point tax collection. This is
why VAT does not have a cascading effect’ on the prices of goods it does not increase inflation— and is therefore
suitable for an economy like India.

149. General Anti-Avoidance Rules (GAAR) may be applicable if

1) A tax officer believes that the main purpose or one of the main purposes of a transaction arrangement is to obtain a
tax benefit
2) A foreign investment has been made with the main aim of expanding the business of its subsidiaries domestically
3) A corporation has not cleared its tax liabilities since several financial years

Select the correct answer using the codes below.

a) 1 and 2 only
b) 1 only
c) 2 and 3 only
d) 1 and 3 only

Solution: b)
Justification: 'The objective of the GAAR provisions is to codify the doctrine of 'substance overform ' where the real
intention of the parties and purpose of an arrangement is taken into account for determining the tax consequences,
irrespective of the legal structure of the concerned transaction or arrangement. It essentially comes into effect
where an arrangement is entered into with the main purpose or one of the main purposes of obtaining a tax benefit
and which also satisfies at least one of the following four tests:

The arrangement creates rights and obligations that are not at arm's length,

it results in misuse or abuse of provisions of tax laws,

lacks commercial substance or is deemed to lack commercial substance, or


it is not carried out in a bona fide manner.

Such a transaction can be declared as impermissible under the avoidance arrangement.

150. Which of the following statements is correct about Indirect Tax?

a) The incidence and impact of taxation are at different points.


b) Only producers pay indirect taxes.
c) Interest tax is a form of indirect tax.
d) None of the above

Solution: a)
Justification: The point where tax makes its effect felt is known as the impact of tax—the after effect of tax
imposition.
The tax which has incidence and impact at the different points is the indirect tax—the person who is hit does not
bleed’ someone else bleeds. As, for example, excise, sales tax, etc., which are imposed on either the producers or
the traders, but it is the general consumers who bear the burden of tax. The tax which has incidence and impact
both at the same point is the direct tax—the person who is hit, the same person bleeds. As for example income tax,
interest tax, etc.

151. An inverted duty structure for a particular product will tend to discourage its

1) Domestic value addition


2) Associated Foreign Direct Investment
3) Import as finished goods as compared to its raw material

Select the correct answer using the codes below.

a) 1 and 2 only
b) 2 and 3 only
c) 1 and 3 only
d) 2 only

Solution: a)
Justification: Inverted duty structure is a situation where import duty on finished goods is low compared to the
import duty on raw materials that are used in the production of such finished goods. For example, suppose the tariff
(import tax) on the import of tires is 10% and the tariff on the imports of natural rubber which is used in the
production of tires is 20%; this is a case of inverted duty structure.

Statement 1: When the import duty on raw materials is high, it will be more difficult to produce the concerned
good domestically at a competitive price. Several industries depend on imported raw materials and components.
High tax on the raw materials compels them to raise price. The disadvantage of the inverted duty structure
increases with the increased use of imported raw materials. An inverted duty structure discourages domestic value
addition.

Statement 2: On the other hand, foreign finished goods will be coming at a reduced price because of low tax
advantage. In conclusion, manufactured goods by the domestic industry becomes uncompetitive against imported
finished goods. In such a case, even foreign investors would not be interested in setting up a firm for production in
the country.

Statement 3: It will be just the opposite.


152. Countervailing duty, a term often heard in news, is related to

a) Import duties
b) Excise duties
c) Corporation tax
d) Stamp duty

Solution: a)
Justification: Duties that are imposed in order to counter the negative impact of import subsidies to protect
domestic producers are called countervailing duties. In cases foreign producers attempt to subsidize the goods
being exported by them so that it causes domestic production to suffer because of a shift in domestic demand
towards cheaper imported goods, the government makes mandatory the payment of a countervailing duty on the
import of such goods to the domestic economy. This raises the price of these goods leading to domestic goods again
being equally competitive and attractive. Thus, domestic businesses are cushioned. These duties can be imposed
under the specifications given by the WTO (World Trade Organization) after the investigation finds that exporters
are engaged in dumping. These are also known as anti- dumping duties.

153. Goods and Services Tax Network (GSTN) is

1) is a non-profit company that serves as the interface between the government and the taxpayers
2) is owned by the Government as a major stakeholder
3) provides IT infrastructure and services to the Central and State Governments with regard to GST

Select the correct answer using the codes below.

a) 1 and 2 only
b) 2 and 3 only
c) 1 and 3 only
d) 1, 2 and 3

Solution: c)
Justification: S1: Goods and Services Tax Network (GSTN) is a Section 8 (under new companies Act, not for profit
companies are governed under section 8), non-Government, private limited company.
The government owns 49 per cent of the share in GSTN and rest of the 51 per cent stake is owned by private players
such as - LIC Housing Finance, HDFC Bank, ICICI Bank and NSE Strategic Investment Co.
S2 and S3: The Company has been set up primarily to provide IT infrastructure and services to the Central and State
Governments, tax payers and other stakeholders for implementation of the Goods and Services Tax (GST).
To be able to pay taxes through GSTN, the taxpayers need to register themselves on GST portal and get a Goods and
Services Tax Identification Number (GSIN). The 15 digits GSTIN serves as the one identification number for
taxpayers.

154. A tax is buoyant when

a) Tax revenue collection registers a steady annual rise irrespective of other economic changes
b) Tax revenues increase by more than 1 per cent for a 1 per cent increase in Tax base
c) Tax revenues are relatively unaffected by economic slowdown or a phase of boom
d) Tax revenues increase by more than 1 per cent for a 1 per cent increase in GDP

Solution: d)
Justification: Tax buoyancy is an indicator to measure efficiency and responsiveness of revenue mobilization in
response to growth in the Gross domestic product or National income. A tax is said to be buoyant if the tax
revenues increase more than proportionately in response to a rise in national income or output. A tax is buoyant
when revenues increase by more than, say, 1 per cent for a 1 per cent increase in GDP. Usually, tax elasticity is
considered a better indicator to measure tax responsiveness.

Tax elasticity is a measure designed for this purpose since it measures the responsiveness of tax revenue to a
change in national income or output after controlling for exogenous influences such as discretionary changes in tax
policy. If a tax is elastic, a one percent increase in GNP or GDP results in a greater than one percent increase in
revenue from the tax holding constant for discretionary tax changes

155. Consider the following statements. Tax policy of a government directly affects the

1) Level of savings and investment in the economy


2) Attractiveness of foreign investment

Which of the above is/are correct?

a) 1 only
b) 2 only
c) Both 1 and 2
d) None

Solution: c)
Justification: Taxes directly affect the savings of individuals because high taxes erode income, and low taxes help
the individual have more savings. If firms pay less taxes, they save more and invest more which affects the
aggregate level of investment in the economy. As investment affects the output (GDP), taxes also have an influence
over the per capita income and thus the attractiveness of the foreign investors who look for good returns in the
economy. Taxes also affect the prices of goods and services as factor cost (production cost) is affected thereby
affecting incentives and behaviour of the economic activities, etc.

156. Consider the following statements.

1) India’s tax revenue as a percentage of GDP has been steadily increasing in the past decade.
2) General Government Debt as a percentage of GDP has steadily reduced in the past decade.

Which of the above is/are correct?

a) 1 only
b) 2 only
c) Both 1 and 2
d) None

Solution: d)
Justification: It is striking that the center’s tax-GDP ratio is no higher than it was in the 1980s, despite average
economic growth of 6.5 percent, the most rapid in India’s history. General government debt-to-GDP ratio is the
amount of a country's total gross government debt as a percentage of its GDP. It is an indicator of an economy's
health and a key factor for the sustainability of government finance.
India recorded a government debt equivalent to 69.50 percent of the country's Gross Domestic Product in 2016.
Government Debt to GDP in India averaged 73.42 percent from 1991 until 2016, reaching an all-time high of 84.20
percent in 2003 and a record low of 66 percent in 1996.

157. Which of the following statements is correct about applicability of income tax provision to sale of farm produce
by farmers to traders?

1) All cash sales of agricultural produce by a cultivator to a trader is taxable.


2) The cultivator and the trader must provide their Permanent Account Number (PAN) in all such transactions.

Which of the above is/are correct?

a) 1 only
b) 2 only
c) Both 1 and 2
d) None

Solution: d)
Justification: Statement 1 and 2: Agriculture in India is generally tax exempt, and dealt largely by state taxes since
agriculture in India is a state subject. However, the Income Tax Department has clarified that the cash sales of
agricultural produce by a cultivator to a trader for less than 2 lakh rupees will not attract tax under the Income Tax
Act.

A circular issued by the Department said, the cultivator and the trader will also not have to provide their Permanent
Account Number (PAN) or furnish Form 60. The clarification came following representations from stakeholders
about applicability of income tax provision to cash sale of farm produce by farmers to traders.

158. Which of the following countries have a system of “Inheritance tax”?

1) USA
2) UK
3) Spain
4) India

Select the correct answer using the codes below.

a) 1, 2 and 3 only
b) 2 and 4 only
c) 3 and 4 only
d) 1, 3 and 4 only

Solution: a)
Justification: Also popularly known as estate tax or estate duty, Inheritance tax was a tax that was levied against a
particular asset during the time of its inheritance. For example, the inheritance of ancestral land.
There are certain countries that practice this form of taxation. Countries like USA, UK, Netherlands, Spain and
Belgium all follow inheritance tax and China had gone to the extent of introducing rules for inheritance tax back in
2002 but was met with heavy opposition to the idea and were not able to implement it.
Statement 4: Inheritance tax is no longer levied in India and was abolished during the time of the Rajiv Gandhi
Government in 1985. Though its intentions were noble, the then finance minister, V.P. Singh was of the opinion that
it had failed to bring about an equilibrium in society and reduce the wealth gap. During its stay, inheritance tax or
estate duty was levied from the period between 1953 and 1985.

159. A Carbon tax imposed by the government necessarily leads to

1) Net revenue loss for the economy


2) Cumulative GDP loss with successive financial years

Which of the above is/are correct?

a) 1 only
b) 2 only
c) Both 1 and 2
d) None

Solution: d)
Justification: A carbon tax is a tax levied on the carbon content of fuels. It is a form of carbon pricing. Since GHG
emissions caused by the combustion of fossil fuels are closely related to the carbon content of the respective fuels,
a tax on these emissions can be levied by taxing the carbon content of fossil fuels at any point in the product cycle
of the fuel.

Statement 1: Carbon tax is revenue positive when it involves no adjustment to other tax rates in the economy. It is
revenue neutral when other tax rates are adjusted so that the revenue Inflow from carbon tax is exactly balanced by
an equal reduction in yields from reduced taxes. So, a carbon tax does not have to lower the revenues.
Statement 2: Carbon taxes offer a potentially cost-effective means of reducing greenhouse gas emissions. It may
actually help the economy in the long-run by promoting sustainable development by promoting environment
friendly methods of growth. It does not have to hurt the GDP growth necessarily even in the short-run because the
benefits from non-pollution may far exceed the economic benefits of an emission laden economy.
From an economic perspective, carbon taxes are a type of Pigovian tax. They help to address the problem of
emitters of greenhouse gases not facing the full social cost of their actions. Carbon taxes can be a regressive tax, in
that they may directly or indirectly affect low-income groups disproportionately. The regressive impact of carbon
taxes could be addressed by using tax revenues to favour low- income groups.
160. Recently, in a bid to curb health hazards originating from bursting of firecrackers, the Pune Bench of the
National Green Tribunal has directed civic bodies to levy ‘green tax’ on sellers. Ecotaxes are examples of ‘Pigouvian
taxes’, which are taxes that
1) Attempt to make the private parties involved feel the social burden of their actions
2) Shift the incidence of a direct tax in the form of an indirect tax
3) Applied only on business firms or commercial entities

Select the correct answer using the codes below.


a) 2 only
b) 2 and 3 only
c) 1 only
d) 1, 2 and 3

Solution: c)
Justification: Ecotax (short for Ecological taxation or Green taxation) refers to taxes intended to promote
environmentally friendly activities via economic incentives. A Pigovian tax is a tax levied on any market activity that
generates negative externalities (costs not internalized in the market price). The tax is intended to correct an
inefficient market outcome, and does so by being set equal to the social cost of the negative externalities. For e.g. if
a factory dumps polluted water in a local stream, it affects the water supply of the local community. But, this social
cost is not included in the company’s cost-benefit analysis. By imposing this tax, the negative externalities are
accounted for.

161. Rebate of State Levies (ROSL) was recently restored by the government for which of the following sectors?

a) Textiles
b) Manufacturing
c) Petroleum
d) Coal

Solution: a)
Justification: In a bid to boost employment generation in the employment intensive textiles and apparel sector the
Government notified the scheme for Remission of State Levies on export of garments and made ups through the
mechanism of rebate. The two schemes have been merged under the Scheme for Rebate of State Levies on Export
of Garments and Made-ups. ROSL, thus, makes the garment export more competitive.

162. With reference to GST, what is Revenue Neutral Rate (RNR)?

a) A tax rate that leaves the tax base unaltered


b) A common rate of tax to both the Central and State governments
c) A GST rate that would keep the revenue of State governments on par with that of the Central government
d) A single rate of tax that would preserve revenue at pre-GST levels

Solution: d)
Justification: The term revenue neutral rate (RNR) refers to that single rate, which preserves revenue at desired
(current) levels. This was one of the major outcomes of the pre-GST negotiation between States and Centre, where
the former were experiencing loss of revenue post-GST. The RNR should be distinguished from the “standard” rate
defined as that rate in a GST regime which is applied to all goods and services whose taxation is not explicitly
specified. Typically, the majority of the base (i.e., majority of goods and services) will be taxed at the standard rate,
although this is not always true
163. Consider the following about Advance Pricing Agreements (APA) scheme.
1) It was introduced by the Income-tax (IT) Act.
2) It cannot be signed with individual companies.
3) It endeavors to provide certainty to taxpayers in the domain of transfer pricing.
4) It is not applicable to international transactions.

Select the correct answer using the codes below.


a) 1 and 3 only
b) 2 and 4 only
c) 2, 3 and 4 only
d) 1, 3 and 4 only

Solution: a)

Justification: Suppose two subsidiaries A and B of the same company X are located in two different tax regimes. Say
A is subjected to more taxes, and B has to pay less tax. If company X transfers most of its profit from A to B, it will
need to less taxes then, because B is taxed at lower rate. This is called transfer pricing, when profits are transferred
to subsidiaries to reduce tax burden.
Statement 2 and 3: It is signed with companies so that this tax evasion can be regulated. It also gives certainty to
the tax department in terms of revenue collection.

164. In taxation, horizontal equity implies

a) Similarly situated people pay the same taxes.


b) Better off people pay more taxes.
c) Everyone pays the same amount of tax.
d) Taxation is independent of income levels.

Solution: a)
Learning: Taxation is based on the idea of Fairness. Though fairness (i.e., the first criteria of a good tax system) is
not always easy to define, economists suggested inclusion of two elements in the tax system to make it fair namely,
horizontal equity and vertical equity. Individuals in identical or similar situations paying identical or similar taxes is
known as horizontal equity. When ‘better off people pay more taxes it is known as vertical equity. When we
combine both equities, we each closer to the Indian taxation system.

165. Which of these taxation systems is most closely related to what India follows?

a) Proportional system
b) Regressive system
c) Progressive system
d) Neither progressive nor proportional

Solution: c)
Justification: Progressive taxation method has increasing rates of tax for increasing value or volume on which the
tax is being imposed. Indian income tax is a typical example of it. The idea here is less tax on the people who earn
less and higher tax on the people who earn more—classifying income earners into different slabs. This method is
believed to discourage more earnings by the individual to support low growth and development unintentionally.
Being poor is rewarded while richness is punished. Tax payers also start evading tax by showing lower unreal
income. But from different angles this taxis pro-poor and taxes people according to their affordability/sustainability.
This is the most popular taxation method in the world and a populist one, too. Regressive system is just the reverse
of this.
Proportional Taxation - In such a taxation method, there is neither progression nor regression from the point of
view rate of taxes point of view. Such taxes have fixed rates for every level of income or production, they are neutral
from the poor or rich point view or from the point of view of the levels of production.

166. Often seen in news, Minimum Alternate Tax (MAT) is

a) A form of direct tax


b) Imposed on High Net Worth Individuals (HNIs)
c) Applied on goods for import and export in foreign trade
d) All of the above

Solution: a)
Justification: Income tax is paid as per the provisions of the Income Tax Act (IT Act) but companies calculate their
profit (through profit and loss account) as per the provisions of the Companies Act. The IT Act allows several kinds of
exemptions and other incentives from total income together with deductions on the gross income. As a result of
these exemptions, deductions and other incentives under IT Act, companies show their taxable income either ‘nil’ or
‘negative’, and this way, the ‘zero tax’ companies emerge and pay no taxes even though they may be paying huge
dividends (under the Companies Act) to their shareholders. To bring such companies under the income tax, MAT
was imposed accordingly. It is applicable on all companies except those engaged in infrastructure and power
sectors, free trade zones, charitable activities, and venture and angel funds. Foreign companies with income sources
in India also come under it

167. The objective of the GST composition scheme is to


1) Reduce duties and taxes on all raw materials to zero
2) Maximize the tax on value added to the final product
3) Reduce the tax compliance cost for small businesses
4) Charge differential taxes for the same product based on different compositions

Select the correct answer using the codes below.

a) 3 only
b) 1 and 2 only
c) 4 only
d) 1, 2 and 3 only

Solution: a)
Justification: The composition scheme is an alternative method of levy of tax designed for small taxpayers whose
turnover is up to Rs 75 lakh — Rs 50 lakh in the case of eight north-eastern states and the hilly state of Himachal
Pradesh. To promote small business, and reduce their tax burden, the scheme provides that:
 While a regular taxpayer has to pay taxes on a monthly basis, a composition supplier is required to file only one
return and pay taxes on a quarterly basis.
 Also, a composition taxpayer is not required to keep detailed records that a normal taxpayer is supposed to
maintain.

As per the Central GST Act, businesses are eligible to opt for the composition scheme if a person is not engaged in
any inter-state outward supplies of goods and not into making any supply of goods through an electronic commerce
operator who is required to collect tax at source. Five lakh businesses have opted for the GST composition scheme,
which allows them to pay taxes at a concessional rate and makes compliance easy.
SUBSIDIES

168. Why do many policymakers advocate withdrawal of public subsidy?

1) They block the market forces from affecting the price of a good or service completely.
2) They induce market distortions and do not allow the market forces to accurately reflect the scarcity or excess of a
particular good.

Which of the above is/are correct?

a) 1 only
b) 2 only
c) Both 1 and 2
d) None

Solution: b)
Justification: Statement 1: Only if prices are administered, the market loses it pricing power, for e.g. in case of
essential medicines. Even when diesel and petrol prices were regulated in India, market had the freedom of pricing.
The state bore the additional burden of subsidies.

Statement 2: For e.g. the price of water does not reflect its scarcity when water is provided for free by public
agencies, even when there is acute shortage of water in the country. When electricity and water are provided at a
subsidized rate or free, they will be used wastefully without any concern for their scarcity.

PUBLIC FINANCE

DISINVESTMENT

169. A strategic sale of a Public Sector Unit (PSU) necessarily involves


1) Complete privatization of a PSU
2) Selling majority of shares to a public listed financial corporation
3) Moving production lines to strategic goods rather than non-strategic goods and services

Select the correct answer using the codes below.

a) 1 only
b) 2 and 3 only
c) 1 and 3 only
d) None of the above

Solution: d)
Justification: There was a major shift in the disinvestment policy from selling small lots of share in the profit-making
PSUs (i.e., token disinvestment) to the strategic sale with change in management control both in profit and loss-
making enterprises. The essence of the strategic disinvestment was—

The minimum shares to be divested will be 51 per cent, and the wholesale sale of shares will be done to a 'strategic
partner' having international class experience and expertise in the sector. This form of disinvestment commenced
with the Modern Food Industries Ltd. (MFIL).
170. The salient features of the Disinvestment Policy are

1) Public Sector Undertakings are the wealth of the Nation and to ensure this wealth rests in the hands of the people
to promote public ownership of CPSEs
2) As far as possible, the Government will undertake strategic sale of CPSEs to ensure greater efficiency and economic
deregulation
3) Funds derived from disinvestment shall strictly not be used to bridge revenue deficit and be deployed for social
security schemes

Select the correct answer using the codes below.

a) 1 only
b) 1 and 2 only
c) 1, 2 and 3
d) 3 only

Solution: a)
Justification: The policy on disinvestment has evolved considerably through President’s address to Joint Sessions of
Parliament and statement of the Finance Minister’s in their Budget Speeches. It includes the following (other than
S1):
While pursuing disinvestment through minority stake sale in listed CPSEs, the Government will retain majority
shareholding, i.e. at least 51 per cent of the shareholding and management control of the Public Sector
Undertakings;
Strategic disinvestment by way of sale of substantial portion of Government shareholding in identified CPSEs up to
50 per cent or more, along with transfer of management control.
Strategic Disinvestment Policy:

I. To be undertaken through a consultation process among different Ministries/Departments, including NITI Aayog.
II. NITI Aayog to identify CPSEs for strategic disinvestment and advice on the mode of sale, percentage of shares to be
sold of the CPSE and method for valuation of the CPSE.
III. The Core Group of Secretaries on Disinvestment (CGD) to consider the recommendations of NITI Aayog to facilitate
a decision by the Cabinet Committee on Economic Affairs (CCEA) on strategic disinvestment and to
supervise/monitor the process of implementation.

171. Which of these authorities approves the sale of government stake in the Public Sector Units (PSUs)?

a) Cabinet Committee on Economic Affairs (CCEA)


b) Governor, Reserve Bank of India
c) Finance Secretary, Ministry of Finance
d) Department of Industrial Policy and Promotion (DIPP)

Solution: a)
Justification: Recently, the CCEA has approved sale of government’s more than 51% stake along with management
control in HPCL (Hindustan Petroleum) to ONGC. HPCL will continue as PSU after the acquisition. A majority stake
sale, if offered to a private enterprise, is termed as a strategic sale that usually involves shedding management
control by the government. On the other hand, in a minority stake, only part of the stake is offered, and the
government retains both the ownership and the management control.

172. ‘Bharat 22’, recently seen in news, is expected to

a) Hasten Government’s disinvestment programme


b) Improve conservation of 22 scheduled languages in the Constitution of India
c) Increase maritime and land security of all coastal and border states of India
d) Reduce red tapism in the delivery of welfare schemes

Solution: a)
Justification: This is an exchange traded fund (ETF) launched by Union Finance Ministry.

 It will help to speed up disinvestment programme budgeted to raise a record Rs 72,500 crore in the FY 2018.
 Bharat 22 spans six sectors — basic materials, energy, finance, industrials, FMCG and utilities.

The sector wise weightage in the Bharat 22 Index is basic materials (4.4%), energy (17.5%), finance (20.3%), FMCG
(15.2%), industrials (22.6%), and utilities (20%).

MISC

173. Consider the following statements.


Assertion (A): Most of India’s Foreign exchange reserves is maintained in the form of Gold.
Reason (R): India is the world's largest gold consuming nation.

In the context of the above, which of these is correct?

a) A is correct, and R is an appropriate explanation of A.


b) A is correct, but R is not an appropriate explanation of A.
c) A is incorrect, but R is correct.
d) A is correct, but R is incorrect.

Solution: c)
Justification: The Foreign exchange reserves of India consists of below four categories: Foreign Currency Assets,
Gold bullion, Special Drawing Rights (SDRs), Reserve Tranche Position in the IMF. As of September 2017, India's
foreign exchange reserves are mainly composed of US dollar in the forms of US government bonds and institutional
bonds with nearly 5% of forex reserves in gold. India is, coincidentally the world's largest gold consuming nation
mainly due to the cultural importance attached to Gold and the large population. The reserves are managed by the
Reserve Bank of India and the main component is foreign currency assets.

Foreign exchange reserves act as the first line of defense for India in case of BoP crisis. While forex reserves act as
insurance when the rupee tends to be volatile against the dollar, there are costs attached to it. When RBI purchases
dollars in the spot, it leads to infusion of rupee into the system which leaves inflationary effect on the economy.
Since the RBI does not want such actions to create inflationary pressure, so, it converts spot purchases into
forwards. 'This way, it is a direct cost because of the forward premiums. If RBI opts for open market operations
(OMOs) to mop up excess liquidity, that also involves costs. Also, the idle forex can be used to finance infrastructure
and other developmental projects and reduce their cost of upkeep.

174. Why do several Indian economists caution against printing currency?

1) It is inflationary in that it may not be accompanied with simultaneous expansion in productive capacity of the
economy.
2) It requires Gold backing which pushes the price of Gold and thus the import bill of India.

Which of the above is/are correct?

a) 1 only
b) 2 only
c) Both 1 and 2
d) None

Solution: a)
Justification: Statement 1: When we print currency, we inject additional demand in the economic system, and this
increased demand need not be accompanied by increased supply of goods. This may be inflationary in most cases. It
brings in regular pressure and obligation on the government for upward revision in wages and salaries of
government employees— ultimately increasing the government expenditures necessitating further printing of
currency and further inflation—a vicious cycle into which economies entangle themselves. Hence, printing currency
is the last resort for the government in managing its deficit.

Statement 2: The system of gold backing has been repealed, and the RBI manages printing of currency. This is
because the additional currency is a liability on RBI and it must ensure that it has sufficient reserves to clear this
liability.

175. Public Procurement is one of the top priorities of the present Government. In this context what are the benefits
of Government e-Marketplace (GeM)

1) GeM considerably reduces human interface in order placement and payment processing for the Government
departments thus cutting down corrupt practices.
2) All the documents on GeM are e-Signed at various stages by the buyers and sellers that makes it secure and
transparent.

Which of the above is/are correct?

a) 1 only
b) 2 only
c) Both 1 and 2
d) None

Solution: c)
Justification: Gem provides an end-to-end online Marketplace for Central and State Government Ministries /
Departments to the Government, Sellers and the Indian industry and economy. GeM is a completely paperless,
cashless and system driven e-market place that enables procurement of common use goods and services with
minimal human interface. It is a very bold step of the Government with the aim to transform the way in which
procurement of goods and services is done by the Government Ministries/Departments, PSUs, autonomous bodies
etc.

Benefits are:

Transparency: GeM eliminates human interface in vendor registration, order placement and payment processing, to
a great extent. Being an open platform, GeM offers no entry barriers to bonafide suppliers who wish to do business
with the Government

Efficiency: Direct purchase on GeM can be done in a matter of minutes and the entire process in online, end to end
integrated and with online tools for assessing price reasonability.

Secure and safe: GeM is a completely secure platform and all the documents on GeM are e-Signed at various stages
by the buyers and sellers.
Potential to support Make in India: On GeM, the filters for selecting goods which are Preferential Market Access
(PMA) compliant and those manufactured by Small Scale Industries(SSI), enables the Government buyers to procure
Make in India and SSI goods very easily.

Savings to the Government: The transparency, efficiency and ease of use of the GeM portal has resulted in a
substantial reduction in prices on GeM, in comparison to the tender, Rate Contract and direct purchase rates.

176. In India, E-way Bill is needed when

a) Moving cargo over long distances


b) Entering an Inner Line Permit (ILP) state
c) Crossing to High Seas from territorial waters
d) Navigating a protected area

Solution: a)
Justification: E-way is an electronic billing system for traders who are moving their goods for sale beyond 10 km
radius. E-way is valid for all-India movement and transporters can generate the bill electronically and in self-service
mode. Under the system, there is no need to visit any tax office and check post. While the mandatory compliance of
inter-state E-way bill may roll-out soon once again, however, for intra-state, the date is June 1 2018. States such as
Kerala, Rajasthan, Uttarakhand and Karnataka have already implemented the e-way bill system. However, some
traders are not happy with the move. Traders are demanding simplification of the process for generation of these
electronic receipts. Explaining that it will be more of a hindrance than compliance, traders say the move will force
them to shut shops.

177. Capital Adequacy Ratio (CAR) is decided by central banks and bank regulators to

a) Ensure that a greater part of the bank’s lending is allocated to long gestation period infrastructure projects
b) Reduce the exposure of Indian banking to the Global economy
c) Prevent Commercial banks from taking excess leverage and becoming insolvent in the process
d) All of the above

Solution: c)
Justification: It is the ratio of a bank's capital in relation to its risk weighted assets and current liabilities. As credit
creation (i.e. loan disbursals) of banks is highly risky business, the depositors’ money depends on the banks’ quality
of lending. If the bank funding goes largely in risky projects such as infrastructure, chances of bank insolvency is
high. Having adequate bank capital (compared to liabilities) helps to prevent bank failure, which arises in case the
bank cannot satisfy its obligations to pay the depositors and other creditors. It is also a part of the overall Basel
norms. Since the whole payment system, public as well as private, depends on banks, safeguarding banks is crucial
to sound economic health.

178. Marginal Cost of funds-based Lending Rate (MCLR) is based/tied on

a) The risk taking capacity of borrowers


b) The willingness of the Bank
c) The Government Security Interest Rate
d) Prevalent security market rates for Commercial bills

Solution: a)
Justification: A large portion of bank loans remain linked to the base rate despite the introduction of the MCLR in
April 2016. Weak monetary transmission during a rate cut cycle has been one of the central bank’s pet peeves. RBI
has proposed to link the base rate for loans with the marginal cost of funds-based lending rate (MCLR) from 1 April
to improve monetary policy transmission. Unlike base rate regime, these rates are expected to get revised on
monthly basis along with the repo rate including other borrowing rates. Banks decide the actual lending rate based
on the floating rate by adding the component of spread to MCLR which becomes the final lending rate. The MCLR
system was introduced by the Reserve Bank to provide loans on minimal rates as well as market rate fluctuation
benefit to customers. MCLR is calculated on the basis of incremental cost of funds, making it a more reliable
benchmark rate as compared to the base rate, usually calculated by taking into account average cost of funds based
on the borrower’s risk profile.

179. The Market Stabilization Scheme (MSS) was launched with the objective of strengthening RBI's ability to
1) Control liquidity in the market
2) Regulate government borrowing

Which of the above is/are correct?

a) 1 only
b) 2 only
c) Both 1 and 2
d) None

Solution: a)
Justification: Historically, the RBI had been sterilizing the effects of significant capital inflows on domestic liquidity
by offloading parts of the stock of Government Securities held by it. The MSS was devised since continuous resort to
sterilization by the RBI depleted its limited stock of Government Securities and impaired the scope for similar
interventions in the future.
Statement 1: Under this scheme, the GoI borrows from the RBI (such borrowing being additional to its normal
borrowing requirements) and issues Treasury-Bills/Dated Securities that are utilized for absorbing excess liquidity
from the market. Therefore, the MSS constitutes an arrangement aiding in liquidity absorption, in keeping with the
overall monetary policy stance of the RBI, alongside tools like the Liquidity Adjustment Facility (LAF) and Open
Market Operations (OMO). The securities issued under MSS, termed as Market Stabilization Scheme (MSS)
Securities/Bonds, are issued by way of auctions conducted by the RBI and are done according to a specified ceiling
mutually agreed upon by the GoI and the RBI.
Statement 2: It was not launched with the objective of regulating or controlling government borrowing. It is just that
the government securities are involved in the operation for offloading liquidity from the market.

180. Consider the following statements.

1) India presently follows the International Financial Reporting Standards (IFRS) developed by the Financial Stability
Board (FSB).
2) Companies and Banks in India have been directed to follow the Indian Accounting Standards by the Ministry of
Finance.

Which of the above is/are correct?

a) 1 only
b) 2 only
c) Both 1 and 2
d) None

Solution: d)
Justification: Statement 1: Banks and non-banking financial companies currently follow Indian generally accepted
accounting principles (GAAP) standards. Other corporate entities started complying with IndAS with effect from 1
April 2016. The Financial Accounting Standards Board (FASB) issues GAAP. The international alternative to GAAP is
the International Financial Reporting Standards (IFRS).
Statement 2: Since Companies already follow IndAS, it is the banks, and especially Public Sector Banks that need to
follow IndAS. Ind AS or Indian Accounting Standards govern the accounting and recording of financial transactions as
well as the presentation of statements such as profit and loss account and balance sheet of a company.

Indian Accounting Standard (abbreviated as Ind-AS) is the Accounting standard adopted by companies in India and
issued under the supervision and control of Accounting Standards Board (ASB). For long, there has been a heated
debate about Indian companies moving to the globally accepted International Financial Reporting Standards (IFRS)
for their accounts. But firms have resisted this shift, stating that this will lead too many changes in the capture and
reporting of their numbers. Ind AS has been evolved as a compromise formula that tries to harmonise Indian
accounting rules with the IFRS.
International Financial Reporting Standards (IFRS) is a set of accounting standards, developed by the International
Accounting Standards Board (IASB) that is becoming the global standard for the preparation of public company
financial statements. The IASB is an independent accounting standards body, based in London

181. Real Interest Rate is Nominal Interest rate adjusted for

a) Risk
b) Inflation
c) External interest rates
d) Economic Growth

Solution: b)
Justification: The real interest rate is the nominal interest rate – inflation rate. For example, if base rates is 5.5% and
the CPI inflation rate is 3.4%. Then the real interest rates is said to be 2.1%. A higher real interest rate is good for
savers and bad for borrowers. Note, even if nominal interest rates were high e.g. 11%, savers would see a decline in
their real value of money if inflation was 12%. This is why the real interest rate is important. Real interest rates can
be negative if inflation is higher than nominal interest rates.

182. With reference to Public Financial Management System (PFMS), consider the following statements.

1) Government has decided to universalise the use of PFMS to cover all transactions and payments under the Central
Sector Schemes.
2) PFMS can provide real time information on financial resource flows and actual utilization in government schemes.
3) It is being administered by National Payments Corporation of India (NPCI).

Select the correct answer using the codes below.

a) 1 and 2 only
b) 3 only
c) 2 and 3 only
d) 1, 2 and 3

Solution: a)
Justification: Statement 1 and 2: It is an end-to-end solution for processing payments, tracking, monitoring,
accounting, reconciliation and reporting. It provides the scheme managers a unified platform for tracking releases
and monitoring their last mile utilization. Mandatory use of Public Finance Management System (PFMS) will help
monitor the flow of funds to beneficiaries of different government welfare schemes. The actual status of utilization
of funds by the multiple implementing agencies of the Central and the State Governments can be known.

Statement 3: The Department of Expenditure is administering the PFMS.


It has tremendous potential to improve programme/financial management, reduce the float in the financial
systems by enabling ‘just in time’ releases and also the Government borrowings with direct impact on interest costs
to the Government.

183. Public goods provide which of the following advantages to the general population?

1) A number of public goods are affordable which when privately provided are expensive.
2) Public goods create a social security net for the downtrodden and poor, especially in times of economic recession.

Which of the above is/are correct?

a) 1 only
b) 2 only
c) Both 1 and 2
d) None

Solution: c)
Justification: Statement 1: Sanitation, education, drinking water, health etc. come under the category of public
goods and services. When provided privately, they often become unaffordable for the poor masses. When they are
provided at a lower price, people can afford them and the money that is spare can be spent in the market.

Statement 2: During recession, often the demand for labour goes down and the first ones to be laid off from the job
are the contract workers who are usually at the lower rung of the society. If public goods like health, education,
drinking water etc. are provided at low cost or free of cost, it gives them a safety net even as they are unemployed.
The money saved from spending on these public goods can be used for private consumption

184. Consider the following statements about Bharat Bill Payment System (BBPS).

1) Under BBPS, a customer can pay several bills at a single location.


2) BBPS transactions can be initiated only through Internet for enhanced security.

Which of the above is/are correct?


a) 1 only
b) 2 only
c) Both 1 and 2
d) None

Solution: a)
Justification: Statement 1: It is a RBI conceptualized system driven by National Payments Corporation of India
(NPCI). It is a one-stop payment platform for all bills providing an interoperable and accessible “Anytime Anywhere”
bill payment service to all customers across India with certainty, reliability and safety of transactions.
Statement 2: BBPS transaction can be initiated through multiple payment channels like Internet, Internet Banking,
Mobile, Mobile-Banking, POS (Point of Sale terminal), Mobile Wallets, MPOS (Mobile Point of Sale terminal), Kiosk,
ATM, Bank Branch, Agents and Business Correspondents.

UNIT-VI
AGRICULTURE AND FOOD MANAGEMENT

AGRICULTURE AND ALLIED ACTIVITIES


185. The Ministry of Consumer Affairs, Food and Public Distribution launched Electronic Negotiable Warehouse
Receipt (e-NWR). What are the benefits of e-NWR?

1) It can be traded, sold, swapped and used as collateral to support borrowing or loans from banks.
2) It will allow more convenient trading of produce on the electronic National Agriculture Markets (e- NAM).

Which of the above is/are correct?

a) 1 only
b) 2 only
c) Both 1 and 2
d) None

Solution: c)
Justification: It aims at simplifying Warehouse Registration Rules, digitizing entire process of registration,
monitoring and surveillance as well as creation and management of NWRs in electronic form through two
repositories. NWRs are documents issued by warehouses to depositors against commodities deposited in
warehouses for which warehouse is bailee. NWRs can be traded, sold, swapped and used as collateral to support
borrowing or loans from banks. These receipts were made negotiable under Warehouse (Development and
Regulation) Act, 2007 and are regulated by Warehousing Development and Regulatory Authority (WDRA). e-NWR
will have no chances of any tempering, mutilation, fudging, loss or damage and also no possibility of any multiple
financing. e-NWR will also help to save expenditure in logistics as stocks could be traded through multiple buyers
without physical movement and can be even split for partial transfer or withdrawal.

186. Consider the following statistics.

1) India ranks first among the world’s milk producing Nations.


2) Most of the milk produced in India comes from cooperatives.

Which of the above is/are correct?


a) 1 only
b) 2 only
c) Both 1 and 2
d) None

Solution: a)
Justification: Statement 1: India continued to be the largest milk producing nation (since 1998) with milk
production of nearly 155 million tonnes during 2015-16. Milk production in the country is 165.4 million tonnes
during 2016-17 and 176.35 million tonnes (provisional) during 2017-18. India also has the largest bovine population
in the World. About 54% of the milk produced in the country is surplus for marketing in the domestic market, of
which only about 20% is procured/processed by the organized sector being equally, shared by Cooperatives and
Private dairy organizations.
Statement 2: The dairy cooperatives collectively procure around 16 million tonnes of milk. Government is mandated
to double milk producers’ income at farm level by providing rural milk producers with greater access to the
organized milk processing sector. Department of Animal Husbandry, Dairying & Fisheries has formulated a Draft
National Action Plan for Dairy Development recently.

187. With reference to High-yielding varieties (HYVs) crops, consider the following statements.

1) Such crops do not require any reliance on irrigation and fertilizers.


2) Yield can be high in these varieties only if they are allowed to mature later than normal crops.
Which of the above is/are correct?

a) 1 only
b) 2 only
c) Both 1 and 2
d) None

Solution: d)
Justification: High-yielding varieties (HYVs) of agricultural crops are usually characterized by a combination of
the following traits in contrast to the conventional varieties:

 higher crop yield per area (hectare)


 dwarfness
 improved response to fertilizers
 high reliance on irrigation and fertilizers - see intensive farming
 early maturation
So, both 1 and 2 are wrong.

Most important HYVs can be found among wheat, corn, soybean, rice, potato, and cotton. HYVs become popular
in the 1960s and play an important role in the green revolution.

188. Most of the cultivable land in India is owned by

a) Small and marginal farmers


b) Medium and Large farmers
c) Scheduled tribes
d) Land share is nearly equally divided among all.

Solution: b)
Justification: These are some of the important agricultural statistics:

The average holding in India is tiny and often split into scattered pieces. Five major categories of land holding
are: marginal (below 1 ha), small (1-2 ha), semi-medium (2-4 ha), medium (4-10 ha), large (10ha and above).
According to Census 2011, 67 per cent of holdings were classified as marginal (less than one hectare) and 18 per
cent were classified as small (one-two hectare). Nearly 80% of the farmers are small and marginal farmers, yet
they hold a tiny fraction of the total landholding.

189. With reference to agricultural statistics in India, Consider the following statements.
1) The production of food grains in India has steadily increased post green revolution since the 1960s.
2) India has presently turned out to host the largest land area certified as organic wild culture in the world.

Which of the above is/are correct?

a) 1 only
b) 2 only
c) Both 1 and 2
d) None
Solution: d)
Justification: Statement 1: Production of food grains has not steadily increases year on year instead there
were years (drought years) were the food grains production was less than the previous years like in 2002-03
where the production was around 150 million tones which was less by about 40 million tonnes from the
previous years

Statement 2: India has 4 million hectares of land certified as organic wild culture, which is third in the world
(after Finland and Zambia). We are yet to realize the full potential of organic culture. North-eastern states,
especially Sikkim, have covered it up well. In fact, Sikkim has become India's first fully organic state.

190. In many areas, the introduction of Green Revolution was associated with

1) Loss of soil fertility


2) Depletion of groundwater table
3) Widespread fungal infections in surrounding crops
4) Reduction of Seed replacement ratio to zero

Select the correct answer using the codes below.

a) 1, 2 and 4 only
b) 3 and 4 only
c) 1 and 2 only
d) 1,2,3 and 4

Solution: c)
Justification: Statement 1 and 2: This was due to increased use of chemical fertilizers. Also, continuous use of
groundwater for tube well irrigation has reduced the water-table below the ground.
Statement 3: An absurd statement. In fact, many HVVs helped reduce fungal infections.
Statement 4: Seed Replacement Ratio is a measure of how much of the total cropped area was sown with certified
seeds in comparison to farm saved seeds.

This has certainly increased post green revolution as farmers started using certified seeds more and more.

191. Consider the following statements. Chemical fertilizers:

1) Provide minerals that are retained in the soil forever


2) Help in the rapid growth of bacteria and other microorganisms to restore the natural fertility of soil

Which of the above is/are correct?

a) 1 only
b) 2 only
c) Both 1 and 2
d) None

Solution: d)
Justification: Statement 1: They provide minerals which dissolve in water and are immediately available to
plants. But these may not be retained in the soil for long. They may escape from the soil and pollute
groundwater, rivers and lakes.
Statement 2: Chemical fertilizers can also kill bacteria and other microorganisms in the soil. This means some
time after their use, the soil will be less fertile than ever before.
The continuous use of chemical fertilizers has led to degradation of soil health.

192. Consider the following statements.

1) Real agricultural growth in India was the highest in the decade of the first Green Revolution.
2) The volatility of agricultural growth in India has increased substantially over the last four decades.

Which of the above is/are correct?

a) 1 only
b) 2 only
c) Both 1 and 2
d) None

Solution: d)
Justification: Statement 1: Real agricultural growth since 1960 has averaged about 2.8 percent in India. The period
before the Green Revolution saw growth of less than 2 percent; the following period until 2004 yielded growth of 3
percent; in the period after the global agricultural commodity surge, growth increased to 3.6 percent (Figure 1).
China’s annual agricultural growth over the long run has exceeded that of India by a substantial 1.5 percentage
points on average.

Statement 2: The volatility of agricultural growth in India has declined substantially over time: from a standard
deviation of 6.3 percent between 1960 and 2004 to 2.9 percent since 2004. In particular, production of cereals has
become more robust to drought.
193. E-RaKAM is a first-of-its-kind initiative that

1) Leverages technology to connect farmers of the smallest villages to the biggest markets of the world
2) Establishes grading and standardization centers for agricultural produce in the remotest locations of the
country to promote food processing

Which of the above is/are correct?

a) 1 only
b) 2 only
c) Both 1 and 2
d) None

Solution: a)
Justification: E-RaKAM is a digital initiative bringing together the farmers, FPOs, PSUs, civil supplies and buyers
on a single platform to ease the selling and buying process of agricultural products. Under this initiative, e-
RaKAM centers are being developed in a phased manner throughout the country to facilitate farmers for
online sale of their produce. The portal is a joint initiative by state-run-auctioneer MSTC and Central
Warehousing Corporation arm CRWC.

194. The main functions of Small Farmers’ Agri-Business Consortium (SFAC) established by the Government
include

1) Helping formation and growth of Farmer Producer Organizations (FPOs)


2) Implementation of National Agriculture Market (e-NAM) Electronic Trading platform
3) Improving availability of working capital and development of business activities of Farmer Producer Companies
(FPCs)
Select the correct answer using the codes below.
a) 2 only
b) 1 and 3 only
c) 3 only
d) 1, 2 and 3

Solution: d)
Justification: It was established as a Society in 1994 to facilitate agri-business ventures by catalyzing private
investment through Venture Capital Assistance (VCA) Scheme in close association with financial institutions.
The role of State SFACs is to aggressively promote agribusiness project development in their respective States.

195. CHAMAN project of the Union Agriculture Ministry focuses on

1) Developing the horticulture sector using remote sensing technology and geo-informatics
2) Using optimal luminescent dating technique to date agricultural fossils for documentation and ex- situ
preservation

Which of the above is/are correct?

a) 1 only
b) 2 only
c) Both 1 and 2
d) None

Solution: a)
Justification: It is CHAMAN - Coordinated Horticulture Assessment and Management using geoinformatics. It is
being implemented by the Delhi-based Mahalanobis National Crop Forecast Centre. Under the project, the ministry
will use remote sensing and geoinformatics data to integrate information on weather, soil, land-use, and crop
mapping to prepare horticulture development plans. The idea is to use space technology to identify crops suitable
to different areas and raise production of horticulture crops. The project will help states develop horticulture
clusters and related infrastructure like cold chains. March 2018 is the deadline to completing the ambitious project.

196. With reference to the Fisheries sector, consider the following statements.

1) The sector contributes nearly 1% of the GDP of India.


2) It accounts for around 10% of the total export value of the country.

Select the correct answer using the codes below.

a) 1 only
b) 2 only
c) Both 1 and 2
d) None

Solution: c)
Justification: Statement 1: Constituting about 6.3% of the global fish production, the sector contributes to 1.1% of
the GDP and 5.15% of the agricultural GDP. The total fish production of 10.07 million metric tonnes presently has
nearly 65% contribution from the inland sector and nearly the same from culture fisheries.

Statement 2: Fish and fish products have presently emerged as the largest group in agricultural exports of India,
with over 10.5 lakh tonnes in terms of quantity and over Rs. 33,000 crores in value. This accounts for around 10% of
the total exports of the country and nearly 20% of the agricultural exports. More than 50 different types of fish and
shellfish products are exported to 75 countries around the world
With diverse resources ranging from deep seas to lakes in the mountains and more than 10% of the global
biodiversity in terms of fish and shellfish species, the country has shown continuous and sustained increments in
fish production since independence. With high growth rates, the different facets of marine fisheries, coastal
aquaculture, inland fisheries, freshwater aquaculture, coldwater fisheries to food, health, economy, exports,
employment and tourism of the country.

197. AGRI-UDAAN Food and Agribusiness Accelerator 2.0 programme will

1) Finance Gross Capital Formation (GCF) at subsidized rates for large land holdings
2) Setup Mega Food Parks in all smart villages of India
3) Mentor related start-ups and help them connect with potential investors

Select the correct answer using the codes below.

a) 1 only
b) 2 and 3 only
c) 3 only
d) 1, 2 and 3

Solution: c)
Justification: It intends to promote innovation and entrepreneurship in agriculture. It will mentor start-ups
and help them connect with potential investors and thus convert innovative ideas from India’s rural youth into
viable businesses. It is managed by ICAR-NAARM’s (Indian Council of Agricultural Research-National Academy
of Agricultural Research Management).Under the programme, start-ups will get incubation space to run their
businesses and have access to research laboratories and libraries.

198. Under the Pashu Sanjivni component of the National Mission on Bovine Productivity scheme cattle and
buffaloes will be identified using polyurethane tags. This scheme will help

1) Prevent cattle trafficking


2) Improve vaccination practices amongst cattle
3) Breeding processes to be improved

Select the correct answer using the codes below.

a) 1 and 2 only
b) 3 only
c) 1 only
d) 1, 2 and 3

Solution: d)
Justification: Statement 1: Under the scheme, 88 million milk producing animals out of 300 million cattle and
buffaloes are being identified using polyurethane tags with 12-digit unique identification (UID) number.
Polyurethane (PUR and PU) is a polymer composed of organic units joined by carbamate (urethane) links.
Statement 2 and 3: The polyurethane tags, which will last for a few year, will be affixed by the technicians
using a special applicator.

 Next, the technicians will, using a tablet, upload details of the cow including its vaccination and deworming
status into an online database.

 The purpose of the exercise, in the long run, is to improve vaccination practices, and ensure that scientific
interventions can be made to improve both breeding and milk production.
 This would help prevent illegal transportation of cattle and livestock to Bangladesh.

199. The concept of a ‘Seed Village’ revolves around

1) Cutting down the seed replacement rate


2) Replacing artificial exotic HYV seeds with existing local seed varieties
3) Engaging trained group of farmers in production of seeds

Select the correct answer using the codes below.

a) 1 and 2 only
b) 3 only
c) 2 only
d) 1, 2 and 3

Solution: b)
Justification: A seed village is one wherein trained group of fanners are involved in production 'of seeds of various
crops and cater to the needs of themselves, fellow fanners of the village and fanners of neighbouring villages in
appropriate time and at affordable cost is called "a seed village".

Major focus areas are:


 Organizing seed production in cluster (or) compact area
 Replacing existing local varieties with new high yielding varieties.
 Increasing the seed production
 To meet the local demand, timely supply and reasonable cost
 Self-sufficiency and self-reliance of the village
 Increasing the seed replacement rate
This allows seed to be available at the door steps of farms at an appropriate time and at an affordable cost. It also
facilitates fast spread of new cultivars of different kinds.

200. The Cabinet Committee on Economic Affairs has given its approval for the Umbrella Scheme, "Green
Revolution – Krishonnati Yojana" in agriculture sector beyond 12th Five Year Plan for the period from 2017-18
to 2019-20. The Schemes that are part of the Umbrella Schemes are?

1) Integrated Scheme on Agricultural Marketing (ISAM)


2) National Food Security Mission (NFSM)
3) Mission for Integrated Development of Horticulture (MIDH)

Select the correct answer using the codes below.

a) 1 only
b) 1 and 2 only
c) 2 and 3 only
d) 1, 2 and 3

Solution: d)
Justification: The Umbrella scheme comprises of 11 Schemes/Missions. These schemes look to develop the
agriculture and allied sector in a holistic and scientific manner to increase the income of farmers by enhancing
production, productivity and better returns on produce.

Statement 1: Integrated Scheme on Agricultural Marketing (ISAM) with a total central share of 3863.93 crore. ISAM
aims to develop agricultural marketing infrastructure; to promote innovative and latest technologies and
competitive alternatives in agriculture marketing infrastructure; to provide infrastructure facilities for grading,
standardization and quality certification of agricultural produce; to establish a nationwide marketing information
network; to integrate markets through a common online market platform to facilitate pan-India trade in agricultural
commodities, etc.

Statement 2: National Food Security Mission (NFSM), including National Mission on Oil Seeds and Oil Palm
(NMOOP), with a total central share of Rs.6893.38 crore. It aims to increase production of rice, wheat, pulses,
coarse cereals and commercial crops, through area expansion and productivity enhancement in a suitable manner in
the identified districts of the country, restoring soil fertility and productivity at the individual farm level and
enhancing farm level economy. It further aims to augment the availability of vegetable oils and to reduce the import
of edible oils.

Statement 3: Mission for Integrated Development of Horticulture (MIDH) with a total central share of Rs. 7533.04
crore, MIDH aims to promote holistic growth of horticulture sector; to enhance horticulture production, improve
nutritional security and income support to farm Households.

Other schemes include, inter alia:

National Mission for Sustainable Agriculture (NMSA) with a total central share of Rs.3980.82 crore. NMSA aims at
promoting sustainable agriculture practices best suitable to the specific agro-ecology focusing on integrated
farming, appropriate soil health management and synergizing resource conservation technology.

Submission on Agriculture Extension (SMAE) with a total central share of Rs.2961.26 crore. SMAE aims to strengthen
the ongoing extension mechanism of State Governments, local bodies etc., achieving food and nutritional security
and socio-economic empowerment of farmers, to institutionalize programme planning and implementation
mechanism, to forge effective linkages and synergy amongst various stake-holders, to support HRD interventions, to
promote pervasive and innovative use of electronic / print media, inter-personal communication and ICT tools, etc.

201. Rashtriya Krishi Vikas Yojana – Remunerative Approaches for Agriculture and Allied sector Rejuvenation
(RKVY-RAFTAAR) focuses on

1) Promoting agribusiness entrepreneurship through financial support


2) Skill development of farmers so as to make farming a remunerative economic activity

Which of the above is/are correct?

a) 1 only
b) 2 only
c) Both 1 and 2
d) None

Solution: c)
Justification: The CCEA cleared the continuation of RKVY as RKVY-RAFTAAR for three years till 2019-20.

Aims:
 It is aimed at making farming a remunerative economic activity through strengthening the farmer’s effort, risk
mitigation and promoting agribusiness entrepreneurship
 The scheme will incentivise states in enhancing more allocation to agriculture and allied sectors.
 This will also strengthen farmer’s efforts through creation of agriculture infrastructure that help in supply of quality
inputs and market facilities.
 This will further promote agri-entrepreneurship and support business models that maximise returns to farmers.

202. The National Seeds Policy, 2002, envisages timely availability of quality seeds to achieve which it suggests

1) Compulsory registration of seeds


2) Creation of Seed Banks
3) Establishment of a license regime for all seed dealers
4) Banning import of seeds
5) Creating a national seed grid

Select the correct answer using the codes below.

a) 2, 3 and 4 only
b) 1, 2 and 5 only
c) 1, 4 and 5 only
d) All of the above

Solution: b)
Justification: The National Seeds Policy, 2002, envisages timely availability of quality seeds, compulsory
registration of seeds, quality assurance, promotion of the seeds industry, abolition of licenses for seed dealers,
import of best quality seeds and creation of Seed Banks and National Seeds Grid. The policy seeks to
encourage investment in research and development to ensure availability of high yielding varieties of seeds.
National Seeds Grid was set up by linking all seeds producing agencies the national and particularly to ensure
that seeds are available on demand by States or farmers. The Grid would provide district wise information
regarding the requirement, production and distribution of seeds through a computerized network.

203. What is FarmerZone, that was recently seen in news?

a) It is an open-source data platform for smart agriculture.


b) It is an Agricultural mandi which will not be regulated by APMC Acts.
c) It is a food processing zone that will be heavily subsidized by the state.
d) It is a farmer distress service to counsel farmers against suicides.

Solution: a)
Justification: FarmerZone has been envisioned by the DBT and aligns with our PM’s call for effective decision-
making in agriculture that integrates science, technology, innovation and the farm ecosystem.

 It is a collective open-source data platform for smart agriculture which will use biological research and data to
improve the lives of small and marginal farmers.

 It is envisaged that “FarmerZone” will help cater to all needs of the farmer, from dealing with climate change,
weather predictions and soil, water, and seed requirements to providing market intelligence.

The FarmerZone platform will connect farmers and scientists, government officials, thought leaders in
agriculture, economists and representatives from global companies who work in the big-data and e-commerce
space to bring about technology-based localized agri-solutions. The platform will work on getting relevant
quality data related to agriculture into the cloud, develop sentinel sites to help link with farmers and evolve
PPP based enterprises for data delivery.

204. Consider the following statements about Urea Subsidy in India.

1) It is a part of Central Sector Scheme (CSS).


2) It is financed partly by the Centre and partly by the States.
3) It includes Imported Urea subsidy to bridge domestic supply-demand gap.

Select the correct answer using the codes below.

a) 1 only
b) 2 and 3 only
c) 1 and 3 only
d) 1, 2 and 3

Solution: c)
Justification: Urea Subsidy is part of Central Sector Scheme (CSS) of Department and is wholly financed by Central
Government of India through budgetary support. It also includes Imported Urea subsidy which is directed towards
import to bridge gap between assessed demand and indigenous production of urea in the country. It also includes
freight subsidy for movement of urea across the country. The Urea subsidy scheme was approved for continuation
recently. The continuation of the urea subsidy scheme will ensure timely payment of subsidy to urea manufacturers
resulting in timely availability of urea to farmers at statutory controlled price. Implementation of DBT in Fertilizer
Sector will reduce diversion of fertilizers to non- agricultural use and plug leakages. Department of Fertilizers is in
process to roll out DBT in fertilizer sector nationwide.

205. Which of the following is/are ‘state intervention(s)’ in the field of agricultural marketing?

1) Provision of physical Infrastructure


2) Cooperative marketing for realizing fair prices for farmers’ products
3) Assurance of minimum support prices (MSP)
4) Maintenance of food buffer stocks by Food Corporation of India (FCI)

Select the correct answer using the codes below.

a) 2 and 4 only
b) 3 and 4 only
c) 1 and 2 only
d) 1, 2, 3 and 4

Solution: d)
Justification: Statement 1: Government has been providing physical infrastructure in form of roads, railways,
warehouses, godowns, cold storages and processing units to improve agricultural marketing. So, 1 is correct.
Statement 2: The government has been promoting cooperative marketing, for e.g. the year 2014 was declared by
the year of Farmer Produce Organizations (FPOs) by the Planning Commission.
Statement 3: MSP incentivizes the farmers to adjust their crop output and reduces chances of shortage of food
grains in the market. So, 3 is also correct.
Statement 4: Offloading food buffer stocks helps n fighting short-term supply shortages of food grains in markets
and tackling inflation. So, 4 is correct.
206. The agriculture sector in India is witnessing many structural changes. As per the estimates of NSSO, which of
these presently contributes to most of the farmer’s income?

a) Non-farm employment
b) Cultivation
c) Livestock
d) Wages received

Solution: b)
Justification: The structural transformation is manifested in the farm incomes of the households. The decrease in
share of crop sector in the total gross value added of the agriculture and allied sector has impacted the sources of
incomes of the farm households. As can be seen from Figure 2 in 2002-03 the share of livestock in total farm
incomes was just 4 per cent which increased to 13 per cent by 2012-13.

These structural changes necessitate re-orientation in policies towards this sector in terms of strengthening the
agricultural value chain by focusing on allied activities like dairying and livestock development along with gender-
specific interventions

207. Consider the following about Deep sea fishing.


Assertion (A): Undertaking deep sea fishing in EEZ and high seas is prohibited under International Maritime
Convention (IMO).
Reason (R): Bottom trawling is an ecologically destructive practice which leads to exhaustion of maritime species
resources.

In the context of the above, which of these is correct?

a) A is correct, and R is an appropriate explanation of A.


b) A is correct, but R is not an appropriate explanation of A.
c) A is correct, but R is incorrect.
d) A is incorrect, but R is correct.
Solution: d)
Justification: Deep sea fishing has been an integral part of the country’s Blue Revolution vision to exploit fishing
resources to the maximum within the 200 nautical mile exclusive economic zone (EEZ). However, deep sea fishing is
not to be equated with Bottom trawling. Bottom trawling involves trawlers dragging weighted nets along the sea-
floor, causing great depletion of aquatic resources. Bottom trawling captures juvenile fish, thus exhausting the
ocean’s resources and affecting marine conservation efforts. This practice was started by Tamil Nadu fishermen in
Palk Bay and actively pursued at the peak of the civil war in Sri Lanka. The Palk Bay fishing conflict has figured
prominently in high-level meetings between India and Sri Lanka.

208. Consider the following statements about National Agricultural Policy, 2000

1) This was the first ever National Agricultural Policy announced in India.
2) The Policy aims at a growth rate in excess of four per cent per annum in the agriculture sector.
3) It explicitly prohibits the use of contract farming and land leasing in India.

Select the correct answer using the codes below.

a) 1 only
b) 2 and 3 only
c) 1 and 2 only
d) 1, 2 and 3

Solution: d)
Justification: Statement 2: It seeks to realize the growth potential of Indian agriculture, strengthen rural
infrastructure for faster agricultural development, promote value addition, accelerate the growth of agro business,
and create employment in rural areas. This is in order to secure a fair standard of living for farmers and
agricultural workers and their families, discourage migration to urban areas and face the challenges arising out of
economic liberalization and globalization.

Statement 3: As per the policy, contract farming and land leasing is done to promote private sector participation, to
allow accelerated technology transfer, capital inflow, assured markets for crop production, especially of oilseeds,
cotton and horticulture crops.

MSP\

209. What factors are taken in account by Commission on Agricultural Prices (CACP) when calculating the Minimum
Support Price (MSP) in India considering its inputs and effects?

1) Cost of crop production and changes in input prices


2) Effect on industrial cost structure
3) Effect on cost of living and general price level
4) Effect on issue prices and implications for subsidy
5) Inter-crop price parity

Select the correct answer using the codes below.

a) 1, 2 and 3 only
b) 1, 4 and 5 only
c) 2, 3 and 4 only
d) All of the above
Solution: d)

Justification: The Commission on Agricultural Prices (CACP) makes use of both micro-level data and aggregates at
the level of district, state and the country. The information/data used by the Commission, inter-alia include the
following Cost of cultivation per hectare and structure of costs in various regions of the country and changes there
in; Cost of production per quintal in various regions of the country and changes therein; Prices of various inputs and
changes therein; Market prices of products and changes therein; Prices of commodities sold by the farmers and of
those purchased by them and changes therein; Supply related information - area, yield and production, imports,
exports and domestic availability and stocks with the Government/public agencies or industry; Demand related
information - total and per capita consumption, trends and capacity of the processing industry;

Also, the following is taken into account:

Prices in the international market and changes therein, demand and supply situation in the world market; Prices of
the derivatives of the farm products such as sugar, jaggery, jute goods, edible/non- edible oils and cotton yarn and
changes therein; Cost of processing of agricultural products and changes therein; Cost of marketing - storage,
transportation, processing, marketing services, taxes/fees and margins retained by market functionaries; and
Macro-economic variables such as general level of prices, consumer price indices and those reflecting monetary and
fiscal factors.

210. Consider the following statements about the Minimum Support Price (MSP).

1) It is directed at providing insurance to agricultural producers against any sharp fall in farm prices.
2) State governments cannot offer MSP.
3) MSP operations are restricted to food crops only.

Select the correct answer using the codes below.

a) 1 and 2 only
b) 1 only
c) 2 and 3 only
d) 1 and 3 only

Solution: b)
Justification: Statement 2: While the Centre supports the procurement of wheat and rice, if states want to ensure
their farmers can access minimum support prices for other crops, they need to use their own funds. They can also
declare a bonus above the central minimum support price to encourage farmers to grow more of a certain crop.
Maharashtra, for example, offered a bonus of Rs 500 per quintal for tur dal, above the declared price of Rs 5,050 in
April 2017. In fact, Centre and states are even cooperating to make MSP effective

Statement 3: It covers other items as well, for e.g. minor forest produce such as honey.

211. The Minimum Support Price (MSP) is declared by the government every year

a) Before the sowing season


b) Every week at all major mandis
c) At the end of every month
d) After the sowing season

Solution: a)
Justification: MSP is like a price signal to provide incentives to the farmers for raising the production of particular
crops whose MSP has been set higher. MSP is decided by Cabinet Committee on Economic Affairs (CCEA). However,
a buffer stock is created by the government to distribute foodgrains in the deficit areas and among the poorer strata
of society at a price lower than the market price also known as Issue Price.

212. With reference to the Commission on Agricultural Costs and Prices (CACP), consider the following statements.

1) CACP submits its recommendations to the government in the form of Price Policy Reports every year.
2) The Commission makes visits to states for on-the-spot assessment of the various constraints that farmers face in
marketing their produce.
3) CACP was established autonomously and functions independently from the government.

Select the correct answer using the codes below.

a) 1 and 2 only
b) 2 and 3 only
c) 1 and 3 only
d) 1, 2 and 3

Solution: a)
Justification: Statement 3: CACP is an attached office of the Ministry of Agriculture and Farmers Welfare,
Government of India. It came into existence in January 1965.

Statement 1 and 2: It is mandated to recommend minimum support prices (MSPs) separately for five groups of
commodities namely Kharif crops, Rabi crops, Sugarcane, Raw Jute and Copra.

This to incentivize the cultivators to adopt modern technology, and raise productivity and overall grain production in
line with the emerging demand patterns in the country. However, the Cabinet Committee on Economic Affairs
(CCEA) of the Union government takes a final decision on the level of MSPs and other recommendations made by
CACP. Currently, the Commission comprises a Chairman, Member Secretary, one Member (Official) and two
Members (Non-Official). The non-official members are representatives of the farming community and usually have
an active association with the farming community. The Commission also makes visits to states for on-the-spot
assessment of the various constraints that farmers face in marketing their produce, or even raising the productivity
levels of their crops. Based on all these inputs, the Commission then finalizes its recommendations/reports, which
are then submitted to the government. The government, in turn, circulates the CACP reports to state governments
and concerned central Ministries for their comments.

213. With reference to the North Eastern Regional Agricultural Marketing Corporation Limited (NERAMAC), consider
the following:

1) It facilitates processing of horticulture produce.


2) It assists Self-Help Groups to serve as an aggregator and provide marketing support to them.

Which of the above is/are correct?

a) 1 only
b) 2 only
c) Both 1 and 2
d) None

Solution: c)
Justification: Statement 1: NERAMAC, inter alia, facilitates processing of horticulture produce; assists processing
units to market their products; and helps in developing linkages of farmers with the market.

Statement 2: NERAMAC in pursuance of the objectives mentioned above, has provided support to large number of
farmers, particularly those growing pineapple, ginger and large cardamom. It has also undertaken marketing of
processed items sourced from local entrepreneurs.

214. Prime Minister Krishi Sampada Yojana (PMKSY) focusses on which of the key sectors of agriculture and allied
sectors?

a) Food processing
b) Fertilizer production
c) Development of Irrigation
d) High quality seed development

Solution: a)
Justification: PMKSY is an umbrella scheme incorporating ongoing schemes of the Ministry like Mega Food Parks,
Integrated Cold Chain and Value Addition Infrastructure, Food Safety and Quality Assurance Infrastructure, etc. It
also covers new schemes like Infrastructure for Agro-processing Clusters, Creation of Backward and Forward
Linkages, Creation / Expansion of Food Processing & Preservation Capacities. The objective of PMKSY is to
supplement agriculture, modernize processing and decrease Agri- Waste.

215. Which of these are important government initiatives/measures to boost food processing sector in India?

1) Food and agro-based processing units and cold chain infrastructure have been brought under the ambit of Priority
Sector Lending (PSL)
2) 100% FDI in trading, including e-commerce, of food products manufactured in India
3) Establishment of a Special Fund in NABARD to make available affordable credit at concessional rate of interest to
designated food parks

Select the correct answer using the codes below.


a) 1 only
b) 2 and 3 only
c) 2 only
d) 1, 2 and 3

Solution: d)
Justification: Statement 1: Bringing them under Priority Sector Lending (PSL) will help provide additional credit for
food processing activities and infrastructure thereby, boosting food processing, reducing wastage, create
employment and increasing farmers' income.

Statement 2: To provide impetus to investment in food processing and retail sector, govt. has allowed cent per cent
FDI in trading including through e-commerce, in respect of food products manufactured and / or produced in India.
This will benefit farmers immensely and will create back - end infrastructure and significant employment
opportunities.

Statement 3: A Fund of Rs. 2000 crore in NABARD has been established to make available affordable credit at
concessional rate of interest to designated food parks and agro processing units in the designated food parks.
AGRICULTURE CREDIT

EXTENSION

216. Consider the following about Krishi Vigyaan Kendras.

1) KVKs act as knowledge and resource center in the field of agriculture in the district.
2) They are financed on a shared funding model between Centre and States.
3) KVKs produce quality technological products and make it available to farmers.
4) They identify and document selected farm innovations.
5) They come under the ambit of the Indian Council of Agricultural Research (ICAR).

Select the correct answer using the codes below.

a) 1, 3, 4 and 5 only
b) 1, 2 and 5 only
c) 1, 2, 3 and 4 only
d) 2, 3, 4 and 5 only

Solution: a)
Justification: Statement 1: KVKs are supposed to act as knowledge and resource center in the field of agriculture in
the district and would build models of technology uptake and farmers' empowerment leading to support
Government of India’s initiative of doubling farmers' income.

Statement 2 and 5: The KVK scheme is functioning on 100% Central funding pattern under the ambit of the Indian
Council of Agricultural Research (ICAR), Department of Agricultural Research and Education (DARE). Till 31.3.2017,
669 KVKs have been established, which are functioning as district level Farm Science Centers, the mandate being,
Technology Assessment and Demonstration for its Application and Capacity Development.

Statement 3 and 5: The activities include on-farm testing of technologies and demonstrations, capacity
development of farmers and extension personnel, functioning as a knowledge and resource center of agricultural
technologies, providing farm advisories using ICT and other media means on varied subjects of interest to farmers.

217. Consider the following statements.

1) Agri-Clinics are envisaged to provide expert advice and services to farmers on various aspects to enhance
productivity of crops or animals and increase the incomes of farmers.
2) Agri-Business Centers are commercial units of agri-ventures established by trained agriculture professionals.

Which of the above is/are correct?

a) 1 only
b) 2 only
c) Both 1 and 2
d) None

Solution: c)
Justification: Agriclinic and Agribusiness Centers Scheme aims to promote the establishment of Agri-Clinics and
Agri-Business Centers (ACABC) all over the country.
Statement 1: Agri-clinics provide support in the following areas:

 Soil health
 Cropping practices
 Plant protection
 Crop insurance
 Post-harvest technology
 Clinical services for animals, feed and fodder management
 Prices of various crops in the market, etc.

Statement 2: Such ventures may include maintenance and custom hiring of farm equipment, sale of inputs and
other services in agriculture and allied areas, including post-harvest management and market linkages for income
generation and entrepreneurship development. The scheme by NABARD concerning these centers covers full
financial support for training and handholding, provision of loan and credit-linked back ended composite subsidy.

218. The government has taken several initiatives in recent times to bring useful information through the help of
modern information technology to the farming community in the shortest possible time. These include

1) Farmers Portal
2) Kisan Call Center
3) Fasal Bima Portal
4) Bol Kisan

Select the correct answer using the codes below.

a) 1 and 3 only
b) 1, 2 and 3 only
c) 2 and 4 only
d) 1, 2, 3 and 4 only

Solution: b)
Justification: Others are Rice Export App, Pusa Krishi App, Agro Collect- Krishi Gyan App, e-NAM Portal, etc.
Statement 1: Once in the Farmers’ Portal, a farmer will be able to get all relevant information on specific subjects
around his village/block /district or state. This information will be delivered in the form of text, SMS, email and
audio/video in the language he or she understands. Farmers will also be able to ask specific queries as well as give
valuable feedback through the Feedback module specially developed for the purpose.

219. Ministry of Agriculture & Farmers Welfare had launched the KISAN Project. The project uses which of these
technologies to improve yield estimation in farms?

a) Herd sampling
b) GIS and GPS
c) Centralized Soil Health Card inputs
d) Ultraviolet Night Beaming

Solution: b)
Justification: The project envisages use of Space Technology and geoinformatics (GIS, GPS and Smartphone)
technology along with high resolution data from UAV/Drone based imaging for improvement in yield estimation and
better planning of Crop Cutting Experiments (CCEs), needed for crop insurance programme. A green cover can be
monitored using GIS based maps and based on these findings appropriate policy responses can be designed for
specific areas.

220. World Bank aided SAMETIs in India are concerned with

a) Capacity building in agricultural management and extension training


b) Financing Farmer Producer Organization (FPOs)
c) Providing irrigation and fertilizer management facilities
d) Sponsoring Krishi Vigyan Kendras

Solution: a)
Justification: In order to develop a competent structure for providing demand driven consultancy services and
capacity building of the stake holders in the agricultural extension management, one SAMETI in each of the seven
States has been established. SAMETIs are to provide HRD/Training support to organize programmes for capacity
building in agricultural management and extension training for the stake holders of NATP. SAMETIs are declared
autonomous with greater flexibilities in structure and functioning and responsible for organizing need-based
training programmes for project functionaries from different line departments as well as the farming community.

Mandate of the SAMETIs

 To function as agricultural management and training institute at State level.


 To provide consultancy in areas like project planning, appraisal and in agricultural management and extension
training.
 To develop and promote the application of management tools for effective agricultural extension services.
 To organize need-based training programmes for middle level and grass-root level agricultural extension
functionaries.
 To conduct problem oriented studies in agricultural management, communication, participatory methodologies,
post-harvest technology and marketing.
 To take up HRD/training activities including preparation of SREP for ATMAs.

MARKETING AND APMC ACT

221. AGMARKNET is a/an

a) Commodity certification platform


b) Grading and standardization facility for primary food processing
c) Online portal for crop price information
d) Agriculture sustainability indicator

Solution: c)
Justification: Agricultural Marketing Information Network (AGMARKNET) was launched in 2000 by the Union
Ministry of Agriculture. The Directorate of Marketing and Inspection (DMI), under the Ministry, links around 7,000
agricultural wholesale markets in India with the State Agricultural Marketing Boards and Directorates for effective
information exchange. This e-governance portal AGMARKNET, implemented by National Informatics Centre (NIC),
facilitates generation and transmission of prices, commodity arrival information from agricultural produce markets,
and web-based dissemination to producers, consumers, traders, and policy makers transparently and quickly. This
G2C e-governance portal caters to the needs of various stakeholders such as farmers, industry, policy makers and
academic institutions by providing agricultural marketing related information from a single window. The portal has
helped to reach farmers who do not have sufficient resources to get adequate market information. It facilitates
web- based information flow, of the daily arrivals and prices of commodities in the agricultural produce markets
spread across the country.

222. What is/are the objective(s) of National Agriculture Market (NAM)?

1) To serve as platform for transparent sale transactions and price discovery initially in regulated markets
2) Ensure one license for a trader valid across all markets in the State.
3) Harmonisation of quality standards of agricultural produce and provision for assaying
4) Provision of Soil Testing Laboratories in/or near the selected mandis

Select the correct answer using the codes below.

a) 1 only
b) 2 and 4 only
c) 1 and 3 only
d) All of the above

Solution: d)
Justification: NAM is a pan-India electronic trading portal which networks the existing APMC mandis to create a
unified national market for agricultural commodities. The NAM Portal provides a single window service for all APMC
related information and services. Other objectives are:
 Liberal licensing of traders / buyers and commission agents by State authorities without any pre-condition of
physical presence or possession of shop /premises in the market yard.
 One license for a trader valid across all markets in the State.
 Harmonisation of quality standards of agricultural produce and provision for assaying (quality testing) infrastructure
in every market to enable informed bidding by buyers. Common tradable parameters have so far been developed
for 69 commodities.
 Single point levy of market fees, i.e. on the first wholesale purchase from the farmer.
 Provision of Soil Testing Laboratories in/ or near the selected mandi to facilitate visiting farmers to access this
facility in the mandi itself.

223. The model Agriculture Produce and Livestock Marketing (Promotion and Facilitation) Act, 2017, has which of
the following provisions?

1) Establish a pan-nation deregulated and de-licensed wholesale agri-market


2) Devolving all regulatory powers of agricultural marketing in the state to the Local self-governing bodies
3) Allowing private players and traders to set up local wholesale markets

Select the correct answer using the codes below.

a) 1 and 3 only
b) 2 and 3 only
c) 1 and 2 only
d) 3 only

Solution: d)
Justification: Traditionally the agricultural produce was sold by the farmers in the village or nearby places. However,
the system was characterized by malpractices by middlemen, inadequate price to the farmers etc. So the State’s
enacted their APMC acts to set up regulated agricultural markets so as to ensure that farmers get a fair price for
their produce. But even this arrangement led to a lot of problems.

The model Agriculture Produce and Livestock Marketing (Promotion and Facilitation) Act, 2017 was
presented in April 2017 and seeks to replace APMC act 2003. It is seen as a major agriculture marketing reform to
help farmers directly connect with the different buyers and enable them to discover optimum price for their
commodities. The purpose is to create a single agriculture market with a single license wherein agriculture produce
as well as livestock could be traded. Some of the important provisions:

 The new model law seeks to establish a regulated wholesale agri-market at a distance of every 80 km.
 To enable this, it has proposed to issue licenses to new private players and traders who wish to set up a wholesale
market. Even private market yards, warehouses and cold storages will be allowed to act as regulated markets.
 Unlike the current system, now only by paying unified single fees, farmer/trader will be able to transact in all such
regulated agri-markets within the state. There will be no separate fees for individual markets.
 It caps market fee (including developmental and other charges) at not more than 1 per cent for fruit and vegetables,
and two per cent for food grain. It caps commission agents’ fee at not more than two per cent for non-perishables
and four per cent for perishables.
 Besides, it stipulates a single license for trading within the state and at the national level.
 All regulatory powers will lie with the office of the director of agricultural marketing in the state, who will also issue
licenses to traders and new private players. As of now this power lies with the mandis managed by the Board of
directors.
 It also has the provision for promoting online or spot (e-national agriculture market) agriculture market platforms

224. The Prime Minister recently signaled that India is ready for Contract Farming. What is/are the advantages of
Contract farming?

1) It makes small scale farming competitive.


2) Farming contracts provide complete legal protection in India.
3) It assures the farmer that that all farm produce will be procured at Minimum Support Price (MSP) by government
agencies.
4) It can open up new markets which would otherwise be unavailable to small farmers.

Select the correct answer using the codes below.

a) 1, 2 and 4 only
b) 4 only
c) 1, 2, 3, 4
d) 1 and 4 only

Solution: d)
Justification: Contract farming can be defined as agricultural production carried out according to an agreement
between a buyer and farmers, which establishes conditions for the production and marketing of a farm product or
products. Typically, the farmer agrees to provide agreed quantities of a specific agricultural product. These should
meet the quality standards of the purchaser and be supplied at the time determined by the purchaser. In turn, the
buyer commits to purchase the product and, in some cases, to support production through, for example, the supply
of farm inputs, land preparation and the provision of technical advice.

Statement 1: It makes small scale farming competitive - small farmers can access technology, credit, marketing
channels and information while lowering transaction costs. It gives assured market for their produce at their
doorsteps, reducing marketing and transaction costs. It reduces the risk of production, price and marketing costs.
Statement 2: Contracting agreements are often verbal or informal in nature, and even written contracts often do
not provide the legal protection in India that may be observed in other countries.
There is no dedicated law or regulation in India concerning contract farming.
Lack of enforceability of contractual provisions can result in breach of contracts by either party.
Statement 3: Contract farming will make sure that the contracting party will purchase all the produce at a
predetermined price. The MSP operations are organized independently of such contracts, and state agencies do not
necessarily purchase all the farm output at MSP in India.
Statement 4: Contract farming can open up new markets which would otherwise be unavailable to small farmers
because the corporate or contracting party might otherwise not purchase from those farms. It also ensures higher
production of better quality, financial support in cash and /or kind and technical guidance to the farmers. In case of
agri-processing level, it ensures consistent supply of agricultural produce with quality, at right time and lesser cost.

225. Private Entrepreneurs Guarantee (PEG) Scheme concerns

a) Manufacturing MSMEs
b) Warehousing capacity for food storage
c) Venture Capital Funds that have been established by the Government
d) Guaranteed Land acquisition for mega projects

Solution: b)
Justification: Under PEG, storage capacity is created by private parties, Central Warehousing Corporation (CWC) and
State Agencies for guaranteed hiring by FCI. This is because the state alone cannot provide such facilities at all
locations. It makes economic sense for private players to finance the construction of these warehouses and then
lease it out for rent.

IMPORTS AND EXPORTS

FOOD –DISTRIBUTION, SAFETY AND STANDARDS

226. A series of sector specific Food Safety Management System (FSMS) Guidance Documents have been developed
for the food businesses by the FSSAI. Consider the following about them.

1) They intend to provide implementation guidance to food businesses involved in manufacturing, packing, storage
and transport of food to ensure critical food hygiene and safety.
2) These are sub-rules and regulations issued under the Food Safety & Standards (Licensing & Registration of Food
Businesses) Regulation, 2011.
3) They are implemented by the Food Safety Knowledge Assimilation Network (FSKAN).

Select the correct answer using the codes below.

a) 1 only
b) 1 and 2 only
c) 1, 2 and 3
d) 2 and 3 only

Solution: a)
Justification: Statement 2: The documents are recommendatory in nature. They are not rules or regulations.

Statement 1: They provide implementation guidance to food businesses (especially the small and medium
businesses) involved in manufacturing, packing, storage and transportation to ensure that critical food safety
related aspects are addressed throughout the supply chain. These documents are based on Schedule 4 of Food
Safety & Standards (Licensing & Registration of Food Businesses) Regulation, 2011. They also provide the basic
knowledge and criteria for implementation of Hazard Analysis and Critical Control Point (HACCP) system by the food
businesses. Sample HACCP Plans have been taken from some established practicing industries. These plans could be
used as reference by the industry and modified or altered based on their operations.

227. Consider the following statements. ‘FoSCoRIS’, recently seen in news, is a/an

1) Online grievance portal to report food poisoning cases to the district administration
2) Verification and compliance system of food safety and hygiene standards by food businesses as per government
norms
3) Crisis protocol to be followed by exporters and importers in the event of food crises
4) Set of norms to regulate nutrition standards in school going girl children under the Mid-Day Meal Scheme

Select the correct answer using the codes below.

a) 1 and 2 only
b) 1, 3 and 4 only
c) 2 only
d) 2 and 3 only

Solution: c)
Justification: To bring in transparency in food safety inspection and sampling, FSSAI has put in place a nationwide
online platform called ‘FoSCoRIS’. ‘FoSCoRIS’ will help verify compliance of food safety and hygiene standards by
food businesses as per the government norms. The new system will bring together all key stakeholders — food
businesses, food safety officers (FSOs), designated officers, state food safety commissioners — on a nation-wide IT
platform and data related to inspection, sampling and test result data will be shared seamlessly by all the officials.
This system will give a clear picture to the FSSAI on the real-time basis and helps eliminate any discrepancy, hence
inspection is accountable. The system will ease out the process of sample collection, make it transparent and
traceable and controls the quality of compliances.

228. The Scheme for Creation of Backward and Forward Linkages under the Ministry of Food Processing Industries
(MoFPI)

1) Provides for financial assistance for setting up of primary processing centers at farm gate and modern retail outlets
2) Is applicable to both perishable and non-perishable horticulture and non-horticulture produce
3) Enables linking of farmers to processors and the market for ensuring remunerative prices for agricultural produce
4) Is implemented by organizations such as PSUs as well as NGOs

Select the correct answer using the codes below.

a) 1, 2 and 3 only
b) 1, 3 and 4 only
c) 2 and 3 only
d) 1, 2 and 4 only

Solution: b)
Justification: The objective of the scheme is to provide effective and seamless backward and forward integration for
processed food industry by plugging the gaps in supply chain in terms of availability of raw material and linkages
with the market. Under the scheme, financial assistance is provided for setting up of primary processing centers/
collection centers at farm gate and modern retail outlets at the front end along with connectivity through insulated/
refrigerated transport.
S2 and 3: The Scheme is applicable to perishable horticulture and non-horticulture produce such as fruits,
vegetables, dairy products, meat, poultry, fish, Ready to Cook Food Products, Honey, Coconut, Spices, Mushroom,
Retails Shops for Perishable Food Products etc. The Scheme would enable linking of farmers to processors and the
market for ensuring remunerative prices for agri produce based on such an infrastructure.
S4: The scheme is implemented by agencies/ organizations such as Govt./ PSUs/ Joint Ventures/ NGOs/
Cooperatives/ SHGs / FPOs / Private Sector / individuals etc. The Ministry has engaged Technical Agencies (TAs) for
assisting farmer/ producer groups including Farmer Producer Companies, Farmer Producer Organization, Self Help
Groups to facilitate their participation under the Scheme. The TAs are responsible for preparation of Business Plan,
Detail Project Report, Capacity Building, Trainee and other related support services.

229. India’s ambitious Zero Hunger Programme launched on occasion of World Food Day (October 16).
Consider the following about it.

1) This is a dedicated farm-based programme in sync with Sustainable Development Goals (SDGs) to end hunger by
2030.
2) The programme consists of organizations of farming system for nutrition and setting up genetic gardens for bio-
fortified plants.
3) The Programme is initiated by the Ministry of Food Processing Industries (MoFPI).

Select the correct answer using the codes below.

a) 1 and 2 only
b) 1 and 3 only
c) 2 and 3 only
d) 1, 2 and 3

Solution: a)
Justification: This ambitious Zero Hunger Programme focuses on agriculture, nutrition and health. The Programme
is initiated by the Indian Council of Agricultural Research (ICAR) in association with the Indian Council of Medical
Research (ICMR), the M S Swaminathan Research Foundation and the Biotechnology Industry Research Assistance
Council (BIRAC). The programme consists of organizations of farming system for nutrition, setting up genetic
gardens for bio-fortified plants and initiation of ‘Zero Hunger’ training. The Programme initially was launched in
three districts – Gorakhpur (Uttar Pradesh), Koraput (Odisha) and Thane (Maharashtra). More districts will be
covered in future. These three districts will act as model of integrated approach to deal with hunger and
malnutrition by adopting suitable agricultural or horticultural practices

230. The Food Standards Programme is an initiative of

a) World Health Organization (WHO) and FAO (Food and Agricultural Organization) jointly
b) United Nations Development Programme (UNDP)
c) World Economic Forum (WEP)
d) United Nations Children's Fund (UNICEF)

Solution: a)
Justification: The Codex Alimentarius Commission is the central part of the Food Standards Programme. The Codex
alimentarius is a collection of internationally adopted food standards presented in a uniform manner. These food
standards aim at protecting consumer’s health and ensuring fair practices in the food trade.
231. Food Safety & Standards Authority of India (FSSAI) under its mandate as specified in Section 16, 3(e) of the FSS
Act, established the Food Safety Knowledge Assimilation Network (FSKAN). Concerning food safety, FSKAN intends
to

1) Build a network of scientific organizations


2) Organize and lead World Food India (WFI) festivals in India and abroad
3) Certify the food quality of export vendors

Select the correct answer using the codes below.

a) 1 only
b) 2 and 3 only
c) 1 and 3 only
d) 1 and 2 only

Solution: a)
Justification: To achieve food safety, strong, functional links must be built between the regulators, experts
(Institutions/Universities/Organizations) and other sectors to ensure effective cross-sectoral collaboration. The
regulator must play a pivotal role in building a community of various scientific experts, who share common goals
and responsibilities. This partnership should encompass consumer, food industry as well as national authorities. Key
players need to have timely access to reliable and up-to-date information including emergency alerts so that
collective actions can be taken. It was the mandate of FSSAI to establish a system of network of organizations
with the aim to facilitate a scientific co-operation framework by the co-ordination of activities, the exchange of
information, the development and implementation of joint projects, the exchange of expertise and best practices in
the fields within the Food Authority’s responsibility.

Two Components of FSKAN:


 Assimilation of information with scientific experts nationally or internationally on emerging food safety issues and
best practices in the fields within the Food Authority’s responsibilities
 Formulation and execution of joint projects (pull and push type) where gaps or uncertainty in risk assessment for
food safety exist

232. Food Safety Display Boards (FSDBs) are informative board which primarily display food safety and hygiene
practices to be followed by food business operator (FBO) in their establishment. Consider the following colour codes
for FSDBs:

1) Restaurant: Green
2) Milk Retail: Blue
3) Street Food: Purple
4) Retail Store: Grey

Select the correct answer using the codes below.

a) 1 and 2 only
b) 2 and 4 only
c) 2, 3 and 4 only
d) 1 and 3 only

Solution: c)
Justification: As per the FSS Regulation, there is a mandatory requirement of displaying FSSAI License/Registration
Number at food premises. Usually, the FSSAI license number is not visible to the consumers. Thus, to change the
overall consumer perceptibility and to strengthen food safety, FSSAI has introduced Food Safety Display Boards
(FSDBs) for various food businesses. Henceforth, in addition to the existing mandatory requirement of displaying
FSSAI License/ Registration Number, it will also be mandatory (in a phased manner) for FBOs to display these Food
Safety Display Boards at FBO premise. The FSDBs are colour coded for different kind of food businesses for ease of
recognition by the consumers. The colour designation as per kind of business is as follows:-

 Restaurant (Purple)
 Fruit & Vegetable Retail (Green)
 Meat Retail (Red)
 Milk Retail (Blue)
 Street Food (Purple)
 Retail Store (Grey)

233. Consider the following statements:

1) Under the Participatory Guarantee System for India (PGS-India), FAO bureaucracy certifies organic products after
due inspection.
2) National Programme for Organic Production (NPOP) intends to facilitate certification of organic products in
conformity with the importing countries organic standards.

Which of the above is/are correct?


a) 1 only
b) 2 only
c) Both 1 and 2
d) None

Solution: b)
Justification: At present, the consumers do not have any method to check the authenticity of the organic food
products due to the absence of a regulatory framework. FSSAI has come out with draft regulations for organic food
products to ensure the authenticity of organic foods. FSSAI’s regulation mandates that labelling of organic foods
should provide full and accurate information regarding the organic status of the organic foods. Organic food
products must carry a certification mark or a quality assurance mark provided by the notified certification bodies.

Statement 1: Organic foods will have to comply with the government’s provisions under the National Programme
for Organic Production (NPOP) or the Participatory Guarantee System for India (PGS-India) run by the Agriculture
Ministry or any other standards notified by FSSAI. The standards of Organic Crop Production have been described in
NPOP document. The production standards include crop production plan, conversion requirement, nutrient
management, disease management etc. for agricultural crops. Details will be covered in I-Learning (new initiative on
website).
Statement 2: The PGS is an internationally applicable organic quality assurance system [like ISO 9000] implemented
and controlled by the committed organic farmer-producers through active participation, along with the consumers,
in the process based on verifiable trust.
It is not an “inspection raj” certification system but, rather, one that is based on personal integrity and peer
pressure. Integrity is honesty when no one is looking over your shoulder to see what you are doing. So, 1 is wrong.
http://pgsindia-ncof.gov.in/pgs_india.aspx

234. E-GRAINS is an initiative of the

a) Food Policy Foundation (FPF)


b) Down to Earth
c) Food Corporation of India (FCI)
d) Akshay Patra

Solution: c)
Justification: It is the IISFM (Integrated Information System for Foodgrains Management for Food Corporation of
India). It provides the online Stock Position of buffer and procurement by FCI. The FCI purchases wheat and rice
from the farmers in states where there is surplus production to ensure food security. E-grains improves
accountability and transparency in this operation.

235. With reference to governance of the Public Distribution System (PDS) system, consider the following
statements.

1) The central and state governments share the joint responsibility of regulating the PDS.
2) State governments are responsible for distributing grains to the consumers through a network of Fair Price Shops
(FPSs).
3) The grain quota of Below poverty Line (BPL) and above poverty line families has been fixed at the same level to
control leakages.
4) States/UTs are forbidden to distribute additional items of mass consumption through the PDS outlets such as edible
oils or iodized salt.

Select the correct answer using the codes below.

a) 1 and 2 only
b) 2, 3 and 4 only
c) 1, 2 and 4 only
d) 1 and 3 only

Solution: a)
Justification: Statement 1 and 2: While the central government is responsible for procurement, storage,
transportation, and bulk allocation of food grains, state governments distributes the same through the FPSs.
State governments are also responsible for operational responsibilities including allocation and identification
of families below poverty line, issue of ration cards etc.

Statement 3: A below poverty line (BPL) card holder should be given 35 kg of food grain and the card holder
above the poverty line should be given 15 kg of food grain as per the norms of PDS. However, there are
concerns about the efficiency of the distribution process.
Statement 4: Under the PDS, presently the commodities namely wheat, rice, sugar and kerosene are being
allocated to the States/UTs for distribution.

INTERNATIONAL EFFORTS

236. With reference to ANUGA 2017, consider the following:

1) It is an inter-governmental platform organized by UNSC for promoting defence technology cooperation.


2) India will be a co-Partner Country in 2017 edition of the event.
3) Agreements signed under the event are mandated to take the place of national laws and impose regulatory burdens
on party nations.

Select the correct answer using the codes below.

a) 1 only
b) 2 and 3 only
c) 2 only
d) 3 only
Solution: c)
Justification: Statement 1: ANUGA is the world’s biggest and most important trade fair for Food and beverage trade
held in Germany. It takes place biennially (every 2 years). ANUGA 2017 is the 34th edition. ANUGA offers an
extensive supporting programme with Lectures, Special exhibition and attractive industry events More than fifty per
cent of the industry visitors are foreigners.
Statement 2: India is to become a co-Partner Country in ANUGA 2017. It gives India an opportunity to showcase its
strengths and opportunities to an international audience in the food sector and invite investments.
Statement 3: There is no such provision.

UNIT-VII
INFRASTRUCTURE, INDUSTRY AND SERVICES

INFRASTRUCTURE

237. The Logistics Sector has been granted Infrastructure status by the Finance Ministry. What are the implications
of this move for this sector?

1) It becomes eligible to borrow from India Infrastructure Financing Company Limited (IIFCL).
2) It can now access External Commercial Borrowings (ECB) which is unavailable to non- infrastructure sectors.

Which of the above is/are correct?

a) 1 only
b) 2 only
c) Both 1 and 2
d) None

Solution: a)
Justification: The need for integrated Logistics sector development has been felt for quite some time in view of the
fact that the logistics cost in India is very high compared to developed countries. High logistics cost reduces the
competitiveness of Indian goods both in domestic as well as export market. It will enable the Logistics Sector to avail
infrastructure lending at easier terms with enhanced limits, access to larger amounts of funds as External
Commercial Borrowings (ECB), access to longer tenor funds from insurance companies and pension funds and be
eligible to borrow from India Infrastructure Financing Company Limited (IIFCL).

238. Delhi-Mumbai Industrial Corridor (DMIC) is a mega infra-structure project. It does NOT span the state of

a) Uttar Pradesh
b) Haryana
c) Madhya Pradesh
d) Rajasthan

Solution: c)
Justification: The project would include six mega investment regions of 200 square kilometres each and will run
through six states Delhi, Western Uttar Pradesh, Southern Haryana, Eastern Rajasthan, Eastern Gujarat, and
Western Maharashtra. The project aims to develop an environmentally sustainable, long lasting and technological
advanced infrastructure utilizing cutting age Japanese technologies and to create world class manufacturing and
investment destinations in this region. Funds from a Japanese government loan will soon be utilised for the first
time in the $100 billion, Delhi-Mumbai Industrial Corridor (DMIC) project. So far, the mega-project was being
developed only with the Indian government’s financial assistance.
LAND ACQUISITION AND REAL ESTATE

ROADS

239. With reference to Bharatmala project, consider the following statements.

1) It is an umbrella program for the highways sector that focuses on optimizing the efficiency of road traffic movement
across the country.
2) It is an entirely private, both domestic and foreign, funded road project being supervised by NHAI.
3) It envisions the creation of various economic corridors throughout the country connecting several districts with
national highways.

Select the correct answer using the codes below.

a) 1 and 2 only
b) 3 only
c) 1 and 3 only
d) 1, 2 and 3

Solution: c)
Justification: Statement 1: It will do so by bridging critical infrastructure gaps. The project includes construction of
feeder routes alongside national highways.
Statement 2: Around 80% of Bharatmala will be based on a government funded, engineering procurement and
construction (EPC) model while the rest will be a hybrid-annuity public private partnership.
Statement 3: Bharatmala envisions 44 economic corridors across the country at a cost of at least Rs 5 lakh crore.
With this, 70 – 80 percent of freight will move along NH as against the 40 percent Bharatmala project will help to
connect 550 Districts in the country through NH linkages. Currently, only around 300 Districts have NH linkages.
Bharatmala is also expected to have a positive impact on the Logistic Performance Index (LPI) of the country. Special
attention has been paid to fulfill the connectivity needs of backward and tribal areas, areas of economic activity,
places of religious and tourist interest, border areas, coastal areas and trade routes with neighbouring countries
under the programme.

240. The benefits of National Common Mobility Card (NCMC) are

1) Eliminating transport license regime by allowing interoperability of private transport operators who are authorized
to run at least one mode of transport
2) Interoperability for passenger between various modes of transport such as bus, train and metro in the urban area
3) Single e-purse with access to multiple transport products

Select the correct answer using the codes below.

a) 1 and 2 only
b) 1, 2 and 3
c) 2 and 3 only
d) 3 only

Solution: c)
Justification: Ministry of Urban Development has taken initiatives for implementation of an interoperable
Automatic Fare Collection (AFC) system for public transport in accordance with National Urban Transport Policy
(NUTP), 2006 which envisages that various urban Public Transport Systems are well integrated and offer a seamless
system to the users. A user should be able to pay one fare and travel in all the required modes to reach the
destination. National Payments Corporation of India (NPCI) has been entrusted to prepare the standards &
specifications of the National Common Mobility Card (NCMC). The scheme has been initiated since 2016. The card
will be of an open system based on EMV standards. The model will have a stored value, helpful in maintaining the
identity.

241. Consider the following: "Value Engineering Program" will

1) Be implemented by Department of Science and Technology


2) Look to use new technologies and materials in highway construction projects

Which of the above is/are correct?

a) 1 only
b) 2 only
c) Both 1 and 2
d) None

Solution: b)
Justification: Statement 1: The Ministry of Road Transport and Highways (MoRTH) has decided to implement
"Value Engineering Program" to promote use of new technologies, materials and equipment in highways projects
executed either under Public Private partnership (PPP) mode or public funding mode.
Statement 2: It also aims to ensure that roads or bridges and other assets get constructed much faster, are
structurally stronger and more durable. The MoRTH also has reconstituted nine-member National Panel of Experts
(NPE) for approving proposals for use of new technologies, material and equipment in highway sector.

RAILWAYS

242. Mission 41k, recently seen in news, is related to developments in which of these sectors of the Indian
economy?

a) Micro, Small and Medium Enterprises (MSMEs)


b) Indian Railways
c) Food Processing sector
d) Information Technology (IT)

Solution: b)
Justification: Learning: Ministry of Railways has come up with Mission 41k to save Rs. 41000 crores in the next
decade in Railways’ energy costs. This includes many initiatives such as:

 All electrification works done in the last decade would be doubled and this would change the energy mix of Indian
Railways. Indian Railways has set a target of 1000 MW of solar power and 200 MW of wind energy.

 At present 70% traffic runs on electric traction. In next 6 -7 years target has been set to run 90% of the traffic on
electric traction.
 By procuring electricity through open access, the cost of electricity procurement can come down drastically which
contributes 25% of the working expenses.

243. RITES, recently seen in Indian news, is a/an

a) Association of consumer rights organization advocating a stronger consumer law


b) Export arm of the railways undertaking various projects worldwide
c) Festivals Abroad Initiative (FAI) of the Ministry of External Affairs
d) A Hindu organization pushing for de-brahmanization of major Vedic rituals

Solution: b)
Justification: RITES is a Public Sector Enterprise under the Ministry of Railways. It is the export arm of the railways
undertaking various projects worldwide. RITES has engaged in the supply of Railway rolling stock (locomotives,
coaches, wagons and permanent way maintenance vehicles), maintenance of railway infrastructure, technical
studies, training of Myanmar Railway officials. Indian Railways recently handed over 18 microprocessor controlled
Indian locomotives to Myanmar Railways. They are supplied by RITES and manufactured by Diesel Locomotive
Works, Varanasi, India.

244. The largest rail network in Asia is in which of the countries?

a) China
b) India
c) Russia
d) Malaysia

Solution: b)

Justification: It is also the world’s second largest network operated under a single management. It has 115,000 km
of track length. It runs more than twelve thousand trains to carry over 23 million passengers daily – equivalent to
moving the entire population of Australia. It has entered a select club of railways that includes China, Russia and
USA in carrying more than one billion tons of freight annually.

245. Indian Railways Organization for Alternate Fuel (IROAF) has been recently awarded the coveted National level
“Golden Peacock Award for the Year 2017 for Eco-Innovation”. The award was given for

1) Substitution of Diesel in passenger trains by environment friendly Compressed Natural Gas (CNG).
2) Installation of open source licensing system for electricity generation and procurement

Which of the above is/are correct?

a) 1 only
b) 2 only
c) Both 1 and 2
d) None

Solution: a)
Justification: Indian Railways has achieved a major landmark in the field of Eco friendly fuel technologies by this
innovation. Use of CNG in Train Sets for passenger transportation has been done for the first time in the world. This
innovation (20% substitution of diesel by CNG) will reduce emissions considerably by bringing down NOx, CO2 and
Particulate Matter besides achieving economy in fuel cost by eight per cent. It could save up to over thirteen
hundred crores annually if implemented over entire fleet of Diesel Locomotives of Indian Railways. An improved
technology of 40% substitution is at present being developed by IROAF. This will enhance the potential of saving of
fuel cost of IR to about Rs. thirty four hundred crore per annum.

246. What is the major source of revenue for Indian Railways?


a) Heritage railways
b) Freight traffic
c) Ticket sales
d) Outsourcing services

Solution: b)
Justification: Only one-third of the 13000 trains running daily on IR are freight trains, but it accounts 65% of total
revenue of IR. Railway Freight traffic is vital for economic and industrial progress of the country. With regards to rail
tickets, on an average, the railways does not even recover the total cost of rail tickets. To help plan the traffic flows
and optimize freight operations, Ministry of Railways have launched Smart Freight Operation Optimization & Real
Time Information (SFOORTI) App for Freight Managers which provides features for monitoring and managing freight
business using Geographic Information System (GIS) Views and Dashboard.

247. Government has approved formation of a Rail Development Authority (RDA) comprising Chairman and three
Members. Consider the following reference to it.

1) RDA will decide pricing of major rail services commensurate with costs.
2) It will suggest measures for enhancement of Non-Fare Revenue.
3) RDA will focus on protection of consumer interests by ensuring quality of service and cost optimization.
4) It will approve non-discriminatory open access to the Dedicated Freight Corridor (DFC) infrastructure for all MNCs

Select the correct answer using the codes below.

a) 1, 2 and 3 only
b) 2 and 3 only
c) 1 and 4 only
d) 2, 3 and 4 only

Solution: b)
Justification: The objective underlying the setting up of RDA is to get expert advice on (RDA is not the apex
level decision making body it is an advisory body, which is why statement 1 and 4 are wrong):
 Encouraging market development and participation of stakeholders in the rail sector and for ensuring a fair
deal to the stakeholders and customers.
 Benchmarking of service standards against international norms and specify and enforce standards with respect
to the quality, continuity and reliability of services provided by them.
 Providing framework for non-discriminatory open access to the Dedicated Freight Corridor (DFC) infrastructure
and others in future.
 Suggesting measures to absorb new technologies for achieving desired efficiency and performance standards.

Statement 2 and 3 (objectives of RDA) are omitted in the above explanation.

248. With reference to the Railway Board, consider the following statements.

1) It is the apex body of the Indian Railways.


2) It reports to the Parliament through the Ministry of Railways.
3) The Chairman of the Board is the Union Minister of Railways.

Select the correct answer using the codes below.


a) 1 and 2 only
b) 2 and 3 only
c) 1 and 3 only
d) 1, 2 and 3

Solution: a)
Justification: Statement 1 and 3: Minister of Railways heads Indian Railways, but the Board is headed by a
Chairman appointed by the government. He has the status of a Secretary to the Government. The Chairman is
responsible for decisions on technical and policy matters of Indian Railways.
Statement 2: This is done to ensure the direct accountability of the board to the legislators.

PORTS&WATERWAYS

249. Consider the following statements.

Assertion (A): Each major port is governed by a Board of Trustees appointed by the Government of India. Reason
(R): Major Ports are under the Union list of the Indian Constitution.

In the context of the above, which of these is correct?

a) A is correct, and R is an appropriate explanation of A.


b) A is correct, but R is not an appropriate explanation of A.
c) A is correct, but R is incorrect.
d) Both A and R are incorrect.

Solution: a)
Justification: Ports in India are classified as Major and Minor Ports according to the jurisdiction of the Central and
State government as defined under the Indian Ports Act, 1908. Major Ports are under the Union list of the Indian
Constitution and are administered under the Indian Ports Act 1908 and the Major Port Trust Act, 1963. Each major
port is governed by a Board of Trustees appointed by the Government of India. The tariffs for major ports are fixed
by the Tariff Authority for Major Ports (TAMP). Minor operates are managed at the State level by the department in
charge of ports or the State Maritime Board, if created, as is the case in Gujarat, Maharashtra and Tamil Nadu.

The functions of the State maritime boards are similar to those of port trusts, and also include the authority to set
tariffs. They also focus on attracting private investment by awarding concession contracts, providing incentives,
exclusivity rights and assuring land acquisition.

250. Which of these have a bearing on inland waterway governance in India?

1) National Waterways Act, 2016


2) Inland Vessels (Amendment) Act, 2007
3) Inland Waterways Authority of India (IWAI)

Select the correct answer using the codes below.

a) 1 and 2 only
b) 2 only
c) 2 and 3 only
d) 1, 2 and 3
Solution: d)
Justification: Inland water traffic amounts to only 0.17% of total inland traffic in India. These acts and bodies
promote such transport.
Statement 1: A national waterway can only be declared by an act of Parliament. So, this Act merges 5 existing
Acts which have declared the 5 National Waterways and proposes 106 additional National Waterways.
Statement 2: It was enacted to facilitate the extension and usage of inland waterways by vessels. It is an
amendment to an original Act passed in 1917.
Statement 3: IWAI is the statutory authority in charge of the waterways in India. It does the function of
building the necessary infrastructure in these waterways, surveying the economic feasibility of new projects
and also administration.

CIVIL AVIATION

251. SkyRev360 is a project concerning

a) Global Satellite Navigation


b) Internet penetration
c) Airline industry
d) Paragliding promotion in hilly regions

Solution: c)
Justification: It is a fully automated and comprehensive data gathering, invoicing and collection service which
eliminates inaccuracies, errors and delays for better revenue management. SkyRev360 has been developed in
collaboration with the International Air Transport Association (IATA) under the Government of India’s Make in India
initiative. The solution gathers 100% of your flight data and produces comprehensive billing data. SkyRev360
automates data processing, helping ANSPs overcome strenuous manual processes which are prone to mistakes. In
this way: The IATA team handles all queries and disputes related to billing and collection from your air operators.
Incident reports, airspace usage, traffic volumes, economic values and more are always at your fingertips.

252. India and Japan have agreed to an open sky arrangement under which

1) The airlines of both countries can operate an unlimited number of flights in between
2) 100% FDI mutually has been allowed in the airline sector for investors of both countries under automatic route
3) Japanese and Indian flights are exempt from aviation tax and other cargo duties in both nations

Select the correct answer using the codes below.

a) 1 only
b) 2 and 3 only
c) 2 only
d) 1, 2 and 3

Solution: a)
Justification: Open sky agreement was signed in accordance with the National Civil Aviation Policy (NACP), 2016.
The NACP permits the government to enter into an ‘open sky’ air services agreement on a reciprocal basis with
SAARC nations as well as countries with territory located entirely beyond a 5,000 kilometer radius from New Delhi.
In 2003, India had signed an open sky agreement with the 10-member Association of South East Asian Nations
(ASEAN). Last year, India signed an open sky agreement with Greece, Jamaica, Guyana, Czech Republic, Finland,
Spain and Sri Lanka. India also signed a bilateral open sky agreement with the US in 2005. The agreement will not
only encourage connectivity and passenger travel between the two countries, but will also result in reduction in
airfares on these routes.

DIGITAL INFRASTRUCTURE

253. Consider the following with regard to the Unified Payments Interface (UPI).

1) It has been built by National Payments Corporation of India (NPCI).


2) It is outside the regulation window of the Reserve Bank of India (RBI).
3) It facilitates instant fund transfer between two bank accounts on mobile platform using unique virtual address.
4) A SWIFT code is mandatory for routing UPI payments.

Select the correct answer using the codes below.

a) 1, 3 and 4 only
b) 1 and 3 only
c) 1, 2 and 3 only
d) 2 and 4 only

Solution: b)
Justification: Statement 1, 2 and 3: UPI is a payments protocol built by government-backed NPCI and regulated by
the RBI. It facilitates fund transfer without putting details of card details, net banking, IFSC code, wallet password
etc.

Statement 4: SWIFT code is used for international banking transactions. It is not mandatory for domestic
transactions.

254. With reference to the Bharat Bill Payment System (BBPS), consider the following statements.

1) BBPS is an integrated bill payment system which will offer interoperable bill payment service to customers
online as well as through a network of agents on the ground.
2) Banks and non-bank entities, both, engaged in any of the bill payment activities falling under the scope of
BBPS are eligible to offer services covered under the system.

Which of the above is/are correct?

a) 1 only
b) 2 only
c) Both 1 and 2
d) None

Solution: c)
Justification: Statement 1: Bharat Bill Payment System (BBPS) system provides multiple payment modes and instant
confirmation of payment. The policy guidelines for the BBPS system were issued by the Reserve Bank of India on
November 28, 2014. The BBPS will operate as a tiered structure with a single Bharat Bill Payment Central Unit
(BBPCU) and multiple Bharat Bill Payment Operating Units (BBPOUs). National Payment Corporation (NPCI) has been
identified to act as BBPCU. It will be an authorized entity under the Payment and Settlement Systems Act, 2007.

Statement 2: Banks and non-bank entities presently engaged in any of the bill payment activities falling under the
scope of BBPS and desirous of continuing the activity are mandatorily required to apply for approval / authorisation
to Reserve Bank of India under the Payment and Settlement Systems (PSS) Act 2007. The entity should be a
company incorporated in India and registered under the Companies Act 1956/ Companies Act 2013. In case of any
Foreign Direct Investment (FDI) in the applicant entity, necessary approval from the competent authority.

URBAN INFRASTRUCTURE

255. With reference to the initiatives under or supported by the UN-Habitat New Urban Agenda, consider the
following statements.

1) Cities and Climate Change Initiative (CCCI) involves formulation of a composite index to define targets that can
support the formulation of evidence-based policies for cities.
2) City Prosperity Initiative (CPI) supports, facilitates and regulates funding of city development projects in LDCs
by developed countries.
3) Global Land Tool Network (GLTN) is an alliance of global, regional and national partners contributing to
poverty alleviation through the promotion of land reform.

Select the correct answer using the codes below.

a) 1 and 2 only
b) 2 and 3 only
c) None of the above
d) 3 only

Solution: d)
Justification: Statement 1: UN-Habitat’s Cities and Climate Change Initiative (CCCI) is the Agency’s flagship Initiative
supporting cities in emerging and developing countries to address the climate challenge. The Initiative supports
cities responding to the negative impact of climate change that is already being felt worldwide, to put in place
appropriate mitigation measures. CCCI places emphasis on participatory processes, sound analysis, sustainable
urban planning, good governance, responsive leadership, and practical initiatives at all levels.

Statement 2: UN-Habitat’s City Prosperity Initiative (CPI) is a global initiative that enables city authorities, as well as
local and national stakeholders, to identify opportunities and potential areas of intervention for their cities to
become more prosperous. Its composite index made of six dimensions serves to define targets and goals that can
support the formulation of evidence-based policies, including the definition of city-visions and long- term plans that
are both ambitious and measurable. The CPI is both a metric and a policy dialogue, which offers cities from
developed and developing countries the possibility to create indicators and baseline information, often for the first
time.

Statement 3: Facilitated by UN-Habitat, Global Land Tool Network (GLTN) is an alliance of global, regional and
national partners contributing to poverty alleviation through the promotion of land reform, improved land
management and security of tenure by developing and disseminating pro- poor and gender-responsive land tools.

ENERGY

256. India Energy Security Scenarios, 2047 housed under the India Energy Portal (IEP) is flagship initiative of the

a) NITI Aayog
b) The Energy and Resources Institute (TERI)
c) Centre for Science and Environment (CSE)
d) Greenpeace, India

Solution: a)
Justification: The India Energy Portal (IEP), an offering of the Energy Division of NITI Aayog, is intended to provide a
common platform for all energy related data and research for different sources. It aims to reach not only policy
makers but also those who use energy in their day to day lives and want to develop a holistic understanding of the
sector. It aims at the cohort of researchers and novices alike where they can gain a basic understanding of India’s
energy scenario and get access to Indian energy databases and reports.

257. The Government of India had decided to set up Strategic Petroleum Reserve (SPR) to

1) Reduce the loss born by Oil Marketing Companies (OMCs) on a day-to- day basis due to international oil price
movement
2) Ensure energy security of India
3) Boost export of petroleum products of India
4) Ease down petroleum prices in the domestic market on a day-to-day basis

Select the correct answer using the codes below.

a) 2 only
b) 2, 3 and 4 only
c) 1 and 4 only
d) All of the above

Solution: a)
Justification: To ensure energy security, the Government of India had decided to set up 5 million metric tons (MMT)
of strategic crude oil storages at three locations namely, Visakhapatnam, Mangalore and Padur (near Udupi). These
strategic storages would be in addition to the existing storages of crude oil and petroleum products with the oil
companies and would serve as a cushion during any external supply disruptions. In the 2017-18 budget, it was
announced that two more such caverns will be set up Chandikhole in Jajpur district of Odisha and Bikaner in
Rajasthan as part of the second phase. The construction of the Strategic Crude Oil Storage facilities is being
managed by Indian Strategic Petroleum Reserves Limited (ISPRL), a Special Purpose Vehicle, which is a wholly
owned subsidiary of Oil Industry Development Board (OIDB) under the Ministry of Petroleum & Natural Gas.

258. In the transportation sector, among the following, which of these is the biggest consumer of energy in India?

a) Indian railways
b) Coastal Shipping
c) Industrial Conveyer belts
d) Aviation sector

Solution: a)
Justification: Transport sector is the biggest consumer of energy, especially railway as the single largest consumer
with share of about 2% of National energy consumption. Rail being the most energy efficient mode of transport is
further working to change the energy mix towards greener sources. Electrification is a powerful measure to achieve
cost effectiveness. It improves mobility by use of high power energy efficient Locomotives, facilitate shift of traffic
from road to rail and also to reduce carbon footprints. In addition, electrification will also reduce traction energy bill
substantially. With its rising energy needs, India has emerged the fourth largest energy consumer of the world after
the US, China and Russia, but its per capita energy consumption remains lower than that of developed countries
259. Consider the following statements. Petrol and Diesel prices in India are

1) Determined by State governments based on the intimations given by the Oil Marketing Companies (OMCs)
2) Revised on a fortnightly basis in alignment with international crude prices

Which of the above is/are correct?

a) 1 only
b) 2 only
c) Both 1 and 2
d) None

Solution: d)
Justification: Statement 1 and 2: They are market determined. Moreover, since June 16 this year, petrol and diesel
prices across the country have been revised on a daily basis, against the previous policy of revising prices every
fortnight. By opting for daily pricing, India has joined advanced countries like the United States and others which
follow the practice. Amid protests against the recent spike in petrol and diesel prices, the government has ruled out
the possibility of an end to the recently introduced policy of revising fuel prices daily.

260. The government considers a village electrified, if

a) it has basic electrical infrastructure and all of its households and some public places have power
b) it is connected to the local power grid or has an independent power generation source
c) it has basic electrical infrastructure and 10% of its households and some public places have power
d) it has been classified so based on energy intensity usage

Solution: c)
Justification: India is the world's third largest producer and consumer of power. But poor distribution has been one
of the biggest challenges. The government had allocated Rs75, 893 crore for the Deen Dayal Upadhyaya Gram Jyoti
Yojana with the aim of providing electricity access to all villages. A village is considered electrified if 10% of its
homes and all public buildings are connected to the grid. All villages in India now have access to electricity. This was
achieved recently when a remote village in the north-eastern state of Manipur became the last to be connected to
the grid. But, World Bank figures show around 200 million people in India still lack access to electricity.

261. In India, among the following states, the percentage of rural households electrified would be the least in

a) Rajasthan
b) Odisha
c) Telangana
d) Goa

Solution: b)
Justification:
262. Consider the following about Power Grid in India.

1) A Power grid helps transfer power from the surplus regions to the deficit ones.
2) National Power Grid of India connects all major geographical regions of India except North-eastern India.
3) Power Grid Corporation of India Limited (POWERGRID) is an Indian state-owned electric utilities company.

Select the correct answer using the codes below.

a) 1 only
b) 2 and 3 only
c) 1 and 3 only
d) 1, 2 and 3

Solution: c)
Justification: Statement 1: In this way, power grid manages the peak deficit problems.
Statement 2: A National Power Grid was completed by linking all the five Regions-North, South, East, West and
North East. It is the largest single system in the world both in terms of grid size and a system capacity of around
200000 MW. This ensured complete integration of India into seamless network for delivering electricity.
Statement 3: POWERGRID is an Indian state-owned electric utilities company headquartered in Gurugram, India.
POWERGRID transmits about 50% of the total power generated in India on its transmission network.

MINES AND MINERALS

263. Consider the following statements.

1) In India the division of major and minor minerals is based on the availability of these minerals.
2) Under the Mines and Minerals (Development and Regulation) Act, 1957, mining of minor minerals is regulated
by the Centre.

Which of the above is/are correct?

a) 1 only
b) 2 only
c) Both 1 and 2
d) None

Solution: d)
Justification: Statement 1: In India, the minerals are classified as minor minerals and major minerals. Central
Government by notification in the Official Gazette declares minerals to be a minor or major mineral. The major-
minor classification has nothing to do with the quantum /availability of these minerals, though it is correlated with
the relative value of these minerals. Further, this classification is based more on their end use, rather than level of
production, level of mechanization, export and import etc. (e.g. Sand can be a major mineral or a minor mineral
depending on where it is used; same is the case for limestone.) Major minerals are those specified in the first
schedule appended in the Mines and Minerals (Development and Regulation) Act, 1957 (MMDR Act 1957) and the
common major minerals are Lignite, Coal, Uranium, iron ore, gold etc. It may be noted that there is no official
definition for “major minerals” in the MMDR Act. Hence, whatever is not declared as a “minor mineral” may be
treated as the major mineral. Major minerals are exploited by the Centre, and minor by the State.
INDUSTRY

GENERAL

264. With reference to the Limited Liability Partnership Act, 2008, consider the following statements about
Limited Liability Partnership (LLP).

1) LLP can enter into contracts and hold properties.


2) LLP can continue its existence irrespective of changes in partners.

Which of the above is/are correct?

a) 1 only
b) 2 only
c) Both 1 and 2
d) None

Solution: c)
Justification: Statement 1 and 2: LLP is a hybrid of companies & partnerships providing benefits of limited liability
while allowing its members the flexibility for organizing their internal structure as a partnership. The Act was
enacted by the Parliament of India to introduce and legally sanction the concept of LLP in India. Unlike the general
partnerships in India, LLP is a body corporate and legal entity separate from its partners, have perpetual succession
and any change in the partners of a LLP shall not affect the existence, rights or liabilities of the LLP.

Statement 2: LLP registered in India will be a resident even if only a part of control and management is in India.
Profits distributed by LLP exempt in the hands of the partners.

265. Consider the following with reference to the Phased Manufacturing Programme (PMP) in India.

1) It was launched for promoting the growth of domestic manufacturing of Cellular mobile handsets and solar
cells.
2) The central feature of PMP is to make interest free credit available to budding entrepreneurs to cut down
domestic cost of production.

Which of the above is/are correct?

a) 1 only
b) 2 only
c) Both 1 and 2
d) None

Solution: d)
Justification: At present, India imports basic chipset for mobile handsets but there has been a spurt in the
production of other mobile components. CBEC has recently imposed a 10% duty on imports of key smartphone
components such as populated printed circuit boards (PCBs), camera modules and connectors. At present there is
zero import duty imposed on the three components. This decision will push local assembly or manufacturing of
these components as companies who make here will get a price advantage over those who don’t. This decision
follows the government’s announcement to impose 20% basic customs duty (BCD) on fully built mobile phones,
which came into effect from February 1, 2018 as part of its phased manufacturing program (PMP). Since 2014,
import duties have acted as catalyst to grow investment into mobile phone manufacturing in India. Over the next
10-12 years, PMP aims to make India a manufacturing hub of mobile components. PMP was launched for promoting
the growth of domestic manufacturing of Cellular mobile handsets only. It has been proposed to extend this
program to other sectors as well. This programme is under Ministry of Electronics and Information Technology
(MieTY). Its overall aim is to impose duties (differential duty regime) and give tax reliefs and incentives on select
products involved in domestic manufacturing of cellular handsets. It is called phased manufacturing programme
because it will give fiscal benefits to domestic manufacturing of various components of cellular handsets in different
fiscals.

266. Which one of the following is an example of a ‘natural monopoly’?

a) Mahanagar Sanchar Nigam Limited (MTNL)


b) Indian Railways (IR)
c) Air India (Ai)
d) Steel Authority of India (SAIL)

Solution: b)
Justification: A natural monopoly is a distinct type of monopoly that may arise when there are extremely high fixed
costs of distribution, such as exist when large-scale infrastructure is required to ensure supply. Examples of
infrastructure include cables and grids for electricity supply, pipelines for gas and water supply, and networks for rail
and underground. These costs are also sunk costs, and they deter entry and exit. It may be more efficient to allow
only one firm to supply to the market because allowing competition would mean a wasteful duplication of
resources.

267. Which of these is/are NOT included in the Eight Core Sector Industries?

1) Iron Ore
2) Electricity
3) Fertilizers

Select the correct answer using the codes below.

a) 1 only
b) 1 and 3 only
c) 1 and 2 only
d) All of them are included.

Solution: a)
Justification: The Eight Core Industries comprise 40.27 per cent of the weight of items included in the Index of
Industrial Production (IIP).
Statement 1: It is steel that is included, not iron ore production. Steel production (weight: 17.92 per cent) increased
by 4.7 per cent in March, 2018 over March, 2017.
Statement 2: Electricity generation (weight: 19.85 per cent) increased by 4.5 per cent in March, 2018 over March,
2017. Its cumulative index increased by 5.2 per cent during April to March, 2017-18 over the corresponding period
of previous year.
Statement 3: Fertilizers production (weight: 2.63 per cent) increased by 3.2 per cent in March, 2018 over March,
2017. Its cumulative index increased by 0.03 per cent during April to March, 2017-18 over the corresponding period
of previous year.

268. eBiz is one of the integrated services projects and part of the 31 Mission Mode Projects (MMPs) under the
National E-Governance Plan (NEGP). It aims to
1) Create an investor-friendly business environment in India by making all regulatory information easily available
to the various stakeholders concerned
2) Provide licenses and permits for the firms across their business life-cycle without the dilatory intervention of
the Ministries concerned

Which of the above is/are correct?

a) 1 only
b) 2 only
c) Both 1 and 2
d) None

Solution: a)
Justification: eBiz is being implemented by Infosys Technologies Limited (Infosys) under the guidance and aegis of
DIPP, Ministry of Commerce & Industry. The vision of eBiz is to be the entry point for all individuals, businesses and
organizations (local and international) who would like to do business or have any existing business in India by
creating a one-stop-shop of convenient and efficient online G2B services to the business community, by reducing
the complexity in obtaining information and services related to starting businesses in India, and dealing with
licenses and permits across the business life-cycle. The focus of eBiz is to improve the business environment in the
country by enabling fast and efficient access to Government-to-Business (G2B) services through an online portal.
This will help in reducing unnecessary delays in various regulatory processes required to start and run businesses.
This project aims at creating an investor-friendly business environment in India by making all regulatory information
starting from the establishment of a business, through its ongoing operations, and even its possible closure - easily
available to the various stakeholders concerned. In effect, it aims to develop a transparent, efficient and convenient
interface, through which the government and businesses can interact in a timely and cost effective manner, in the
future.

269. Which of the following do NOT characterize Corporate Governance?

a) Maximizing the shareholder value in a corporation ethically


b) Benefitting the society-at-large
c) The ideal of trusteeship
d) All of the above characterize corporate governance.

Solution: d)
Justification: As per Narayan Murthy, Corporate governance is maximizing the shareholder value in a corporation
while ensuring fairness to all stakeholders, customers, employees, investors, vendors, the government and the
society-at-large. Corporate governance is about transparency and raising the trust and confidence of stakeholders in
the way the company is run. It is about owners and the managers operating as the trustees on behalf of every
shareholder - large or small.

270. Industrial licensing is compulsory for which of the following industries in India?

1) Tobacco, cigarette and related products


2) Semiconductor manufacturing
3) Drugs and pharmaceuticals

Select the correct answer using the code below.


a) 1 and 2 only
b) 2 and 3 only
c) 1 and 3 only
d) 1, 2 and 3

Solution: c)
Justification: In India, there are some regulations and restrictions with regard to establishing industries in certain
categories. This is done by making it mandatory to obtain licenses before setting up such an industry. Some
industries still carry that burden, for e.g.: Aerospace and defence related; Explosives etc.; dangerous chemicals;
tobacco etc.; alcoholic drinks; and drugs and pharmaceuticals. All these industries are directly related to public
health and security.

271. A wide range of issues have negatively impacted the growth of capital goods production in India. These
include

1) Zero duty imports of capital goods


2) Non-tariff barriers in export markets of capital goods denying market access
3) Low end user acceptance of new Indian capital goods technology

Select the correct answer using the codes below.

a) 2 and 3 only
b) 3 only
c) 1 only
d) 1, 2 and 3

Solution: d)
Justification: The capital goods sector production has grown at a rate of 1.1% p.a over the last 3 years. This is in
stark contrast to the Planning Commission targeted growth rate of 16.8% p.a. for production of capital goods during
the 12th Five Year Plan period.
Statement 1: Capital goods imports have been growing at a rate of 9.8% p.a. over the last 5 years. Issues affecting
domestic demand creation: The lack of positive bias towards domestic value addition in public procurement
policies, difficult contract conditions, persistent import and use of second-hand machinery with no incentive for
replacement, zero duty import under 'Project Imports' and delays in project implementation are the key factors
limiting domestic demand.
Statement 2: Issues affecting exports: Key challenges faced by Indian capital goods exporters are the inadequate
availability of competitive short and long-term financing, non-tariff barriers in export markets denying market
access and limited understanding of international market requirements especially
Statement 3: Issues affecting technology depth: Significant challenges and gaps exist in high-end, heavy-duty, high-
productivity and high precision technologies across sub-sectors. Contributors to these gaps include low end user
acceptance of new Indian technology, lack of skill availability, weak support infrastructure and low Indian
participation in developing international standards. Further, patent processing takes very long and fiscal incentives
for R&D are still inadequate.
Learning: Other issues are:
Issues affecting cost competitiveness: Indian manufacturers are still challenged with respect to cost competitiveness
compared to their global peers due to a skewed and state-wise variation in tax and duty structure, prevalence of
inverted duty structure for several products and high infrastructure and logistics cost.
Issues related to SMEs: SMEs still face challenges in developing new products and processes due to their smaller
scale and inadequate institutional mechanisms, limited access to capital and low awareness and compliance with
international standards.
272. The organized sector of India shares which of the following characteristics?

1) Recognition and registration by the government


2) Security of employment for the employed based on a contract
3) Terms of employment are regular and negotiated between the employer and employee
4) The sector is mandated to comply with labour laws in India

Choose the answer from the codes below:

a) 1 and 2 only
b) 2, 3 and 4 only
c) 3 only
d) 1, 2, 3 and 4

Solution: d)
Justification: The organized sector gives fixed wages, gives job security, pensions to workers, have assured regular
work in employment and provides other facilities like provident fund, paid wages and medical benefits. This growth
of this sector is crucial for the growth of India as this sector contains high paid and skilled jobs, as against uncertain
low-skilled and poorly paid jobs in the unorganized sector.

273. Arrange the following core Industries in decreasing order of their weightage under the Index of Industrial
Production (IIP).

a) Coal> Crude Oil> Natural Gas> Electricity> Cement


b) Electricity> Crude Oil > Coal> Cement > Natural Gas
c) Crude Oil > Electricity> Cement > Coal> Natural Gas
d) Natural Gas > Electricity> Cement > Coal> Crude Oil

Solution: d)
Justification: Core industries together have a combined weight of 37.90 per cent in the Index of Industrial
Production (IIP). Individual percentages of them are: Coal (weight: 4.38 per cent); Crude Oil (weight: 5.22 per cent);
Natural Gas (weight: 1.71 per cent); Petroleum refinery (weight: 5.94 per cent); Fertilizer (weight: 1.25 per cent);
Steel (weight: 6.68 per cent); Cement (weight: 2.41 per cent); and Electricity (weight: 10.32 per cent).

274. A rapid expansion of the manufacturing sector has been a key element of the growth experience of successful
developing countries, especially labour-abundant ones like India. Consider the following about the state of
manufacturing in India.

1) Since last one decade, the contribution of manufacturing sector to GDP has remained stagnant at one-fourth of
GDP.
2) Only a small share of employment in manufacturing is in organized manufacturing.
3) Employment is heavily concentrated in small firms.

Select the correct answer using the codes below.

a) 1 and 2 only
b) 2 and 3 only
c) 3 only
d) 1 and 3 only

Solution: b)
Justification: The Indian manufacturing sector exhibits many peculiarities that are quite puzzling:
i. It contributes around 17-18% of GDP, a small and stagnant share to GDP. It is envisaged that this share be increased
to 25%.
ii. Its composition is more skewed towards skill and capital intensive activities compared to countries at similar levels
of development;
iii. Only a small share of employment in manufacturing is in organized manufacturing. The unorganized manufacturing
sector accounted for almost 70 per cent of total manufacturing employment in 2009—10.
Employment is heavily concentrated in small firms. The degree of concentration is much higher than in other Asian
countries.

275. Which of the following is NOT an example of ‘sunrise’ industry?

a) Information Technology Industry


b) Wellness and Hospitality
c) Renewable energy industry
d) Automobile industry

Solution: b)
Justification: A sunrise industry is typically characterized by high growth rates, numerous start- ups and an
abundance of venture capital funding. Sunrise industries generally have plenty of "buzz" surrounding them as public
awareness about the sector increases and investors get attracted to its long-term growth prospects. Automobile
industry is a well-established industry, but renewable energy industry holds a lot of promise, which is why D is the
answer.

276. Consider the following about Industries (Development and Regulation) Act, 1951 (IDRA).

1) The Act establishes a 'Central Advisory Council' for the purpose of advising the Central Government on matters
concerning the development of the industries.
2) The Act is administered by the Ministry of Commerce through its Department of Industrial Policy & Promotion
(DIPP).
3) The IDRA empowers the Central Government to regulate the development of industries by means of licensing with
suitable exemptions as decided by the Government.

Select the correct answer using the codes below.

a) 1 only
b) 2 and 3 only
c) 3 only
d) 1, 2 and 3

Solution: d)
Justification: IDRA was enacted in pursuance of the Industrial Policy Resolution, 1948. The Act was formulated for
the purpose of development and regulation of industries in India by the Central Government. The main objectives of
the Act is to empower the Government: -
 to take necessary steps for the development of industries;
 to regulate the pattern and direction of industrial development;
 to control the activities, performance and results of industrial undertakings in the public interest.

The Act applies to the 'Scheduled Industries' listed in the First Schedule of the Act. However, small scale industrial
undertakings and ancillary units are exempted from the provisions of this Act.

277. Technology and Innovation Support Centre (TISC) are being setup by Department of Industrial policy and
promotion (DIPP). Services offered by TISCs include:

1) Access to online patent resources and intellectual property (IP)-related publications


2) Assistance in searching and retrieving technology information
3) Design, incubation and execution support and finance for start-ups
4) Plug and play special economic zones (SEZs) inside TISC

Select the correct answer using the codes below.

a) 1 and 2 only
b) 3 and 4 only
c) 1, 2 and 4 only
d) 2, 3 and 4 only

Solution: a)
Justification: The objective of the TISC is to stimulate a dynamic, vibrant and balanced IPRs to promote
entrepreneurship and enhance social, economic and cultural development by establishing a network of TISCs in
India.
It provides innovators in developing countries with access to locally based, high quality technology information and
related services, helping them to exploit their innovative potential and to create, protect, and manage their IPRs.
Statement 3 and 4: There are no such provisions, however, other services given by TISC include:
 Monitoring technology and competitors.
 Basic information on industrial property laws, management and strategy, and technology commercialization and
marketing.

278. Consider the following about the Modified Industrial Infrastructure Upgradation Scheme (MIIUS).

1) All States are covered under the scheme.


2) It focuses on providing quality infrastructure through public private partnership.
3) Greenfield Projects are undertaken in backward areas and North Eastern Region (NER).

Select the correct answer using the codes below.

a) 1 only
b) 2 and 3 only
c) 1 and 3 only
d) 1, 2 and 3

Solution: d)
Justification: Industrial Infrastructure Upgradation Scheme (IIUS) was launched in 2003 with the objective of
enhancing industrial competitiveness of domestic industry. This is a modified version of the scheme (started in
2013). Under MIIUS, projects have been undertaken to upgrade infrastructure in existing Industrial Parks/ Estates/
Areas. Projects are being implemented by the State Implementing Agency (SIA) of the State Government. Central
Grant up to 50% of the project cost with a ceiling of Rs.50 crore is provided under MIIUS with at least 25%
contribution of State Implementing Agency. In case of North Eastern States, the central grant and minimum
contribution of the SIA are up to 80% and 10% respectively.

279. Annual Survey of Industries (ASI) is the principal source of industrial statistics in India. Consider the following
with reference to it.

1) It does not cover unorganized or unregistered or informal sector enterprises.


2) It is conducted by Central Statistics Office (CSO)
3) Data from ASI comes with a lag of two years.

Select the correct answer using the codes below.

a) 1 and 2 only
b) 2 and 3 only
c) 3 only
d) 1, 2 and 3

Solution: d)
Justification: Statement 1: For the other category of factories/establishments, which are not covered under the
ASI, the information is collected through the unorganized sector surveys conducted by National Sample Survey
Organisation (NSSO) every 5 years. Therefore, the ASI and the unorganized sector surveys together cover the
complete manufacturing sector.
Statement 3: Reference Year for ASI 2013-14 is the accounting year for the factory which is ending on FY 2014, while
the survey would have been conducted in 2014-15.

STEEL

280. Consider the following statements.

1) India is currently world’s largest steel producer after China.


2) Steel industry has been de-licensed and de-controlled in India.
3) To promote steel export there is no export duty on steel items.

Select the correct answer using the codes below.

a) 1 only
b) 2 and 3 only
c) 1 and 3 only
d) 1 and 2 only

Solution: b)
Justification: Statement 1: India is the world’s third-largest steel producer for the third year in a row in 2017. In
2017, China and Japan have occupied top two positions respectively.
Statement 2: Steel industry was de-licensed and de-controlled in 1991 & 1992 respectively. As a facilitator, the
Government monitors the steel market conditions and adopts fiscal and other policy measures based on its
assessment.
Statement 3: Currently, GST of 18% is applicable on steel and there is no export duty on steel items. The
government has also imposed export duty of 30% on all forms of iron ore except low grade

PSUs
281. Prompt Corrective Action (PCA) framework of the Reserve Bank of India intends to ensure that banks do not
collapse. Banks placed under PCA have to face which of the following?

1) Restriction on the amount of loans banks can extend


2) Restrictions on branch expansion plans - domestic or overseas
3) Monitoring of bank’s non-performing assets and return on assets
4) Restrictions on entering into new lines of business
5) Restrictions on capital expenditure, other than for technological upgradation within Board approved limits

Select the correct answer using the codes below.

a) 1, 2 and 3 only
b) 2, 3 and 4 only
c) 1, 4 and 5 only
d) All of the above

Solution: d)
Justification: PCA is process or mechanism to ensure that banks don’t go bust. Under it, RBI has put in place some
trigger points to assess, monitor, control and take corrective actions on banks which are weak and troubled. It was
first introduced after global economy incurred huge losses due to failure of financial institutions during 1980s-90s.
According to latest PCA framework, banks to be placed under it are assessed on three parameters viz. Capital ratios,
Asset Quality and Profitability. Indicators to be tracked for these three parameters are CRAR (Capital to Risk
weighted Assets Ratio)/Common Equity Tier I ratio, Net NPA (non-performing assets) ratio and Return on Assets
(RoA) respectively. If banks breach of any risk threshold mentioned above, it results in invocation of PCA against
them. RBI enforces these guidelines to ensure banks do not go bust and follow prompt measures to put their house
in order. It had tightened its PCA framework in April 2017 to turn around lenders with weak operational and
financial metrics. The Reserve Bank of India (RBI) has placed 11 public sector banks (PSBs) out of 21 State-owned
banks under its Prompt Corrective Action (PCA) framework because of deteriorating performance. Since PCA
framework restricts amount of loans banks can extend, placing 11 PSBs under it will put pressure on credit being
made available to companies especially MSMEs.

282. In India, the organization SCOPE is

a) A Federation of business pressure groups


b) An informal group of think tanks geared towards public policy
c) The standing conference of Public bodies working towards ecological conservation
d) The apex body of Central Government owned Public Enterprises

Solution: d)
Justification: Standing Conference of Public Enterprises (SCOPE) is the apex body of Central Government owned
Public Enterprises. SCOPE has all the Central Public Enterprises, a few State Government Enterprises and some
nationalized banks as its members. It is the only organization representing the entire spectrum of public sector
enterprises (PSEs) in India. SCOPE, with a vision to enable member enterprises be globally competitive in a market
driven environment, facilitates the endeavors of its members in improving their overall performance and promote
their legitimate aspirations by strengthening their effective and sustained engagement with stakeholders. It has
been advocating concerns and issues of the PSEs and is engaged in an ongoing dialogue with the government in
order to have conducive public sector policy framework. Its activities are aimed at strengthening the PSEs by making
them more efficient and effective and at the same time encouraging them to explore new horizons in the global
market.
283. The PSUs were considered by the government as the focus of the 'trickle-down effect'. How PSUs were intended
to bring trickle-down effect?

1) Generating employment for the masses


2) Initiating industrial development in backward regions
3) Raise the purchasing power of the masses

Select the correct answer using the codes below.

a) 1 and 2 only
b) 3 only
c) 2 and 3 only
d) 1, 2 and 3

Solution: d)
Justification: Trickle-down economics assumes investors, savers and company owners are the real drivers of growth.
It thus believes that if investment flows to the higher echelons of the economy, the benefits slowly percolate to the
poor and the lower echelons of the economy. Employment would raise purchasing power and increase demand for
certain goods and services. This would push the demand for new industries which would again generate
employment and tackle poverty. This creates a virtuous cycle.

TEXTILES INDUSTRY

284. What is the purpose behind awarding “India Handloom” tags by the government?

a) To create a brand for promoting traditional hand woven heritage of India


b) To allow only certain category of India handicrafts to be exported abroad
c) To protect selected tribal handloom mills in remote areas of the country
d) To reduce dependence on powerlooms in view of the employment generation potential of handlooms

Solution: a)
Justification: The “India Handloom” Brand (IHB) was launched by the Prime Minister of India on the occasion of first
handloom day in August 2015 to endorse the quality of the products in terms of raw material, processing, weaving
and other parameters besides social and environmental compliances for earning the trust of the customers. The
“India Handloom” brand would be given only to high quality defect free authentic handloom products for catering
to the needs of those consumers who are looking for niche handmade products. The “India Handloom” Brand is
aimed at generating a special market space and increased earnings to the weavers.

285. The Ministry of Textiles proposes to organize Pan India camps in handlooms and handicrafts clusters, under the
initiative “Hastkala Sahyog Shivir”. Which of these services will be provided to the weavers and artisans in these
camps?

1) Issuance of credit facilities through MUDRA scheme


2) Assistance for technological upgradation under Hathkargha Samvardhan Sahayata
3) Distribution of modern tool kits and equipments

Select the correct answer using the codes below.

a) 1 only
b) 2 and 3 only
c) 3 only
d) 1, 2 and 3

Solution: d)
Justification: The camps will be organized for ten days during October this year. This initiative is dedicated to Pandit
Deendayal Upadhyay Garib Kalyan Varsh, the birth centenary year of Pandit Deendayal Upadhayay. Other services
that will be provided are:
 Issuance of Yarn Pass Book
 Enrollment under IGNOU/NIOS courses
 Access to Common Service Centre facilities
 Buyer-seller meets and expos

It is proposed to organize the camps in over two hundred handloom Block Level Clusters & Weavers’ Service Centers
in collaboration with State Governments.

286. Welfare schemes for textile workers are being implemented by the Ministry of Textiles in which of the following
areas?

1) Health and accident insurance of weavers and artisans


2) Credit Gaurantee Fund for Micro and small enterprises for the coverage of handicraft artisans
3) Mudra loan for Handicraft Artisans in line with Pradhan Mantri Mudra Yojana

Select the correct answer using the codes below.

a) 1 only
b) 2 and 3 only
c) 3 only
d) 1, 2 and 3

Solution: d)
Justification: Statement 1: Insurance schemes are:
 Rajiv Gandhi Shilpi Swasthaya Bima Yojana for Health Insurance, currently under merger with Rashtriya Shilpi
Swasthya Bima Yojana
 Aam Admi Bima Yojana, providing for life insurance and accident Insurance
Statement 2 and 3: The National Handloom Development Programme (NHDP) is being implemented in the country.

Main components of NHDP are Concessional Credit, Handloom Marketing Assistance, and Block Level Clusters.

COOPERATIVE SECTOR INDUSTRIES

STARTUPs

287. Startupblink, recently seen in news, is a/an

a) Indian MNC that was declared a Unicorn start-up by FICCI


b) Global startup ecosystem map with several registered start-ups and accelerators
c) Initiative of the Department of Industrial Promotion to establish quick single window mechanisms for start-up
grievance redressal.
d) Employment portal of start-ups in India to recruit talent from top Universities

Solution: b)
Justification: Startupblink is a global startup ecosystem map with tens of thousands of registered startups,
coworking spaces, and accelerators. Its report prepared in association with ANSYS Startup Program measures
startup ecosystem strength and activity. It includes global ranking index of 125 countries and 900 cities, measuring
their startup ecosystem strength and activity. According to report released by Startupblink, India was ranked 37th
out of 125 countries in global startup ecosystem in 2017. The rankings of country are based on thousands of data
points gathered from various sources, such as incubators and accelerators that appear on StartupBlink global
ecosystem. Top 5 Countries in Global startup ecosystem in 2017 are United States (1st), United Kingdom (2nd),
Canada (3rd), Israel (4th) and Germany (5th).

MSMEs

288. Zero Effect, Zero Defect is a policy initiative to enhance energy efficiency and resources efficiency in

a) Rural Cottage industries


b) Medium & Small Industries
c) Information Technology Parks
d) Special Economic Zones (SEZs)

Solution: b)
Justification: The scheme is an extensive drive of the Government of India to enhance global competiveness of
MSMEs by providing them financial support in assessment, rating & handholding of its manufacturing processes on
quality and environment aspects. The activities include nationwide awareness of the ZED Model, building capacities
of accredited agencies or bodies by training & certifying assessors & consultants for site-assessment & handholding.
The scheme also includes an e-platform for assessments and e-content for capability building. The major objectives
of the ZED Scheme are:

 To create proper awareness in MSMEs about ZED manufacturing ad motivate them for assessment of their
enterprise for ZED rating.
 To drive manufacturing with adoption of Zero Defect production processes without impacting the environment
(Zero Effect).
 To encourage MSMEs to constantly upgrade their quality standards in products and processes.
 To support “Make in India” campaign.

289. CriSidEx that was recently launched is actually a

a) Solar power augmentation Index


b) Medium and Small scale enterprises Sentiment Index
c) Green City Development Index
d) Credit rating index

Solution: b)
Justification: It is India’s First MSE Sentiment Index, and has been launched by CRISIL and SIDBI. CriSidEx is a
composite index based on a diffusion index of 8 parameters and measures MSE business sentiment on a scale of 0
(extremely negative) to 200 (extremely positive). CriSidEx will have 2 indices, one for the ‘survey quarter’ and
another for the ‘next quarter’ once a trend emerges after few rounds of the survey, providing independent time
series data. The crucial benefit of CriSidEx is that its readings will flag potential headwinds and changes in
production cycles and thus help improve market efficiencies. And by capturing the sentiment of exporters and
importers, it will also offer actionable indicators on foreign trade. MSME sector is backbone of the economy. It is
one of the largest employers in the country and with the vast population where employment either in government
or in the large industry itself has limited potential.

290. What is/are the benefits of Udyog Aadhar Registration for MSMEs?

1) The applicant will receive financial support for participating in foreign expos to showcase their products.
2) After registering their MSME, the applicant will receive the benefits of government schemes such as a loan without
guarantee or with subsidized rates of interest.
3) Registration would facilitate hassle-free opening of current bank accounts in the name of the business.

Select the correct answer using the codes below.

a) 2 only
b) 2 and 3 only
c) 1 and 3 only
d) 1, 2 and 3

Solution: d)
Justification: A large chunk of enterprises in India are simply not registered due to the cumbersome paperwork
involved in the process and, therefore, can't tap the government schemes for them. The Ministry for Micro, Small
and Medium Enterprises had in 2015 notified Udyog Aadhaar to ensure ease of registration and wider coverage of
MSMEs to avail the benefits under various Schemes of Central/ State governments. Udyog Aadhaar (UA) has been
introduced for running units. Udyog Aadhaar Memorandum (UAM) can be filled online on the portal created by
Ministry of MSME i.e. http://udyogaadhaar.gov.in.

The Udyog Aadhaar Memorandum is filed on self-declaration basis, and no supporting document is required to be
uploaded or submitted while filing the Udyog Aadhaar Memorandum, but the Central Government or the State
Government or such person as may be authorized in this behalf may seek documentary proof of information
provided in the Udyog Aadhaar Memorandum, wherever necessary.

Benefits of Udyog Aadhar Registration

 After registering their MSME, the applicant will receive the benefits of all the government schemes such as an easy
loan, loan without guarantee, loans with subsidized rates of interest etc.
 The applicant will receive financial support for participating in foreign expos to showcase their products.
 The applicant will also be eligible for government subsidies.
 Registration would facilitate hassle-free opening of current bank accounts in the name of the business.
 It would also allow businesses to apply for government micro business loans and other such related beneficial
schemes

291. The Government has launched a new scheme namely “Financial Support to MSMEs in ZED Certification
Scheme”. The objective(s) of the scheme includes

1) Promote adaptation of energy efficient manufacturing


2) Drive manufacturing with adoption of Zero Defect production processes and without impacting the environment.
3) Establish a MSME Product Certifying Authority (MPCA) to assess quality of their products
4) Support ‘Make in India’ campaign.

Select the correct answer using the codes below.

a) 1 and 3 only
b) 2 and 4 only
c) 1, 2 and 4 only
d) D. 1, 2, 3 and 4

Solution: c)
Justification: ZED strives for quality manufacturing. The objective of the scheme for promotion of ZED
manufacturing amongst MSMEs and ZED Assessment for their certification so as to (these are other than what is
mentioned in the question):
 Develop an Ecosystem for Zero Defect Manufacturing in MSMEs.
 Encourage MSMEs to constantly upgrade their quality standards in products and processes. Develop professionals in
the area of ZED manufacturing and certification.

292. The project "Financing of Energy Efficiency at MSMEs" in India is an initiative of

a) World Bank and UN Energy Council


b) Global Environmental Facility (GEF) and UNDP
c) Bureau of Energy Efficiency (BEE), GEF and World Bank
d) Green Climate Fund (GCF) and World Economic Forum (WEF)

Solution: c)
Justification: The objective of the India-MSME Energy Efficiency Project is to improve efficiency and reduce GHG
emissions through commercial investments in energy efficiency goods and services in target Small and Medium
Enterprise clusters. The project aspires to address the current gap in understanding between energy auditors and
bank loan officers.

MISCELLANEOUS

293. Recently, the Ease of Doing Business report based on an Enterprise Survey of over three thousand
manufacturing firms across Indian states and union territories was recently released by

a) NITI Aayog
b) WEP- India chapter
c) Department of industrial Policy and Promotion (DIPP)
d) Industrial Development Bank of India (IDBI)

Solution: a)
Justification: The survey has been conducted, along with the IDFC Institute, to assess the business regulations and
enabling environment across India from firms’ perspective. The Survey was released by the Union Minister of
Commerce and Minister of Law & Justice.
The major findings of this report are as follows:

 A higher level of economic activity and better performance on a range of doing business indicators are strongly
correlated.
 Newer and younger firms report a more favorable business environment in that they take less time in obtaining
approvals than older firms, suggesting an improvement in the business environment.
 Labor regulations are a bigger constraint for labor intensive firms
 The experience of firms with fewer employees is different from that of larger firms. In some cases, large firms face
more regulatory barriers than smaller firms.

294. A ‘Task Force on Shell Companies’ under the Joint Chairmanship of Revenue Secretary and Secretary, Ministry
of Corporate Affairs was constituted in February, 2017. What are ‘Shell Companies’?

a) Oil companies that cause heavy ecological damage


b) Fly by night operators
c) A corporate entity without active business operations
d) A company engaged in deep sea precious stones extraction
Solution: c)
Justification: Shell Company is a corporate entity without active business operations or significant assets. It can’t be
asserted that shell corporations are illegal. They are deliberate financial arrangements by many big companies to
avoid taxes without attracting legal actions. Tax avoidance is not illegal, though it is not desirable. But many shell
companies park black money, carryout illegal transactions and sometimes act as facilitators of money laundering.

 Often, shell companies remain untraceable and happen to be the vehicle of choice for money launderers, bribe
givers and takers, tax evaders and financiers of terrorism.
 Most of the shell companies are registered in tax havens like British Virgin Islands or Cayman Islands.
 Recently it was found that a single address hosted 75 firms and no employees! The taskforce will look into such
issues.

295. Social Entrepreneurship is running

a) Any non-profit business.


b) An ecologically sustainable business.
c) An inclusive business that has participation from major social groups and local community.
d) A business that makes its money in a socially responsible way.

Solution: d)
Justification: It is the use of the techniques by start-up companies and other entrepreneurs to develop, fund and
implement solutions to social, cultural, or environmental issues. Muhammad Yunus (Grameen Bank) was a famous
social entrepreneur. Another example can be a crowd funding internet platform like https://milaap.org .These
ventures are not necessarily formed to reinvest all profits into the communities. So, these firms apply commercial
strategies to maximize improvements in human and environmental well-being.

296. Consider the following about India Brand Equity Foundation (IBEF) and associated provisions.

1) It is a non-profit NGO certifying khadi and handloom products.


2) Its objective is to promote and create international awareness of the ‘Made in India’ label in markets overseas.
3) India’s Foreign Trade Policy Statement released in 2015 emphasises the need of a branding strategy.
4) Activities of IBEF are limited to plantations, textiles and leather sectors.

Select the correct answer using the codes below.

a) 2 and 3 only
b) 1 and 4 only
c) 1, 2 and 4 only
d) 1, 2 and 3 only

Solution: d)
Justification: Statement 1: It is a trust established by the Department of Commerce, Ministry of Commerce and
Industry.
Statement 2: IBEF's primary objective is to promote and create international awareness of the Made in India label in
markets overseas and to facilitate dissemination of knowledge of Indian products and services. Towards this
objective, IBEF works closely with stakeholders across government and industry
Statement 3: This is to promote Indian exports as well as push Make in India campaign.
Statement 4: IBEF has undertaken focused branding activities for sectors namely engineering, pharma, plantations
(tea, coffee and spices), services, textiles and leather.

297. Global Entrepreneurship Summit (GES)


1) was first organized by India following the years of the 1991 economic reforms
2) focusses exclusively on commercial as well as social ventures by women

Which of the above is/are correct?

a) 1 only
b) 2 only
c) Both 1 and 2
d) None

Solution: d)
Justification: Statement 1: The summit organized annually since 2010, is the preeminent annual entrepreneurship
gathering that convenes over one thousand emerging entrepreneurs, investors, and supporters from around the
world.
This year marks the first GES held in South Asia, and the event underscores the broad and enduring partnership with
India.
Statement 2: The summit of 2017 will focus on four key industry sectors: Energy and Infrastructure, Healthcare and
Life Sciences, Financial Technology and Digital Economy, and Media and Entertainment.
 The US and India will co-host the Global Entrepreneurship Summit (GES) in Hyderabad, India.
 GES 2017 will create an environment that empowers innovators, particularly women, to take their ideas to the next
level.

SERVICES SECTOR

CORPORATE GOVERNANCE

INTELLECTUAL PROPERTY RIGHTS

CONCEPTS

POLICIES

298. Countries of Particular Concern (CPC) and Priority Watch List are often seen in news with reference to

1) Indicators of Religious freedom


2) Intellectual property rights deficiencies
3) Openness of the Economy
4) Zones prone to civil wars

Select the correct answer using the codes below.

a) 1 and 2 only
b) 2 and 3 only
c) 1 and 4 only
d) 3 and 4 only

Solution: a)
Justification: CPC: The US State Department has unveiled its list of countries designated as the worst offenders
against religious liberty. These countries are labelled as CPC. A country is labelled as a CPC, by USA, after it engages
in “systemic, ongoing, [and] egregious” violations of religious liberty. This declaration is in accordance with the
International Religious Freedom Act of 1998. The list includes Burma, China, Eritrea, Iran, North Korea, Sudan, Saudi
Arabia, Tajikistan, Turkmenistan, and Uzbekistan. The governments that have been designated as CPCs are subject
to ‘Presidential Actions’, such as sanctions, which may include economic or political measures directed against a
government to encourage it to improve the state of religious freedom in its country.

299. The International Treaty on Plant Genetic Resources for Food and Agriculture

1) Calls for protecting the traditional knowledge of these farmers.


2) Prevents the recipients of genetic resources from claiming intellectual property rights over those resources in the
form in which they received them
3) Facilitates access to the genetic materials of various crops for research, breeding and training for food and
agriculture

Select the correct answer using the codes below.

a) 1 only
b) 2 and 3 only
c) 1 and 3 only
d) 1, 2 and 3

Solution: d)
Justification: It was adopted by a Conference of the Food and Agriculture Organization (FAO) in 2001. The Treaty
aims at: recognizing the enormous contribution of farmers to the diversity of crops that feed the world; establishing
a global system to provide farmers, plant breeders and scientists with access to plant genetic materials; ensuring
that recipients share benefits they derive from the use of these genetic materials with the countries where they
have been originated.
Statement 1: The Treaty recognizes the enormous contribution farmers have made to the ongoing development of
the world’s wealth of plant genetic resources. It calls for protecting the traditional knowledge of these farmers,
increasing their participation in national decision-making processes and ensuring that they share in the benefits
from the use of these resources
Statement 2 and 3: The Treaty facilitates access to the genetic materials of the 64 crops in the Multilateral System
for research, breeding and training for food and agriculture. Those who access the materials must be from the
Treaty’s ratifying nations and they must agree to use the materials totally for research, breeding and training for
food and agriculture. The Treaty prevents the recipients of genetic resources from claiming intellectual property
rights over those resources in the form in which they received them, and ensures that access to genetic resources
already protected by international property rights is consistent with international and national laws.

300. Consider the following statements.

1) Protecting a title or slogan requires the use of ‘patents’.


2) A trademark registration prevents others from producing the same good.

Which of the above is/are correct?

a) 1 only
b) 2 only
c) Both 1 and 2
d) None

Solution: d)
Justification: S1: Protecting a title, slogan, or other short word phrase, generally requires trademark, not Patent.
Both require registration, else trademark or patent cannot be claimed over the product (phrase, logo etc.). A patent
for an invention is the grant of a property right to the inventor issued by the government. It is the right to exclude
others from making, using, offering for sale, or selling. Thus, it is more restrictive than both copyrights and
trademark.
S2: In trademark the same good can be made. However, it does not restrict others from innovating in the patented
subject-matter, as also in the case of copyrights (where the subject matter can be written upon by others too).

301. What is NOT included in the meaning of ‘Design’ under the Designs Act, 2000?

1) Mode of construction of the product


2) Trade mark of the product
3) Shape and pattern of the product

Select the correct answer using the codes below.

a) 1 only
b) 1 and 2 only
c) 2 and 3 only
d) 1 and 3 only

Solution: b)
Justification: ‘Design’ means only the features of shape, configuration, pattern or ornament or composition of lines
or colour or combination thereof applied to any article whether two dimensional or three dimensional or in both
forms, by any industrial process or means, whether manual, mechanical or chemical, separate or combined, which
in the finished article appeal to and are judged solely by the eye. It does not include any mode or principle or
construction or anything which is in substance a mere mechanical device, and does not include any trade mark, as
defined in the Trade and Merchandise Marks Act, 1958, property mark or artistic works as defined under the
Copyright Act, 1957. Object of the Designs Act is to protect new or original designs so created to be applied or
applicable to particular article to be manufactured by Industrial Process or means. Sometimes purchase of articles
for use is influenced not only by their practical efficiency but also by their appearance. But, Stamps, Labels, tokens,
cards cannot be considered an article for the purpose of registration of Design because once the alleged Design i.e.,
ornamentation is removed only a piece of paper, metal or like material remains and the article referred ceases to
exist. Article must have its existence independent of the Designs applied to it.

302. Which of the following governmental departments administers matters related to patents?

a) Department of Industrial Policy and Promotion


b) Department of Science and Technology
c) Department of Commerce
d) Department of Scientific and Industrial research

Solution: a)
Justification: The Office of the Controller General of Patents, Designs & Trade Marks (CGPDTM) is located at
Mumbai, under DIPP, under Ministry of Commerce. Department of Commerce is another department of Ministry of
Commerce. Patent is a set of exclusive rights granted by a sovereign state to an inventor or assignee for a limited
period of time in exchange for detailed public disclosure of an invention. An invention is a solution to a specific
technological problem and is a product or a process. Patents are a form of intellectual property. The Controller
General supervises the working of the Patents Act, 1970, as amended, the Designs Act, 2000 and the Trade Marks
Act, 1999 and also renders advice to the Government on matters relating to these subjects. In order to protect the
Geographical Indications of goods a Geographical Indications Registry has been established in Chennai to administer
the Geographical Indications of Goods (Registration and Protection) Act, 1999 under the CGPDTM.
INSTITUTIONS

303. Cell for IPR Promotion and Management (CIPAM) is a professional body under

a) Ministry of Corporate Affairs


b) Department of Industrial Policy & Promotion (DIPP)
c) NITI Aayog
d) Department of Economic Affairs

Solution: b)
Justification: CIPAM is a professional body under the aegis of the DIPP, Ministry of Commerce & Industry. Its
mandate is to effectively implement National Intellectual Property Rights (IPR) Policy adopted in May 2016 with
slogan – “Creative India; Innovative India”. It has undertaken several measures to strengthen IP ecosystem in the
country. It has been mandated to undertake:

 Simplifying and streamlining of IP processes by formulating and implementing a focused strategy for each policy
objective
 Coordination with State level agencies and Ministries/ Departments of the Government of India, industry bodies as
well as international agencies;
 IP cells to be set up
 IPR awareness campaign across the country in schools, colleges/universities and the industry
 Training and sensitization porgrammes for enforcement agencies and Judiciary; coordination for effective
enforcement of IPR rights.

Study and facilitate implementation of best practices for promotion and commercialization of IP within the country.

UNIT-VIII
INVESTMENT, MONEY AND CAPITAL MARKETS

INVESTMENT-THEORY

INVESTMENT -GOVERNMENT INITIATIVES

304. The National Investment and Infrastructure Fund (NIIF) of India accepts investment from

1) Indian Government
2) Sovereign wealth funds
3) Insurance and pension funds
4) Endowments

Select the correct answer using the codes below.

a) 2 and 3 only
b) 1 and 4 only
c) 1, 2 and 4 only
d) 1, 2, 3 and 4.

Solution: d)
Justification::NIIF was set up in 2015 as an investment vehicle for funding commercially viable greenfield,
brownfield and stalled projects in the infrastructure sector. NIIF will invest in areas such as energy, transportation,
housing, water, waste management and other infrastructure-related sectors in India. The corpus of the fund is
proposed to be around Rs 40,000 crore, with the government investing 49% and the rest to be raised from third-
party investors such as sovereign wealth funds, insurance and pension funds, endowments etc. Recently, the
National Investment and Infrastructure Fund (NIIF) of India has signed an investment agreement worth $1 billion
with a wholly owned unit of the Abu Dhabi Investment Authority (ADIA).

305. What is a Systemically Important Core Investment Company (CIC-ND-SI), as per the guidelines of the Reserve
Bank of India?

1) It is a Scheduled Bank, with a large asset base, registered with the RBI.
2) Infrastructure Finance Companies (IFCs) are eligible to called CICs.
3) A robust financial company that does not carry the business of acquisition of shares and securities can be called as
CIC.

Select the correct answer using the codes below.

a) 1 only
b) 1 and 2 only
c) 2 only
d) 2 and 3 only

Solution: c)
Justification: Over last some years, RBI has carved out some specialized NBFCs like Core Investment Companies
(CICs), NBFC- Infrastructure Finance Companies (IFCs), Infrastructure Debt Fund- NBFCs, NBFC-MFIs and NBFC-
Factors being the most recent one. The Systematically Important Core Investment Company (CIC-ND-SI) is a financial
company that is principally engaged in the business of acquisition of shares and securities. A CIC-ND-SI is a Non-
Banking Financial Company (only some conditions are mentioned here) with asset size of Rs 100 crore and above
carrying on the business of acquisition of shares and securities and which satisfies the following conditions as on the
date of the last audited balance sheet: - it holds not less than 90% of its net assets in the form of investment in
equity shares, preference shares, bonds, debentures, debt or loans in group companies; it accepts public funds.
Thus, all these conditions must be met by any non-banking finance company to trade in shares and securities
business. Due to systemic implications on account of access to public funds (such as funds raised through
Commercial Paper, debentures, inter-corporate deposits and borrowings from banks/FIs), CICs having asset size of
100 crore or above are categorized as Systemically Important Core Investment Companies (CICs-ND-SI) and are
required to obtain Certificate of Registration from the Reserve Bank.

306. Consider the following statements.

1) India Infrastructure Finance Company Ltd (IIFCL) is a Public Private Partnership (PPP) based Special Purpose Vehicle
(SPV) company set up to provide long term finance to viable infrastructure projects
2) Industrial Finance Corporation of India (IFCI) is a Systemically Important Non-Deposit taking Non- Banking Finance
Company (NBFC-ND-SI), registered with the RBI.

Which of the above is/are correct?

a) 1 only
b) 2 only
c) Both 1 and 2
d) None

Solution: b)
Justification: Statement 1: IIFCL is a wholly-owned Government of India company set up in 2006 to provide long
term finance to viable infrastructure projects through the Scheme for Financing Viable Infrastructure Projects
through a Special Purpose Vehicle called India Infrastructure Finance Company Ltd (IIFCL), broadly referred to as
SIFTI. The sectors eligible for financial assistance from IIFCL are as per the Harmonized list of Infrastructure Sub-
Sectors as approved by the Government and RBI and as amended from time to time. These broadly include
transportation, energy, water, sanitation, and communication, social and commercial infrastructure. IIFCL has been
registered as a NBFC-ND-IFC with RBI since September 2013.

Statement 2: IFCI Ltd. was set up in 1948 as Industrial Finance Corporation of India, a Statutory Corporation,
through ‘The Industrial Finance Corporation of India Act, 1948’ of Parliament to provide medium and long term
finance to industry. After repeal of this Act in 1993, IFCI became a Public Limited Company registered under the
Companies Act, 1956. The shareholding of the Government of India in paid-up share capital of IFCI has been
increased to 51.04% and IFCI has become a Government Company under Section 2(45) of the Companies Act, 2013.
The primary business of IFCI is to provide medium to long term financial assistance to the manufacturing, services
and infrastructure sectors.

307. Hybrid Annuity Model is a public-private partnership model that is characterized by

1) Delinking of performance of the structure with the cost compensation of payment


2) Absence of competitive bidding when awarding contracts

Which of the above is/are correct?

a) 1 only
b) 2 only
c) Both 1 and 2
d) None

Solution: d)
Justification: The Government of India had accorded Cabinet approval to Hybrid Annuity-PPP model in 2016 with
100% central sector funding.

Statement 1 and 2: Under this model, the development, operation and maintenance of the sewage treatment STPs
will be undertaken by a Special Purpose Vehicle (SPV) to be created by the winning bidder at the local level. As per
this model, 40% of the Capital cost quoted would be paid on completion of construction while the remaining 60% of
the cost will be paid over the life of the project as annuities along with operation and maintenance cost (O&M)
expenses. One of the most important features of this model is that both the Annuity and O&M payments are linked
to the performance of the STP. This will ensure continued performance of the assets created due to better
accountability, ownership and optimal performance. A tripartite agreement has been signed between National
Mission for Clean Ganga (NMCG) and the state level executing agencies Uttarakhand Pey Jal Nigam and Uttar
Pradesh Jal Nigam, with concessionaires for setting up of India’s first Sewage Treatment Plants under Hybrid
Annuity mode, at Haridwar and Varanasi.

MONEY AND CAPITAL MARKETS

308. With reference to InvITs, consider the following statements.

1) The objective of InvIT is to facilitate investment into the infrastructure sector in India.
2) To encourage investments, InvITs are exempt from regulation by the Securities and Exchange Board of India (SEBI).
3) InvITs are designed to pool small sums of money from a number of investors to invest in assets that give cash flow
over a period of time.

Select the correct answer using the codes below.


a) 1 and 3 only
b) 2 and 3 only
c) 1 only
d) 1, 2 and 3

Solution: a)
Justification: Often, infrastructure projects such as roads or highways take some time to generate steady cash flows.
Meanwhile, the infrastructure company has to pay interest to banks for the loans taken by it. An InvIT essentially
gives the company the leeway to fulfil its debt obligations quickly. InvITs are mutual fund like institutions that
enable investments into the infrastructure sector by pooling small sums of money from multitude of individual
investors for directly investing in infrastructure so as to return a portion of the income (after deducting
expenditures) to unit holders of InvITs, who pooled in the money. InvITs are set up as a trust and registered with
SEBI. Since the minimum investment amount in an IPO is Rs 10 lakh, InvITs are suitable for high networth
individuals, institutional and non-institutional investors like pension funds, foreign portfolio investors, mutual funds,
banks and insurance firms. InvITs are listed on exchanges just like stocks — through IPOs.

309. Infrastructure Debt Funds (IDF) is an innovative attempt for addressing the issue of sourcing long term debt for
infrastructure projects. Who among the following are eligible to invest in IDF?

1) Institutional investors such as insurance and pension funds


2) Off-shore high net worth individuals (HNIs)
3) Foreign Institutional Investors (FIIs)

Select the correct answer using the codes below.

a) 1 and 2 only
b) 2 and 3 only
c) 1 and 3 only
d) 1, 2 and 3

Solution: d)
Justification: Potential investors under IDFs may include off-shore institutional investors, off-shore high net worth
individuals and other institutional investors (insurance funds, pension funds, sovereign wealth funds, etc.). IDFs can
be set up either as a trust or as a NBFC. The income of Infrastructure Debt Funds has been exempted from income
tax. So far, three IDF-NBFCs and three IDF -MFs have been operationalized.

310. Consider the following statements. Initial Coin Offerings (ICO)

1) Can be converted into crypto currencies


2) Can be used to raise funds by start-up firms
3) Are regulated by SEBI in India

Select the correct answer using the codes below.

a) 2 only
b) 1 and 3 only
c) 1 and 2 only
d) 1, 2 and 3

Solution: c)
Justification: An ICO, like an equity initial public offer (IPO), is an issuance of digital tokens that can be converted
into crypto currencies and are mostly used to raise funds by start-up firms dealing in block chain technology and
virtual currencies like bitcoins and ethereum. Unlike an IPO, which is governed by SEBI regulations, there is no
regulatory body for ICOs in India. Start-ups like Zebpay, Unocoin, Coinsecure, Searchtrade, Belfrics and Bitxoxo are
some of the well- known players in the bitcoin and blockchain segment in India. China recently banned such
offerings after its central bank said that ICOs are “illegal public finance” mechanism used for issue of securities and
money laundering.

311. Stock markets in India are regulated by

a) Reserve Bank of India (RBI)


b) Ministry of Corporate Affairs (MoCA)
c) Unified Financial Agency (UFA)
d) Securities and Exchange Board of India (SEBI)

Solution: d)
Justification: The overall responsibility of development, regulation and supervision of the stock market rests with
the Securities & Exchange Board of India (SEBI), which was formed in 1992 as an independent authority. Since then,
SEBI has consistently tried to lay down market rules in line with the best market practices. It enjoys vast powers of
imposing penalties on market participants, in case of a breach.

312. Black-box trading, sometimes seen in news, is

a) Financial investment activities by shadow banks


b) High frequency trading using computer algorithms
c) Buying and selling of coal in auction markets
d) Trading in illegal commodities

Solution: b)
Justification: It is also known algorithmic trading or automated trading. It is the process of using computers
programmed to follow a defined set of instructions for placing a trade in order to generate profits at a speed and
frequency that is impossible for a human trader.
Issues: It is one of the most debated issues impacting the way the securities transactions are being conducted in the
world. Speedy execution, accuracy, reduced costs and avoiding the errors of human emotions are some of the
reasons for its increasing popularity. At the same time, the development of such technology raises several
regulatory challenges, particularly with respect to market manipulation and ensuring equity and integrity of the
markets.

313. The main difference between money market and capital market pertains to

a) Short-term vs. long-term borrowing of funds


b) Regulation vs. non-regulation
c) National vs. international character
d) All of the above

Solution: a)
Justification: Financial markets in every economy are having two separate segments today, one catering to the
requirements of short-term funds and the other to the requirements of long-term funds. The short-term financial
market is known as the money market, while the long-term financial market is known as the capital market. The
money market fulfils the requirements of funds for the period up to 364 days (i.e., short term) while the capital
market does the same for the period above 364 days (i.e., long term).

314. With reference to the directions issues by the Reserve Bank of India on Exchange Traded Currency Derivatives
(ETCD), consider the following:
1) Non-Resident Indians and Foreign Portfolio Investors will not be allowed to invest in ETCDs.
2) They cannot be used to hedge exposure or speculate on financial assets such as commodities and currencies.

Which of the above is/are correct?

a) 1 only
b) 2 only
c) Both 1 and 2
d) None

Solution: d)
Justification: ETCD is financial instrument that trades on regulated exchange, and whose value is based on value of
another asset. These derivatives are traded in a regulated fashion. They can be used to hedge exposure or speculate
on wide range of financial assets like commodities, currencies, equities and even interest rates. RBI recently raised
exposure limit under exchange traded currency derivatives (ETCD) trading for residents and foreign portfolio
investors (FPIs) to $100 million across all currency pairs involving Indian rupee. This decision aims to help entities
engaged in forex transactions to maintain their currency risks in better manner. Earlier, the limit was of $15 million
for US Dollar-Rupee and $5 million for other currency pairs of Indian rupee with Japanese Yen, Euro and British
Pound. The raised exposure limit permits persons resident in India and FPIs to take positions (long or short), without
having to establish existence of underlying exposure, up to single limit of $100 million equivalent across all currency
pairs involving Indian rupees, put together, and combined across all exchanges.

315. CNX Nifty is used extensively by investors in India and around the world as a barometer of the Indian capital
markets. It is prepared by

a) Reserve Bank of India (RBI)


b) Securities and Exchange Board of India (SEBI)
c) National Stock Exchange of India
d) Department of Financial Services, Union Government

Solution: c)
Justification: The NIFTY 50 covers 22 sectors of the Indian economy and offers investment managers exposure to
the Indian market in one portfolio. The base period for the CNX Nifty index is 1995. NSE was the first exchange in
the country to provide a modern, fully automated screen-based electronic trading system which offered easy
trading facility to the investors spread across the length and breadth of the country. NSE was set up by a group of
leading Indian financial institutions at the behest of the government of India to bring transparency to the Indian
capital market

316. What is the importance of Treasury Bills (T-Bills) in the economy?

1) They provide long-term capital to the government especially for infrastructure projects.
2) They satisfy the CRR and SLR requirements of the banking institutions that invest in them.

Which of the above is/are correct?

a) 1 only
b) 2 only
c) Both 1 and 2
d) None

Solution: b)
Justification: Started in 1986, they are instruments of money market used by the Central Government to fulfil its
short-term liquidity requirement up to the period of 364 days. At present only the 91—day TBs, 182—day TBS and
the TBS are issued by the government. The TBS other than providing short-term cushion to the government, also
function as short-term investment avenues for the banks and financial institutions, besides functioning as
requirements of the CRR and SLR of the banking institutions.

317. Consider the following statements. Indian Depository Receipt (IDR)

1) It is a financial instrument denominated in foreign currency issued by an Indian Depository


2) Every IDR represents an ownership interest in one Share.

Which of the above is/are correct?

a) 1 only
b) 2 only
c) Both 1 and 2
d) None

Solution: b)
Justification: It is a financial instrument denominated in Indian Rupees in the form of a depository receipt. The IDR
is a specific Indian version of the similar global depository receipts. It is created by a Domestic Depository (custodian
of securities registered with the Securities and Exchange Board of India) against the underlying equity of issuing
company to enable foreign companies to raise funds from the Indian securities Markets. The foreign company IDRs
will deposit shares to an Indian depository. The depository would issue receipts to investors in India against these
shares. Each IDR represents any securities, cash or other property attributable to the Deposited Shares that has
been deposited with the Custodian or the Depository but has not been directly distributed to the IDR Holders.

318. The Government of India has recently announced the sale of “Government of India Floating Rate Bonds 2024”.
With reference to government securities, consider the following:

1) Floating Rate Bonds are sold by the Reserve Bank of India.


2) RBI issues Cash Management Bills (CMBs) to meet long-term borrowing needs of the Government of India.
3) Treasury bills are short term debt instruments issued by the Government of India.

Select the correct answer using the codes below.

a) 1 and 2 only
b) 1 and 3 only
c) 2 only
d) 2 and 3 only

Solution: b)
Justification: Government security is a tradable instrument issued by the Central Government or the State
Governments. It acknowledges the Government’s debt obligation (to raise money from the market for various
purposes). Such securities are short term (usually called treasury bills, with original maturities of less than one year)
or long term (usually called Government bonds or dated securities with original maturity of one year or more).
Besides providing a return in the form of coupons (interest), Government securities offer the Maximum safety as
they carry the Sovereign’s commitment for payment of interest and Repayment of principal.

Statement 1: There are two types of bonds:


Fixed Rate Bonds – These are bonds on which the coupon rate is fixed for the entire life of the bond. Most
Government bonds are issued as fixed rate bonds.
Floating Rate Bonds – Floating Rate Bonds are securities which do not have a fixed coupon rate. The coupon is re-
set at pre-announced intervals (say, every six months or one year) by adding a spread over a base rate. These are
issued by RBI.
Statement 2: Cash Management Bills (CMBs), are a relatively new short-term instrument to meet the temporary
mismatches in the cash flow of the Government. The CMBs have the generic character of T-bills but are issued for
maturities less than 91 days.
Statement 3: Treasury bills or T-bills, which are money market instruments, are short term debt instruments issued
by the Government of India and are presently issued in three tenors, namely, 91 day, 182 day and 364 day.

319. Which of these types of trading/agreements in securities markets is NOT permitted in India?

a) Futures contract
b) Options contract
c) Forward contract
d) All are permitted

Solution: d)
Justification: Option A: Futures Contract has been permitted long back. It is a contract between two parties where
both parties agree to purchase or sell a particular commodity or any other financial instrument at a predetermined
price at a specified time in the future.
Option B: Commodity Exchanges have been requesting SEBI for a long time to permit options trading in
commodities be allowed. It was permitted recently.
 Options Contract is a derivative product that offers an investor the right to purchase without any obligation to buy
at the specified price/date. Suppose you want your dream car which as of now costs RM 100,000 (RM is a currency
unit). You don’t have the money to buy the car now, but you will have after 3 months. A smart move would be to
buy an “option” to purchase this car in the future (after 3 months). You pay the car owner a contract premium (say
RM 1,000) to exercise this option. Remember, you have the option to buy or not buy the car. See the diagram below
now. Suppose after 3 months, car prices increase to RM 150,000. This is a win case for you, since the options
contract was to buy it at RM 100,000. You gain. Suppose car price reduces. Now, you won’t buy the car, because the
contract was not an obligation (only a right) to buy the car. You would now pay the options premium (RM 1000 to
the owner), and may be buy the car from the market.

Option C: You would heard have the Forward Markets Commission that earlier regulated forward markets. Now
SEBI has subsumed the organization.
 A forward contract is a private agreement between two parties giving the buyer an obligation to purchase an asset
(and the seller an obligation to sell an asset) at a set price at a future point in time.

320. India Infrastructure Fund (IIF)

1) is a SEBI-registered domestic venture capital fund


2) sponsored entirely by foreign governments
3) invests only in greenfield projects

Select the correct answer using the codes below.

a) 1 only
b) 2 and 3 only
c) 1 and 3 only
d) None of the above

Solution: a)
Justification: Statement 1: It is focused on long-term equity investments in a diversified portfolio of infrastructure
projects.
Statement 2: IIF has been sponsored by IDFC Limited (IDFC), along with Citigroup Inc. (Citi) and India Infrastructure
Finance Company Limited (IIFCL) as founder investors. The establishment of IIF has received the strong support of
the Government of India.
Statement 3: IIF’s portfolio is expected to comprise greenfield, brownfield and operational assets/projects in core
infrastructure sub-sectors including transport etc. The current size of IIF is INR 38 billion with investor commitments
from institutional investors in India, USA, Canada, Europe, Japan and the Middle-East. China-led Asian Infrastructure
Investment Bank (AIIB) has approved $150 million equity investment loan to the India Infrastructure Fund. This loan
will be the bank’s first equity investment to fund private projects.

321. With reference to the India International Exchange (India INX), consider the following:

1) It is located in International Financial Services Centre (IFSC).


2) It allows international investors to trade from anywhere across globe.
3) The exchange provides competitive advantage against other Indian exchanges in terms of lower security
transactions taxes.
4) It is the first exchange in the world to operate for twenty four hours in the day.

Select the correct answer using the codes below.

a) 1 and 2 only
b) 2, 3 and 4 only
c) 1, 2 and 3 only
d) 1 and 4 only

Solution: c)
Justification: Statement 1 and 2: India-INX is India’s first international exchange at IFSC Gujarat International
Financial Tech (GIFT) City, Gandhinagar. It is wholly-owned subsidiary of Bombay Stock Exchange (BSE), through its
global bourse. It is one of world’s most advanced and fastest trading technology platforms with turn-around time of
4 micro seconds. It trades in equity derivatives, currency derivatives, commodity derivatives including Index and
Stocks. It also offers depository receipts and bonds. Its vision is "Emerge as the leading International Exchange with
best-in-class global practice in technology, products innovation and customer service."
Statement 3: The exchange being located in IFSC, GIFT City, provides competitive advantage in terms of tax
structure and supportive regulatory framework. These include benefits in security transaction tax, commodity
transaction tax, dividend distribution tax and long-term capital gain tax waivers and no income tax.
Statement 4: Following the International trading timings, this unique stock exchange will start when trading at
Japanese stock exchange starts, and will end when US stock exchange stops, from sunrise to sunset. So, it operates
for 22 hours a day allowing international investors and NRIs to trade from anywhere across globe.

322. Consider the following about Multi Commodity Exchange (MCX) of India.
1) MCX is country’s first listed commodity futures exchange that facilitates online trading.
2) It offers futures trading in bullion, ferrous and non-ferrous metals.
3) Futures trading in Energy and agricultural commodities have still not been allowed in India.

Select the correct answer using the codes below.

a) 1 and 2 only
b) 3 only
c) 1, 2 and 3 only
d) 1 only

Solution: d)
Justification: It also provides clearing and settlement of commodity futures transactions, thereby providing platform
for risk management. It was launched in 2003 and operates within regulatory framework of Forward Contracts
Regulation Act, 1952 (FCRA, 1952) under SEBI. MCX offers futures trading in bullion, ferrous and non-ferrous metals,
energy, and a number of agricultural commodities (mentha oil, cardamom, potatoes, palm oil and others). Globally,
MCX ranks no. 1 in silver, no. 2 in natural gas, no. 3 in crude oil and gold in futures trading.

323. Algorithmic trading in financial markets refers to

a) Any high frequency trading (HFT)


b) Automatic transaction orders generated using mathematical models
c) A trade happening outside of the conventional exchanges based on a pre-decided future contract
d) Any trading that does not obey the laws of spot trading

Solution: b)
Justification: Both Algorithmic trading and HFT are often used inter-changeably, but they are not really same. HFT
refers to high-volume orders executed within split-seconds to make immediate gains from market opportunities.
HFT trading are often backed by algo trading, which spot trading opportunity. AT are transaction orders generated
by using advanced mathematical models that involves automated execution of trade. It uses mathematical models
and software codes to make transaction decisions on exchanges and execute them at high speed. This technology-
driven trading enables traders to take advantage of any profit making opportunities arising in the market much
before a human trader can even spot them. It was introduced in India in 2009. At present, on National Stock
Exchange (NSE), algorithm trades accounts close to 16% of all trades. On the Bombay Stock Exchange (BSE), it was
8.56% in January 2017.

324. An Exchange Traded Fund (ETF)

1) Is traded at a fixed price in the market


2) Cannot be used to trade commodities
3) Must be bought over a futures contract

Select the correct answer using the codes below.

a) 1 and 3 only
b) 2 only
c) 3 only
d) None of the above

Solution: d)
Justification: Statement 1 and 2: ETF is like a mixture of funds that is trade in the stock market. It can be based on
an index, a commodity, bonds, or a basket of assets like an index fund; similar to a mutual fund. Unlike mutual
funds, an ETF trades like a common stock on a stock exchange. ETFs experience price changes throughout the day as
they are bought and sold.
Statement 3: There is no such condition. ETFs can be easily bought / sold anytime during market hours like any
other stock on the exchange. The CCEA has recently authorized taking a call on an Alternative Mechanism on
divestment through Exchange Traded Fund (ETF) out of all the listed CPSEs.

325. Alternative Investment Funds (AIFs), as defined in Securities and Exchange Board of India (Alternative
Investment Funds) Regulations, 2012 invests mainly in

1) Start- ups and SMEs


2) Private equity funds
3) Government securities

Select the correct answer using the codes below.

a) 1 only
b) 3 only
c) 1 and 2 only
d) 2 and 3 only

Solution: c)
Justification: AIFs refer to any privately pooled investment fund, (whether from Indian or foreign sources), in the
form of a trust or a company or a body corporate or a Limited Liability Partnership (LLP). AIF does not include funds
covered under the SEBI (Mutual Funds) Regulations, 1996, SEBI (Collective Investment Schemes) Regulations, 1999
or any other regulations of the Board to regulate fund management activities. Hence, in India, AIFs are private funds
which are otherwise not coming under the jurisdiction of any regulatory agency in India. As per Securities and
Exchange Board of India (Alternative Investment Funds) Regulations, 2012, AIFs shall seek registration in one of the
three categories

 Category I: Mainly invests in start- ups, SME's or any other sector which Govt. considers economically and socially
viable.
 Category II: These include Alternative Investment Funds such as private equity funds or debt funds for which no
specific incentives or concessions are given by the government or any other Regulator
 Category III : Alternative Investment Funds such as hedge funds or funds which trade with a view to make short
term returns or such other funds which are open ended and for which no specific incentives or concessions are
given by the government or any other Regulator.
Assets under management by India-focused AIFs have swelled 90% over the past year, thanks to the increasing
interest of high-net worth investors in the unique structures and themes of these funds that also face fewer
regulations than other instruments like mutual funds.

326. Venture capitalists and Angel Investors invest in

a) Only debt-ridden ventures


b) Promising start-ups
c) Overseas companies that round-trip money
d) Corporate bonds issued by blue chip companies

Solution: b)
Justification: Both Angel Investors and Venture Capitalists will hold private equity from having made investments
directly into private companies (start-ups). However, business Angel Investors will be individuals, often successful
business people, who are investing their own personal funds into a potentially rewarding business opportunity.
Whereas Venture Capital is invested by firms or companies that use other people's money. They raise that money
by offering investors a chance to take part in a fund that is then used to buy shares in a private company.

327. Consider the following with reference to Call Money Market:


1) It is where uncollateralized borrowing and lending of funds take place on an overnight basis.
2) It is a market exclusively dedicated to banks.
3) Cooperative institutions are allowed to invest in the market based on a 2017 circular of the Reserve Bank of India
(RBI).

Select the correct answer using the codes below.

a) 1 and 2 only
b) 1 only
c) 1 and 3 only
d) 2 and 3 only

Solution: b)
Justification: The call money market is an important segment of the money market where uncollateralised
borrowing and lending of funds take place on over-night basis. Participants in the call money market in India
currently include scheduled commercial banks regional rural banks), cooperative banks (other than land
development banks), and primary dealers, both as borrowers and lenders. Prudential limits, in respect of both
outstanding borrowing and lending transactions in the call money market for each of these entities, are specified by
the RBI.

328. What kind of assets are permitted to be included in mutual funds (MFs)?

1) Equity
2) Debt bills issued by companies
3) Government bonds

Select the correct answer using the codes below.

a) 1 only
b) 2 and 3 only
c) 1 and 3 only
d) 1, 2 and 3

Solution: d)
Justification: MFs are polled funds of various investors, managed by an active fund manager. There are various type
of mutual funds in the market. Income Funds invest in corporate bonds, government bonds and money market
instruments with long maturities. Money market funds invest in short-term fixed income securities such as
government bonds, treasury bills, bankers’ acceptances, commercial paper and certificates of deposit. There are
MFs dedicated to each of these instruments or handling a mixture of these instruments.

329. Central Counterparty (CCP) is an entity that exists in European countries to

a) Facilitate trading done in derivatives and equities markets


b) Guard joint borders of European Union
c) Coordinate defence equipment related export production
d) Fund mining investments in Least Developed Countries (LDCs)

Solution: a)
Justification: CCP is a corporate entity that helps facilitate trading done in European derivatives and equities
markets.

CCP bear most of the credit risk of buyers and sellers when clearing and settling market transactions.
So, it reduces counterparty, operational, settlement, market, legal and default risk for traders.
It provides efficiency and stability to the financial markets in which they operate.

330. Non-Resident Indians (NRIs) can invest in India as per a regulation of Foreign Exchange Management (Transfer
or issue of Security by a person resident outside India) Regulations, 2000. However, as per the regulation

1) NRIs cannot invest in shares or debentures.


2) NRIs may contribute foreign capital by way of capital contribution

Which of the above is/are correct?

a) 1 only
b) 2 only
c) Both 1 and 2
d) None

Solution: b)
Justification: Statement 1: As per Regulations, NRIs may invest in shares or partly paid up shares of an Indian
company under Foreign Direct Investment (FDI) Scheme subject to certain terms and conditions.
Statement 2: NRIs may even acquire securities or units on a Stock Exchange in India on repatriation basis (i.e. they
can take the investment back abroad) under the Portfolio Investment Scheme. NRIs even hold FCNR deposits in
India banks that earn them interest. We will cover FCNR in details in coming tests.

331. Qualified Institutional Placement (QIP) allows

1) A listed company to issue equity shares for raising capital


2) Government to borrow from the market without causing any ‘crowding out effect’

Which of the above is/are correct?

a) 1 only
b) 2 only
c) Both 1 and 2
d) None

Solution: a)
Justification: Crowding out effect happens when a large borrowing by the government leads to a shortage of
capital for private players in the market. This leads to a rise in interest and thus increases in borrowing costs for
the corporate.
A QIP is a capital raising tool wherein a listed company can issue equity shares, fully and partly convertible
debentures, or any security (other than warrants) that is convertible to equity shares. Apart from preferential
allotment, this is the only other speedy method of private placement whereby a listed company can issue shares or
convertible securities to a select group of investors.
Statement 2: If there is government borrowing, a crowding out effect is most likely to follow because there isn’t
enough capital for everyone in the market.
But unlike in an IPO or an FPO (further public offer), only institutions or qualified institutional buyers (QIBs) can
participate in a QIP issuance.

 QIBs include mutual funds, domestic financial institutions such as banks and insurance companies, venture capital
funds, foreign institutional investors, and others.

 The State Bank of India is planning to complete a planned share sale by year-end, probably through a qualified
institutional placement (QIP).
332. ‘Insider trading’ is considered detrimental to the health of financial systems and has been declared illegal in
several nations because

1) It allows the trading agent to speculate in financial markets.


2) The trading agents have access to valuable non-public information about a corporation.

Which of the above is/are correct?

a) 1 only
b) 2 only
c) Both 1 and 2
d) None

Solution: b)
Justification: Statement 1: Speculation is the practice of engaging in risky financial transactions in an attempt
to profit from fluctuations in the market value of a tradable good. It is not illegal. Also, insider trading can be
more than speculation and can even be a genuine investment. For e.g. an employee knows (confidentially)
that his company’s plans of acquiring another company will raise its stock price, and hence he invests in the
stocks to gain from the rise in price.
Statement 2: When insiders, e.g. key employees or executives who have access to the strategic information
about the company, use the same for trading in the company's stocks or securities, it is called insider trading
and is highly discouraged by the Securities and Exchange Board of India to promote fair trading in the market
for the benefit of the common investor. However, in certain cases if the information has been made public, in
a way that all concerned investors have access to it, but that will not be a case of illegal insider trading.

333. Consider the following statements.

1) A bond is a debt instrument in which an investor loans money to an entity which borrows the funds for a defined
period of time at a variable or fixed interest rate.
2) In India, the Central Government issues both treasury bills and bonds or dated securities while the State
Governments issue only bonds or dated securities.

Which of the above is/are correct?

a) 1 only
b) 2 only
c) Both 1 and 2
d) None

Solution: c)
Justification: Statement 1: Bonds are used by companies, municipalities, states and sovereign governments to
raise money to finance a variety of projects and activities. Owners of bonds are debt holders, or creditors, of
the issuer.
Statement 2: A Government Security (G-Sec) is a tradable instrument issued by the Central Government or the
State Governments. It acknowledges the Government’s debt obligation. Such securities are short term (usually
called treasury bills, with original maturities of less than one year) or long term (usually called Government
bonds or dated securities with original maturity of one year or more).
In India, the Central Government issues both, treasury bills and bonds or dated securities while the State
Governments issue only bonds or dated securities, which are called the State Development Loans (SDLs). G-Secs
carry practically no risk of default and, hence, are called risk-free gilt-edged instruments.

Treasury bills or T-bills, which are money market instruments, are short term debt instruments issued by the
Government of India and are presently issued in three tenors, namely, 91 day, 182 day and 364 day. Treasury bills
are zero coupon securities and pay no interest. They are issued at a discount and redeemed at the face value at
maturity.

334. Recently, the Finance minister launched the first options security with gold as the underlying asset for trading
on Multi Commodity Exchange Ltd. What are the benefits of Gold Options?

1) It allows the investors to hedge volatility in the price of gold in the market.
2) It will help in wider participation of investors in commodity speculation markets.
3) It helps in formalizing Gold trade in India leading to improved transparency in gold trade.

Select the correct answer using the codes below.

a) 1 and 2 only
b) 2 only
c) 1 and 3 only
d) 1, 2 and 3

Solution: d)
Justification: We have covered the concept of Options in an earlier test.
Statement 1: The derivative instrument allows investors to enter into contracts to either buy or sell gold sometime
in the future at a pre-determined price, thus allowing investors to hedge any volatility in the price of the metal, for
a price.
Statement 2: Options usually are cheaper than binding future agreements will help in the wider participation of
investors in the realm of commodity speculation.
Statement 3: The introduction of a new financial instrument in India clears a path to building a vibrant market for
commodities.
The commodity options would be taken up initially by hedgers, professionals and high net worth individuals. Later
retail investors can participate if the bidding size is reduced.

335. Consider the following statements. The Multi Commodity Exchange of India Limited (MCX)

1) Is a listed exchange in India


2) operates under the regulatory framework of SEBI
3) does not offer trading in commodity futures contracts

Select the correct answer using the codes below.

a) 1 only
b) 2 and 3 only
c) 1 and 2 only
d) 1, 2 and 3

Solution: c)
Justification: Statement 1: It is India’s first listed exchange, is a state-of-the-art, commodity derivatives exchange
that facilitates online trading.
Statement 2: Most participants in the security market are under the purview of SEBI.
Statement 3: MCX offers trading in varied commodity futures contracts across segments including bullion, industrial
metals, energy and agricultural commodities.
MCX is India’s leading commodity derivatives exchange with a market share of nearly 90 per cent in terms of the
value of commodity futures contracts traded in Q1, FY 2017-18.

336. With reference to the Indian Commodity Exchange Limited (ICEX), consider the following.

1) It is a screen based on-line derivatives exchange for commodities.


2) It is owned completely by the government.
3) It was established under the Securities Contracts (Regulation) Act.

Select the correct answer using the codes below.

a) 1 and 2 only
b) 3 only
c) 1 and 3 only
d) 1, 2 and 3

Solution: c)
Justification: Statement 1: ICEX is deemed recognized Stock exchange providing a nation-wide on- line trading
platform in commodity derivative.
Statement 2: It is a public-private partnership with Reliance Capital, MMTC Ltd, Indiabulls Housing Finance Ltd,
Indian Potash Ltd, KR18HCO and 'DFC Bank as major shareholders.
Statement 3: It was established under the Securities Contracts (Regulation) Act, 1956 in terms of a section of the
Finance Act, 2015 pursuant to central Government notification issued in 2015.

337. A ‘diamond futures contracts’ has been recently designed to provide Indian exporters with a hedging tool. What
do you understand by hedging in this context?

1) To ensure optimum demand conditions of diamonds abroad


2) To protect them from diamond price fluctuations

Which of the above is/arecorrect?

a) 1 only
b) 2 only
c) Both 1 and 2
d) None

Solution: b)
Justification: A hedge is an investment to reduce the risk of adverse price movements in an asset. This is done by
taking a position in the futures market that is opposite to the one in the physical market with the objective of
reducing or limiting risks associated with price changes.
Statement 1: India is a global diamond polishing hub where most rough diamonds in the world are polished. The
futures contracts will enable companies involving in cut and polished gems business to better hedge price risks. The
exact mechanism through which this happens is not important, and need not be covered here.

UNIT-IX
EXTERNAL SECTOR
 INTERNATIONAL TRADE
 BALANCE OF PAYMENTS
 FDI, FII

INTERNATIONAL TRADE

CONCEPTS

338. Which of the following are barriers to the free adjustments in the currency exchange rate in the Forex market?

1) Frequent sterilization by the Central bank


2) Exporting a commodity that is of the comparative advantage of a nation

Which of the above is/are correct?

a) 1 only
b) 2 only
c) Both 1 and 2
d) None

Solution: a)
Justification: Statement 1: Sterilization involves the infusion and extraction of liquidity in the market by the Central
bank to control the total money supply in the economy. This keeps the exchange rate stable and thus blocks its free
adjustment.

Statement 2: A nation usually exports a commodity of its comparative advantage meaning that it specializes in the
export of those commodities that it can produce cheaply or at more competitive rates than the global economy.
This aids trade and does not block the free adjustment in the exchange rates.

339. Which of the following is NOT a valid argument for advocating ‘free trade’ for a developing economy?

a) It results in more efficient allocation of resources internationally.


b) It improves competition and reduces domestic monopoly in goods and services.
c) It allows exporters to reap economies of scale.
d) It creates a level-playing field between developing and developed economies.

Solution: d)
Justification: Since it allows specialization of production, (a) is correct. Option (b) is a natural consequence of more
producers of the same product or service. Since exporters can tap domestic as well as foreign markets, they can
produce more, reduce costs and thus reap economies of scale. So, (c) is also correct. Option (d) is not a valid
argument out of the all four. Globalization along with free trade can adversely affect the economies of developing
countries by not giving equal playing field by imposing conditions which are unfavourable. With the development of
transport and communication systems goods and services can travel faster and farther than ever before. But free
trade should not only let rich countries enter the markets, but allow the developed countries to keep their own
markets protected from foreign products.

340. Which of the following trends characterize government fiscal deficit since the 2008 global slowdown?

a) Steadily increasing with an all-time low during 2009-10 immediately after recession
b) Steadily decreasing with a major spike last year
c) Sharp spike in post-recession years and moderation later
d) Moderation in post-recession years and sharp increase later registering an all-time low last year

Solution: c)
Justification: The sharp spike in post-recession years was due to fiscal stimulus given by the government to recover
the economy from the shock.
341. The Market Stabilization Scheme (MSS) was launched with the objective of strengthening RBI's ability to
conduct exchange rate and monetary management. Under the scheme

a) RBI issues periodic binding circulars to both public and private sector banks that have the implicit approval of the
government
b) All banking institutions share a part of their risk with RBI by hedging risky assets against RBI issued securities
c) RBI is provided with a stock of securities with which it can intervene in the market for managing liquidity
d) Government floats securities jointly with RBI in the domestic as well international markets
Solution: c)
Justification: Historically, the RBI had been sterilizing the effects of significant capital inflows on domestic liquidity
by offloading parts of the stock of Government Securities held by it. The MSS was devised since continuous resort to
sterilization by the RBI depleted its limited stock of Government Securities and impaired the scope for similar
interventions in the future. Under this scheme, the GoI borrows from the RBI (such borrowing being additional to its
normal borrowing requirements) and issues Treasury-Bills/Dated Securities that are utilized for absorbing excess
liquidity from the market. Therefore, the MSS constitutes an arrangement aiding in liquidity absorption, in keeping
with the overall monetary policy stance of the RBI, alongside tools like the Liquidity Adjustment Facility (LAF) and
Open Market Operations (OMO). The securities issued under MSS, termed as Market Stabilization Scheme (MSS)
Securities/Bonds, are issued by way of auctions conducted by the RBI and are done according to a specified ceiling
mutually agreed upon by the GoI and the RBI.

342. Taking which of the following actions would render the system of flexible exchange rate in India dysfunctional
and ineffective?

a) Dominance of Foreign Direct Investment (FDI) over Foreign Institutional Investment (FII)
b) Stopping Current Account Convertibility (CAC)
c) India shedding SDRs from the forex reserve
d) Increasing exports vis-à-vis imports

Solution: b)
Justification: Option B: For a Flexible exchange rate system to work, the inflow and outflow of foreign currency
should be largely uninhibited. When this is severely curtailed, the system can’t work. This is because for the
exchange rate to be determined, there must be free buying and selling of the domestic currency in the international
market. CAC means inflow and outflow of foreign currency for the purpose of trade, remittances etc. is freely
allowed. When CAC is stopped, the buying and selling of currency is stopped, and hence its value determination.
Flexible exchange rate system becomes dysfunctional.

Option A, C and D: In rest of the options, this movement of currency is not stopped, and it continues, hence the
flexible exchange rate system remains intact

343. Consider the following statements.

1) Indian Individuals are allowed to invest in foreign assets and shares.


2) Investment by Limited Indian companies in foreign countries is allowed.
3) Unlimited amount of gold is allowed to be imported from any country in India.

Select the correct answer using the codes below.

a) 1 and 2 only
b) 2 and 3 only
c) 1 and 3 only
d) 1, 2 and 3

Solution: a)
Justification: Statement 1 and 2: Our economy is in the process of allowing full capital account convertibility, and
options 1 and 2 come under capital account. Option 3 comes under current account. For e.g. Indian corporate are
allowed full convertibility in the automatic route up to $ 500 million overseas ventures (Investment by Ltd.
companies in foreign countries allowed). Indian corporate are allowed to prepay their external commercial
borrowings (ECBs) via automatic route if the loan is above $ 500 million. As in 2015, Indian Individuals were allowed
to invest in foreign assets, shares, etc., up to the level of $2, 50,000 per annum.
Statement 3: In recent years, the imports of Gold have tripled from $5 billion to $15 billion. In view of this, the
Government has restricted the import of gold. For e.g. India has restricted all forms of gold imports from South
Korea on the grounds that it is not a gold producing country. However, the commerce department is against the use
of any such measure across the board.
344. Governments maintain a capital account that shows the capital kind of transactions of the economy with the
outside economies. Which of the following form a part of the capital account?

1) External lending or borrowing


2) Interest payments
3) Foreign Direct investment
4) Export and Import of Services

Select the correct answer using the codes below.

a) 1 and 3 only
b) 2 and 4 only
c) 1, 2 and 3 only
d) 1 only

Solution: a)
Justification: Issuing of external bonds, long-term investment abroad or by foreign residents (FDI), FII etc. form part
of the capital account. Capital account shows the net change in asset ownership for a nation. S2 and S4 form part of
the current account. Since large capital inflows or outflows can have destabilizing effects on a nation's economy,
many countries have controls in place to regulate capital account flows.

345. Which of the following characterizes a managed exchange rate?

1) Determined by the IMF based on market movements


2) Value is affected by the Balance of Payments of a nation
3) Central Bank intervenes to manage the value of the currency

Select the correct answer using the codes below.

a) 1 only
b) 2 and 3 only
c) 1 and 3 only
d) 1 and 2 only

Solution: b)
Justification: Exchange rates – They link national currencies for purposes of international trade. There are broadly
two kinds of exchange rates: fixed exchange rate and floating exchange rate
Fixed exchange rates – When exchange rates are fixed and governments intervene to prevent movements in them
Flexible or floating exchange rates – These rates fluctuate depending on demand and supply of currencies in foreign
exchange markets, in principle without interference by governments.
Statement 1: IMF does not peg a currency’s level. Pegging of currencies by a central authority was done earlier in
the Gold system where currencies were fixed in value.
Statement 2: In this system, currencies freely float apart from some occasional interventions. The more a nation’s
currency is in demand, the higher will be its value in forex market.
If a nation follows managed floating system, all its external transactions are based on this system. It cannot fix its
currency level in opposition to the market.

346. Consider the following statements. Technical barriers to trade (TBTs)

1) are a category of nontariff barriers to trade


2) recognizes countries’ rights to adopt standards they consider appropriate for protection of the environment
3) regulates financial assistance given by a developed to a developing country to improve the technical
competitiveness of its good

Select the correct answer using the codes below.

a) 1 only
b) 1 and 2 only
c) 2 and 3 only
d) 1, 2 and 3

Solution: b)
Justification: The TBT exists to ensure that technical regulations, standards, testing, and certification procedures do
not create unnecessary obstacles to trade. The agreement prohibits technical requirements created in order to limit
trade, as opposed to technical requirements created for legitimate purposes such as consumer or environmental
protection. At the same time, it recognizes WTO members' right to implement measures to achieve legitimate policy
objectives, such as the protection of human health and safety, or protection of the environment. Other examples of
TBTs are rules for product weight, size, or packaging; ingredient or identity standards; shelf-life restrictions; and
import testing and certification procedures. A separate agreement on food safety and animal and plant health
standards (the Sanitary and Phytosanitary Measures Agreement or SPS) sets out the basic rule for regulating
international trade based on SPS.

347. A long-term decline in the value of the Rupee reflects

1) A relative decline in global competitiveness of India’s goods and services


2) Sustained low inflation and low demand for goods in the Indian economy

Which of the above is/are correct?

a) 1 only
b) 2 only
c) Both 1 and 2
d) None

Solution: a)
Justification: Statement 1: A consequence of poor competitiveness and high demand for imports is a current
account deficit. This means India is purchasing more imports of goods and services than it is exporting. A large
current account deficit tends to put downward pressure on a currency. This is because more currency is leaving the
country to buy imports than is coming in to buy exports.
Statement 2: Low demand for goods in the economy would actually cut down imports and lead to appreciation of
the rupee. This may, however, also lead to a cut back on exports leading to depreciation and balancing the exchange
rate. But, if this balancing does not happen, it means that imports are rising faster than exports. Also, in particular,
India has a higher inflation rate than its international competitors. In 2013, Indian inflation reached 11.24%.
Therefore, there is relatively less demand for the rising price of Indian goods; this reduction in demand causes a fall
in the value of the Rupee. Inflation has been moderated in recent years, but remains above the comfort level.

348. Consider the following statements. As rupee depreciates

1) Other country residents can import more from India for the same amount of foreign currency.
2) A foreign investor would find it more profitable to invest in India due to the increased value of his investment
budget in foreign currency

Which of the above is/are correct?


a) 1 only
b) 2 only
c) Both 1 and 2
d) None

Solution: c)
Justification: Statement 1: Suppose 1 Kg. of Rice was Rs. 120 in India, and the Dollar-Rupee conversion rate is 60:1,
a Us citizen can import 1 Kg of rice from India for $2. If rupee depreciates to the rate of 120:1, the same US resident
can now buy 2 kg of rice for the same $2.
Statement 2: Same applies for the foreign investor. If he was willing to invest $1 million which would have given him
60 million rupees of worth, after rupee depreciation, his value of investment in India would be 120 million rupees.

349. Which of the following forms a part of the capital account of India?

1) Remittances received from NRIs living in Saudi Arabia


2) A loan given to Myanmar
3) Foreign exchange earned from trade in Invisibles

Select the correct answer using the codes below.

a) 1 and 3 only
b) 2 only
c) 1, 2 and 3 only
d) 1 only

Solution: b)
Justification: Capital account shows the net change in asset ownership for a nation.
Statement 2: Issuing of external bonds, long-term investment abroad or by foreign residents (FDI), or loans given to
foreign countries FII etc. form part of the capital account.
Statement 1 and 3: Trade of any sort, Interest payments and remittances form part of the current account.
Since large capital inflows or outflows can have destabilizing effects on a nation's economy, many countries have
controls in place to regulate capital account flows.

350. Real Effective Exchange Rate (REER) is said to be a better indicator of the competitiveness of the country in
terms of exchange rates. This is because it takes into account

1) Relative price differential between the domestic and foreign countries


2) Relative rate of Inflation in the domestic country as compared to foreign countries

Which of the above is/are correct?

a) 1 only
b) 2 only
c) Both 1 and 2
d) None

Solution: c)
Justification: Real effective exchange rate is defined as “a weighted average of nominal exchange rates adjusted for
relative price differential between the domestic and foreign countries, relates to the purchasing power parity (PPP)
hypothesis”. As the definition highlights, REER takes price differential and inflation into account and, therefore, is
said to be a better indicator of the competitiveness of the country in terms of exchange rates. In India, Reserve Bank
of India (RBI) complies with REER indices. The first one is based on six country’s trade-based weights and the second
on 36-currencies’ export and trade-based weights. The indices are also a better reflection of the position of a
currency in comparison with the countries in which India has large export and trade interest. The base is taken as
100 and currently the base year is 2004-2005. A rise in the level of index indicates appreciation of currency and vice-
versa. Even though rising and falling currency means different things to different people, economists insist that
people should not look at the movement of nominal rates and focus on real effective exchange rate (REER).

351. Under ‘Import substitution’

1) Government protects the domestic industries from foreign competition


2) Exports are discouraged since they reduce domestic consumption and thus production.

Which of the above is/are correct?

a) 1 only
b) 2 only
c) Both 1 and 2
d) None

Solution: a)
Justification: This policy is aimed at replacing or substituting imports with domestic production.
Statement 1: For e.g. in India, the government protected the domestic industries from foreign competition, so that
the same goods that are now imported can be produced domestically.
Protection from imports can take two forms: tariffs and quotas.
Statement 2: The best example would be ‘Make In India’ rather than ‘Import to India’. Firms that used to export
goods and services like mobile phones would manufacture in India itself after FDI. This would reduce imports and
build domestic manufacturing base.
Export based industries are meant to generate an export surplus in the goods they export (e.g. Gem and
Jewelleries). If such industries are subsidized, they will not substitute imports from abroad (as they don’t
manufacture those goods), and would actually boost exports

352. Over the last decade, the country that is the World’s largest receiver of remittances is

a) USA
b) India
c) Russia
d) Canada

Solution: b)
Justification: According to UN International Fund for Agricultural Development (IFAD), Indians working across the
world had sent home USD 62.7 billion in 2016 making India the top remittance receiving country surpassing China.
The study is the first ever study conducted that analysed a 10-year trend in migration and remittance flows over the
period 2007-2016.
In the period between 2007 and 2016, India has surpassed China to become the world’s largest receiver of
remittances. Asia remains the main remittance-receiving region with 55% of the global flows. This increase in
remittances is expected to lift millions out of poverty and in attaining the Sustainable Development Goals (SDGs).
Top Remittance Receiving Countries: India (USD 62.7 billion), China (USD 61 billion), the Philippines (USD 30 billion)
and Pakistan (USD 20 billion).

353. What do you understand by ‘protectionism’, a term often seen in newspapers?

a) Imposing trade restrictions to protect domestic industry


b) Entering into alliances with foreign militaries to safeguard one’s borders
c) Mandating military conscription for better defence capabilities
d) Protecting one’s citizen from influence of foreign cultures

Solution: a)
Justification: In economics, protectionism is the economic policy of restraining trade between states (countries)
through methods such as tariffs on imported goods, restrictive quotas, and a variety of other government
regulations. Protectionist policies protect the producers, businesses, and workers of the import-competing sector in
a country from foreign competitors. However, they hurt consumers in general, and the producers and workers in
export sectors, both in the country implementing protectionist policies, and in the countries protected against.

WTO

354. Some nations have been pushing for talks to end barriers to international e-commerce via WTO negotiations.
What is India’s position in this regard?

a) India supports further negotiations beyond the 1998 Work programme on E-commerce of the WTO.
b) India is against any negotiation on opening cross border digital trade.
c) India supports the principle of zero tariffs on international e-commerce trade.
d) India has not officially stated any position with regard to such an issue.

Solution: b)
Justification: India has maintained that e-commerce per se may be good for development but it may not be prudent
to begin talks since many countries don’t fully understand the implications of negotiating binding rules. India has for
the first time submitted a formal document opposing any negotiations on e-commerce at the WTO. The country has
said that it would “continue the work under the Work Programme on electronic commerce... based on the existing
mandate and guidelines”, referring to the programme on e- commerce adopted by the WTO countries in 1998. E-
commerce entered the WTO in 1998, when member countries agreed not to impose customs duties on electronic
transmissions, and the moratorium has been extended periodically. However, since last year, many countries have
made submissions on various aspects of digital trade such as cross-border data flows, server localization, technology
transfer, source code, consumer protection, intellectual property rights and trade facilitation aspects of e-
commerce. While the United States last year floated a proposal on e-commerce prohibiting digital customs duties,
enabling cross-border data flows, promoting a free and open internet and preventing localization barriers, China
wants easier norms for goods ordered over the internet but physically delivered. The European Union, Japan, Korea,
Pakistan, Nigeria and Singapore too want outcomes in e- commerce disciplines. Intense efforts are now being made
by some countries to secure a mandate to initiate comprehensive negotiations on e-commerce.

355. Under the WTO agreements, countries cannot normally discriminate between their trading partners – a
treatment which is known as “Most favoured Nation (MFN)” status. However some discrimination is allowed when

1) Countries can set up a free trade agreement that applies only to goods traded within the group
2) They can give developing countries special access to their markets
3) A country can raise barriers against products that are considered to be traded unfairly from specific countries

Select the correct answer using the codes below.

a) 1 and 2 only
b) 2 and 3 only
c) 1 and 3 only
d) 1, 2 and 3

Solution: d)
Justification: Exceptions to MFN principle: For example, countries can set up a free trade agreement that applies
only to goods traded within the group — discriminating against goods from outside, for e.g. SAFTA or TPP. Or they
can give developing countries special access to their markets, for e.g. agricultural markets. Or a country can raise
barriers against products that are considered to be traded unfairly from specific countries, for e.g. guns and
ammunition. And in services, countries are allowed, in limited circumstances, to discriminate, as under TRIMS. But
the agreements only permit these exceptions under strict conditions. In general, MFN means that every time a
country lowers a trade barrier or opens up a market, it has to do so for the same goods or services from all its
trading partners — whether rich or poor, weak or strong.

356. Special safeguard mechanism (SSM) as under WTO relates to

a) Most Favoured Nation treatment dispute clause


b) WTO’s financial assistance to farmers that is not linked to prices or production
c) Licensing companies or individuals other than the patent owner to use the rights of the patent
d) Allowing developing countries to raise tariffs temporarily to deal with import surges or price falls

Solution: d)
Justification: WTO’s Special Safeguard Mechanism (SSM) is a protection measure allowed for developing countries
to take contingency restrictions against agricultural imports that are causing injuries to domestic farmers. The
contingency measure is imposition of tariff if the import surge causes welfare loss to the domestic poor farmers.
The design and use of the SSM is an area of conflict under the WTO. At the Doha Ministerial Conference, the
developing countries were given a concession to adopt a Special Safeguard Mechanism (SSM) besides the existing
safeguards (like the Special Agricultural Safeguard or the SSG). This SSM constituted an important part of the
promises offered to the developing world at Doha (known as Doha Development Agenda) and the Doha MC became
known as a development round. The SSG was available to all countries- both developing and developed whereas the
SSM is allowable only to the developing countries.

357. There is a growing consciousness of climate change in trade and commerce. In the context, consider the
following statements.
Assertion (A): There are no WTO rules specific to climate change.
Reason (R): Trade is not related to carbon emissions.

In the context of the above, which of these is correct?

a) A is correct, and R is an appropriate explanation of A.


b) A is correct, but R is not an appropriate explanation of A.
c) A is correct, but R is incorrect.
d) Both A and R are incorrect.

Solution: c)
Justification: As per WTO’s website, “The issue of climate change, per se, is not part of the WTO's ongoing work
programme and there are no WTO rules specific to climate change. However, the WTO is relevant because climate
change measures and policies intersect with international trade in a number of different ways.” However, the WTO
offers a powerful supporting framework for sustainable development and green economy. It provides an enabling
environment through its objectives, institutions and monitoring of potential trade protectionism, enforcement
mechanism, toolbox of rules, and growing case law in the environment area.

358. The ‘Peace Clause’, often seen in news is related to which of following issues under discussion World Trade
Organization (WTO)?

a) Food security of developing nations


b) Economic sanctions on Iran by P5 1 nations
c) U.S. withdrawal from Trans-Pacific Partnership (TPP)
d) Protection of Island nations from onslaught of Climate change related trade policies

Solution: a)
Justification: The issue has been discussed several times at our current affairs sections and editorials. At the
upcoming WTO’s Ministerial Conference, India has decided not to agree to severe restrictions on its right to give
price subsidies to farmers through the Minimum Support Price (MSP) to procure grains from them for food security
purposes. Food security and protection of low-income and resource-poor farmers are top priority items for India at
the WTO meet. Currently, an interim mechanism called the ‘Peace Clause’ is in place, as per which WTO members
had agreed not to challenge developing nations at the WTO Dispute Settlement Mechanism if they breached the
cap of the product-specific domestic support (which is 10% of the value of production). The ‘Peace Clause’ is
available to developing nations, including India, till a permanent solution is found to public stockholding for food
security purposes.

359. With reference to the Generalized System of Preferences (GSP), consider the following statements.

1) It is a system of exemption from the most favored nation principle (MFN) of WTO.
2) It is a provision in UNCTAD to digitize the trading system for improved access and efficiency.

Which of the above is/are correct?

a) 1 only
b) 2 only
c) Both 1 and 2
d) None

Solution: a)
Justification: GSP is a preferential tariff system which provides for a formal system of exemption from the more
general rules of the WTO (formerly, the General Agreement on Tariffs and Trade or GATT). Specifically, it's a system
of exemption from the most favored nation principle (MFN) that obliges WTO member countries to treat the
imports of all other WTO member countries no worse than they treat the imports of their "most favored" trading
partner. In essence, MFN requires WTO member countries to treat imports coming from all other WTO member
countries equally, that is, by imposing equal tariffs on them, etc. GSP exempts WTO member countries from MFN
for the purpose of lowering tariffs for the least developed countries, without also lowering tariffs for rich countries.

360. Trade-Related Investment Measures (TRIMs) are rules that

1) Restrict preference of domestic firms vis-à-vis foreign firms


2) Mandate compensating investment in LDCs by developed countries for sustainable development

Which of the above is/are correct?

a) 1 only
b) 2 only
c) Both 1 and 2
d) None

Solution: a)
Justification: TRIMs are rules that restrict preference of domestic firms and thereby enable international firms to
operate more easily within foreign markets. Policies such as local content requirements and trade balancing rules
that have traditionally been used to both promote the interests of domestic industries and combat restrictive
business practices are now banned. The agreement was agreed upon by all members of the World Trade
Organization. Trade-Related Investment Measures is one of the four principal legal agreements of the WTO trade
treaty
361. The Generalized System of Preferences (GSP) is heard in the context of

a) World Bank
b) United Nations Environment Programme
c) World Trade Organization
d) International Monetary Fund

Solution: c)
Justification: It is a preferential tariff system which provides for a formal system of exemption from the more
general rules of the World Trade Organization (WTO). Specifically, it is a system of exemptions from the most
favoured nation principle (MFN) that obliges WTO member countries to treat the imports of all other WTO member
countries no worse than they treat the imports of their "most favoured" trading partner. In essence, MFN requires
WTO member countries to treat imports coming from all other WTO member countries equally, that is, by imposing
equal tariffs on them. GSP exempts WTO member countries from MFN for the purpose of lowering tariffs for the
least developed countries, without also lowering tariffs for rich countries. For e.g. the European Union (EU) has
amended its Generalized System of Preferences (GSP) Scheme by introducing the concept of self-certification from
2017 onwards for which the office of the DGFT has taken out Public Notice providing the details of the
implementation of the EU GSP self-certification scheme in India.

362. With reference to WTO, where are the Rules of Origin (RO) used?

1) To implement measures and instruments of commercial policy such as anti-dumping duties and safeguard measures
2) To determine whether imported products shall receive most-favoured-nation (MFN) treatment or preferential
treatment;
3) For the purpose of determining government procurement

Select the correct answer using the codes below.

a) 1 only
b) 2 and 3 only
c) 1 and 3 only
d) 1, 2 and 3

Solution: d)
Justification: Determining where a product comes from is no longer easy when raw materials and parts crisscross
the globe to be used as inputs in scattered manufacturing plants. Rules of origin are therefore needed to attribute
one country of origin to each product. They are the criteria used to define where a product was made and are
important for implementing other trade policy measures, including trade preferences, quotas, anti-dumping
measures and countervailing duties. They are also used for the purpose of trade statistics and for the application of
labelling and marking requirements.

363. The Information Technology Agreement (ITA) of the WTO intends to

a) Completely eliminate tariffs on IT products covered by the Agreement


b) Increase the share of IT based goods vis-à-vis non-IT goods in international trade
c) Give developing countries special concessions in the development of IT industry
d) Use IT to facilitate trade smoothly across borders

Solution: a)
Justification: Entered into force in 1997, it is being expanded ever since and is often seen in news. The agreement
was expanded in 2015. Since 1997 a formal Committee under the WTO watches over the following of the
Declaration and its Implementations. The aim of the treaty is to lower all taxes and tariffs on information technology
products by signatories to zero. According to a 2017 study in the World Trade Review, the 2015 ITA expansion is
"the most successful attempt at trade liberalization under the auspices of the WTO since its inception in 1995." For
e.g. India’s plans to impose import duties on mobile phones under the Goods and Services Tax (GST) regime created
a flutter among manufacturing countries, including Japan, Korea, the US and the EU, at the WTO and led to protests.
The EU said India was bound by a commitment of zero per cent duty under the ITA (part of General Agreement on
Tariffs & Trade), and its argument that mobile phones can be considered as “new products” not covered by the pact
was invalid. However, India stressed that IT and telecom technologies have evolved with new applications and
equipment that did not exist, and had not even been conceived, at the time of signing the ITA. Therefore, the new
products cannot be considered as falling under the scope of the agreement as they did not exist at the time of
granting concessions

364. In the context of International trade, ‘Aggregate Measure of Support’ is often heard concerning which of the
following sectors?

a) Banking
b) Information Technology
c) Agriculture
d) Manufacturing

Solution: c)
Justification: Aggregate measurement of support (AMS) is the indicator on which the domestic support discipline
for the Uruguay Round Agreement on Agriculture is based. It is determined by calculating a market price support
estimate for each commodity receiving such support, plus non-exempt direct payments or any other subsidy not
exempted from reduction commitments, less specific agricultural levies or fees paid by producers. The main
conceptual consideration is that there are basically two categories of domestic support — support with no, or
minimal, distortive effect on trade on the one hand (often referred to as “Green Box” measures) and trade-
distorting support on the other hand (often referred to as “Amber Box” measures). For example, government
provided agricultural research or training is considered to be of the former type, while government buying-in at a
guaranteed price (“market price support”) falls into the latter category. Under the Agreement on Agriculture, all
domestic support in favour of agricultural producers is subject to rules. In addition, the aggregate monetary value of
Amber Box measures is, with certain exceptions, subject to reduction commitments as specified in the schedule of
each WTO Member providing such support.
365. I-TIP portal of WTO provides information on

a) tariff and non-tariff measures (NTMs) applied by WTO members in merchandise trade
b) the progress of trade in Least Developed Countries (LDCs)
c) intellectual property violation in international trade
d) the list of major and minor custom ports around the world

Solution: a)
Justification: The Integrated Trade Intelligence Portal (I-TIP) provides a single entry point for information compiled
by the WTO on trade policy measures. Containing information on over 25,000 measures. I-TIP covers both tariff and
non-tariff measures affecting trade in goods as well as information on trade in services, trade in government
procurement markets, regional trade agreements and the accession commitments of WTO members. Its aim is to
serve the needs of those seeking detailed information on trade policy measures as well as those looking for
summary information. For e.g. Trade defence measures, technical measures and their Specific Trade Concerns
(STCs), quantitative restrictions, the NTMs from the Agreement on Agriculture and information on state trading
enterprises are covered in I-TIP.
366. Special safeguard mechanism (SSM) as under WTO relates to

a) Governments recognizing other countries’ measures as acceptable even if they are different from their own, so long
as an equivalent level of protection is provided
b) Support for farmers that is not linked to prices or production
c) Licensing companies or individuals other than the patent owner to use the rights of the patent
d) Allowing developing countries to raise tariffs temporarily to deal with import surges or price falls

Solution: d)
Justification: WTO’s Special Safeguard Mechanism (SSM) is a protection measure allowed for developing countries
to take contingency restrictions against agricultural imports that are causing injuries to domestic farmers. The
contingency measure is imposition of tariff if the import surge causes welfare loss to the domestic poor farmers.
The design and use of the SSM is an area of conflict under the WTO. At the Doha Ministerial Conference, the
developing countries were given a concession to adopt a Special Safeguard Mechanism (SSM) besides the existing
safeguards (like the Special Agricultural Safeguard or the SSG). This SSM constituted an important part of the
promises offered to the developing world at Doha (known as Doha Development Agenda) and the Doha MC became
known as a development round. The SSG was available to all countries- both developing and developed whereas the
SSM is allowable only to the developing countries.

367. Under WTO rules, a member country cannot discriminate between its trade partners. Hence

1) A country cannot enter into a free trade agreement with another country without the authorization of WTO
2) If a special status is granted to a trade partner, it should usually be extended to all members of the WTO.

Which of the above is/are correct?

a) 1 only
b) 2 only
c) Both 1 and 2
d) None

Solution: b)
Justification: Statement 1: Countries don’t require the permission of WTO for their trade related activities.
However, they can move WTO if a country violates the rules of trade set by WTO. They are free to enter into FTAs
with other countries.

Statement 2: This is called as MFN status. As per a WTO rule, every member of WTO requires to accord this status to
other member countries. India has already granted this status to all WTO members including Pakistan. Pakistan is
yet to reciprocate. Most Favoured Nation is a treatment accorded to a trade partner to ensure non- discriminatory
trade between two countries vis-a-vis other trade partners. MFN, however, allows for countries to enter into
regional trade agreements. It also relaxes such requirements in case of LDCs and other developing countries.

INDIA’S FOREIGN TRADE POLICY&SCHEMES

368. The incentives and facilities offered to the units in SEZs for attracting investments into the SEZs, including
foreign investment include

1) Duty free import/domestic procurement of goods for development, operation and maintenance of SEZ units
2) Single window clearance for Central and State level approvals
3) 100% Income Tax exemption on export income for SEZ units under certain conditions

Select the correct answer using the codes below.


a) 1 only
b) 2 and 3 only
c) 1 and 2 only
d) 1, 2 and 3

Solution: d)
Justification: This was the status before the implementation of GST: Cent per cent Income Tax exemption on export
income for SEZ units under Section 10AA of the Income Tax Act for first 5 years, 50% for next 5 years thereafter and
50% of the ploughed back export profit for next 5 years. Exemption from minimum alternate tax under section
115JB of the Income Tax Act. External commercial borrowing by SEZ units up to US $ 500 million in a year without
any maturity restriction through recognized banking channels.

 Exemption from Central Sales Tax.


 Exemption from Service Tax.
 Single window clearance for Central and State level approvals.
 Exemption from State sales tax and other levies as extended by the respective State Governments.

369. In Special Economic Zones (SEZs) business and trades laws differ from the rest of the country. These relaxations
are provided in order to

1) Make SEZs Export Processing Zones thereby promoting exports


2) Establish industrial townships in SEZs

Which of the above is/are correct?

a) 1 only
b) 2 only
c) Both 1 and 2
d) None

Solution: a)
Justification: The term special economic zone (SEZ) is commonly used as a generic term to refer to any modern
economic zone. The relaxed policies typically regard investing, taxation, trading, quotas, customs and labour
regulations. Additionally, companies may be offered tax holidays. The aims of the zones include: increased trade,
increased investment, job creation and effective administration. To encourage businesses to set up in the zone,
financially libertarian policies are introduced.

370. Mutual Agreement Procedure (MAP) was recently seen in news with reference to

a) Double taxation
b) Safeguarding Marine Biosphere Reserve (MBP)
c) Smart Cities
d) Cyber Security of critical systems

Solution: a)
Justification: MAP is an alternative available to taxpayers to resolve disputes giving rise to double taxation, whether
juridical or economic in nature. An agreement for avoidance of double taxation between countries would give
authorization for assistance of Competent Authorities (CAs) in the respective jurisdiction under MAP. The main
benefit of pursuing MAP is the elimination of double taxation (either juridical or economic). The MAP resolution,
once accepted, eliminates protracted litigation.

371. How is an Early Harvest Scheme/Programme (EHS) different from a Free Trade Agreement (FTA)?

a) EHS is a Trade facilitation agreement (TFA) to ease the administrative implementation of an FTA.
b) EHS supplements an FTA by extending its scope to encompass a Comprehensive Economic Partnership Agreement
(CEPA).
c) EHS is restricted to terms and conditions of the FTA related to agricultural commodities.
d) EHS is a precursor to a free trade agreement (FTA) between two trading partners.

Solution: d)
Justification: Early harvest scheme is a precursor to a free trade agreement (FTA) between two trading partners.
This is to help the two trading countries to identify certain products for tariff liberalization pending the conclusion of
FTA negotiation. It is primarily a confidence building measure. A good example of an EHS is between India and
Thailand signed in October 2003, wherein 83 products were identified to be reduced to zero in a phased manner.
The EHS has been used as a mechanism to build greater confidence amongst trading partners to prepare them for
even bigger economic engagement.

372. With reference to the Trade Promotion Council of India, consider the following statements.

1) It is recognized and supported by the Department of Commerce.


2) It supports India’s trade policy and economic diplomacy through research.
3) Members of TPCI get automated funding for business expansion in India and abroad.

Select the correct answer using the codes below.

a) 1 and 2 only
b) 2 and 3 only
c) 1 and 3 only
d) 1, 2 and 3

Solution: a)
Justification: Statement 1 and 2: TPCI is an apex trade and investment promotion organization notified in the
Foreign Trade Policy. TPCI is also recognized and supported by the Department of Commerce, Govt. of India. We
work towards facilitating the growth of Indian industry with global investment & trade opportunities. The council
provides strategies for expanding business internationally, by organizing specialized business events and
simultaneously working with the Government by providing policy suggestions which are essentially based on inputs
collated from research and industry stakeholders.
Statement 3: Services to Ordinary Members Include Close interaction with TPCI’s members, international agencies,
diplomatic missions in India and Indian missions abroad, academicians and technical experts. Participation in various
trade specific and general business events and gain professional knowledge through seminars, workshops and
round table conferences. Services to Patron members (apart from the above) include:
 Nomination of members on Committees & Advisory Forums constituted by the government of different countries.
 Priority consideration for participation in high level delegations visiting India and those visiting other countries and
meeting senior government officials.
 Patron members are eligible to avail TPCI Business Services on demand.

373. International Transport and Transit Corridor is a regional transport network initiative of the governments of

1) USA
2) Saudi Arabia
3) China
4) India
5) Iran

Select the correct answer using the codes below.

a) 2 and 4 only
b) 1, 2, 3 and 4 only
c) 4 and 5 only
d) 1, 3 and 5 only
Solution: c)
Justification: Also known as the “Chabahar Agreement”, it is a regional transport network initiative of the
governments of Afghanistan, India, and Iran. Aim is to create a reliable transport corridor for the smooth transport
and transit of goods and services through Chabahar Port between Afghanistan, India and Iran. This will also
synchronize with the Ashgabat agreement, a Multimodal transport agreement signed by India, Pakistan, Oman, Iran,
Turkmenistan, Uzbekistan and Kazakhstan, for creating an international transport and transit corridor facilitating
transportation of goods between Central Asia and the Persian Gulf.

374. Assist Focus export promotion programmes have been launched in specific regions abroad by the Government.
It includes

1) Africa
2) ASEAN
3) Latin American countries
4) Commonwealth of Independent States
Select the correct answer using the codes below.

a) 1, 2 and 3 only
b) 2 and 4 only
c) 1 and 3 only
d) 1, 2, 3 and 4

Solution: d)
Justification: To stimulate and diversify the country's export trade, Marketing Development Assistance (MDA)
Scheme is under operation through the Department of Commerce to support the under mentioned activities:

 Assist exporters for export promotion activities abroad


 Assist Export Promotion Councils(EPCs) to undertake export promotion activities for their product(s) and
commodities
 Assist approved organization/trade bodies in undertaking exclusive nonrecurring innovative activities
connected with export promotion efforts for their members
 Assist Focus export promotion programmes in specific regions abroad like FOCUS (LAC), Focus (Africa), Focus
(CIS) and Focus (ASEAN 2) programmes
 Residual essential activities connected with marketing promotion efforts abroad

375. The South Asia Growth Quadrangle (SAGQ) constitutes which of these countries?

1) Nepal
2) Bangladesh
3) Sri Lanka
4) India
5) Afghanistan
Select the correct answer using the codes below.

a) 1, 2 and 4 only
b) 3 and 4 only
c) 1, 2 and 3 only
d) 1, 2, 3, 4 and 5

Solution: a)
Justification: SAGQ was launched in 1997 by the Foreign Ministers of Bangladesh, Bhutan, India, and Nepal
(BBIN). SAARC in 1997 endorsed SAGQ as a sub-regional initiative under SAARC. The goals of SAGQ are:

 to create an enabling environment for accelerating economic growth;


 to overcome infrastructural constraints;
 to make optimal use of and further develop the complementarities in the subregion
 to improve policy coordination

The South Asia Subregional Economic Cooperation (SASEC) Program, set up in 2001, brings together
Bangladesh, Bhutan, India, Maldives, Myanmar, Nepal and Sri Lanka in a project- based partnership to
promote regional prosperity by improving cross-border connectivity, boosting trade among member countries,
and strengthening regional economic cooperation.

376. What is/are the difference(s) between Preferential Trade Agreement (PTA) and Free Trade Agreement (FTA)?

1) While in a PTA there is a positive list of products on which duty is to be reduced; in an FTA there is a negative list on
which duty is not reduced or eliminated making the latter more comprehensive.
2) FTAs consist of an integrated package on goods, services and investment along with other areas such as IPR and
competition; a feature not found in most PTAs.

Which of the above is/are correct?

a) 1 only
b) 2 only
c) Both 1 and 2
d) None

Solution: a)
Justification: Preferential Trade Agreement (PTA): In a PTA, two or more partners agree to reduce tariffs on agreed
number of tariff lines. The list of products on which the partners agree to reduce duty is called positive list. India
MERCOSUR PTA is such an example. However, in general PTAs do not cover substantially all trade.
Free Trade Agreement (FTA): In FTAs, tariffs on items covering substantial bilateral trade are eliminated between
the partner countries; however each maintains individual tariff structure for non-members. India Sri Lanka FTA is an
example.
Statement 1: The key difference between an FTA and a PTA is that while in a PTA there is a positive list of products
on which duty is to be reduced; in an FTA there is a negative list on which duty is not reduced or eliminated. Thus,
compared to a PTA, FTAs are generally more ambitious in coverage of tariff lines (products) on which duty is to be
reduced.
Statement 2: Such extensive coverage is done in a Comprehensive Economic Cooperation Agreement (CECA) and
Comprehensive Economic Partnership Agreement (CEPA), not in FTAs or PTAs.
CECA and CAPC describe agreements which consist of an integrated package on goods, services and investment
along with other areas including IPR, competition etc. The India Korea CEPA is one such example and it covers a
broad range of other areas like trade facilitation and customs cooperation, investment, competition, IPR etc.

INDIA’s TRADE

377. Without the Rules of Origin (ROO), it will be difficult to

1) Decide whether imported products shall receive preferential treatment for tariffs
2) Implement anti-dumping duties and safeguard measures with regards to certain products

Which of the above is/are correct?

a) 1 only
b) 2 only
c) Both 1 and 2
d) None
Solution: c)
Justification: Rules of Origin (ROO) are the criteria needed to determine the identity of a product for purposes of
international trade. Their importance is derived from the fact that duties and restrictions in several cases depend
upon the source of imports. Rules of origin are used:
 to implement measures and instruments of commercial policy such as antidumping duties and safeguard measures;
 to determine whether imported products shall receive most-favoured-nation (MFN) treatment or preferential
treatment;
 for the purpose of trade statistics;
 for the application of labelling and marking requirements; and
 for government procurement.
The Rules of Origin are important in the context of making an assessment on the application of preferential tariff
under an FTA. Hence, without the rules of origin, the preferential tariffs under an FTA cannot be implemented.
Moreover, the non-members to the FTA are not provided with the benefit of the preferential tariffs, agreed
between the FTA partners.

378. Which of the following can be a form of economic “Protectionism” by India?

1) Entry restrictions for foreign nationals


2) Preferential market access policies for domestic industries
3) Increasing custom duties on imported goods and services

Select the correct answer using the codes below.

a) 2 only
b) 2 and 3 only
c) 1 and 3 only
d) 1, 2 and 3

Solution: d)
Justification: Statement 1: Protectionism can be any attempt to protect domestic industries from global
competition. Restricting movements of people (labour), goods, services will all be considered under economic
protectionism.
Statement 2: Preference to domestic industries creates barriers for other MNCs that desire investing in India.
Statement 3: This renders Indian goods to be more competitive than goods imported from abroad.

379. India’s trade volume (in terms of market value) with SAARC neighbours is roughly what percentage of India’s
global trade?

a) 2-3%
b) 15-20%
c) 33%
d) 45%

Solution: a)
Justification: India's global trade was worth an estimated $800 billion in 2012-13, of which only about $17 billion
was with the South Asian Association for Regional Cooperation (SAARC) partners. SAARC includes Bangladesh,
Bhutan, India, the Maldives, Nepal, Pakistan, Sri Lanka and Afghanistan. The reason is that a) these are competing
economies, and b) the FTAs and bilateral negotiations have not been settled properly due to several disagreements.
Most of India’s foreign trade is carried through sea and air routes. However, a small portion is also carried through
land route to neighbouring countries like Nepal, Bhutan, Bangladesh and Pakistan

380. Which of the following modes of supply as considered by India as international ‘trade’ in services?

1) A Non-Resident Indian (NRI) Chartered Accountant giving paid services in USA


2) An Indian teacher sending paid material to students in other countries
3) An Indian bank opening its branch in another country

Select the correct answer using the codes below.

a) 1 only
b) 1 and 2 only
c) 1, 2 and 3
d) 2 and 3 only

Solution: c)
Justification: These are the four modes of supply under trade in services, as defined under General Agreement on
Trade in Services (WTO).
Mode 1: Cross border supply (supply from the territory of a Party into the territory of the other Party). For example
an architect can send his architectural plan through electronic means; a teacher can send teaching material to
students in any other country; a doctor sitting in Germany can advise his patient in India through electronic means.
In all these cases, trade in services takes place and this is equivalent to cross-border movement of goods.
Mode 2: Consumption abroad (consumption in the territory of a Party by the service consumer of the other Party).
For example a tourist using hotel or restaurant services abroad; a ship or aircraft undergoing repair or maintenance
services abroad.
Mode 3: Commercial presence (by a service supplier of a Party, through commercial presence in the territory of the
other Party). In this case, the service supplier establishes a legal presence in the form of a joint venture/
subsidiary/representative/branch office in the host country and starts supplying services. For example a bank opens
its branch in another country.
Mode 4: Presence/movement of natural persons (by a service supplier of a Party, through presence of natural
persons of a Party in the territory of the other Party). For example Independent service suppliers or ISS (e.g. doctors,
engineers, individual consultants, accountants, etc.) who supply services in another country.
However, GATS covers only temporary movement and not citizenship, residence or employment on a permanent
basis in the foreign country.

381. Which of the following factors have recently caused ballooning of India’s trade deficit with China?

1) Pharmaceutical sector exports from India faces difficulties in getting access to the Chinese market
2) China’s ‘Sanitary and Phytosanitary measures’ is hurting Indian farm products exports
3) High anti-dumping duty imposed by India, on Chinese imports, to safeguard domestic industries balloons India’s
trade deficit

Select the correct answer using the codes below.

a) 1 and 2 only
b) 2 only
c) 1 and 3 only
d) 2 and 3 only

Solution: a)
Justification: The bilateral trade between the countries stood at around USD 71.42 billion in 2016-17. India's trade
deficit with China stood at USD 36.73 billion during the April-October period of the FY 2017-18. The deficit had
marginally dipped to USD 51 billion in 2016-17 from USD 52.69 billion in the FY 2016-17. India has raised the
concerns of ballooning trade deficit due to the growing difference between imports and exports. China has agreed
to take concrete steps such as giving more market access to Indian products and services, and setting up industrial
parks here to narrow the widening trade deficit
Statement 2: It has agreed to address India’s concerns regarding China’s ‘Sanitary and Phytosanitary measures’
(regarding ‘application of food safety, and animal and plant health norms’) hurting Indian farm products exports to
that country.
Statement 1: Chinese authorities have also decided to look into India’s request to remove the ‘curbs’ on Indian
pharmaceutical companies/products — especially those having the approval of American, European and Japanese
regulators. Besides, Beijing would also soon take a call on removing the difficulties faced by the Indian IT/business
process management sector in getting greater market access in China.
Statement 3: Anti-dumping duty is in force on 93 products concerning imports from China, covering products in
broad groups of chemicals and petrochemicals, products of steel and other metals, fibres and yarn, machinery
items. But, these actually help cut down trade deficit due to all these imports. So, 3 is wrong.

382. India has signed Preferential Trade Agreements (PTAs) or Free Trade Agreements (FTAs) with which of the
following trade blocs?

1) MERCOSUR
2) ASEAN
3) European Union (EU)

Select the correct answer using the codes below.

a) 2 only
b) 1 and 2 only
c) 1 and 3 only
d) 2 and 3 only

Solution: b)
Justification: Statement 1: India has signed a PTA with MERCOSUR since 2004. It is presently limited to just 450
products. PTA is a limited free trade agreement where partner countries reduce import duties on a few identified
products for the other.
Statement 2: This was signed in 2009. Here is a list of all major FTA/PTA entered by India.
Statement 3: This is still under negotiations due to several sticking issues such as automobile imports, patent and IP
protection etc.

383. Consider the following with reference to India’s trade composition.

1) Manufactured goods constitute the bulk of exports in terms of value.


2) Petroleum, Oil and Lubricant (POL) Imports constitute more than fifty per cent of imports in terms of value.
3) United States is the major source of India’s imports.
4) China is the largest export partner of India.

Select the correct answer using the codes below.

a) 1, 2 and 4 only
b) 2 and 3 only
c) 1 only
d) 1, 3 and 4

Solution: c)
Justification: Statement 1: Manufactured goods constitute the bulk of exports—over 63 per cent in recent years,
followed by crude and petroleum products (including coal) with a 20 per cent share, and agriculture and allied
products with a share of 13.7 per cent share.
Statement 2: One of the major items in India's import basket is the POL group, which accounted for 36.6 per cent of
India's total imports. POL imports surged with a growth of 46.2 per cent in 2011—12, mainly on account of
significant increase in global crude oil prices. Capital goods imports are another major group which declined
continuously from 2011—12 onwards. Within capital goods, imports of machinery registered positive growth in
2014—15. Gold and silver imports accounted for 11.4 per cent of India's total imports in 2012—13 and 7.4 per cent
in 2013—14.
Statement 3: The following table shows India's 11 largest sources of imports:

384. Since 2012-13, which of the following sectors have been experiencing trade deficit in the external sector?

1) Manufacturing
2) Software plus Non-Factor Service Balance
3) Oil and Gold

Select the correct answer using the codes below


a) 1 and 2 only
b) 2 and 3 only
c) 1 and 3 only
d) 3 only

Solution: c)
Justification: The current account deficit can be split into a manufacturing trade deficit, an oil and gold deficit, a
services deficit, and a remittances deficit (Figure 25). In the first half of 2017-18, the oil and gold balance has
improved (smaller deficit of $47 billion) but this has been offset by a higher trade deficit ($18 billion) and a reduced
services surplus ($37 billion), the latter two reflecting a deterioration in the economy’s competitiveness

385. For Imported and exported Goods, which of these authorities determines the rate of exchange of conversion of
Australian Dollar currency to Indian currency?

a) Trade Division, Reserve Bank of India


b) Secretary, Department of Commerce
c) Central Board of Excise and Customs
d) It is an entirely market determined rate

Solution: c)
Justification: India follows a managed exchange rate system where currency exchange rate is market determined by
occasional interventions from RBI. However, there are certain cases where the Customs Act, 1962 and associated
notifications grant the CBEC the power to determine exchange rates of certain currencies for e.g. Bahrain Dinar,
Canadian Dollar.
386. Consider the following with respect to the trade statistics between India-China:

1) India runs a trade deficit with China.


2) China’s exports to India account for a majority of China’s exports.
3) Chinese companies dominate the telecom sector in India in terms of total equipment imported.
4) India’s pharma sector has critical dependence on some Chinese imports for use in drugs manufacturing.

Select the correct answer using the codes below.

a) 1 and 2 only
b) 1, 3 and 4 only
c) 3 and 4 only
d) 2, 3 and 4 only

Solution: b)
Justification: A trade war between China and India seems to be looming after the imposition of anti-dumping duties
on many Chinese products by India. But, India cannot afford to fight a trade war with China at this juncture. The
reasoning is as below:
Statement 1: India’s trade deficit with China rose to $46.56 billion last year.
Statement 2: China’s exports to India account for only 2% of its total exports. So even if Indians boycott all the goods
imported from China, it will not make as big an impact on China as to bring it to its knees before India.
Statement 3: India today imports telecom gear worth over Rs 70,000 crore annually, much of it from Chinese firms
like Huawei and ZTE. Chinese companies dominate the telecom sector in India.
Statement 4: Some active pharmaceutical ingredients (APIs) can be provided only by China to India.

Power is another sector where India has come to be dependent on Chinese imports. In the 12th Plan alone, almost
30% of the generating capacity was imported from China.

387. India has become the 71st nation to join the United Nations TIR (Transports Internationaux Routiers)
Convention. How will this be helpful to India?

1) It will eliminate all border taxes, customs and duties between member countries.
2) It will enable India to move cargo along the International North-South Transport Corridor via Chabahar port in Iran.

Which of the above is/are correct?


a) 1 only
b) 2 only
c) Both 1 and 2
d) None

Solution: b)
Justification: TIR Carnets, 1975 (TIR Convention) is an international customs transit system under the auspices of
the United Nations Economic Commission for Europe (UNECE) with the widest geographical coverage. It is more
than a transport convention.

 The TIR is managed and developed by International Road Transport Union (IRU) which is the world road transport
organisation.

 The TIR procedure facilitates movement of goods under customs control across international borders without the
payment of the duties and taxes.
Justification: Statement 1: The TIR system secures customs duties and taxes and provides a robust guarantee
mechanism, thereby reducing trade transaction costs, and facilitating higher growth of intra-regional and inter-
regional trade. It does not eliminate those taxes and duties, which are otherwise dealt by trade agreements and
WTO rules.
Statement 2: On the eastern front, it will help India integrate with Myanmar and Thailand, as well as Bangladesh,
Bhutan and Nepal. On the western front, it will enable India to move cargo along the International North-South
Transport Corridor via Chabahar port in Iran, to access landlocked Afghanistan and the energy-rich Eurasian region.

TRADE BLOCs

388. The South Asian Free Trade Area (SAFTA) intends to

1) Gradually reduce customs duties of all traded goods to zero


2) Recognize the special needs of the Least Developed Contracting States and provide an agreement on concrete
preferential measures in their favour

Which of the above is/are correct?

a) 1 only
b) 2 only
c) Both 1 and 2
d) None

Solution: c)
Justification: The basic principles underlying SAFTA are as under overall reciprocity and mutuality of advantages so
as to benefit equitably all Contracting States, taking into account their respective level of economic and industrial
development, the pattern of their external trade, and trade and tariff policies and systems; negotiation of tariff
reform step by step, improved and extended in successive stages through periodic reviews; recognition of the
special needs of the Least Developed Contracting States and agreement on concrete preferential measures in their
favour; inclusion of all products, manufactures and commodities in their raw, semi-processed and processed forms.
In 2011, Afghanistan joined SAFTA. SAFTA requires the developing countries in South Asia (India, Pakistan and Sri
Lanka) to bring their duties down to 20 percent in the first phase of the two-year period ending in 2007. In the final
five-year phase ending 2012, the 20 percent duty will be reduced to zero in a series of annual cuts. The least
developed nations in South Asia (Nepal, Bhutan, Bangladesh, Afghanistan and Maldives) have an additional three
years to reduce tariffs to zero.

389. The year of 2017 marks 25 years of dialogue partnership between India and ASEAN and 15 years of summit-
level interaction. With reference to India-ASEAN relations, consider the following:

1) India's relationship with ASEAN is a key pillar of our Act East Policy.
2) AIFTA is a free trade area among the member states of the ASEAN and India.
3) India and ASEAN are yet to upgrade their relationship to a ‘strategic partnership’.

Select the correct answer using the codes below.

a) 2 only
b) 1 and 3 only
c) 1 and 2 only
d) 3 only

Solution: c)
Justification: Statement 1: India’s commitment to strengthen its ties with ASEAN countries is reflected by its ‘Look
East Policy’ becoming ‘Act East Policy.’ Three Cs are the focus -Commerce, Connectivity and Culture.
Statement 2: ASEAN became ASEAN Plus Six with additional countries: Australia, New Zealand and India.
Codification of the relations between these nations has seen progress through the development of the Regional
Comprehensive Economic Partnership (RCEP), a proposed free-trade agreement involving the 16 countries of ASEAN
plus six.
Statement 3: India and ASEAN had upgraded their relationship to a strategic partnership in 2012. ASEAN consists of
10-member countries- Brunei, Cambodia, Indonesia, Laos, Malaysia, Myanmar, Philippines, Singapore, Thailand and
Vietnam. It is a regional organisation that aims to promote intergovernmental cooperation and economic
integration amongst its members.

390. With reference to unique BRICS countries initiatives, consider the following statements:

1) BRICS Network University (NU) is an open educational project and does not involve establishment of rigid
organizational forms such as a secretariat.
2) BRICS Think Tanks Council (BTTC) comprises research organizations from BRICS countries including Observer
Research Foundation (ORF) from India.

Which of the above is/are correct?

a) 1 only
b) 2 only
c) Both 1 and 2
d) None

Solution: c)
Justification: Recently at the 5th BRICS Education ministers meeting, Beijing declaration on education was adopted
which advocated support for both these organizations.
Statement 1: NU is a consortium of the universities, elaborating Master and PhD programmes in different
disciplines. It was established in 2015 in BRICS meeting at Moscow. NU follows principles of: Openness, which
enables various universities to join the network respecting national criteria. Focus on educational programmes that
can be supplemented with network research and innovation projects. There is also a BRICS Universities League
which is a consortium of leading research universities from BRICS countries.
Statement 2: The BTTC (established in 2013) comprises the Institute for Applied Economic Research (IPEA) (Brazil),
the National Committee for BRICS Research (NRC/BRICS) (Russia), the Observer Research Foundation (ORF) etc. The
objective behind creating such a Council was to establish a permanent entity that would enhance cooperation in
research, knowledge sharing, capacity building, and policy advice between think tanks in BRICS countries. The main
purposes of its establishment was to form a platform for the exchange of ideas and present policy
recommendations for BRICS leaders.

391. Consider the following statements about the recently established Global Foreign Exchange Committee (GFXC).

1) It was established as a research arm of International Monetary Fund (IMF).


2) GFXC will enforce a FX Global Code that will legally regulate the manipulation of floating foreign exchange by
Central Banks.
3) It will promote collaboration and communication among the local foreign exchange committees (FXCs) of member
nations.
4) It will replace the institution of Bank of International Settlements (BIS).
5) Indian government has boycotted GFXC due to its non-democratic governance structure.

Select the correct answer using the codes below.

a) 2, 3 and 4 only
b) 1, 2 and 5 only
c) 3 only
d) 1, 2, 3 and 4 only

Solution: c)
Justification: This is a very important development, hence the long explanation.
Statement 1: GFXC was established in 2017 as a forum bringing together central banks and private sector
participants with the aim to promote a robust, liquid, open, and appropriately transparent FX market.
Statement 2: FX Global Code is a set of global principles of good practice in the foreign exchange market, developed
to provide a common set of guidelines to promote the integrity and effective functioning of the wholesale foreign
exchange market. The Global Code does not impose legal or regulatory obligations on Market Participants, nor does
it substitute for regulation. It is rather intended to serve as a supplement to any and all local laws, rules and
regulations by identifying global good practices and processes.
Statement 3: The objectives of the GFXC are:
 to promote collaboration and communication among the local foreign exchange committees (FXCs) and non-GFXC
jurisdictions with significant FX markets;
 to exchange views on trends and developments in global FX markets, including on the structure and functioning of
those markets, drawing on information gathered at the various FXCs; and
 to promote, maintain and update on a regular basis the FX Global Code (the Code) and to consider good practices
regarding effective mechanisms to support adherence.
Statement 4: The GFXC committee has been set up under guidance of the Bank for International Settlements (BIS),
an international financial organisation.
Statement 5: India will soon join Global Foreign Exchange Committee (GFXC).

392. Regional Comprehensive Economic Partnership (RCEP) is a proposed free trade agreement (FTA) between

1) India
2) Australia
3) China
4) Japan
5) Russia

Select the correct answer using the codes below.

a) 1, 3 and 4 only
b) 1, 2,3 and 4 only
c) 2, 3 and 5 only
d) None of the above

Solution: b)
Justification: RCEP is between the ten member states of the Association of Southeast Asian Nations (ASEAN)
(Brunei, Burma (Myanmar), Cambodia, Indonesia, Laos, Malaysia, the Philippines, Singapore, Thailand, Vietnam) and
the six states with which ASEAN has existing FTAs (Australia, China, India, Japan, South Korea and New Zealand).
RCEP negotiations were formally launched in November 2012 at the ASEAN Summit in Cambodia. At the
forthcoming ministerial-level meeting on the proposed Free Trade Agreement (FTA) involving 16 Asia-Pacific
nations, India has decided to raise concerns regarding the ‘slow’ pace of negotiations on services trade liberalization
as opposed to ‘higher priority’ being accorded to commitments to open up goods trade in the region.

393. The European Free Trade Association (EFTA) is an intergovernmental organisation consisting of which of these
countries?

1) Switzerland
2) Norway
3) Germany
4) France

Select the correct answer using the codes below.

a) 1 and 2 only
b) 3 and 4 only
c) 2, 3 and 4 only
d) 1 and 3 only

Solution: a)
Justification: EFTA is a regional trade organisation and free trade area consisting of four European states: Iceland,
Liechtenstein, Norway, and Switzerland. EFTA was established to serve as an alternative trade bloc for those
European states that were unable or unwilling to join the then European Economic Community (EEC) (which
subsequently became the European Union). The organisation operates in parallel with the European Union (EU), and
all four members participate in the European single market. India and the European Free Trade Association (EFTA)
are planning to shortly conclude their long pending negotiations on FTA. So far, several rounds of negotiations have
taken place, talks for which started in 2008. The proposed agreement covers trade in goods and services,
investments, trade facilitation, Customs cooperation, protection of intellectual property and public procurement.

THE BALANCE OF PAYMENTS

394. Balance of Payments (BoP) of a nation

1) Reflects the total transactions of an economy with the outside world


2) Is the net outcome of the current and capital accounts of the economy

Which of the above is/are correct?

a) 1 only
b) 2 only
c) Both 1 and 2
d) None

Solution: c)
Justification: Statement 1: It is the net outcome of the current and capital accounts of an economy that reflect all
international transactions. It might be favourable or unfavourable for the economy. However, negativity of the BOP
does not mean it is unfavourable. A negative BOP is unfavourable for an economy if only the economy lacks the
means to fill the gap of negativity. 'The BOP of an economy is calculated on the principles of accountancy (double-
entry book- keeping) and looks like the balance sheet of a company—every entry shown either as credit (inflow) or
debit (outflow). If there is a positive outcome at the end of the year, the money is automatically transferred to the
foreign exchange reserves of the economy. And if there is any negative outcome, the same foreign exchange is
drawn from the country's forex reserves. If the forex reserves are not capable of fulfilling the negativity created by
the BOP, it is known as a BOP crisis and the economy tries different means to solve the crisis in which going for forex
help from the IMF is the last resort.

395. Consider the following statements. In a closed economy


Assertion (A): Balance of Payments is zero.
Reason (R): No external transactions from outside the nation’s economy take place.

In the context of the above, which of these is correct?

a) A is correct, and R is an appropriate explanation of A.


b) A is correct, but R is not an appropriate explanation of A.
c) A is correct, but R is incorrect.
d) Both A and R are incorrect.

Solution: a)
Justification: A closed economy is self-sufficient, meaning that no imports are brought in and no exports are sent
out. The goal is to provide consumers with everything that they need from within the economy's borders. A closed
economy is the opposite of an open economy, in which a country will conduct trade with outside regions.
So, if no capital or goods/services are imported, exported, the BoP will be zero.

396. In a pure flexible exchange rate system, foreign exchange reserves of India will increase necessarily if

a) Balance of Payments (BoP) is in surplus


b) Current Account Deficit (CAD) is in surplus
c) Trade is in surplus
d) Capital account is in surplus

Solution: a)
Justification: BoP is composed of current account as well as capital account. Current account deficit is shown as CAD
which comprises of trade surplus/deficit; invisibles surplus/deficit, remittances outflow/inflow etc. Capital account
consists of external commercial borrowings, loans taken by government, long-term investments abroad, FDI, FII etc.
So, even if CAD is in surplus, it is possible that foreign debt and capital outflows (capital account) are more than
CAD. This makes the BoP negative, and there is a net outgo of foreign exchange reserves in this case. So, option (b)
is not always correct. Same is true for option (c). Even if trade is in surplus, total CAD and BoP may be negative. So,
option (c) is also incorrect. Similarly even if capital account is in surplus, it is possible that CAD is highly in deficit.
Thus, there may be a net outgo of foreign exchange. So, option (d) is also incorrect.

THE FOREIGN EXCHANGE MARKET

BASIC CONCEPTS

397. Inflow of foreign investments in India lead to

1) Increase in Current Account Deficit (CAD) in the short-term


2) Crowding out in financial markets in the short-term

Which of the above is/are correct?

a) 1 only
b) 2 only
c) Both 1 and 2
d) None

Solution: d)
Justification: Statement 1: It actually bridges the CAD in the short-run. In the long-run, however, India has to pay
more due to an outgo of both the principle and earned returns on investment in the long run.
Statement 2: Crowding out happens due to shortage of funds and a hike in interest rates following the shortage.
This usually happens when government borrows in large quantities from markets. But, increase foreign exchange
influx would actually help reduce crowding out by increasing the fund flow and lowering the interest rates.

398. Anti-dumping duty is usually applied in order to

1) Contain trade distortion


2) Increase imports of capital goods
3) Sterilize foreign capital inflows

Select the correct answer using the codes below.

a) 1 and 2 only
b) 3 only
c) 1 only
d) 2 and 3 only

Solution: c)
Justification: Statement 1: Suppose a nation like China is dumping (exporting) its goods at exceptionally cheap
prices so that it can capture the market. Since these cheap prices can’t be justified by market forces alone, the
Indian government must apply a duty (like a tax) on the product to raise its price. This is to shield the domestic
producers.
Statement 2: Capital goods are usually not dumped by nations, since the industry is highly segmented and it’s hard
to dump products at cheap prices.
Statement 3: It has nothing to do with FDI or FII.
India has initiated over two hundred anti-dumping investigations between 2012 and 2017 against various countries,
including China and Indonesia. During the period, maximum number of cases were against China. Product categories
on which the levy was imposed include chemicals, fibre boards, glass & glassware, pharmaceuticals and steel.

FOREX RESERVES

ARBITRAGE

FDI&FII
399. As per the Consolidated FDI Policy (Effective from August 28, 2017) recently published by Department of
Industrial Policy and Promotion, Ministry of Commerce and Industry

1) Erstwhile Overseas Corporate Bodies (OCBs) have been recognized as a class of investors in India.
2) A company incorporated outside India and owned and controlled by NRIs can invest in India.
3) A Foreign Venture Capital Investor (FVCI) cannot invest in India.
4) NRIs may subscribe to the National Pension System (NPS).

Select the correct answer using the codes below.

a) 1 and 3 only
b) 2 and 4 only
c) 1, 3 and 4 only
d) 2 only

Solution: b)
Justification: Statement 1: OCBs have been derecognized as a class of investors in India with effect from 2003.
Erstwhile OCBs which are incorporated outside India and are not under the adverse notice of RBI can make fresh
investments under FDI Policy as incorporated non-resident entities, with approval of concerned authorities.
Statement 2: A company, trust and partnership firm incorporated outside India and owned and controlled by NRIs
can invest in India with the special dispensation as available to NRIs under the FDI Policy. Moreover, Indian
companies can issue capital against FDI.
Statement 3: A SEBI registered Foreign Venture Capital Investor (FVCI) may contribute up to 100% of the capital of
an Indian company, including startups irrespective of the sector in which it is engaged, under the automatic route.
Statement 4: A Non- Resident Indian may subscribe provided such subscriptions are made through normal banking
channels and the person is eligible to invest as per the provisions of the PFRDA Act. The annuity/accumulated saving
will be repatriable (i.e. can be taken back abroad).
400. As per extant FDI policy in India

1) Foreign investment up to 100 percent is permitted for a company engaged in the business of manufacturing mobile
phones.
2) There is a 30 per cent local sourcing of component for any FDI related manufacturing activity undertaken in India.

Which of the above is/are correct?

a) 1 only
b) 2 only
c) Both 1 and 2
d) None

Solution: a)
Justification: Statement 1: This is under the automatic route where investment in the country can be made without
prior approval either of the Government or the Reserve Bank of India. FDI is largely a matter of private business
decisions, and therefore, no such details regarding efforts made by foreign companies to set up for e.g. Apple i-
phone facilities are required.
Statement 2: This isn’t applicable to all manufacturing units, for e.g. it is a restriction for mobile phone
manufacturing units in India. Recently Apple submitted a representation to waive off this requirement because it
would affect their profit margins. However, in case of ‘state-of-art’ and ‘cutting-edge technology’, sourcing norms
can be relaxed subject to Government approval. This relaxation will be valid for an initial period of three years from
the opening of first store, and thereafter, such entities will be required to meet the domestic sourcing norms.

EXTERNAL DEBT
UNIT-X
CHALLENGES: POVERTY, SOCIAL INFRASTRUCTURE, SKILLING AND EMPLOYMENT

LABOUR

401. If Labour Force Participation rate (LFP) of a country increases, it indicates that the

a) Skilled population in the country is increasing.


b) Country’s adult population is increasing.
c) Number of people who are either employed or are actively looking for work is increasing.
d) Number of people who are employed is increasing.

Solution: c)
Justification: It is a measure of the active portion of an economy's labour force. It refers to the number of people
who are either employed or are actively looking for work. The number of people who are no longer actively
searching for work would not be included in the participation rate. During recession many workers often get
discouraged and stop looking for employment that is why the participation rate decreases. Analysis of National
Sample Survey data shows that India has been experiencing declining and low rates of female labour force
participation. This is concentrated mostly among married women in rural areas. Some suggest that this is partly
because women with medium levels of education choose to spend more time on child care and domestic work.

402. The Ministry of Labour has categorised the unorganized labour force under four groups depending on
occupation, nature of employment, specially distressed categories and service categories. Bonded labourers come
under which category?

a) Under Terms of Occupation


b) Under Terms of Nature of Employment
c) Under Terms of Specially Distressed Category
d) Under Terms of Service Category

Solution: b)
Justification: These are the categories: Under Terms of Occupation: Small and marginal farmers, landless
agricultural labourers, share croppers, fishermen, those engaged in animal husbandry, beedi rolling, labelling and
packing, building and construction workers, leather workers, weavers, artisans, salt workers, workers in brick kilns
and stone quarries, workers in saw mills, oil mills, etc. come under this category.
Under Terms of Nature of Employment: Attached agricultural labourers, bonded labourers, migrant workers,
contract and casual labourers come under this category.
Under Terms of Specially Distressed Category: Toddy tappers, scavengers, carriers of head loads, drivers of animal
driven vehicles, loaders and unloaders come under this category.
Under Terms of Service Category: Midwives, domestic workers, fishermen and women, barbers, vegetable and fruit
vendors, newspaper vendors, etc., belong to this category.

403. Which of the following schemes is targeted towards the workers in the Unorganized sector?

1) Aam Aadmi Bima Yojana


2) Rashtriya Swasthya Bima Yojana (formerly)
3) Pradhan Mantri Suraksha Bima Yojana

Select the correct answer using the codes below.

a) 1 and 2 only
b) 2 and 3 only
c) 1 and 3 only
d) 1, 2 and 3

Solution: a)
Justification: This scheme covers an accidental death and disability cover of Rs 2 lakh. Premium is Rs 12 per year.
Anybody who has a savings account in the banks that offer this scheme.

404. What are the advantages of investing in Public Provident Fund (PPF) Account?

1) Premature closure is allowed.


2) Deposits qualify for tax deduction.
3) Interest earned is completely tax-free.
Which of the above is/are correct?

a) 1 and 2 only
b) 2 and 3 only
c) 2 only
d) 1 and 3 only

Solution: b)
Justification: Major features of PPF are:

 Deposits can be made in lump-sum or in installments. Joint account cannot be opened.


 Maturity period is 15 years but the same can be extended within one year of maturity for further 5 years and so on.
 Maturity value can be retained without extension and without further deposits also.
 Premature closure is not allowed before 15 years. But, withdrawal is permissible every year from 7th financial year
from the year of opening account.
Other advantages are:

 No attachment under court decree order.


 Deposits qualify for deduction from income under Sec. 80C of IT Act. Interest is completely tax-free.

LABOUR REFORMS

405. The Union Ministry of Labour and Employment has launched the PENCIL Platform for effective enforcement of

a) Skill India campaign


b) National Child Labour Policy
c) International labour convention
d) Right to Education, Sarva Shiksha Abhiyaan

Solution: b)
Justification: It is the Platform for Effective Enforcement for No Child Labour (PENCIL) Portal. The PENCIL portal is an
electronic platform that aims at involving Centre, State, District Governments, civil society and general public in
achieving the target to a child labour free society. It encompasses various components- Child Tracking System,
Complain Corner, State Government, National Child Labour Project Convergence. Each district will nominate District
Nodal Officers (DNOs) who will receive the complaints. Within 48 hours of receiving complaints. DNOs will check
genuineness of complaint and take rescue measures it coordination with police.

406. The contractual workers working in establishments having ten or more workmen avail social security and other
benefits under which of the following acts?

1) Employees’ Compensation Act (1923)


2) Industrial Dispute Act (1947)
3) Maternity Benefit Act (1961)
4) Unorganised Workers’ Social Security Act, 2008

Select the correct answer using the codes below.

a) 3 and 4 only
b) 2, 3 and 4 only
c) 1 and 2 only
d) 1, 2, 3 and 4

Solution: d)
Justification: Consideration of demands of trade unions and making of further improvement in the benefits
available to workers by the Government is a continuous process. The contractual workers working in establishments
having ten or more workmen avail social security and other benefits under the Employees’ Compensation Act
(1923), the Employees’ State Insurance Act (1948), the Industrial Dispute Act (1947), the Minimum Wages Act
(1948), the Provident Funds Act (1925), the Employees Provident Fund and Miscellaneous Provisions Act (1952), the
Maternity Benefit Act (1961), the Payment of Gratuity Act (1972), etc. Further, it has been the constant endeavor of
the Government to ensure that the unorganized workers, including contractual workers engaged in establishments
having less than ten workers, get social security benefits as available to the workers in the organized sector. With
this objective, the Government is implementing the Unorganised Workers’ Social Security Act, 2008. The 2008 Act
stipulates formulation of suitable welfare schemes for unorganized workers on matters relating to life and disability
cover, health and maternity benefits, old age protection.

407. Many of the State Governments like Rajasthan, Kerala, Punjab, Tamilnadu and Tripura have included domestic
workers in the schedule of the Minimum Wages Act. What is/are the implications of this inclusion?
1) The state government can regulate the minimum wages to be given to domestic workers similar to other
contractual and informal employment in the state.
2) Domestic workers are entitled to file cases before the concerned authorities in case of any failure on the part of
their employer to give them minimum wages.

Which of the above is/are correct?

a) 1 only
b) 2 only
c) Both 1 and 2
d) None

Solution: c)
Justification: Statement 1 and 2: Both statements are correct, and logically follow from the question statement.
The registration of the placement agencies is a State subject. An advisory has been issued to all the State
Government requesting them to take steps for formulation of a policy at their own level for regulating these
placement agencies.
The Central government has also been pushing states to regulate this issue because there have been repeated
demands to regulate their employment given repeated cases of exploitation and harassment.

408. Government of India plans to do away with all employer codes being issued by separate labour enforcement
agencies such as ESIC, EPFO, CLC(C) and DGMS etc. by replacing them with a/an

a) Aadhar Number
b) Universal Account Number (UAN)
c) Labour Identification Number (LIN)
d) Permanent Account Number (PAN)

Solution: c)
Justification: A unit registered with different labour enforcement agencies is identified uniquely and allotted a
single unique LIN. LIN is the pivot around which all other services work. All the units need to verify their data and
report any alteration or correction to us for modification through “LIN Verification” Service that has been made
available on the ShramSuvidha Portal. All the units need to verify their data and report any alteration or correction
to us for modification through “LIN Verification” Service that has been made available on the ShramSuvidha Portal.

409. Minimum rates of wages can be fixed by the

a) Central Government
b) State Government
c) Local government
d) Both (a) and (b)

Solution: d)
Justification: The issue is frequently in news due to protests and strikes on minimum wages and contract labour
such as ASHAs. Under The Minimum Wages Act, 1948 both Central and state governments are “Appropriate
Governments” for fixation/revision of minimum rates of wages for employments covered by the Schedule to the
Act. Under the Contract Labour (Regulation and Abolition) Act, 1970:

 The Central and state governments are empowered to prohibit employment of contract labour in any activity in an
establishment.
 The Act contains specific provisions to ensure payment of wages and certain amenities to be provided by the
contractor to the contract labour.
410. The Government has introduced ‘The Code on Wages’ Bill in the Lok Sabha that will consolidate which of the
following laws?

1) Payment of Wages Act, 1936


2) Minimum Wages Act, 1998
3) Payment of Bonus Act, 1923
4) Equal Remuneration Act, 1919
Select the correct answer using the codes below.

a) 1 only
b) 2, 3 and 4 only
c) 3 only
d) 1, 2, 3 and 4

Solution: a)
Justification: Solve by elimination. If you notice, both 3 and 4 are written as pre-independence acts, which isn’t
the case. It is actually - Payment of Bonus Act 1965 and Equal Remuneration Act 1976. Both progressive measures
were passed post-independence and could not have been expected in a colonial setup. Statement 2: It was passed
right after independence to ensure social justice, and the year of passing of this act should be common knowledge
to you. It was “Minimum Wages Act 1948”. The bill seeks to amend the laws relating to wages and bonus which
also seeks to empower the Centre to fix a “universal minimum wage” aimed to benefit over 40 crore unorganized
sector workers.
 The bill seeks to amalgamate four laws — Payment of Wages Act 1936, Minimum Wages Act 1948, Payment of
Bonus Act 1965 and Equal Remuneration Act 1976.
 The Code provides for the government to determine the minimum wages every five years using factors like skills
required for the job, arduousness of work, geographical location of work place and other aspects.

POVERTY

411. A general rise in Gini Coefficient indicates that

1) Income inequality is decreasing between the nation's residents


2) Income level of the General public is showing recurrent increase.

Which of the above is/are correct?

a) 1 only
b) 2 only
c) Both 1 and 2
d) None

Solution: d)
Justification: In economics, the Gini coefficient (sometimes expressed as a Gini ratio or a normalized Gini index) is a
measure of statistical dispersion intended to represent the income or wealth distribution of a nation's residents,
and is the most commonly used measure of inequality. Gini Coefficient is a popular statistical measure to gauge the
rich-poor income or wealth divide. It measures inequality of a distribution — be it of income or wealth — within
nations or States. Its value varies anywhere from zero to 1; zero indicating perfect equality and one indicating the
perfect inequality. Gini Coefficients can be used to compare income distribution of a country over time as well. An
increasing trend indicates that income inequality is rising independent of absolute incomes.

412. A high Gini score for a nation means that


1) The nation is highly economically developed
2) The nation has high economic inequality

Which of the above is/are correct?

a) 1 only
b) 2 only
c) Both 1 and 2
d) None

Solution: b)
Justification: Gini coefficient is intended to represent the income distribution of a nation's residents, and is the
most commonly used measure of inequality. A nation with a Gini score of 0 has perfect income equality, and score
of 100 has perfect inequality (i.e. all the wealth is owned by a single person). If Gini coefficient of a Nation is high,
there is no guarantee that the Nation will be wealthy or poor.

413. The ‘International Poverty Line’ is published by

a) World Bank
b) United Nations Development Program (UNDP)
c) Oxfam International
d) International Labour Organization (ILO)

Solution: a)
Justification: The World Bank has been publishing poverty line since 1990 World Development Report (WDR) on
Poverty. It has anchored its international poverty lines to the national poverty lines used in the poorest countries. In
2015 World Bank updated international poverty line to US $1.90 a day. Determining the poverty line is usually done
by finding the total cost of all the essential resources that an average human adult consumes in one year.

414. Ration cards are an official document entitling the holder to a ration of food, fuel, or other goods. Consider the
following about them.

1) One card per family is issued by the Central government.


2) Getting a ration card is mandatory in India.
3) The ration card allocated to Below Poverty Line (BPL) people is black in colour.

Select the correct answer using the codes below.

a) 1 and 2 only
b) 1 only
c) 3 only
d) None of the above

Solution: d)
Justification: Statement 2: Ration cards are voluntary in India; they are needed to obtain subsidized food and fuel.
Statement 1 and 3: One card per family is issued by the state government. Three card categories are issued:
extreme poverty level (Antyodaya), below poverty line (BPL), and above poverty line (APL). Antoydaya cards are red
in colour.

415. Poverty line in India is calculated based on which of these bare minimum parameters?

1) Minimum level of food requirement


2) Clothing and footwear
3) Absence of land ownership
4) Educational and medical requirements
5) Fuel and light
Select the correct answer using the codes below.

a) 1, 2, 4 and 5 only
b) 1 and 4 only
c) 2, 3 and 5 only
d) 1, 2, 3, 4 and 5

Solution: a)
Justification: While determining the poverty line in India, a minimum level of food requirement, clothing, footwear,
fuel and light, educational and medical requirement etc. are determined for subsistence. These physical quantities
are multiplied by their prices in rupees.
Statement 1: The present formula for food requirement while estimating the poverty line is based on the desired
calorie requirement. The accepted average calorie requirement in India is 2400 calories per person per day in rural
areas and 2100 calories per person per day in urban areas.
Statement 3: A person is not declared poor if he does not possess land. However, if he has land, he will be
automatically excluded from the poverty line.

416. Gini coefficient is a measure of

a) Constitutional democracy
b) Spread of biodiversity
c) Income inequality
d) Social diversity
Solution: c)
Justification: The Gini coefficient provides an index to measure inequality. It is a way of comparing how distribution
of income in a society compares with a similar society in which everyone earned exactly the same amount. This
number, which ranges between 0 and 1 and is based on residents' net income, helps define the gap between the
rich and the poor, with 0 representing perfect equality, and 1 representing perfect inequality i.e. all wealth
concentrated in just one person.
417. Wealth redistribution is poor in a capitalist economy because of

1) Lack of any economic regulations by the State


2) Lack of centralized economic planning that may seem to bring about greater equity

Which of the above is/are correct?

a) 1 only
b) 2 only
c) Both 1 and 2
d) None

Solution: b)
Justification: By the very nature of a capitalistic economy, the goods are bought by the consumers and money flows
to the entrepreneurs. As there is little state intervention or taxes; there is little redistribution.
Statement 1: But, states regulate economies by way of laws, rules and regulations, for e.g. licensing of certain
industries or labour laws that prevent the exploitation of labour.
Statement 2: If the state plans for more equitable growth, it can for e.g. provide greater incentives for capitalists to
setup industries in rural or backward areas so that rural-urban disparity can be minimized.

418. With reference to poverty related statistics in India, Incidence of poverty is higher in

1) Rural areas as compared to urban areas


2) Eastern states as compared to Southern states

Which of the above is/are correct?

a) 1 only
b) 2 only
c) Both 1 and 2
d) None

Solution: c)
Justification: Statement 1: Nearly, every fourth person in India is poor. This means, roughly 270 million (or 27
crore) people in India live in poverty 2011-12. Only old estimates are available, new ones will be available after the
updated results of sample surveys.

Statement 1
Statement 2

DEMOGRAPHIC DIVIDEND

419. What are the possible ways to increase the human capital of the country?

1) Increase the population of the nation


2) Invest in health expenditure

Select the correct answer using the codes below.

a) 1 only
b) 2 only
c) Both 1 and 2
d) None of the above

Solution: c)
Justification: Statement 1: Human capital can be increased in two important ways. One that we increase the
population and thus increase the labour force and second if the population is fixed or declining, increase
expenditure on human resources. Health expenditure directly increases the supply of healthy labour force and is,
thus, a source of human capital formation. Digital literacy builds IT skills, increases productivity and efficiency of
labour and digital information is necessary to make decisions regarding investments in human capital as well as for
efficient utilization of the acquired human capital stock.

420. As per Census 2011, consider the following with reference to the demography of senior citizens in India.

1) Sex ratio is skewed in the favour of females.


2) Larger population resides in urban areas compared to rural areas.

Which of the above is/are correct?

a) 1 only
b) 2 only
c) Both 1 and 2
d) None

Solution: a)
Justification: According to Population Census 2011 there are nearly 104 million elderly persons (aged 60 years or
above) in India; 53 million females and 51 million males. A report released by the United Nations Population Fund
and HelpAge India suggests that the number of elderly persons is expected to grow to 173 million by 2026. Both the
share and size of elderly population is increasing over time. From 5.6% in 1961 the proportion has increased to 8.6%
in 2011. For males it was marginally lower at 8.2%, while for females it was 9.0%. As regards rural and urban areas,
71% of elderly population resides in rural areas while 29 % is in urban areas. The life expectancy at birth during
2009-13 was 69.3 for females as against 65.8 years for males

421. Immigration in a country can

1) Slow down the ageing process of the society as a whole


2) Augments the human capital of the society

Which of the above is/are correct?

a) 1 only
b) 2 only
c) Both 1 and 2
d) None

Solution: c)
Justification: Statement 1: Globally, population aged 60 or over is growing faster than all younger age groups. In
2017, there are an estimated 962 million people aged 60 or over in the world, comprising 13 per cent of the global
population. The population aged 60 or above is growing at a rate of about 3 per cent per year. Currently, Europe has
the greatest percentage of population aged 60 or over (25 per cent). While declining fertility and increasing
longevity are the key drivers of population ageing globally, international migration has also contributed to changing
population age structures in some countries and regions. In countries that are experiencing large immigration flows,
international migration can slow the ageing process, at least temporarily, since migrants tend to be in the young
working ages. However, migrants who remain in the country eventually will age into the older population.
Statement 2: This concept has been covered in an earlier test and need not be explained again. Societies like USA
have considerable number of immigrants augmenting their human capital especially in industries like IT.

422. The demographic target of the ‘Mentor India campaign’ of NITI Aayog are

a) Geriatric population
b) International Leaders
c) Senior Government Employees
d) School going children

Solution: d)
Justification: Atal Innovation Mission (AIM), Government of India’s flagship program to promote innovation and
entrepreneurship, is inviting applications for ‘Mentor India’. Mentor India is as a strategic nation building initiative
to engage leaders who can guide and mentor schools students in 900 Atal Tinkering Labs (ATL) established by AIM in
schools across India. Through Mentor India, AIM is looking to engage leaders who can dedicate 1 – 2 hours every
week in one or more such labs and enable school students to experience, learn and practice future skills such as
design and computational thinking.

SOCIAL INFRASTRUCTURE

423. The concept of inter-generational equity has a bearing on

1) Principles of taxation and public borrowing


2) Sustainable Development

Which of the above is/are correct?

a) 1 only
b) 2 only
c) Both 1 and 2
d) None
Solution: c)
Justification: Statement 1: “Ricardian equivalence” says that if government borrows too much from the public, it
can repay this debt only by increasing the taxes, on either the present generation or the coming generations.
Therefore, there should be a sense of equity in public borrowing programs that do not put a lot of burden on the
coming generations.
Statement 2: This is clear as more pollution and consumption will only deplete resources for the coming
generations.
The essential idea is that we should not compromise on the needs of the future generations against the present
generation.

424. Rich countries are rich because

a) All of them have greater reserves of natural resources than any developing country.
b) They have made adequate investment in the health, education and training of their workforce.
c) All rich nations are democratic.
d) All of the above.

Solution: b)
Justification: Option A: Africa is resource rich whereas, many rich nations like Scandinavian countries are not
resource rich.
Option B: Human capital (through education, training, medical care) yields a return just like investment in physical
capital. This can be seen directly in the form of higher incomes earned because of higher productivity of the more
educated or the better trained persons, as well as the higher productivity of healthier people.
Option C: Many poor nations are democratic, e.g. Zimbabwe, whereas many rich nations like Saudi Arabia are
monarchies.
425. A natural resource rich country will not be economically developed if

1) It is an open economy.
2) It does not have the required human capital to transform the natural resources.

Which of the above is/are correct?

a) 1 only
b) 2 only
c) Both 1 and 2
d) None

Solution: b)
Justification: Statement 1: Open economy can be helpful to development in that it allows the country to borrow
best practices and capital from across the globe. The nation can also increase its income through exports.

Statement 2: The resource rich Africa and Latin America have some of the poorest countries in the world, whereas
many prosperous countries have scarcely any natural wealth. The economic wealth, or well-being, of a country thus
does not necessarily depend on the mere possession of resources; it is how these resources are used in generating a
flow of production and how, as a consequence, income and wealth are generated from that process. It is called as
demographic dividend, but its fruition depends on how skilled, healthy and well- managed is the labour force. India
is a well-known case.

426. SATH - ‘Sustainable Action for Transforming Human capital’ initiative of NITI Aayog focuses on

a) Creating ‘role model’ states for health systems


b) Revamping higher education
c) Building workforce for the future in areas of cutting edge technology
d) Eradicating manual scavenging
Solution: a)
Justification: This was launched with the State Governments for furthering the agenda of cooperative federalism.
NITI Aayog under the SATH initiative plans to identify and build three future ‘role model’ states for health systems.
After identification of three states, NITI Aayog will work in close collaboration with the state machinery of these
three states. It will help in designing a robust roadmap, develop a program governance structure, establish
monitoring and tracking mechanisms, and provide support to the state institutions to achieve the end objectives.
SATH program will be implemented by NITI Aayog along with McKinsey & Company and IPE Global consortium.
Parameters such as MMR, IMR, and incidence of malaria will be considered for determining potential impact.

427. What s/are the differences between physical capital and human capital?

1) Human capital cannot be used as factor of production whereas physical capital can be.
2) A nation’s physical capital cannot grow over time whereas human capital can.
3) Physical capital cannot be organized unlike human capital.

Select the correct answer using the codes below.

a) 1 only
b) 2 and 3 only
c) 2 only
d) None of the above

Solution: d)
Justification: Statement 1: Every production is organized by combining land, labour, physical capital and human
capital, which are known as factors of production.
Statement 2: Both grow. You may have head of Gross physical capital formation term in economic surveys often.
This essentially means development in roads, bridges, capital good etc. that are helpful in increasing production
capabilities.
Statement 3: Physical capital is the variety of inputs required at every stage during production, such as machinery.
They can be very well organized and maintained for e.g. at different factory locations.

428. Consider the following about the Senior Citizens Welfare Fund.

1) It was established along with the Central Social Welfare Board (CSWB).
2) It will be administered by an Inter-Ministerial Committee.
3) The fund shall not be audited by the CAG due to its fundamental character as a welfare fund.

Select the correct answer using the codes below.

a) 2 only
b) 1 and 3 only
c) 1 and 2 only
d) 2 and 3 only
Solution: d)
Justification: Statement 1: CSWF was established in the 1950s, so 1 is wrong. You can read about CSWB here
http://www.cswb.gov.in/index1.asp?linkid=230&langid=1 The Centre brought in Senior Citizens’ Welfare Fund Act,
2015 (SCWF) as part of the Finance Act, 2015, which mandates transfer of unclaimed amounts of policyholders to
the fund (SCWF) after a period of 10 years.
Statement 2: The fund will be administered by an Inter-Ministerial Committee, headed by a Chairperson. The
Committee will be competent to spend money from the fund for satisfying various objectives.
Statement 3: The accounts of the fund will be open to audit by CAG, regularly. The Central Government will present
the annual report and the one furnished by CAG to be laid before the Parliament.

HEALTH

429. The government has launched Intensified Mission Indradhanush (IMI). Consider the following about it.

1) The programme is focused on natals over two years of age.


2) It would be reviewed by the Cabinet Secretary at the National level.
3) Japanese Encephalitis and Rubella diseases are covered by the scheme.
4) Injectable Polio Vaccine has been kept out of the scheme for which a separate nationwide strategy will be devised.

Select the correct answer using the codes below.

a) 1 and 3 only
b) 2 and 4 only
c) 2 and 3 only
d) 1, 2, 3 and 4

Solution: c)
Justification: Statement 1: Through this programme, the Government aims to reach each and every child under two
years of age and all those pregnant women who have been left uncovered under the routine immunization
programme.
The special drive will focus on improving immunization coverage in select districts and cities to ensure full
immunization to more than 90% by December 2018.
Statement 2: It would be reviewed by the Cabinet Secretary at the National level and will continue to be monitored
at the highest level under a special initiative ‘Proactive Governance and Timely Implementation (PRAGATI)’.
Statement 3 and 4: Mission Indradhanush aims to immunize all children under the age of 2 years, as well as all
pregnant women, against seven vaccine preventable diseases. The diseases being targeted are diphtheria,
whooping cough, tetanus, poliomyelitis, tuberculosis, measles and Hepatitis B. In addition to these, vaccines for
Japanese Encephalitis and Haemophilus influenzae type B are also being provided in selected states. In 2016, four
new additions have been made namely Rubella, Japanese Encephalitis, Injectable Polio Vaccine Bivalent and
Rotavirus. Same will be continued by this scheme.

430. Consider the following statements about Malaria and its incidence in India.

1) The majority of malaria reporting districts are in India’s eastern and central parts.
2) The Government has set malaria elimination deadline as 2027.
3) Malaria is most commonly transmitted by a male mosquito or female sand-flies.

Select the correct answer using the codes below.

a) 1 and 2 only
b) 3 only
c) 1 and 3 only
d) 2 only

Solution: a)
Justification: According to the World Malaria Report 2016, India accounts for 89% of the incidence of malaria in the
South-East Asia region.
Statement 1: In India, malaria parasites are found more in the forest areas and plains. Most malaria cases are mainly
concentrated in tribal and remote areas of the country. Six states — Odisha (40%), Jharkhand (20%), Chhattisgarh
(20%), Meghalaya, Arunachal Pradesh and Mizoram (5-7%) report most of the malaria cases in India. These states,
along with tribal areas of Madhya Pradesh and Maharashtra account for 90% of India’s malaria burden.
Statement 2: The Union Health Ministry and Family Welfare for first time has unveiled National Strategic Plan (NSP)
for Malaria Elimination (2017-22). It has set malaria elimination deadline as 2027, three years ahead of the global
deadline (under SDGs).
Statement 3: Malaria is a vector borne disease caused by parasitic protozoans belonging to the Plasmodium type. It
is most commonly transmitted by an infected female Anopheles mosquito.

431. The National Nutrition Mission (NNM) consists of which of the following initiatives?

1) Incentivizing States/UTs for meeting the targets


2) Free vaccination and health checkups for students up to primary school level
3) Setting-up Nutrition Resource Centers
4) Gradually replacing the system of Anganwadi Workers (AWWs) with a Public Private Partnership (PPP) based
contractual system

Select the correct answer using the codes below.

a) 1 and 2 only
b) 1, 3 and 4 only
c) 1 and 3 only
d) 2, 3 and 4 only
Solution: c)
Justification: There are a number of schemes directly/indirectly affecting the nutritional status of children (0-6 years
age) and pregnant women and lactating mothers. Inspite of these, level of malnutrition and related problems in the
country is high. The mission targets to reduce stunting, under-nutrition, anaemia among young children, women
and adolescent girls and reduce low birth weight by at least 2% per annum. Though target to reduce stunting is at
least 2% annum, the mission will strive to achieve reduction in stunting from 38.4% (NFHS-4) to 25% by 2022. The
NNM, as an apex body, will monitor, supervise, fix targets and guide the nutrition related interventions across the
Ministries. The proposal consists of mapping of various Schemes contributing towards addressing malnutrition and
introducing a very robust convergence mechanism, ICT based Real Time Monitoring system. Incentivizing States/UTs
for meeting the targets, incentivizing Anganwadi Workers (AWWs) for using IT based tools, eliminating registers
used by AWWs, Social Audits, setting-up Nutrition Resource Centers and involving masses through Jan Andolan for
their participation on nutrition through various activities, are some of the other systems.

432. The cabinet has approved the setting up of a National Nutrition Mission (NNM) to reduce stunting, under-
nutrition, anaemia among young children, women and adolescent girls. This approved proposal includes mapping of
various Schemes contributing towards addressing malnutrition and

1) Adding a strong ICT based Real Time Monitoring system


2) Incentivizing States/UTs for meeting the targets
3) Setting-up Nutrition Resource Centers
4) Involving masses through Jan Andolan for their participation on nutrition through various activities

Select the correct answer using the codes below.

a) 1 and 2 only
b) 2, 3 and 4 only
c) 1 and 4 only
d) 1, 2, 3 and 4

Solution: d)
Justification: There are a number of schemes directly/indirectly affecting the nutritional status of children (0-6 years
age) and pregnant women and lactating mothers. Inspite of these, level of malnutrition and related problems in the
country is high. There is no dearth of schemes but lack of creating synergy and linking the schemes with each other
to achieve common goal. NNM through robust convergence mechanism and other components would strive to
create the synergy.
A three-year budget of over Rs 9,000 crore has been approved to rein in malnourishment and stunted growth. The
proposal consists of (other than the ones mentioned above):
 introducing a very robust convergence mechanism across various schemes
 incentivizing Anganwadi Workers (AWWs) for using IT based tools
 eliminating registers used by AWWs
 introducing measurement of height of children at the Anganwadi Centres (AWCs)
 Social Audits

433. Which of these is the most accurate ratio of MMR and IMR numbers, with respect to India?

a) Six
b) One by seven
c) One by ten
d) Four

Solution: d)
Justification: IMR is 40 as of 2013, and MMR is 167. So, D is the right answer. We had covered the concepts of IMR
and MMR in the last test.
434. The Ministry of Health and Family Welfare (MOHFW), Government of India, designated which of these as the
nodal agency, responsible for providing coordination and technical guidance for the National Family Health Survey
(NFHS)?

a) International Institute for Population Sciences (IIPS)


b) United Nations Children's Fund (UNICEF)
c) NITI Aayog
d) Directorate General of Health and Nutrition, MoHFW

Solution: a)
Justification: NFHS is a large-scale, multi-round survey conducted in a representative sample of households
throughout India. NFHS is a collaborative project of IIPS, Mumbai and others institutions like. NFHS was funded by
the United States Agency for International Development (USAID) with supplementary support from United Nations
Children's Fund (UNICEF).

435. Family Participatory Care (FPC) as an important concept of health care mainly focuses on

a) Newborn healthcare
b) Geriatric population
c) Urban sanitation and hygiene
d) Health insurance in India

Solution: a)
Justification: Sick and new-born are highly vulnerable and require careful nurturing in order to survive the neonatal
period and first year of life. This approach provides for partnership between health care staff and families in care of
sick newborns admitted in the Special Newborn Care Units (SNCU). Under it, the capacities of parents-attendants
are built in newborn care through a structured training programme.

 MoHFW has recently released Operational Guidelines on FPC.


 The guidelines will be shared with the states for implementation to further improve the quality of care provided in
the SNCUs across the country.

436. In India, Nutrition Rehabilitation Centre (NRC) is a health facility where

1) Children with Severe Acute Malnutrition (SAM) are admitted and managed
2) Institutional deliveries are executed by Auxiliary Nurse Mid-wives (ANMs)
3) The Primary Health Centre of a cluster of villages belonging to a Panchayat is located

Select the correct answer using the codes below.

a) 1 and 2 only
b) 2 and 3 only
c) 1 only
d) 1 and 2 only

Solution: c)
Justification: It has a steady linkage with ICDS that identifies and refers severely malnourished children in the
community.
Children are admitted in NRC as per the defined admission criteria adopted in line with IAP 2006 and new WHO
2009 recommendations and provided with medical and nutritional therapeutic care. Once discharged from the NRC,
the child continues to be in the Nutrition Rehabilitation program till she/he attains the defined discharge criteria
from the program. In addition to curative care special focus is given on timely, adequate and appropriate feeding for
children; and on improving the skills of mothers and caregivers on complete age appropriate caring and feeding
practices. In addition, efforts are made to build the capacity of mothers/caregivers through counselling and support
to identify the nutrition and health problems in their child. NRCs are essentially set up in district hospital campus
and Community Health Centers with NRC ward of bed strength of 20 or 10, a kitchen with proper cooking and feed
demonstration space and attached toilets/bathrooms.

437. LaQshya certification, that was recently seen in news relates to

a) Make in India Campaign


b) International Trade
c) Fisheries quality monitoring
d) Maternity care

Solution: d)
Justification: The program aims to improve quality of care for pregnant women in labour room, maternity
Operation Theatre and Obstetrics Intensive Care Units (ICUs) and High Dependency Units (HDUs). Under it, multi-
pronged strategy has been adopted such as improving infrastructure up-gradation, ensuring availability of essential
equipment, providing adequate human resources, capacity building of health care workers and improving quality
processes in labour room. The NQAS (National Quality Assurance Standards) will monitor quality improvement in
labour room and maternity OT. Every facility achieving 70% score on NQAS will be certified as LaQshya certified
facility.

438. What is ‘Hidden Hunger’?

a) When economically well off lack nutrition


b) Unaccounted hungry people in National Sample Surveys
c) Malnourishment of foetus during pregnancy
d) Deficiency of micronutrients, proteins and vitamins despite getting a calorie rich diet

Solution: d)
Justification: More than 840 million people in the world do not have adequate food to meet their daily food and
nutritional requirements. A far greater number– three billion people – suffer from micronutrient, protein and
vitamin deficiencies or ‘hidden hunger’ because they cannot afford to buy enough fruits, vegetables, legumes, fish
and meat. Diets lacking essential micronutrients – particularly iron, vitamin A, iodine and zinc – increase the risk for
disease, reduce lifespan and reduce mental abilities.

439. With reference to the Pradhan Mantri Matritva Vandana Yojana (PMMVY), consider the following:

1) It is a conditional cash transfer scheme for pregnant and lactating women and adolescent girls
2) It provides a partial wage compensation to women for wage-loss during childbirth and childcare

Which of the above is/are correct?

a) 1 only
b) 2 only
c) Both 1 and 2
d) None

Solution: a)
Justification: Previously Indira Gandhi Matritva Sahyog Yojana (IGMSY), is a maternity benefit program run by the
government of India.
Statement 1: It is only for pregnant and lactating women of 19 years of age or above for first two live births.
Adolescents are legally not permitted to marry and bear children.
Statement 2: This is for childbirth and childcare and to provide conditions for safe delivery and good nutrition and
feeding practices.
440. With reference to the steps taken by the Government to regulate prices of Drugs and Diagnostics, consider the
following statements.

1) Under National Health Mission (NHM), Government is supporting States through National Free Diagnostic Service.
2) National Free Drug Initiative under NHM aims at expanding the availability of free drug provision in all public health
facilities.
3) As per statutory provisions, the clinical establishments are required to display the rates charged for each type of
services provided and facilities available.

Select the correct answer using the codes below.

a) 2 only
b) 1 and 3 only
c) 1, 2 and 3
d) 2 and 3 only

Solution: c)
Justification: Statement 1: This is an initiative to provide essential diagnostic services in public health facilities.
Government of India has brought out guidelines in July 2015 to provide states with a broad framework for
implementing free drug initiatives. The number of tests varies from State to State. An amount of Rs 759 crore has
been approved for free diagnostic service initiative under NHM for 29 States/UTs in 2017-18.
u The initiative would not only provide support to States for purchase of drugs but enabling States to place
transparent system of procurement and quality assurance, robust supply management and logistics that would
ensure highest level of safety and quality of drugs. All States have notified free drug policy. Over 25 States are
implementing IT based supply chain management of drugs.
Statement 3: Under Clinical Establishments (Registration and Regulation) Act, 2010 and Clinical Establishments
(Central Government) Rules, 2012, the clinical establishments (in the States / Union Territories where the Act is
applicable) shall charge the rates for each type of procedure and services within the range of rates determined by
the Central Government from time to time in consultation with the State Governments. The clinical establishments
are also required to display the rates charged for each type of services provided and facilities available, at a
conspicuous place both in the local language and English. The National Council for Clinical Establishments has
approved a standard list of medical procedures and a standard template for costing of medical procedures and
shared the same with the States and UTs.

Medical Council of India (MCI) has amended the Indian Medical Council (Professional Conduct, Etiquette and Ethics)
Regulations, 2002 vide notification dated 21.09.2016, which stipulates that ‘every physician should prescribe drugs
with generic names legibly and preferably in capital letters and he/she shall ensure that there is a rational
prescription and use of drugs’. All the Registered Medical Practitioners under the Indian Medical Council (IMC) Act
have been directed to comply with the aforesaid provisions

441. The India Newborn Action Plan (INAP) was launched with the aim of

1) Ending preventable newborn deaths and stillbirths by 2030


2) Reducing fertility rate to zero by 2030
3) Increasing Aanganwadi enrolment of children of 0-6 years of age to cent per cent

Select the correct answer using the codes below.

a) 1 and 2 only
b) 1 only
c) 2 only
d) 2 and 3 only

Solution: b)
Justification: The salient features are:
 INAP has set the goals for neonatal mortality and stillbirths. The goal is to attain Single Digit Neonatal Mortality and
Stillbirth Rates by 2030.
 INAP is to be implemented within the existing Reproductive, Maternal, Newborn, Child and Adolescent health
(RMNCH A) framework of the National Health Mission (NHM).

442. India has launched one of the world’s largest vaccination campaign against measles and congenital rubella
syndrome (CRS). These diseases are

1) Highly contagious and caused by bacteria


2) Responsible for irreversible birth defects in children even causing death

Which of the above is/are correct?

a) 1 only
b) 2 only
c) Both 1 and 2
d) None

Solution: b)
Justification: These are vaccine preventable diseases.
Statement 1: Measles is a highly contagious disease caused by a virus. Measles can occur in a person who has never
had measles and has no immunity against it.
Statement 2: Congenital rubella syndrome (CRS) is an illness resulting from rubella virus infection during pregnancy.
When rubella infection occurs during early pregnancy, serious consequences—such as miscarriages, stillbirths, and a
constellation of severe birth defects in infants can result.
The campaign aims to vaccinate more than 35 million children in the age group of nine months to 15 years with MR
(measles and rubella) vaccine. For the MR campaign to be effective, it is important that throughout its duration, and
in routine immunization thereafter, no child is left behind.

EDUCATION

443. Annual Status of Education Report (ASER) is published by

a) Ministry of Human Resources and Development


b) Akshay Patra
c) UNICEF India
d) Pratham NGO

Solution: d)
Justification: ASER is India’s largest NGO-run annual survey conducted by Pratham since 2005 to evaluate the
relevance and impact of its programs. Findings are disseminated at national, state, district and village levels, and
influence education policies at both state and central levels. The ASER survey reaches almost every rural district in
the country and covers more than 15,000 households and 650,000 children each year (the sample size is more than
twice that of the survey rounds conducted by the government's National Sample Survey Organization or NSSO). The
ASER model has been adapted for use by thirteen other countries across three continents. These countries came
together organically to form the People's Action for Learning (PAL) Network

444. The National Institutional Ranking Framework (NIRF) outlines a methodology to rank institutions across the
country. It includes which of the following parameters?

1) Research and Professional Practices


2) Global Expansion
3) Graduation Outcomes
4) Perception

Select the correct answer using the codes below.

a) 1, 2 and 3 only
b) 1, 3 and 4 only
c) 2, 3 and 4 only
d) 1 2 and 4 only

Solution: b)
Justification: The parameters for ranking broadly cover “Teaching, Learning and Resources,” “Research and
Professional Practices,” “Graduation Outcomes,” “Outreach and Inclusivity,” and “Perception”. The 2018 rankings
include nine different categories-overall, universities, engineering, colleges, management, pharmacy, medical,
architecture, and law. Out of which sections like medical, architecture and law have been introduced this year. The
Indian Institute of Science in Bengaluru and Delhi’s Miranda House are India’s best university and college this year.

445. Consider the following statements. Revitalising Infrastructure and Systems in Education (RISE) scheme

1) Targets Central Universities in India


2) Provides grants for improvement in quality of infrastructure and research

Which of the above is/are correct?

a) 1 only
b) 2 only
c) Both 1 and 2
d) None

Solution: a)
Justification: S2: It is a loan, not an obligation free grant.

Under RISE, all centrally-funded institutes (CFIs), including central universities, IITs, IIMs, NITs and IISERs, can borrow
from a Rs 1, 00,000 crore corpus over the next four years to expand and build new infrastructure. The initiative aims
to step up investments in research and related infrastructure in premier educational institutions, including health
institutions. Higher Education Financing Agency (HEFA) would be suitably structured for funding this initiative. The
manner in which investment in institutions is provided is likely to be the same as is practiced in HEFA, but there may
be different windows for different institutions.

446. The government has announced a new initiative called Revitalising Infrastructure and Systems in Education
(RISE). Consider the following about it.

1) The scheme emphasizes on Primary Education system of India.


2) The Budget for the current financial year has introduced a Special Educational Cess (SEC) to finance the scheme.
3) Akshay Patra is the lead implementing agency for this scheme in rural India.

Select the correct answer using the codes below.

a) 1 only
b) 2 only
c) 1 and 3 only
d) None of the above

Solution: d)
Justification: The initiative aims to step up investments in research and related infrastructure in premier
educational institutions, including health institutions. It will have a total investment of ₹1, 00,000-crore in the next
four years. Higher Education Financing Agency (HEFA) would be suitably structured for funding this initiative. The
manner in which investment in institutions is provided is likely to be the same as is practiced in HEFA, but there may
be different windows for different institutions. HEFA will leverage the equity to raise up to ₹20,000 crore for the
funding of world-class infrastructure at the IITs, IIMs, the National Institutes of Technology (NITs) etc.

447. The Institutes of eminence scheme under the Union human resource development (HRD) ministry aims to

1) Increase the number and presence of deemed universities in India


2) Project Indian institutes to global recognition
3) Draw global expertise from the best universities abroad to India

Select the correct answer using the codes below.

a) 1 only
b) 2 only
c) 2 and 3 only
d) 1 and 3 only

Solution: b)
Justification: The scheme would select 20 institutes, and these selected institutes will enjoy complete academic and
administrative autonomy. The government will run 10 of these and they will receive special funding. The selection
shall be made through challenge method mode by the Empowered Expert Committee constituted for the purpose.
Only higher education institutions currently placed in the top 500 of global rankings or top 50 of the National
Institutional Ranking Framework (NIRF) are eligible to apply for the eminence tag. The private Institutions of
Eminence can also come up as greenfield ventures provided the sponsoring organisation submits a convincing
perspective plan for 15 years.

448. DIKSHA Portal initiative of HRD ministry focusses for providing a digital platform for which of the following
groups?

a) Students in scheduled areas


b) Girl children in backward and remote regions
c) Teachers, to make their teaching styles more digital
d) Differently abled children with special needs

Solution: b)
Justification: Diksha portal launched with a tagline “National Digital Infrastructures for Our Teacher Our Teacher”.
The portal will consist the whole teacher’s life cycle – from the time they were enrolled as student teachers in
Teacher Education Institutes (TEIs) to after they retire as teachers. Teacher can learn and train themselves for which
assessment resources will be available. The complete work and accomplishment of teachers in Teacher’s
educational institutes will be recorded from start to end point till their retirement. It will help teachers boost their
teaching skills and create their own profile with their skills and knowledge. Not only the government, private
institutes and NGO’s are also allowed to participate in the Diksha initiative.

449. With reference to the National Mission on Education through ICT (NMEICT), consider the following:

1) Certification of attainments of any kind at any level acquired through formal or non-formal means in conventional
or non-conventional fields
2) Platform for sharing of ideas and techniques and pooling of knowledge resources
3) Scholarship and Talent management including identification, nurturing and disbursement electronically
Which of the above is/are the objectives of NMEICT?
a) 1 and 2 only
b) 2 only
c) 1 and 3 only
d) 1, 2 and 3

Solution: d)
Justification: These are the guiding philosophies for NMECIT –

 no talent of the country should be allowed to go waste,


 all the services available through the content delivery portal Sakshat (NMEICT) should be free and freely available
material on the web should be used so as to avoid reinventing the wheel. It has been envisaged as a Centrally
Sponsored Scheme to leverage the potential of ICT, in teaching and learning process for the benefit of all the
learners in Higher Education Institutions in any time anywhere mode. The Mission has two major components:
 providing connectivity, along with provision for access devices, to institutions and learners;
 Content generation.
The content portion of this Mission would have an ambitious vision of catering to the learning needs of more than
50 crore Indians (working population) and of providing a one stop solution to all the requirements of the learning
community.

450. The Objective of the SANKALP project supported by the World Bank is to

a) Enhance institutional mechanisms for skills development


b) Provide drinking water and sanitation to remote habitations in India
c) Increase penetration of primary health care in tribal societies
d) Push up the literacy rate of girl children in India by social and economic empowerment

Solution: a)
Justification: The project also envisages increased access to quality and market-relevant training for the work force.
A Financing Agreement for IBRD loan of USD 250 million (equivalent) for the “Skills Acquisition and Knowledge
Awareness for Livelihood Promotion (SANKALP) Project was signed by India with the World Bank. The Key result
areas for the project include:

 Institutional Strengthening at the National and State Levels for Planning, Delivering, and Monitoring High-Quality
Market-Relevant Training;
 Improved Quality and Market Relevance of Skills Development Programs;
 Improved access to and completion of skills training for female trainees and other disadvantaged groups;
 Expanding skills training through private-public partnerships (PPPs).

451. With reference to various initiatives of the Ministry of Human Resource Development (MHRD), consider the
following statements.

1) E-pathshala is a single point repository of e-resources such as NCERT text-books and various other learning
resources.
2) Padhe Bharat Badhe Bharat focuses on digital literacy.
3) National Reading Initiative intends to organize regular book fairs in major Indian languages.
4) Saransh is an online self-review tool for schools affiliated to the Central Board of Secondary Education (CBSE).

Select the correct answer using the codes below.

a) 1, 2 and 3 only
b) 3 and 4 only
c) 1 and 4 only
d) 1, 3 and 4 only

Solution: c)
Justification: Statement 1: As a part of the Digital India Campaign, the Ministry of HRD has launched ‘e- pathshala’
which is a single point repository of e-resources containing, NCERT text-books and various other learning resources.
Statement 2: Padhe Bharat Badhe Bharat is a sub-programme of Sarva Shiksha Abhiyan launched in 2014 with
special focus on improving language development and to create interest in mathematics.
Statement 3: As a follow up to the above foundational programme, in 2015-16 a programme called the National
Reading Initiative was launched to develop and promote the habit of reading among students in elementary
schools, thereby extending the programme up to class 8. CBSE prescribes textbooks published by National Council of
Educational Research and Training (NCERT) for classes IX to XII.
Statement 4: It helps the schools to look at their performance online at an aggregate level and at the level of each
student. It is an online self-review tool. It allows schools to identify areas of improvement in students, teachers &
curriculum and take necessary measures to implement change. It also provides schools with a view of overall and
individual student’s performance in academic & extra-curricular activities.

452. An Institute of National Importance (INI) is

1) Recognized by Government of India


2) Any international parent institution of a child institution located in India
3) Generally supported financially by the government
4) Restricted to the domains of engineering and medical institutions

Select the correct answer using the codes below.

a) 1, 2 and 3 only
b) 1 and 3 only
c) 2 and 4 only
d) 1, 2 and 4 only

Solution: b)
Justification: An Institute of National Importance, in India is defined as one which serves as a pivotal player in
developing highly skilled personnel within the specified region of the country/state. Only a chosen few institutes
make it to this coveted list and are usually supported by the Government of India or even any other international
institutes to develop centers of excellence in research, academics, and other such elite schools of education. In
India, all of the IITs, NITs, AIIMS, NIPERs, ISI and some other institutes as Institutes of National Importance. It is also
proposed to add to the list IIMs, IISERs, IIESTs and the new AIIMS under PMSSY scheme of GOI once they are
empowered by the Government of India by enacting an act in the Parliament.

453. The committees that have worked or are working towards evolving the National Education Policy lately are

1) Gadgil committee
2) Kasturirangan committee
3) Subramanium Panel
4) Shome Committee

Select the correct answer using the codes below.

a) 1 and 4 only
b) 2 and 3 only
c) 1. 2 and 3 only
d) 3 and 4 only
Solution: b)
Justification: Statement 1: This was setup in 2013 to evolve a framework for the sustainable governance of
Western Ghats.
Statement 2: One such committee was setup on Western Ghats, and another has been setup on educational policy
recently.
Statement 3: This was setup to give a shape to the draft national education policy 2016.
Statement 4: This was on retrospective tax amendments and GAAR.

454. Consider the following statements.

Assertion (A): The literacy rates in India have increased from nearly 50% in 1951 to about 75% in 2010-11.
Reason (R): Public expenditure on education as a percentage of GDP has consistently increased from 1951 levels in
India.

In the context of the above, which of these is correct?

a) A is correct, and R is an appropriate explanation of A.


b) A is correct, but R is not an appropriate explanation of A.
c) A is incorrect, but R is correct.
d) Both A and R are incorrect.

Solution: d)
Justification: The literacy rates in 1951 was only 18% which has increased to 74% in 2010-11. The expenditure on
education as a percentage of GDP rose from 0.64% in 1951–52 to 3.3% in 2013–14 (B.E.). However, there have been
many ups and downs. See the attached graph (Source: World Bank). Y axis shows percentage of GDP, and X axis
shows the years.

455. The recent passing of The Right of Children to Free and Compulsory Education (Amendment) Bill, 2017 by Lok
Sabha was centered on

a) Teacher training
b) Infrastructure development
c) Digitization of schools
d) No detention provision

Solution: a)
Justification: The Bill amends the RTE, 2009 to extend the deadline for teachers to acquire the prescribed minimum
qualifications for appointment. It will enable the in-service untrained elementary teachers to complete their training
and ensure that all teachers at the elementary level have certain minimum standard of qualifications.

SKILLING

456. NABSKILL is a digital interface to promote Skill INDIA. It is an initiative by

a) Reserve Bank of India


b) National Skill Development Corporation of India
c) National Agricultural Bank for Rural Development
d) Labour Bureau

Solution: c)
Justification: The portal capture the details of all stakeholders (Training Seeker, Training Provider, Placement
Agencies and NABARD) involved in skill development initiatives of NABARD. The eligible training provider can check
for their eligibility criteria under “Guidelines” in home page and upon successful registration the training provider
can apply online for seeking grant support from NABARD. The portal captures the details of training related data
viz., trainees profiles, training programme details, post training settlement rate etc. The registered placement
agencies can view the details of successful trainees (Name, Type of training obtained, Details of Training Institute)
who have undergone skill training supported by NABARD. The placement agencies can approach the training
providers for further information on placement, etc.

457. With reference to the Green Skill Development Programme (GSDP), consider the following statements.
1) The programme aims to train scholars with green skills who have considerable experience in the field of sustainable
development.
2) The programme is being implemented by the Ministry of Environment, Forests & Climate Change (MoEF&CC) with
the help of ENVIS portal.
3) The GSDP guarantees an acclaimed job with public agencies in the environmental sector.

Select the correct answer using the codes below.

a) 1 and 2 only
b) 2 only
c) 1 only
d) 1, 2 and 3

Solution: b)
Justification: Statement 1 and 3: The programme aims to train youth who have not been able to continue higher
education due to different financial or social constraints but has an urge to learn new things and do something
fruitful. The government will provide the training in these sectors which are highly sought after, but jobs cannot be
guaranteed. India would need around 10.4 crores of new workforces by the year 2022, in various sectors and hence
skill development is essential to meet the demand. The green skilled workforce having technical knowledge and
commitment to sustainable development will help in the attainment of SDGs, INDCs and National Biodiversity
Targets.

Statement 2: Green skills are those skills needed to adapt processes, services and products to climate change and
the environmental regulations and requirements related to it. The 'green skills' under the programme include --
biodiversity conservationists, Effluent Treatment Plant operators, air pollution abatement workers, equipment
operators, waste handling and candidates for plantation sectors among others.

458. The National Skills Qualifications Framework (NSQF)

1) Organizes qualifications according to a series of levels of knowledge, skills and aptitude


2) Recognizes skills regardless of whether they were acquired through formal, non-formal or informal learning

Which of the above is/are correct?

a) 1 only
b) 2 only
c) Both 1 and 2
d) None

Solution: c)
Justification: Statement 1 and 2: These levels are defined in terms of learning outcomes which the learner must
possess regardless of whether they were acquired through formal, non-formal or informal learning. In that sense,
the NSQF is a quality assurance framework. It is, therefore, a nationally integrated education and competency based
skill framework that will provide for multiple pathways, horizontal as well as vertical, both within vocational
education and vocational training and among vocational education, vocational training, general education and
technical education, thus linking one level of learning to another higher level. This will enable a person to acquire
desired competency levels, transit to the job market and, at an opportune time, return for acquiring additional skills
to further upgrade their competencies.

459. Atal Tinkering Labs are dedicated works spaces for

a) Space scientists
b) Students from Class 6th to Class 12th
c) Social entrepreneurs
d) Diplomats and politicians

Solution: b)
Justification: Atal Tinkering Labs are spaces where students from Class 6th to Class 12th learn innovation skills and
develop ideas that will go on to transform India. The labs are powered to acquaint students with state-of-the-art
equipment such as 3D printers, robotics & electronics development tools, Internet of things & sensors etc. NITI
Aayog will launch the Mentor India Campaign, a strategic nation building initiative to engage leaders who can guide
and mentor students at hundreds of Atal Tinkering Labs, established across the country as a part of the Atal
Innovation Mission.

EMPLOYMENT

460. Disguised unemployment naturally leads to

a) High unemployment rate


b) Low productivity per worker
c) Lower economic growth
d) Decline in product quality

Solution: b)
Justification: Option C and D: These are mere possibilities, and not necessarily flow from disguised unemployment.
In case of disguised unemployment people appear to be employed. If a work requiring at most 5 people employs 8
people, then the extra 3 do not add to the output and are essentially unemployed in disguise, i.e. underutilization of
the extra labour. In India, this is more common in agriculture and low productivity sectors like small households
establishments. For e.g. 90% of industrial units in India face the issue of disguised unemployment. This leads to low
productivity per worker. Factors that contribute to it are: a) large unemployment: b) low skill base of labour; c) less
efficient technology and capital utilization; d) sub-optimal production management etc.

461. What is the most common form of unemployment in urban areas?

a) Seasonal employment
b) Disguised unemployment
c) Educated unemployment
d) No unemployment

Solution: c)
Justification: In case of India we have different nature of unemployment in rural and urban areas. In case of rural
areas, there is seasonal and disguised unemployment. Urban areas have mostly educated unemployment. Seasonal
unemployment happens when people are not able to find jobs during some months of the year. People dependent
upon agriculture usually face such kind of problem. In case of disguised unemployment people appear to be
employed. But, their contribution to work is marginal.

462. Assuming full employment initially, which of the following factors can prevent the restoration of full
employment in an economy in the event of a major macroeconomic shock?

1) Flexibility in the prices of goods


2) Rigidity in labour wage

Which of the above is/are correct?

a) 1 only
b) 2 only
c) Both 1 and 2
d) None

Solution: b)
Justification: After a negative shock, demand for goods and services suddenly reduces due to much lower aggregate
demand (for e.g. recession). Lower demand needs fewer goods and thus fewer labour to produce the goods. This
demand pulls down prices (if prices are flexible) and thus increases demand for goods and services for which more
people need to be employed. If wages are flexible, they will reduce following a period of low demand. Firms would
like to hire more labour if wages are lower. Thus, a combination of lower prices (that pushes up demand) and lower
wages (that pushes up demand for employment by firms) leads to restoration of full employment in the economy. If
the wages and prices are rigid, the abovementioned mechanism fails to operate and thus full employment is difficult
to restore in the economy. The analysis presented above is of course simplistic and much more elaborated models
are given in Economics to analyse them, but it serves our purpose here.

463. The Indian job landscape is in a transitory phase. In such a scenario which of the following can potentially lead
to an increase in employment levels in India?

1) Promoting cottage industries


2) Persistent rise in the Interest rates
3) Enacting stricter labour laws as demanded by the Industry

Select the correct answer using the codes below.

a) 2 only
b) 1 only
c) 1 and 3 only
d) 1, 2 and 3

Solution: b)
Justification: Statement 1: This would give a push to rural and informal sector jobs that contribute to the majority of
employment in India.
Statement 2: This would actually cut down investment. Lower interest rates promote investment.
Statement 3: Industry demands relaxed labour laws with laxer provisions of hiring and firing people. Stricter laws
would actually cut down hiring by firms, as pointed by several economists.

464. Champions of Change" initiative organized by NITI Aayog deals with

a) Promotion of entrepreneurship
b) Eradication of untouchability
c) Protection of Indian diaspora abroad
d) Skill development in Self-Hel Groups (SHGs)

Solution: a)
Justification: It is an institutional initiative to bring together diverse strengths for the benefit of the nation and
society. In Prime Minister Narendra Modi interacted with two hundred CEOs as part of the ‘Champions of Change’
programme. In the programme, groups of Young Entrepreneurs made presentations before the PM on themes such
as - Soft Power: Incredible India 2.0; Education and Skill Development; etc.

465. In case of India we have unemployment both in rural and urban areas. However, the nature of unemployment
differs in both with respect to

1) Year-round availability of employment in urban areas


2) Disguised unemployment that is more prevalent in urban areas
Which of the above is/are correct?

a) 1 only
b) 2 only
c) Both 1 and 2
d) None

Solution: a)
Justification: In case of rural areas, there is seasonal and disguised unemployment. Urban areas have mostly
educated unemployment.
Statement 1: Seasonal unemployment happens when people are not able to find jobs during some months of the
year, mainly due to agriculture.
Statement 2: In case of disguised unemployment people appear to be employed. They have agricultural plot where
they find work. This usually happens among family members engaged in agricultural activity. The work requires the
service of five people but engages eight people. This reduces productivity per person.

466. What are the functions and objectives of the National Employment Service (NES)?

1) Compile and disseminate information on vacancies notified on employment in various industries


2) Assessment of demand and supply of labour in the Labour Market and adjust them
3) Provide Career Counselling and Vocational Guidance
4) Generate internal revenue to finance young entrepreneurs in their ventures

Select the correct answer using the codes below.

a) 1, 2 and 3 only
b) 2 and 4 only
c) 1 and 3 only
d) 1, 2, 3 and 4

Solution: a)
Justification: It is operated by the Directorate General of Employment and Training, Ministry of Labour, and it runs
several Employment Exchanges in order to bring about a better matching of supply-demand of work opportunities.
The other objective is to create a requisite data base for manpower planning and management, career counselling
and vocational guidance with a view to effectively guiding the job-seekers.

Functions (other than those mentioned in the question):

 Collect comprehensive Employment Market Information on a quarterly basis


 Conduct area specific specialized study/surveys to have an assessment of skills available and the marketable
skills required for encouraging the job-seekers for self-employment particularly in rural informal sector

Some of the State Governments arrange disbursement of unemployment allowance to certain specific
categories of jobseekers out of their own resources through the employment exchanges as registered with
them.

467. With reference to the Unorganised Workers' Social Security Act 2008, consider the following:
1) It covers self-employed workers as well as daily-wage workers.
2) It provides for constitution of National Social Security Board.
3) It issues a universal labour identification number (ULIN) to each labour in the unorganized sector.
Select the correct answer using the codes below.
a) 1 and 2 only
b) 3 only
c) 2 and 3 only
d) 1 only

Solution: a)
Justification: Statement 1: It is for the welfare of the unorganized workers (meaning home-based workers, self-
employed workers or daily-wage workers).
Statement 2: The National Social Security Board at the Central level shall recommend formulation of social security
schemes viz life and disability cover, health and maternity benefits, old age protection and any other benefit as may
be determined by the Government for unorganized workers.
Statement 3: LIN has been recently issued under Shramev Jayate Yojana since 2015. It wasn’t under this act.

UNIT-XI
DEVELOPMENT SCHEMES
SCHEMES

468. Nai Manzil Scheme aims at

a) Addressing educational and livelihood needs of minority communities


b) Creating an enabling environment to ensure equal opportunities, equity, social justice and empowerment of
persons with disabilities
c) Generating self-employment to raise the income-generation capacity of target groups among the poor
d) Giving scholarship to encourage students to take up research careers in the areas of basic sciences, engineering and
medicine

Solution: a)
Justification: Option (b) is scheme Deendayal Disabled Rehabilitation Scheme. Option (c) is scheme NRLM.
Option (d) is scheme Kishore Vaigyanik Protsahan Yojana. It aims to provide educational intervention by giving the
bridge courses to the trainees and getting them Certificates for Class XII and X from distance medium educational
system.
It seeks to provide trade basis skill training in four courses at the same time of formal education, in field of (i)
Manufacturing (ii) Engineering (iii) Services (iv) Soft skills. It intends to cover people in between 17 to 35 age group
from all minority communities as well as Madrasa students. This scheme will provide new avenues for minorities for
continuing higher education and also open up employment opportunities in the organized sector.

469. Under the SMARTGRAM initiative of Rashtrapati (President) Bhavan, what is considered as a ‘smart gram
(village)’?

a) A village that is on the transition to become a municipal area


b) A village where a majority of the population is digitally literate
c) A village having required physical and social infrastructure governed with a layer of smart information and
communication systems
d) A village that is close to sub-urban areas of smart cities

Solution: c)
Justification: Under the project the selected villages will see new ventures in agriculture, skill development, energy,
education, heath, employment generation, and entrepreneurship. The model is based on the convergence of
resources and effort by the central government, state government, district administration, PRIs, private sector etc.
to enhance the environment, connectivity and wellbeing of each village.
470. PRAGATI is a unique integrating and interactive platform to

1) Address the grievances of citizens


2) Monitoring and reviewing important government projects

Which of the above is/are correct?

a) 1 only
b) 2 only
c) Both 1 and 2
d) None

Solution: c)
Justification: PRAGATI stands for Pro-Active Governance And Timely Implementation. Key features of the PRAGATI
application are as follows: It is a three-tier system (PMO, Union Government Secretaries, and Chief Secretaries of
the States); Prime Minister will hold a monthly programme where he will interact with the Government of India
Secretaries, and Chief Secretaries through Video-conferencing enabled by data and geo- informatics visuals; Issues
to be flagged before the PM are picked up from the available database regarding Public Grievances, on-going
Programmes and pending Projects; It will also take into consideration various correspondences to PM’s office by
the common people or from high dignitaries of States and/or developers of public projects;

471. Which of the following are key stakeholders in the scheme Udaan which is a Special Initiative for Jammu &
Kashmir.

1) Ministry of Home Affairs (MHA)


2) Corporate training Partners
3) Implementation Agency which is National Skill Development Corporation (NSDC)

Select the correct answer using the codes below.

a) 1 only
b) 2 and 3 only
c) 3 only
d) 1, 2 and 3

Solution: d)
Justification: It is in the nature of partnership between the corporates of India and Ministry of Home Affairs and
implemented by National Skill Development Corporation. The programme aims to provide skills training and
enhance the employability of unemployed youth of J&K. The Scheme covers graduates, post graduates and three-
year engineering diploma holders. It has two objectives:
 To provide an exposure to the unemployed graduates to the best of Corporate India;
 To provide Corporate India, an exposure to the rich talent pool available in the State.
The Scheme aims to cover 40,000 youth of J&K over a period of five years. There is another scheme which is
dedicated to the development of girl child education, so as to promote the admission of girl students. It seeks to
enhance the enrolment of girl students in prestigious technical education institutions through incentives &
academic support.

472. Consider the following about Mission Antyodaya.

1) It encourages partnerships with network of professionals, institutions and enterprises to further accelerate the
transformation of rural livelihoods.
2) District Development Officer (DDO) is the nodal agency for the implementation of the mission.
3) The mission thrusts on not only the physical infrastructure but also social infrastructure with focus on SHG
expansion in selected clusters.

Select the correct answer using the codes below.


a) 1 only
b) 2 and 3 only
c) 1 and 3 only
d) 1, 2 and 3 only

Solution: c)
Justification: In India, 8.88 crore households are found to be deprived and poor households as per Socio Economic
Caste Census (SECC) of 2011 from the perspective of multi-dimensional deprivations such as shelterlessness,
landlessness, households headed by single women, SC/ST household or disabled member in the family. These
households require targeted interventions under government’s various schemes and programmes in areas such as
wage creation, skill generation, social security, education, health, nutrition and livelihood creation. Already, financial
resources to the tune of about Rupees four lakh crore are allocated annually to impact lives of rural poor by several
government ministries/ departments including Ministry of Rural Development in schemes spanning rural wages,
rural roads, skill development, health, education, sanitation, drinking water, electricity, environment, etc. In this
context, ‘Mission Antyodaya’ seeks to converge government interventions with Gram Panchayats as the basic unit
for planning by following a saturation approach by pooling resources - human and financial. Mission Antyodaya
encourages partnerships with network of professionals, institutions and enterprises to further accelerate the
transformation of rural livelihoods. Self Help Groups (SHG) are enablers to convergence approach due to their social
capital and their proven capacity for social mobilisation. The thrust is not only on physical infrastructure but also on
social infrastructure with strengthening of agriculture, horticulture, animal husbandry activities with utmost priority
given to SHG expansion in Mission Antyodaya clusters. Capacity for financial and social audit would also be built at
grassroot level. This would be accomplished by convergence of frontline worker teams, cluster resource persons
(CRPs) and professionals at the Gram Panchayats/ Cluster level. Mission Antyodaya Framework for Implementation
is, thus, based on convergence, accountability and measureable outcomes to ensure that resources are effectively
managed in providing sustainable livelihoods for every deprived household based upon SECC, 2011. The Framework
makes use of technology to ensure that benefits reach those who are most deserving as per SECC data. Backed by a
robust MIS linked to schemes’ data bases using a common Local Governance Directory (LGD) code, it would be
possible to ensure end-to-end targeting against defined set of indicators to measure progress against the baseline.

473. With reference to Pradhan Mantri Vaya Vandana Yojana (PMVVY) which is a pension scheme exclusively for
senior citizens aged 60 years and above, consider the following statements.

1) Senior citizens will get a market dependent rate of interest.


2) National Securities Depository Limited (NSDL) has been given the sole privilege to operate the scheme.
3) It is exempted from goods and services (GST) tax.
4) The scheme offers loan to senior citizens.
5) It does not allow for premature exit.

Select the correct answer using the codes below.

a) 1, 2 and 5 only
b) 2, 3 and 4 only
c) 3 and 4 only
d) D. 1, 2, 3, 4 and 5

Solution: c)
Justification: Statement 1: Under this scheme, senior citizens will get a guaranteed interest of 8% for 10 years
depending upon the investment made by them.
Statement 2: Life Insurance Corporation of India (LIC) has been given the sole privilege to operate the scheme. It can
be purchased offline as well as online through LIC.
Statement 4: The scheme also offers loan up to 75% of the purchase price after 3 policy years (to meet the liquidity
needs). Loan interest will be recovered from the pension instalments and loan will be recovered from claim
proceeds.
Statement 5: The scheme allows for premature exit for the treatment of any critical terminal illness of self or
spouse.

474. Aajeevika Grameen Express Yojana (AGEY) will leverage

a) Community monitored rural transport services


b) Strengths of a Self-help group (SHGs)
c) Both (a) and (b)
d) None of the above

Solution: c)
Justification: It is a sub-scheme, to be launched by MoRD, under Deendayal Antyodaya Yojana – National Rural
Livelihoods Mission (DAY-NRLM). AGEY aims to provide an alternative source of livelihood to members of SHGs by
facilitating them to operate public transport services in backward rural areas. AGEY will provide safe, affordable and
community monitored rural transport services like e- rickshaws, 3 and 4 wheeler motorised transport vehicles to
connect remote villages. These transport vehicles will connect villages with key services and amenities including
access to markets, education and health for the overall economic development of the area. The scheme will be
implemented on a pilot basis. Under it, Community Based Organisation (CBO) is proposed to provide interest free
loan from its own corpus to SHG member for purchase of the vehicle.

475. Adivasi Mahila Sashaktikaran Yojana provides for

1) Grant of mandatory land pattas to tribal women


2) Loans at concessional rates for income generation activities to tribal women
3) Improving basic and digital literacy among tribal women

Select the correct answer using the codes below.

a) 1 only
b) 2 and 3 only
c) 2 only
d) 1, 2 and 3

Solution: c)
Justification: National Scheduled Tribes Finance and Development Corporation (NSTFDC), an apex organisation
under Ministry of Tribal Affairs is implementing this exclusive scheme for tribal women. Under the scheme, ST
women can undertake any income generation activity. Loans up to 90% for scheme costing up to Rs.1 lakh are
provided at a concessional rate of interest of 4% per annum. Ministry of Health & Family Welfare and Ministry of
Women and Child Development have jointly developed Mother and Child Protection card. Consider the following
about it.

476. Every pregnant woman is to be given this Card at the time of registration of pregnancy in all public or private
health facilities. The card would help monitor stages of child development and growth till three years of age.

Which of the above is/are correct?

a) 1 only
b) 2 only
c) Both 1 and 2
d) None

Solution: b)
Justification: Statement 1: This is mandatory only in public health facilities.

Statement 2: It is for documenting and monitoring services for antenatal, intra natal and post-natal care to pregnant
women. It includes information on free transport facility to hospital, institutional delivery, preparation in case of
home delivery for the pregnant women, new born care, immunization and, stages of child development and growth
monitoring of child till 3 year of age. Funds for printing of MCP card are provided to all the states/UTs in the annual
Programme Implementation Plan.

477. The government has launched a web portal called ‘PENCIL’ for effective implementation of the

a) National Child Labour Project (NCLP)


b) Sarva Shiksha Abhiyaan (SSA)
c) Rashtriya Madhmyamika Shiksha Abhiyaan
d) Van Bandhu Scheme in Primary schools

Solution: a)
Justification: We had covered this topic in an explanation in the last test. PENCIL (Platform for Effective
Enforcement for No Child Labour) — an electronic platform for no child labour in the country is being developed by
the Labour Ministry. PENCIL portal has five components — Child Tracking System, Complaint Corner, State
Government, National Child Labour Project and Convergence. The National Child Labour Project (NCLP) Scheme
initiated in the in 1988 to rehabilitate working children by eliminating all forms of child labour through identification
and withdrawal of all children in the Project Area from child labour, Preparing children withdrawn from work for
mainstream education along with vocational training.

478. Under Pradhan Mantri Kisan Sampada Yojana (PMKSY), the Ministry of Food Processing Industries is
implementing various Central Sector Schemes in the food processing sector. The PMKSY has the following schemes?

1) Mega Food Parks


2) Infrastructure for Agro-processing Clusters
3) Food Safety and Quality Assurance Infrastructure
4) Creation of Backward and Forward Linkages

Select the correct answer using the codes below.

a) 1 and 2 only
b) 3 and 4 only
c) 1, 2 and 4 only
d) D. 1, 2, 3 and 4

Solution: d)
Justification: Government is implementing the Central Sector Scheme – PMKSY - with an allocation of Rs. 6,000
crores for the period 2016-20 coterminous with the 14th Finance Commission Cycle. The scheme is expected to
benefit 20 lakh farmers and generate 5, 30,500 direct/ indirect employments by the year 2019-20. It also includes:
 Integrated Cold Chain and Value Addition Infrastructure (on going)
 Creation / Expansion of Food Processing & Preservation Capacities (new)
 Food Safety and Quality Assurance Infrastructure (on going)
 Human Resources and Institutions (on going)
PM Kisan SAMPADA Yojana is expected to leverage investment of Rs. 31,400 crore for handling of 334 lakh MT agro-
produce valued at Rs. 1,04,125 crore, benefiting 20 lakh farmers and generating 5,30,500 direct/indirect
employment in the country by the year 2019-20.

479. Which of these statements about Atal Pension Yojana (APY) is correct?

a) It was started before the 2008 recession.


b) The Central Government contributes 100% of the fund.
c) Only Income Tax payers are eligible under the scheme.
d) None of the above

Solution: d)
Justification: Option A: The Atal Pension Yojana became operational from June 1, 2015 and is available to all the
citizens of India in the age group of 18-40 years.

Option B and C: The Central Government would also co-contribute 50% of the total contribution or Rs. 1000 per
annum, whichever is lower, to each eligible subscriber account, for a period of 5 years, that is, from 2015-16 to
2019-20, to those who join the NPS before 31st December, 2015 and who are not members of any statutory social
security scheme and who are not Income Tax payers. Under the scheme, a subscriber would receive a minimum
guaranteed pension of Rs 1,000 to Rs 5,000 per month, depending upon his contribution, from the age of 60 years.

480. The objective of Comprehensive Handicrafts Cluster Development Scheme (CHCDS) scheme is to

1) To scale up the infrastructural and production chain at Handicrafts clusters using Mega cluster approach
2) Raise the living standards of poor artisans located in far flung areas of the country

Which of the above is/are correct?

a) 1 only
b) 2 only
c) Both 1 and 2
d) None

Solution: c)
Justification: Handicrafts clusters are located in clearly identifiable geographical locations (clusters) that specialize
in specific products, with close linkages and inter dependence amongst the key players in the cluster. The current
need is to have an umbrella cluster development program that drives the development program in an integrated
and holistic manner. In fact these cluster shall inter alia, comprise – Common facility centers; CAD Centers;
Communication Network; Design Banks; Establishment of Resource Centers for major crafts etc. These components
shall help in sorting out the challenges faced by the Handicrafts sector which are complex and numerous.
The broad objectives of the proposed program are as follows:

 To enhance the competitiveness of selected two clusters in terms of increased market share and ensuring
increased productivity by higher unit value realization of the products.
 To ensure effective integration of scattered artisans, building their grass roots enterprises and linking them
to SMEs in the sector to build critical mass for customized interventions and ensure economies of scale in
operations. This will build a supply system that is geared to responding to large-scale orders, adhering to
quality and product standardization, which are pre-requisites of global markets.
 To generate additional livelihood opportunities to the people through specific intervention in segmental sub
sector industry and increase the incomes to the artisans/craftsmen already engaged in this sector.
 To provide requisite support/ linkages in terms of adequate infrastructure, technology, product
diversification, design development, raw material banks, marketing & promotion, social security and other
components that are vital for sustainability of artisans/craftsmen engaged in the Handicrafts sector

481. Consider the following about Jiyo Parsi scheme.

1) It is a Central Sector Scheme focused on increasing employment opportunities for Parsis in India.
2) It provides central assistance for expatriate Parsis to transfer their skills to native Parsis.

Which of the above is/are correct?

a) 1 only
b) 2 only
c) Both 1 and 2
d) None

Solution: d)
Justification: Its main objective is to reverse the declining trend of Parsi population by adopting a scientific protocol
and structured interventions. It has two components: Medical Assistance and Advocacy (Counselling). The first
phase of the scheme was launched initiated in 2013. Minorities are defined both at the Central and State level.
Parsis form a minority community in India, and their welfare is the responsibility of the Central government. For a
state declared minority, state governments provide for welfare plans.
482. Mahila E-Haat is an initiative of the

a) Self Employed Women's Association (SEWA)


b) Kudumbashree, Kerala
c) Ministry of Women and Child Development
d) National Association of Women (NAW)

Solution: c)
Justification: The scheme aims to meet aspirations and needs of women entrepreneurs. This start up at Rashtriya
Mahila Kosh website leverages technology for showcasing products made/manufactured/sold by women
entrepreneurs. They can even showcase their services reflecting their creative potential. This unique e-platform will
strengthen the socio-economic empowerment of women. With the launch of the site itself more than 125000
women are likely to benefit. It is expected to result in a paradigm shift enabling women to exercise control over
their finances by leveraging technology.

483. The objectives of the North East Rural Livelihood Project (NERLP) are

1) To create sustainable community institutions around women Self-Help Groups (SHGs)


2) Increase economic and livelihood opportunities by managing natural resources and improving agriculture for food
security
3) Creating access to finance through linkages with banks and other financial institutions

Select the correct answer using the codes below.

a) 1 only
b) 1, 2 and 3
c) 2 and 3 only
d) 1 and 2 only
Solution: b)
Justification: Online marketplace and hospitality service provider Airbnb has signed four MoUs to boost tourism
North East India. The MoUs were singed under aegis of Ministry of Development of North East Region (DoNER).
They were signed on behalf of North Eastern Council (NEC), North East Tourism Development Council (NETDC),
North East Rural Livelihood Project (NERLP) and North Eastern Region Community Resource Management Project
(NERCORMP).
Objectives of NERLP are:

 Create sustainable community institutions around women Self-Help Groups (SHGs), Youth Groups of men
and women (YG) and Community Development Groups (CDG).
 Build capacity of community institutions for self-governance, bottom up planning, democratic functioning
with transparency and accountability.
 Increase economic and livelihood opportunities by
 Managing natural resources and improving agriculture for food security and income enhancement from
farming and allied activities
 Skill development of youth for employability and establishment of self and/or group managed enterprises
 Establishing backward and forward linkages for economic enterprises
 Creating access to finance through linkages with banks and other financial institutions
 Creating critical infrastructures
 Develop partnership of community institutions for natural resource management, microfinance, market
linkages, and sectoral economic services.
 The project is for a period of five years. It covers two districts each in Mizoram, Nagaland, Sikkim and 5
districts in Tripura.

484. With reference to District Mineral Foundations (DMFs), consider the following statements.

1) It is a trust set up as a non-profit body, in those districts affected by the mining works
2) It was mandated through the Mines and Minerals (Development & Regulation) Amendment Act, (MMDRA) 2015
3) Every holder of a mining lease in the concerned district shall contribute royalty to the DMF.

Select the correct answer using the codes below.

a) 1 only
b) 2 and 3 only
c) 1 and 3 only
d) 1, 2 and 3

Solution: d)
Justification: Statement 1: District Mineral Foundation (DMF) is a trust set up as a non-profit body, in those districts
affected by the mining works, to work for the interest and benefit of persons and areas affected by mining related
operations. It is funded through the contributions from miners.
Statement 2: The Central Government notified on 17 September 2015, the rates of contribution payable by miners
to the DMFs. In case of all mining leases executed before 12th January, 2015 (the date on which MMDR
Amendment Act came into force) miners will have to contribute an amount equal to 30% of the royalty payable by
them to the DMFs. Where mining leases are granted after 12.01.2015, the rate of contribution would be 10% of the
royalty payable (Subsequent to the enactment of MMDR Amendment Act, mining leases are given out after
auctions; hence, a lower levy). Thus, every holder of a mining lease or a prospecting licence-cum-mining lease shall,
in addition to the royalty, pay to the District Mineral Foundation of the district in which their mining operations are
carried on.
485. Aadhaar is presently being used as an identifier in respect of following school education related schemes?

1) Mid-Day Meal Scheme (MDMS)


2) Sarva Shiksha Abhiyan (SSA)
3) Rashtriya Madhyamik Shiksha Abhiyan (RMSA)
4) Inclusive Education for Disabled at Secondary Stage (IEDSS)

Select the correct answer using the codes below.

a) 1 and 2 only
b) 2, 3 and 4 only
c) 3 and 4 only
d) 1, 2, 3 and 4

Solution: d)
Justification: This is to reduce ghost beneficiaries, leakages in distribution of provisions. Also, this enforces
accountability, transparency and efficiency in the governance of the schemes. Other covered schemes are:

 National Scheme of Incentive to Girls for Secondary Education (NSIGSE)


 National Means-cum-Merit Scholarship Scheme (NMMSS)
 Centrally Sponsored Scheme for providing quality education in Madrasa (SPQEM)

486. With reference to the Pt Deen Dayal Upadhayay Vigyan Gram Sankul Pariyojana, consider the following:

1) Clusters of villages will be selected under the scheme for transforming them with the help of S&T interventions.
2) The scheme exhorts local communities to migrate in for acquiring skilled jobs so that they can return and contribute
to the management of technological interventions in the villages.
3) It covers development of manufacturing and services sector exclusively, as agricultural sector modernization would
overlap with other schemes.

Select the correct answer using the codes below.

a) 1 only
b) 2 and 3 only
c) 1 and 3 only
d) 1, 2 and 3

Solution: a)
Justification: Under the scheme, the department of science and technology has conceived to adopt a few clusters of
villages in Uttarakhand and transform them to become self-sustainable in a time bound manner through the tools of
Science and Technology (S&T). The key deliverable in this approach is to utilise local resources and locally available
skill sets and convert them in a manner using science and technology, that substantial value addition takes place in
their local produce and services which can sustain the rural population locally. Areas of interventions in these
selected clusters would be processing and value addition of milk, honey, mushroom, herbal tea, forest produce,
horticulture and local crops, medicinal & aromatic plants and traditional craft and handloom of Uttarakhand.
Practice of agriculture, agro-based cottage industries and animal husbandry in an eco-friendly manner will be
emphasized during the implementation of the project.

487. Under the Pradhan Mantri Rojgar Protsahan Yojana (PMRPY)

1) Government is providing support grants for vocational training under RSETIs


2) State governments are financially supported to contribute fifty per cent of the apprenticeship hiring cost of MSMEs
Which of the above is/are correct?

a) 1 only
b) 2 only
c) Both 1 and 2
d) None

Solution: d)
Justification: PMRPY has been in operation since August 2016. Under it, Government is paying 8.33% contribution
of Employers to Employees’ Pension Scheme (EPS) in respect of new employees (joined on or after 1st April 2016)
having new Universal Account Number (UAN), with salary up to Rs. 15,000/- per month. The scheme has dual
benefit i.e. it incentivize employers for increasing employment base of workers establishments and enables large
number of workers to find jobs in such establishments. Its direct benefit is that these workers get access to social
security benefits of organized sector. Till now, the scheme has produced quite encouraging results and has added
about 31 Lakhs beneficiaries to formal employment involving expenditure of more than Rs. 500 crore.

488. The first state in India to introduce the Mid-Day Meal (MDM) scheme was

a) Kerala
b) Gujarat
c) Tamilnadu
d) Bihar

Solution: c)
Justification: Other than the TN model of MDM, in 2001, the Supreme Court asked all state governments to begin
this programme in their schools within six months. Later, it was made a central programme that today feeds more
than 12 crore children all over India. The Midday Meal Scheme is covered by the National Food Security Act, 2013.
The central and state governments share the cost of the Midday Meal Scheme, with the center providing 60 percent
and the states 40 percent

489. Consider the following about the ‘Saubhagya’ scheme that intends to provide energy access to all.

1) The scheme applies to rural households alone because the electricity development in urban areas is managed by
the municipalities.
2) The scheme would provide free connections and free electricity to all poor rural households.

Which of the above is/are correct?

a) 1 only
b) 2 only
c) Both 1 and 2
d) None

Solution: d)
Justification: Statement 1: It focuses on last mile connectivity and electricity connections to all remaining un-
electrified households in rural as well as urban areas to achieve universal household electrification in the country.

Statement 2: Poor households would be provided electricity connections free of cost. Other households would also
be provided electricity connections under the scheme on payment of Rs.500 only. There is no provision in the
scheme to provide free power to any category of consumers. The cost of electricity consumption shall have to be
paid by the respective consumers as per prevailing tariff of the DISCOM/Power Dept.
490. With reference to key initiatives taken by the Department of Expenditure, Ministry of Finance, in the last few
years, consider the following.

1) All schemes and projects have now an Outcome Framework.


2) Each Scheme will now have a start and a sun-set date co-terminus with the Finance Commission Cycle.
3) Number of both central Sector and Centrally Sponsored Schemes were increased for improved coordination
between implementing agencies.

Select the correct answer using the codes below.

a) 1 only
b) 1 and 2 only
c) 2 and 3 only
d) 3 only

Solution: b)
Justification: Statement 1 and 2: An outcome framework focuses on the impact of the scheme, rather than only the
outlay (finances) of the scheme. Schemes and projects have now an Outcome Framework, which has been
formulated through consultation with the implementing Ministries/ Departments, NITI Aayog and the Department
of Expenditure. A consolidated Outcome Budget 2017-18 was presented in the Parliament as a part of the Budget
Documents. Third party Evaluation has also been formally built into the appraisal and approval framework to ensure
periodic feedback, a more concerted focus on outcomes and enhanced growth performance

Statement 3: Both Central Sector and Centrally Sponsored Schemes were rationalized (reduced and consolidated
significantly in number) in consultation with the line Ministries.

491. Consider the following statements. Deen Dayal Upadhyaya Grameen Kaushalya Yojana (DDU-GKY) is a part of
the

1) National Rural Livelihood Mission (NRLM)


2) Skill India campaign
3) National Employment Exchange Programme (NEEP)

Select the correct answer using the codes below.

a) 1 only
b) 1 and 2 only
c) 2 and 3 only
d) 1, 2 and 3

Solution: b)
Justification: DDU-GKY, administered by MoRD, is a part of the National Rural Livelihood Mission (NRLM), tasked
with the dual objectives of adding diversity to the incomes of rural poor families and cater to the career aspirations
of rural youth. DDU-GKY is uniquely focused on rural youth between the ages of 15 and 35 years from poor families.
As a part of the Skill India campaign, it plays an instrumental role in supporting the social and economic programs of
the government like the Make In India, Digital India, Smart Cities and Start- Up India, Stand-Up India campaigns.

492. Consider the following statements about the National Health Protection Scheme (NHPS).

1) It is a Universal Health Insurance scheme.


2) It is a Central Sector Scheme financed entirely by the Centre and implemented by the States.

Which of the above is/are correct?


a) 1 only
b) 2 only
c) Both 1 and 2
d) None

Solution: d)
Justification: NHPS (Ayushman Bharat Scheme) dubbed ‘Modicare’ seeks to provide health insurance of Rs 5 Lakh to
10 crore poor and vulnerable households. Under it, up to Rs 5 lakh insurance cover will be provided to each family
per year in secondary and tertiary care institutions. NHPS will have 50 crore beneficiaries, making it world’s largest
government-funded healthcare programme to provide quality health cover to population larger than combined
citizenry of US, UK, Germany and France. The Centre had drawn up health plan for the scheme in which 40% of fund
for the scheme has to come from state. Under it, the central government will contribute Rs 2,000 crore to scheme
out of a total cost of Rs 5,500-6,000 crore – the remaining amount is to be paid by the state governments. Health
Ministry is holding consultations with States on NHPS because there is lack of clarity on some aspects of the scheme
concerning the role and participation of states.

493. E-Shakti for digitisation of Self Help Groups (SHGs) is a pilot project of

a) Prime Minister’s Office (PMO)


b) Micro Units Development & Refinance Agency Ltd. (MUDRA)
c) Reserve Bank of India (RBI)
d) National Bank for Agriculture and Rural Development (NABARD)
Solution: d)
Justification: It was initiated to address certain concerns like improving the quality of book keeping of SHGs and
to enable banks to take informed credit decisions about the group through a Management Information
System (MIS). The project aims at digitisation of all the SHG accounts to bring SHG members under the fold of
Financial Inclusion. This increases bankers’ comfort in credit appraisal. Over 1 lakh SHGs have been digitized by
now.

494. Saubhagya Scheme, that was recently in news, intends to achieve

a) Universal vaccination for all students in Primary public schools


b) Universal household electrification in all parts of the country
c) Smokeless chullhas in all rural areas by 2022
d) A direct LPG pipeline distribution network in every panchayat area

Solution: b)
Justification: Also called as Pradhan Mantri Sahaj Bijli Har Ghar Yojna, its objective is to achieve universal household
electrification in all parts of the country. The scheme primarily benefits rural areas, which have vast majority of
households without power connections. It is being funded to extent of 60% by central grants, 30% by bank loans
and 10% by states. Of total outlay of Rs 16,320 crore, rural areas will get Rs 14,025 crore. For urban households
outlay is Rs. 2,295 crore. States will be provided with incentive of 50% of their loan being converted to grants, if
electrification targets under the scheme are met by 31 December 2018. The government will use Socio Economic
and Caste Census (SECC) 2011 data to identify beneficiaries for free electricity connections.

495. Consider the following with reference to Unnat Bharat Abhiyan.

1) It is a flagship programme of the Ministry of Rural Development.


2) It intends to bring in transformational change in rural developmental process.
3) It focuses primarily on the capacity building of public functionaries.

Select the correct answer using the codes below.


a) 1 only
b) 1, 2 and 3
c) 2 and 3 only
d) 2 only

Solution: d)
Justification: It is a flagship programme of the Ministry of Human Resources Development, with the intention to
enrich Rural India. It aims to leverage the knowledge base and resources of the Premier Institutions of the country
to bring in transformational change in rural developmental process. It also aims to create a vibrant relationship
between the society and the higher educational institutes, with the latter providing the knowledge and technology
support to improve the livelihoods in rural areas and to upgrade the capabilities of both the public and private
organizations in the society. Under the Unnat Bharat Abhiyan 2.0, the institutions have been selected on a
Challenge Mode and the scheme has been extended to 750 reputed Higher Educational Institutes (both public and
private) of the country. IIT Delhi has been designated to function as the National Coordinating Institute for this
programme and the Ministry intends to extend the coverage to all the reputed Higher Educational Institutes, in a
phased manner. Each selected institute would adopt a cluster of villages / panchayats and gradually expand the
outreach over a period of time.

496. With reference to North East Road Network Connectivity Project (NERNCP), consider the following
statements:

1) The project will be based on public-private partnership model.


2) It will enhance connectivity with inter-state roads and International Borders concerning the North- eastern region.
3) Recently, India and Japan signed a document on Japanese loan and aid for the project.

Select the correct answer using the codes below.

a) 1 only
b) 2 and 3 only
c) 1 and 2 only
d) 1, 2 and 3

Solution: d)
Justification: Statement 1: The project will be executed in EPC (Engineering, procurement and Construction) Mode.
Statement 2: First phase focuses on development of National Highways in Meghalaya and Mizoram. It can
complement India’s connectivity initiatives in Bangladesh, Myanmar and beyond.
Statement 3: Japan will extend a loan of over two thousand crore rupees to India for ‘North East Road Network
Connectivity Improvement Project’ to improve NH-40 and construct a bypass on NH-54 in the Northeast.
It will be financially supported by loan assistance from Japan International Cooperation Agency (JICA).

UNIT-XII
ENVIRONMENT ECONOMICS

ENVIRONMENT

WATER
UNIT-XIII
ECONOMISTS AND THEIR CONTRIBUTIONS

ECONOMISTS AND THEIR CONTRIBUTIONS


UNIT-XIV
INSTITUTIONS –ROLE AND FUNCTIONS

NATIONAL INSTITUTIONS

497. Which of the following authorities is empowered to compound contraventions under Foreign Exchange
Management Act, 1999 (FEMA)?

a) Securities and Exchange Board of India


b) Department of Economic Affairs, Ministry of Finance
c) Reserve Bank of India
d) Foreign Investment Promotion Board (FIPB)

Solution: c)
Justification: Solve such questions by an informed guess. RBI administers the FEMA act (and not any other body as
mentioned above, with the partial exception of SEBI). Answer must be C. Contravention is a breach of the provisions
of the Foreign Exchange Management Act (FEMA), 1999 and rules/ regulations etc. issued there under.
Compounding refers to the process of voluntarily admitting the contravention, pleading guilty and seeking redressal.
The Reserve Bank is empowered to compound any contraventions as defined under section 131 of FEMA, 1999
except the contravention under section 3(a) 2 ibid, for a specified sum after offering an opportunity of personal
hearing to the contravener. It is a voluntary process in which an individual or a corporate seeks compounding of an
admitted contravention. It provides comfort to any person who contravenes any provisions of FEMA, 1999.

498. The Preamble of the Reserve Bank of India describes the basic functions of the Reserve Bank. Which of the
following is NOT included in its preamble?

1) Regulate the issue of Bank notes


2) Keeping of reserves with a view to securing monetary stability in India
3) Operate the currency and credit system of the country
4) Mapping economic growth and development in India

Select the correct answer using the codes below.

a) 2 and 3 only
b) 3 only
c) 1 and 4 only
d) 4 only

Solution: d)
Justification: The Preamble reads: "to regulate the issue of Bank notes and keeping of reserves with a view to
securing monetary stability in India and generally to operate the currency and credit system of the country to its
advantage; to have a modern monetary policy framework to meet the challenge of an increasingly complex
economy, to maintain price stability while keeping in mind the objective of growth."
Statement 4: While RBI keeps the growth objective in mind, it is not mandated to map or statistically chart the
economic growth pattern of India. This is done by the Ministry of Finance.
499. The Department of Revenue, Ministry of Finance, is mainly responsible for which of the following
functions?

1) All matters relating to levy and collection of Direct and Indirect Taxes
2) Investigation into economic offences and enforcement of economic laws.
3) Prevention and combating abuse of Narcotic drugs and psychotropic substances and illicit traffic therein

Select the correct answer using the codes below.

a) 1 only
b) 1, 2 and 3
c) 2 and 3 only
d) 2 only

Solution: b)
Justification: Statement 3: NARCOTICS CONTROL DIVISION: Framing of licensing policy for cultivation of Opium
poppy, production of opium and export and pricing of opium. Coordination of the working of Committee of
Management and issues relating of UN and International Organisations. Work relating to forfeiture of property
under Smugglers and Foreign Exchange Manipulators (Forfeiture of Property) Act, 1976 and Narcotics Drugs and
Psychotropic Substances Act, 1985.

Other functions include:

 Enforcement of FEMA
 Levy of taxes on sales in the course of inter-state trade or commerce.
 Residual work of Gold Control
 Cadre Control of IRS (Group-A) and IRS (C&CE) (Group-A).

CENTRAL ECONOMIC INTELLIGENCE BUREAU: Coordinating and strengthening of the intelligence gathering
activities, the investigative efforts and enforcement action by various agencies concerned with investigation into
economic offences and enforcement of economic laws.

500. The government is planning to facilitate greater investments from South Korea and would strengthen the
“Korea Plus” cell in this regard. Korea Plus comprises of

1) Chairman, FICCI
2) Representatives from the Korean ministry of industry, trade and energy
3) Representatives from Invest India
4) Secretary, Ministry of Finance

Select the correct answer using the codes below.

a) 2 and 3 only
b) 1 and 4 only
c) 1, 2 and 3 only
d) 1, 2, 3 and 4

Solution: a)
Justification: “Korea Plus” is a strategic initiative between the Governments of India and South Korea “to promote,
facilitate and retain Korean investments in India.” Korea Plus comprises of a representative each from the Korean
ministry of industry, trade and energy, Korea Trade Investment and Promotion Agency as well as three
representatives from Invest India — the national investment promotion and facilitation agency of India. The
mandate of Korea Plus covers the entire investment spectrum including supporting Korean enterprises entering the
Indian market for the first time, looking into issues faced by Korean companies doing business in India and policy
advocacy to the Indian government on their behalf. Korea Plus will act as a mediator in arranging meetings, assisting
in public relations and research, evaluation and provide information and counselling in regard to Korean companies’
investing in India.

501. With reference to the new Bureau of Indian standards (BIS) Act 2016, consider the following:

1) The Act establishes the Bureau of Indian Standards (BIS) as the National Standards Body of India.
2) It makes hallmarking of precious metal articles voluntary based on self-declaration.
3) There is a provision for repair or recall of the products bearing Standard Mark but not conforming to the relevant
Indian Standard.

Select the correct answer using the codes below.

a) 1 and 2 only
b) 2 and 3 only
c) 1 and 3 only
d) 1, 2 and 3

Solution: c)
Justification: Parliament passed this law to replace the BIS Act of 1986. It was brought into force with effect from
12th October, 2017.
Highlights of the act:
National standards body: The Act establishes the Bureau of Indian Standards (BIS) as the National Standards Body
of India.
Compulsory certification: The Act has enabling provisions for the Government to bring under compulsory
certification regime any goods or article of any scheduled industry, process, system or service which it considers
necessary in the public interest or for the protection of human, animal or plant health, safety of the environment, or
prevention of unfair trade practices, or national security.
Hallmarking: Enabling provisions have also been made for making hallmarking of the precious metal articles
mandatory.
Simplified conformity: The new Act also allows multiple type of simplified conformity assessment schemes
including self-declaration of conformity against a standard which will give simplified options to manufacturers to
adhere to the standards and get certificate of conformity.
Authority for verification: The Act enables the Central Government to appoint any authority/agency, in addition to
the BIS, to verify the conformity of products and services to a standard and issue certificate of conformity.
Repair or recall: There is also a provision for repair or recall, including product liability of the products bearing
Standard Mark but not conforming to the relevant Indian Standard.

502. Federation of Indian Export Organizations (FIEO) is a trade promotion organisation in India. It was jointly
established by

a) Jointly by Ministry of Commerce and private trade bodies


b) WTO – India Chapter
c) FICCI
d) Trade Facilitation Agreement (TFA)

Solution: a)
Justification: It was jointly established in 1965 by the Ministry of Commerce of the Government of India and private
trade and industry. The organisation is responsible for representing and assisting Indian entrepreneurs and
exporters in foreign markets. The Federation has evolved into a key player in the promotion of trade, investment
and collaboration. FIEO provides the content, direction and thrust to India’s expanding international trade. FIEO
represents the interest of professional government recognized exporting firms, consultancy firms, service exporters,
banks, export management training institutes etc. FIEO members representing large, medium & small scale
exporting units contribute more around 70% global exports of our country. Its membership comprises of exporting
firms with strong credentials, called Government-recognized Export House, Star Export House, Trading House, Star
Trading House and Premier Trading House besides Consultancy firms.

503. Consider the following statements. The body “Invest India”

1) Was established by Confederation of Indian Industries (CII) as a non-profit entity


2) Guides prospective foreign investors in India
3) Is a component of NeGP (National e-Governance plan)

Select the correct answer using the codes below.

a) 1 only
b) 2 only
c) 1 and 3 only
d) 3 only

Solution: b)
Justification: The government has set up Invest India—a joint-venture company between the Department of
Industrial Policy and Promotion and FICCI, as a not-for-profit, Single window facilitator, for prospective overseas
investors and to act as a structured mechanism to attract investment. In addition, the government has initiated
implementation of the e-Biz Project, a mission mode project under the NeGP (National e-Governance plan) for
promoting an online single window at the national level for business users. The project aims at enhancing India's
business competitiveness through a service oriented, event-driven G2B (Government to Business) interaction.

504. Export Inspection Council (EIC) was established as a statutory authority to ensure sound development of export
trade of India through Quality Control and Inspection and for matters connected thereof. Consider the following
with reference to it.

1) It is empowered to notify commodities which will be subject to quality control and inspection prior to export.
2) It establishes standards of quality for notified commodities that will be subject to quality control and inspection
prior to export.

Which of the above is/are correct?

a) 1 only
b) 2 only
c) Both 1 and 2
d) None

Solution: c)
Justification: ECI was established by Central Government under Section 3 of Export (Quality Control and Inspection)
Act, 1963. It was established to ensure sound development of export trade of India through Quality Control and
Inspection and for matters connected thereof. EIC is advisory body to Central Government. It is empowered to
notify commodities which will be subject to quality control and inspection prior to export. It establishes standards of
quality for such notified commodities. It also specifies type of quality control and inspection to be applied to such
commodities. EIC also exercises technical and administrative control over the five Export Inspection Agencies (EIAs),
one each at Chennai, Delhi, Kochi, Kolkata and Mumbai established by Ministry of Commerce for purpose of
implementing various measures and policies by it.

505. Consider the following about Agrinnovate India Ltd. (AgIn).

1) It is a "Not-for-profit" Company owned by Department of Agricultural Research & Education (DARE).


2) It is mandated to share the products of R&D in agriculture and allied sciences with the stakeholders of agricultural
sector in India and abroad.
Which of the above is/are correct?

a) 1 only
b) 2 only
c) Both 1 and 2
d) None

Solution: b)
Justification: Statement 1: It was incorporated under the Companies Act, 1956 (No. 1 of 1956) on 19th October,
2011. It is a "for profit" Company owned by Department of Agricultural Research & Education (DARE), Ministry of
Agriculture, Government of India. It is to act as an effective interface between Indian Council of Agricultural
Research (ICAR- an autonomous organization under DARE) on one side and the Stakeholders of agricultural sector
(Farmers; Public & Private Sector firms; R&D organizations; Educational Institutions- all of these at National and
International level) on the other side, for a significant purpose of securing, sustaining and promoting global
agricultural development.

Statement 2: AgIn is an endeavour of ICAR through its parent department-- of Agricultural Research and Education,
to share the products of research and development in agriculture and allied sciences with all those who are
stakeholders of agricultural sector both at national and international level.

506. World Sustainable Development Summit (WSDS) is a flagship forum of

a) The Energy and Resources Institute (TERI)


b) International Energy Policy Institute (IEPI)
c) United Nations Environment Programme
d) Convention on Biological Diversity Secretariat

Solution: a)
Justification: WSDS is flagship forum of The Energy and Resources Institute (TERI). It seeks to bring together global
leaders and thinkers in the fields of sustainable development, energy and environment sectors on common
platform. The theme of WSDS 2018, to be held in New Delhi, is ‘Partnerships for a Resilient Planet’. It seeks to
create action frameworks to resolve some of most urgent challenges facing developing economies in backdrop of
climate change. TERI is a non-profit research institution that conducts research work in the fields of energy,
environment and sustainable development. It aims to focus on formulating local and national level strategies for
shaping global solutions to critical issues. It was established in 1974 as Tata Energy Research Institute and was
renamed The Energy and Resources Institute in 2003.

507. With reference to the Competition Commission of India (CCI), consider the following statements.

1) It is a quasi-judicial statutory body.


2) It is required to give opinion on competition issues on reference received from statutory authorities.
3) It has the power of imposing fines of private players for unfair business practices.
4) It jurisdiction does not extend to Public Sector Units (PSUs).

Select the correct answer using the codes below.

a) 1, 2 and 3 only
b) 2, 3 and 4 only
c) 1, 3 and 4 only
d) 2 and 4 only
Solution: a)
Justification: Statement 1: CCI is quasi-judicial statutory body established under The Competition Act, 2002. It
consists of Chairperson and 6 Members appointed by Central Government.
Statement 2: CCI aims to eliminate practices that adversely affect competition in different industries and protect
interests of consumers and ensure freedom of trade. Its mandate is to eliminate practices having adverse effect on
competition, promote and sustain competition, protect interests of consumers and ensure freedom of trade in
markets of India. It is also required to give opinion on competition issues on reference received from statutory
authority established under any law and undertakes competition advocacy, create public awareness and impart
training on competition issues.
Statement 3: For e.g. CCI has imposed fine of more than Rs. 54 crore recently on Jet Airways, InterGlobe Aviation
and SpiceJet for unfair business practices with respect to fixing fuel surcharge on cargo transport.
Statement 4: CCI imposed a penalty of ₹591.01 crore in 2017 upon Coal India Limited (CIL) on finding that CIL and its
subsidiaries violated the Competition Act by imposing unfair and discriminatory conditions in Fuel Supply
Agreements (FSAs) with power producers for supply of non-coking coal.

508. Serious Frauds Investigation Office (SFIO) is a

1) Multi-disciplinary organization under Ministry of Home Affairs


2) Usually takes up complex cases with inter-departmental and multi- disciplinary ramifications
3) Can undertake investigations under the provisions of the Companies Act

Select the correct answer using the codes below.


a) 1 and 2 only
b) 3 only
c) 2 and 3 only
d) 1, 2 and 3

Solution: c)
Justification: It is a multi-disciplinary organization under Ministry of Corporate Affairs, consisting of experts in the
field of accountancy, forensic auditing, law, information technology, investigation, company law, capital market and
taxation for detecting and prosecuting or recommending for prosecution white-collar crimes/frauds.
The SFIO will normally take up for investigation only such cases, which are characterized by:

 Complexity and having inter-departmental and multi- disciplinary ramifications;


 Substantial involvement of public interest to be judged by size, either in terms of monetary
 The possibility of investigation leading to or contributing towards a clear improvement in systems, laws or
procedures.

Statement 2 and 3: The SFIO shall investigate serious cases of fraud received from Department of Company Affairs.
SFIO may also take up cases on its own.
The SFIO would make investigations under the provisions of the Companies Act, 1956 and would also forward the
investigated reports on violations of the provisions of other acts to the concerned agencies for
prosecution/appropriate action.

509. With reference to the Energy Efficiency Services Limited (EESL), consider the following statements.

1) It is a joint venture of National Grid Network (NGN) and Public Energy Service Company (PESC).
2) It was established with the sole objective of providing and generating finance for energy efficiency projects.
3) It acts as the resource centre for capacity building of State DISCOMs.
4) It is owned on a public-private partnership (PPP) basis.

Select the correct answer using the codes below.


a) 1 and 3 only
b) 1, 2 and 4 only
c) 3 only
d) 2, 3 and 4 only

Solution: c)
Justification: It is a joint venture of NTPC Limited, Power Finance Corporation, Rural Electrification Corporation and
POWERGRID set up under Ministry of Power to facilitate implementation of energy efficiency projects. EESL seeks to
unlock energy efficiency market in India, estimated to at US$12 billion that can potentially result in energy savings
of up to 20 per cent of current consumption, by way of innovative business and implementation models. It is the
largest public Super Energy Service Company (ESCO) in the world. EESL has been able to successfully aggregate
demand and has undertaken bulk distribution of LED bulbs that has led to more than 80% reduction of costs.

510. National Pharmaceutical Pricing Authority (NPPA)

1) Fixes the prices of Scheduled medicines


2) Decides the pricing of new drugs introduced in the market
3) Monitors the prices of medicines fixed by the manufacturers
4) Issues notices for the violation of fixed medicine prices
5) Takes action against erring pharmaceutical companies or retailers violating price orders

Select the correct answer using the codes below.


a) 1, 3 and 4 only
b) 2, 3 and 4 only
c) 1, 2 and 5 only
d) 1, 2, 3, 4 and 5

Solution: d)
Justification: The NPPA fixes the prices of Scheduled medicines and new drugs as per the provisions of Drugs (Prices
Control) Order, 2013 (DPCO, 2013). The Minister said that the NPPA monitors the prices of medicines fixed by the
manufacturers and issues notices for the violation of the prices. It verifies the prices of medicines randomly from
the respective websites of the drug manufacturing companies and wherever any violation is found, action is taken
as per the provisions of DPCO, 2013. This is an ongoing process

511. India's Membership for European Bank for Reconstruction & Development (EBRD) will impact India in which of
the following ways?

1) India can leverage the technical assistance and sectoral knowledge of the bank for the benefit of development of its
private sector.
2) It would enable Indian nationals to get the employment opportunity in the Bank.
3) India’s financial obligations to EBRD would come down as members receive grants from EBRD for their membership.
4) India's investment opportunities would get a boost.

Select the correct answer using the codes below.

a) 1, 2 and 4 only
b) 1, 3 and 4 only
c) 2 and 3 only
d) 1, 2, 3 and 4

Solution: a)
Justification: Union Cabinet has approved the membership. Necessary steps will be initiated by the Department of
Economic Affairs, Ministry of Finance to acquire the membership of the EBRD.
Statement 1 and 4: Membership of EBRD would enhance India's international profile and promote its economic
interests. Access to EBRD's Countries of Operation and sector knowledge. It would increase the scope of
cooperation between India and EBRD through co-financing opportunities in manufacturing, services, Information
Technology, and Energy. EBRD's core operations pertain to private sector development in their countries of
operation. The membership would help India leverage the technical assistance and sectoral knowledge of the bank
for the benefit of development of private sector.
Statement 2: Membership entails this provision.
Statement 3: The minimum initial investment towards the membership of EBRD will be approximately €1 (one)
million. However, this assumption is based on India deciding to buy the minimum number of shares (100) required
for obtaining the membership. If India were to buy a higher number of Bank shares, the financial implications could
be higher. In- principle approval of the Cabinet at this stage is being obtained for joining the Bank.

512. Consider the following about Indian Renewable Energy Development Agency Limited (IREDA).

1) It is a Navratna public limited government company.


2) It is headed by the Cabinet Secretary.
3) It is under the administrative control of the Department of Science and Technology (DST).
4) It provides financial support to projects that aim at generating electricity through new and renewable energy
sources.

Select the correct answer using the codes below.

a) 1 and 2 only
b) 2, 3 and 4 only
c) 4 only
d) 2 and 3 only

Solution: c)
Justification: IREDA) is a Mini Ratna (Category – I) Government of India Enterprise under the administrative control
of Ministry of New and Renewable Energy (MNRE). IREDA is a Public Limited Government Company established as a
Non-Banking Financial Institution in 1987 engaged in promoting, developing and extending financial assistance for
setting up projects relating to new and renewable sources of energy and energy efficiency/conservation
Its objectives are, inter alia:
 To give financial support to specific projects and schemes for generating electricity and / or energy through
new and renewable sources and conserving energy through energy efficiency.
 To increase IREDA's share in the renewable energy sector by way of innovative financing.
 Improvement in the efficiency of services provided to customers through continual improvement of
systems, processes and resources.

513. National Agricultural bank for Rural Development (NABARD) provides

1) Loans for Food Parks and Food Processing Units in Designated Food Parks
2) Credit Facilities to Marketing Federations
3) Direct Refinance Assistance to Co-operative Banks
4) Long term irrigation Fund
5) Short Term re-financing for seasonal agricultural operations

Select the correct answer using the codes below.

a) 1, 3 and 4 only
b) 2 and 5 only
c) 1, 2, 3 and 4 only
d) 1, 2, 3, 4 and 5

Solution: d)
Justification: Statement 1: Development of food processing industry in the country is accorded top priority by the
Government of India as it is one of the most critical links in the agri value chain. Taking this agenda further, the
Finance Ministry, in 2014, announced setting up of a Special Fund of Rs. 2,000 crore in NABARD for providing direct
term loans at affordable rates of interest to Designated Food Parks (DFPs) and food processing units in the DFPs.
Statement 2: Marketing federations and cooperatives are playing a very important role in agribusiness and
value/supply chain management of the various agricultural commodities. Major activities undertaken by these
institutions are:

 Procurement of agricultural commodities including milk


 Aggregation, storage and value addition in few select commodities like milk etc.
 Marketing

Large number of farmers, producers’ organizations, and primary societies depend upon these institutions for
marketing of their produce and for value-added services like input supply, value addition and storage facilities. The
marketing operations by these federations and cooperatives require seasonal and timely short-term credit facility to
support their day-to-day operations.
Statement 3: Implementation of the Government of India’s Revival Package of Short Term Co- operative Credit
Structure (STCCS) as per Vaidyanathan Committee recommendations enabled District Central Co-operative Banks
(DCCBs) to borrow funds directly from any financial institution regulated/approved by RBI. As a corollary to this
enabling provision, NABARD developed a progressive product titled ‘Short Term Multipurpose Credit Product’
(STMPCP) to provide financial assistance to Co-operative Banks (StCBs/DCCBs). The primary objective here was to
expand their lendable resources and enable their diversification into a variety of business operations.
Statement 5: Refinancing is the replacement of an existing debt obligation with another debt obligation under
different terms. In agricultural terms, it is often given when a farmer is unable to pay his existing loans, and he opts
for a refinance from NABARD.

514. Green Indian States Trust (GITS) is

a) A non-profit charitable trust that encourages policymakers and governments to measure and manage their natural,
human and social capitals.
b) A trust of World Wide Fund for Nature, Greenpeace and Friends of the Earth in India to finance ecological
conservation projects
c) An informal consortium of the State Environment Ministries of India to monitor inter-state projects
d) None of the above

Solution: a)
Justification: GIST is an NGO that aims to promote sustainable development in India. It was created in July 2004,
with its first project being to research and publish ‘Green Accounts’ to focus on sustainability at the state level in
India. Economic growth does not capture the many vital aspects of national wealth and well-being, such as changes
in the quality of health, the extent of education, or the quality and quantity of natural resources. The 'Green
Accounting for Indian States Project' (GAISP) was undertaken to encourage India’s policymakers to abandon their
dependence on the narrow economic measure of ‘GDP Growth’ with a holistic measure that takes into account all
major externalities

515. With reference to the Prime Minister's National Relief Fund (PMNRF) consider the following statements.

1) PMNRF was constituted by the Parliament.


2) It was established to assist displaced persons from Pakistan.
3) The fund gets budgetary support.
4) The corpus of the fund cannot be invested or appropriated for any other purpose than relief operations.

Select the correct answer using the codes below.

a) 1 and 3 only
b) 2 only
c) 3 and 4 only
d) None of the above

Solution: b)
Justification: Statement 1 and 2: In pursuance of an appeal by the then Prime Minister, Pt. Jawaharlal Nehru in
January, 1948, the Prime Minister's National Relief Fund (PMNRF) was established with public contributions to assist
displaced persons from Pakistan. The resources of the PMNRF are now utilized primarily to render immediate relief
to families of those killed in natural calamities like floods, cyclones and earthquakes, etc. and to the victims of the
major accidents and riots.
Statement 3: The fund consists entirely of public contributions and does not get any budgetary support. It accepts
voluntary contributions from Individuals, Organizations, Trusts, Companies and Institutions etc. All contributions
towards PMNRF are exempt from Income Tax under section 80(G).
Statement 4: The corpus of the fund is invested with scheduled commercial banks in various forms. Disbursements
are made with the approval of the Prime Minister. PMNRF has not been constituted by the Parliament. The fund is
recognized as a Trust under the Income Tax Act and the same is managed by Prime Minister or multiple delegates
for national causes

516. The Financial Stability Board (FSB) is an international body that monitors and makes recommendations about
the global financial system. It was founded in the wake of

a) East Asian Financial crisis


b) 2008 sub-prime Mortgage crisis
c) 2012 Eurozone Crisis
d) 2004 Recession in Asia and Europe

Solution: b)
Justification: It was established after the G20 London summit in April 2009 as a successor to the Financial Stability
Forum (FSF). The Board includes all G20 major economies, FSF members, and the European Commission. Hosted
and funded by the Bank for International Settlements.

Its recent priorities are:

Promote a coordinated program of reforms to deliver resilient sources of market-based finance, including
addressing structural vulnerabilities associated with asset management Develop robust financial market
infrastructure, including assessing policies on central counterparty resilience, recovery, and resolvability, and
recommending any necessary improvements

Support effective macro prudential arrangements by drawing lessons from countries that have applied macro
prudential policy frameworks and tools, working in partnership with the IMF and BIS.

The FSB represented the G20 leaders' first major international institutional innovation. It has been described as "in
effect, a fourth pillar" of the architecture of global economic governance.

517. Consider the following statements about the National Pharmaceutical Pricing Authority (NPPA).

1) It fixes ceiling price of essential medicines of Schedule I under the Drugs (Prices Control) Order (DPCO) 2013.
2) It is entrusted with the task of recovering amounts overcharged by manufacturers for the controlled drugs from the
consumers.
3) It monitors the prices of decontrolled drugs in order to keep them at reasonable levels.

Select the correct answer using the codes below.

a) 1 and 2 only
b) 2 and 3 only
c) 1 and 3 only
d) 1, 2 and 3

Solution: d)
Justification: NPPA is an organization of the Government of India which was established, inter alia, to fix/ revise the
prices of controlled bulk drugs and formulations and to enforce prices and availability of the medicines in the
country, under the Drugs (Prices Control) Order, 1995.
Functions of National Pharmaceutical Pricing Authority
 To implement and enforce the provisions of the Drugs (Prices Control) Order in accordance with the powers
delegated to it.
 To deal with all legal matters arising out of the decisions of the Authority.
 To monitor the availability of drugs, identify shortages, if any, and to take remedial steps.
 To collect/ maintain data on production, exports and imports, market share of individual companies,
profitability of companies etc., for bulk drugs and formulations.
 To undertake and/ or sponsor relevant studies in respect of pricing of drugs/ pharmaceuticals.
 To recruit/ appoint the officers and other staff members of the Authority, as per rules and procedures laid
down by the Government.
 To render advice to the Central Government on changes/ revisions in the drug policy.
 To render assistance to the Central Government in the parliamentary matters relating to the drug pricing.

518. Consider the following statements. Office of the Economic Adviser (OEA)

1) is an attached office of the Ministry of Finance


2) Renders advice to the Government on formulation of Industrial Policy and Foreign Trade policy
3) Compiles and releases monthly Wholesale Price Indices and Index of Core Industries Production

Select the correct answer using the codes below.

a) 1 and 2 only
b) 2 and 3 only
c) 3 only
d) 1, 2 and 3

Solution: b)
Justification: Statement 1: It is under the Ministry of Commerce and Industry. The main functions of the Office of
Economic Adviser include, inter alia the following:
Policy Functions:
 Economic policy inputs on industrial development.
 Rendering advice relating to formulation of Industrial Policy, Foreign Trade Policy with respect to industrial
sector in general with thrust on manufacturing, issues relating to bilateral and multilateral trade, as well as
taxes and duties related to industry, including but not restricted to safeguard and anti-dumping duties.
 Analysis of trends of industrial production and growth.
 Examination of multilateral and bilateral issues and processing Policy Notes with economic implications
referred to the Office.
 Planning and Gender Budgeting on behalf of the Department of Industrial Policy and Promotion.
Statistical Functions:
 Compiling and releasing monthly Wholesale Price Indices
 Compiling and releasing monthly Index of Core Industries Production

519. The Census 2001 and 2011 exercises in India were conducted and administered under
a) Ministry of Planning
b) Ministry of Statistics and Programme Implementation
c) Ministry of Home Affairs
d) Ministry of Social Justice and Empowerment

Solution: c)
Justification: The responsibility of conducting the decennial Census rests with the Office of the Registrar General
and Census Commissioner, India under Ministry of Home Affairs, Government of India. The Census Act was enacted
in 1948 to provide for the scheme of conducting population census with duties and responsibilities of census
officers. Later, the office was also entrusted with the responsibility of implementation of Registration of Births and
Deaths Act, 1969 in the country. The Indian Census is the most credible source of information on Demography
(Population characteristics), Economic Activity, Literacy and Education, Housing & Household Amenities,
Urbanisation, Fertility and Mortality.

520. What is/are the major functions of the Khadi and Village Industries Commission (KVIC)?

1) Building up of a reserve of relevant raw materials and implements for supply to producers
2) Promoting the sale and marketing of khadi or products of village industries or handicrafts
3) Organising programs for the development of Khadi and other village industries in the rural areas

Select the correct answer using the codes below.

a) 1 and 2 only
b) 2 and 3 only
c) 1 and 3 only
d) 1, 2 and 3

Solution: d)
Justification: The broad objectives of KVIC are:

 The social objective of providing employment.


 The economic objective of producing saleable articles.
 The wider objective of creating self-reliance amongst the poor and building up of a strong rural community
spirit.

Some of the major functions of KVIC are:

 The KVIC is charged with the planning, promotion, organisation and implementation of programs for the
development of Khadi and other village industries in the rural areas in coordination with other agencies
engaged in rural development wherever necessary.
 Its functions also comprise building up of a reserve of raw materials and implements for supply to
producers, creation of common service facilities for processing of raw materials as semi-finished goods and
provisions of facilities for marketing of KVI products apart from organisation of training of artisans engaged
in these industries and encouragement of co- operative efforts amongst them.
 To promote the sale and marketing of khadi and/or products of village industries or handicrafts, the KVIC
may forge linkages with established marketing agencies wherever feasible and necessary.

521. The National Skill Development Corporation (NSDC) will become pivotal to realizing the demographic dividend
in India. Which of these terms characterize the corporation?

a) Not-for profit Public Private Partnership (PPP)


b) Non-Government Organization (NGO) registered under the Societies Act
c) For-profit Company under Companies Act
d) Attached government office under the Ministry of Skill Development & Entrepreneurship (MSDE)

Solution: a)
Justification: NSDC is a one-of-its-kind, Public Private Partnership (PPP) model in India, under the MSDE. A not-for-
profit company set up by the Ministry of Finance, under Section 25 of the Companies Act, it has an equity base of
Rs.10 crore, of which the Government of India holds for 49%, while the private sector has the balance 51%. It aims
to promote skill development by catalysing creation of large, quality and for-profit vocational institutions. NSDC
provides funding to build scalable and profitable vocational training initiatives. NSDC acts as a catalyst in skill
development by providing funding to enterprises, companies and organisations that provide skill training. It will also
develop appropriate models to enhance, support and coordinate private sector initiatives.

522. Consider the following statements. Small Farmers Agribusiness Consortium (SFAC)

1) is an Autonomous Society promoted by Ministry of Agriculture, Cooperation and Farmers’ Welfare


2) registered as a Payments Bank by the Reserve Bank of India (RBI)
3) implements the schemes of Government of India for economic inclusion of small and marginal farmers in
agribusiness activities

Select the correct answer using the codes below.


a) 1 only
b) 2 and 3 only
c) 1 and 3 only
d) 1, 2 and 3

Solution: c)
Justification: Statement 1 and 2: It was registered under Societies Registration Act 1860. The Society is also
registered as Non-Banking Financial Institution by Reserve Bank of India. The Society is governed by Board of
Management which is chaired, ex-officio, by Union Minister for Agriculture and Farmers Welfare as the President
and the Secretary, Department of Agriculture, Cooperation and Farmers Welfare, is the ex-officio Vice-President.
Statement 3: SFAC is implementing the central schemes of Government of India namely VCA, EGCGS etc. Society is
pioneer in organising small and marginal farmers as Farmers Interest Groups, Farmers Producers Organisation and
Farmers Producers Company for endowing them with bargaining power and economies of scale. Recently the
Society has been entrusted with the task of implementation of the critically important Delhi Kisan Mandi and
National Agriculture Market Scheme on e-platform to progressively free agricultural trade and offer price discovery
to farmers.

523. India Brand Equity Foundation (IBEF) was established by the Department of Commerce, Ministry of Commerce
and Industry, with the objective of

1) Supporting handlooms and rural artisans with financial leverage and technology upgradation
2) Facilitate dissemination of knowledge of Indian products and services
3) Promoting and creating international awareness of the Made in India label in markets overseas

Select the correct answer using the codes below.

a) 1 and 2 only
b) 2 and 3 only
c) 1 and 3 only
d) 1, 2 and 3

Solution: b)
Justification: India Brand Equity Foundation (IBEF) was established in 2003 with the objective of promoting and
creating international awareness of the Made in India label in markets overseas and to facilitate dissemination of
knowledge of Indian products and services. Towards this objective, IBEF works closely with stakeholders across
government and industry. IBEF endeavours to ensure consistent and effective messaging on the India growth story
through brand building initiatives/campaigns undertaken in partnership with various stakeholders ranging from lead
industry associations to export promotion councils. IBEF is the branding and communication partner for various
trade exhibitions organized under the aegis of the Department of Commerce. IBEF Trust is headed by the Commerce
Secretary and has 14 other members representing the industry, trade, market, academia, media, advertising &
publicity, and the government. Digital media and web technologies are integral part of its nation brand campaigns
both within India and overseas markets.

524. With reference to Tribal Cooperative Marketing Development Federation of India Limited (TRIFED), consider the
following statements.

1) TRIFED is a multi-State Cooperative Society under Ministry of Tribal Affairs.


2) It aims at marketing tribal products within India and abroad by providing marketing support to tribal products
through its network of retail outlets.
3) It facilitates participation of tribal artisans to enable them to interact directly with art lovers to assess market needs.

Select the correct answer using the codes below.

a) 1, 2 and 3
b) 1 only
c) 1 and 2 only
d) 2 and 3 only

Solution: a)
Justification: It was established in August 1987 by then Ministry of Welfare under Multi State Cooperative Societies
Act 1984 (which has now been replaced by Multi-State Cooperative Societies Act, 2002). It is headquartered in New
Delhi. It aims at promoting and marketing tribal products, tribal art and crafts within the country and abroad by
providing marketing support to tribal products through its network of retail outlets. It also organizes exhibitions like
National Tribal Craft Expo called “Aadi Mahotsav” etc. to promote and market tribal products. It also facilitates
participation of tribal artisans to enable them to interact directly with art lovers to assess market needs

525. With reference to National Foundation for Corporate Governance (NFCG), consider the following statements.

1) It is an initiative of the FICCI and CII.


2) It acts as an informal appellate mechanism for corporate frauds in the industry.
3) It publishes a model Corporate Governance Code annually which must be agreed by a firm in order to get
representation in FICCI.

Select the correct answer using the codes below.

a) 1 only
b) 2 and 3 only
c) 1 and 3 only
d) None of the above

Solution: d)
Justification: It was set up in the year 2003 by the Ministry of Corporate Affairs (MCA), in partnership with
Confederation of Indian Industry (CII), Institute of Company Secretaries of India (ICSI) and Institute of Chartered
Accountants of India (ICAI) to promote good Corporate Governance practices both at the level of individual
corporates and Industry as a whole. In the year 2010, Institute of Cost Accountants of India (ICAI) and National Stock
Exchange (NSE) and in 2013 Indian Institute of Corporate Affairs (IICA) were included in NFCG as Trustees.
Mission is to:
 To foster a culture of good governance, voluntary compliance and facilitate effective participation of
different stakeholders;
 To catalyze capacity building in new emerging areas of Corporate Governance.
 To further research, scholarship, and education in corporate governance in India;
 To create a framework of best practices, structure, processes and ethics;

526. National Skill Development Fund (NSDF) entered into an Investment Management Agreement (IMA) with
National Skill Development Corporation (NSDC) recently. The agreement provides for

1) Utilization of the NSDF corpus to meet the desired objectives of National Skill Development Mission
2) Supervisory role of NSDF over NSDC's functions

Which of the above is/are correct?

a) 1 only
b) 2 only
c) Both 1 and 2
d) None

Solution: c)
Justification: NSDC and NSDF were set up by the Ministry of Finance and registered in July, 2008 and January, 2009
respectively for implementing coordinated action for skill development. NSDF trust was incorporated to act as a
receptacle for financial contributions from Governmental sources, bilateral/multilateral and other agencies. Its main
objective is to enhance, stimulate and develop the skills of Indian youth force by various sector specific
programmes. NSDF entered into an Investment Management Agreement (IMA) with NSDC for utilization of its
corpus to meet the desired objectives of National Skill Development Mission and encourage skill development in the
country. Provision of supervisory role of NSDF over NSDC's functions is also included in the IMA between NSDC and
NSDF.

527. National sample Survey (NSS), that gives useful information on consumer expenditure, employment and the
like, is conducted by the

a) NITI Aayog, and formerly Planning Commission


b) Ministry of Statistics and Programme Implementation
c) Department of Economic Affairs, Ministry of Finance
d) Department of Industrial Policy and Promotion, Ministry of Commerce

Solution: b)
Justification: The Employment and Unemployment surveys of National sample Survey (NSS) are the primary sources
of data on various indicators of labour force at National and State levels. These are used for planning, policy
formulation, decision support and as input for further statistical exercises by various Government organizations,
academicians, researchers and scholars. NSS surveys on employment and un-employment with large sample size of
households have been conducted quinquennially from 1972 of NSS onwards.

528. Among the following Acts, an Enforcement Directorate (ED) in India will be most concerned with

a) Co-operative Societies Act


b) Negotiable Instrument Act
c) Public Debt Act
d) Prevention of Money Laundering Act

Solution: d)
Justification::It is a law enforcement agency and economic intelligence agency responsible for enforcing economic
laws and fighting economic crime in India. It is part of the Department of Revenue, Ministry of Finance. The prime
objective of the Enforcement Directorate is the enforcement of two key Acts- the Foreign Exchange Management
Act 1999 (FEMA) and the Prevention of Money Laundering Act 2002 (PMLA). Other objectives are primarily linked to
checking money laundering in India. It comprises officers of the Indian Revenue Service, Indian Police Service and
the Indian Administrative Service.

529. The Energy Efficiency Services Limited (EESL) is implementing World’s largest street light replacement
programme as a part of the Street Lighting National Programme (SLNP). Consider the following about EESL.

1) It is a non-profit Public Private Partnership (PPP).


2) It acts as the resource centre for capacity building of State DISCOMs.
3) It undertakes the manufacture and supply of energy efficiency services and products.

Select the correct answer using the codes below.

a) 1 and 2 only
b) 2 and 3 only
c) 2 only
d) 1 and 3 only

Solution: b)
Justification: EESL was set up under Union Ministry of Power to facilitate implementation of energy efficiency
projects. It is a joint venture of NTPC Limited, Power Finance Corporation (PFC), Rural Electrification Corporation
(REC) and POWERGRID. It leads market-related actions of National Mission for Enhanced Energy Efficiency (NMEEE).
It also acts as the resource center for capacity building of State DISCOMs. It is also implementing Unnat Jyoti by
Affordable LEDs for All (UJALA): World’s largest zero- subsidy domestic LED bulb programme and the World’s largest
Agricultural Demand Side Management programme (AgDSM). It is mandated to carry out and promote the business
of energy efficiency and climate change including manufacture and supply of energy efficiency services and
products. It also provides consultancy services in the field of CDM projects, Carbon Markets, Demand Side
Management, Energy Efficiency, Climate change and other related areas.

530. The Prime Minister Shri Narendra Modi has constituted the Economic Advisory Council to the Prime Minister
(EAC-PM). It consists of

1) Minister of Finance
2) Chairman, NITI Aayog
3) Secretary, Department of Economic Affairs
4) Deputy Governor, RBI

Select the correct answer using the codes below.

a) 1 and 2 only
b) 2, 3 and 4 only
c) 2 only
d) 1, 3 and 4 only

Solution: c)
Justification: The EAC-PM is an independent body to give advice on economic and related issues to the Government
of India, specifically to the Prime Minister. The five-member council consists of economists of high repute and
eminence. The composition of the EAC-PM is as follows:
 Dr. Bibek Debroy, Member, NITI Aayog - Chairman
 Dr. Surjit Bhalla- Part-time Member
 Dr. Rathin Roy- Part-time Member
 Dr. Ashima Goyal- Part-time Member
 Shri Ratan Watal, Principal Advisor, NITI Aayog – Member-Secretary
It will address issues of macroeconomic importance and presenting views thereon to the Prime Minister. This could
be either suo-motu or on reference from the Prime Minister or anyone else.
531. RBI’s affairs are governed by the Central Board of Directors (CBD) which also responsible for its general
superintendence and direction. It consists of

1) Union Minister of Finance as the Chairperson


2) Governor of RBI as the Vice-Chairperson
3) Members nominated by Union Government
4) Nominees from SEBI and IRDA

Select the correct answer using the codes below.

a) 1 and 2 only
b) 2, 3 and 4 only
c) 3 only
d) None of the above

Solution: c)

Justification: Composition: There are 21-members in CBD of RBI. It comprises of Governor, 4 Deputy Governors, 2
Finance Ministry representatives. 10 Directors are nominated by Union Government in order to represent important
elements from India’s economy. 4 directors are from local boards of RBI which are headquartered at Mumbai,
Kolkata, Chennai and New Delhi.

532. Consider the following statements about the Pay Commissions setup by the Government of India from time to
time.

1) Its recommendations cover the pay structure of both civil and military staff under the Government.
2) Its recommendations are binding on both centre and states.

Which of the above is/are correct?

a) 1 only
b) 2 only
c) Both 1 and 2
d) None

Solution: a)
Justification: Statement 1: The best example would be the One Rank One Pension scheme suggested after
discussions in the 7th Pay commission. It is an advisory body. A serving government employee can be its member. It
first submits its report to the Finance Minister who then places it before the Cabinet. One Rank One Pension (OROP)
scheme for central government staffers, para military as well as armed forces personnel was announced by the 7th
Pay Commission. Its recommendations are usually followed up by state governments as they hike their staff wage
structure. However, it is not binding on the states.

533. CRISIL, often seen in news, is a

a) An International NGO focussed on disaster management


b) Credit Rating agency headquartered in India
c) A global Energy finance company
d) Group of LDCs committed to development

Solution: b)
Justification: CRISIL (formerly Credit Rating Information Services of India Limited) is a global analytical company
providing ratings, research, and risk and policy advisory services. CRISIL’s majority shareholder is Standard & Poor's,
a division of McGraw Hill Financial and provider of financial market intelligence. It assesses several aspects of Indian
economy. For e.g. as per its recent estimates, if all affected states also announce farm loan waivers the Uttar
Pradesh, Maharashtra, Karnataka and Punjab did, then the total cost the exchequer could be up to Rs 2.5 lakh crore
or 0.5% of GDP.

534. National Anti-profiteering Authority will look after

a) Enforcing anti-trust or competitive laws in the industry on the lines of Competition Commission of India (CCI)
b) Consumer grievances arising due to complaints raised by unreasonable pricing of goods and services
c) Ensuring that benefits of a reduction in tax on supply of goods or services flow to the consumers
d) None of the above

Solution: b)
Justification: In the GST mechanism, the producers are provided an input tax credit. It works this way. Suppose, a
producer requires INR 80 worth of cotton to produce a shirt, which is finally processed and priced at INR 100 in the
market. In this process, the producer will have to pay taxes at two places – a) while purchasing the raw material,
and b) while selling the final produce – shirt. This double taxation (tax on tax) is avoided by an input tax credit
mechanism. Under this, the producers are compensated for any additional taxes they would have paid. When
constituted by the GST Council, the National Anti-profiteering Authority shall be responsible for applying anti-
profiteering measures in the event of a reduction in rate of GST on supply of goods or services. It will also check if
the benefit of input tax credit is not passed on to the recipients by way of commensurate reduction in prices. In the
event the National Anti-profiteering Authority confirms the necessity of applying anti-profiteering measures, it has
the power to order the business concerned to reduce its prices or return the undue benefit availed along with
interest to the recipient of the goods or services.

 If the undue benefit cannot be passed on to the recipient, it can be ordered to be deposited in the
Consumer Welfare Fund.

 In extreme cases the National Anti-profiteering Authority can impose a penalty on the defaulting business
entity and even order the cancellation of its registration under GST.

535. Consider the following about Financial Intelligence Unit– India (FIU-IN).

1) It is an independent body reporting directly to the Economic Intelligence Council (EIC) headed by the Prime
Minister.
2) FIU-India gathers its intelligence via seeking information directly from individuals, companies and trusts involved in
financial operations.
3) It is responsible for coordinating and strengthening efforts of national and international intelligence agencies in
pursuing the global efforts against money laundering and related crimes.

Select the correct answer using the codes below.

a) 1 and 3 only
b) 2 and 3 only
c) 3 only
d) 1 and 2 only

Solution: c)
Justification: It was set by the Government of India in 2004 as the central national agency responsible for receiving,
processing, analyzing and disseminating information relating to suspect financial transactions. FIU-IND is also
responsible for coordinating and strengthening efforts of national and international intelligence, investigation and
enforcement agencies in pursuing the global efforts against money laundering and related crimes. FIU-IND is an
independent body reporting directly to the Economic Intelligence Council (EIC) headed by the Finance Minister.

Statement 2: FIU-India does not seek any information from individuals. If any person receives any such letter/query,
it is a fraud and it should be immediately reported to the Police and also brought to the notice of FIU-IND.

INTERNATIONAL INSTITUTIONS

536. The International Bank for Reconstruction and Development (IBRD) is an international financial institution that

1) Offers loans to middle-income developing countries


2) Is a part of the World Bank Group (WBG)
3) Funds not only infrastructure but also access to healthcare, food and potable water

Select the correct answer using the codes below.

a) 1 only
b) 2 and 3 only
c) 1 and 3 only
d) 1, 2 and 3

Solution: d)
Justification: The IBRD is the first of five member institutions that compose the World Bank Group. It was
established in 1944 with the mission of financing the reconstruction of European nations devastated by World War
II. The IBRD provides commercial-grade or concessional financing to sovereign states to fund projects that seek to
improve transportation and infrastructure, education, domestic policy, environmental consciousness, energy
investments, healthcare, access to food and potable water, and access to improved sanitation.

537. The Global Forum on Agricultural Research and Innovation (GAFR) is a/an

a) Voluntary forum and a movement for change hosted by FAO


b) Inter-governmental organization affiliated to the WTO
c) NGO that is affiliated to the International Food Policy Research Institute (IFPRI)
d) Wing of the World Economic Forum on Agriculture

Solution: a)
Justification: The Global Forum on Agricultural Research and Innovation (GFAR) is an inclusive global mechanism
enabling all those concerned with the future of agriculture and its role in development around the world to come
together and address key global needs. GFAR provides an open forum for stakeholders across the agricultural
spectrum—from researchers and organizations to farmers—to participate in collaborative discussion and action
around the current and future state of agriculture. Established in 1996, GFAR was formed as a project for resource
sharing—a commitment that remains the essential purpose of the Forum today. GFAR facilitates collaboration,
partnerships and sharing of objectives along the complex pathways from research through to development
outcomes. Its headquarters is in Rome, Italy, where it is hosted by FAO.

538. International Vaccine Institute (IVI)

1) is a Washington based inter-governmental agency of the World Health Organization (WHO)


2) is devoted to developing and introducing new and improved vaccines to protect from infectious diseases
3) has India as a member country
4) was established on the initiatives of the United Nations Development Programme (UNDP)

Select the correct answer using the codes below.


a) 2, 3 and 4 only
b) 1 and 4 only
c) 2 and 3 only
d) 1, 2, 3 and 4

Solution: a)
Justification: Statement 1: IVI is a Seoul (South Korea) based international non-profit organization devoted to
developing vaccines.
Statement 2: Its work is exclusively on vaccine development and introduction specifically for people in developing
countries, with a focus on neglected diseases affecting these regions.
Statement 3 and 4: Currently, IVI has 40 countries and the World Health Organization (WHO) as signatories to its
Establishment Agreement. India officially became a signatory to the IVI in 2012.

539. Which of the following does NOT coincide with the idea of a New International Economic Order (NIEO) as it was
developed in the days of the cold war?
a) Give the Least Developed Countries (LDCs) control over their natural resources exploited by the developed Western
countries
b) Make trade with the western countries more beneficial for the poorer countries
c) Provide the LDCs with a greater role in international economic institutions
d) Give LDCs greater voting rights in the World Trade Organization (WTO)

Solution: d)
Justification: Option D: There was no WTO at the time the idea of NIEO originated. Moreover, even presently all
nations have equal voting rights in WTO. The United Nations Conference on Trade and Development (UNCTAD)
brought out a report in 1972 entitled Towards a New Trade Policy for Development. The report proposed a reform
of the global trading system so as to (apart from the above):
 Help LDCs to obtain access to Western markets so that the LDCs could sell their products and, therefore,
make trade more beneficial for the poorer countries,
 Reduce the cost of technology from the Western countries, and
 Gradually, the nature of nonalignment changed to give greater importance to economic issues.
In 1961, at the first NAM summit in Belgrade, economic issues had not been very important. By the mid-1970s, they
had become the most important issues. As a result, NAM became an economic pressure group.

540. Hard and soft loans, extended by international financial organizations, mainly differ in their

a) Quantum of lending
b) Concessional conditions
c) Source of credit
d) Sovereign v/s non-sovereign nature

Solution: b)
Justification: For e.g. the ADB offers both Hard Loans and Soft loans. ADB offers “hard” loans from ordinary capital
resources (OCR) on commercial terms, and the Asian Development Fund (ADF) affiliated with the ADB extends
“soft” loans from special fund resources with concessional conditions. Soft loans are usually given to developing
countries in special needs such as reconstruction, social security, critical infrastructure etc.

541. With reference to the India-UN Development Partnership Fund, consider the following statements.

1) It will be utilized to carry out country-level projects catalytic towards achieving the SDGs.
2) The Partnership fund would be managed by UN Office for South-South Cooperation (UNOSSC).

Which of the above is/are correct?


a) 1 only
b) 2 only
c) Both 1 and 2
d) None

Solution: c)
Justification: This will be the first time that India has entered into a partnership with the UN in a triangular
cooperation with fellow developing countries. The establishment of the fund is aimed at reducing poverty and
hunger, improving health, education, water, energy and equality etc. in LDCs and other developing countries. The
United Nations Office for South-South Cooperation (UNOSSC) is a knowledge hub providing advisory and consulting
services to all stakeholders on South-South and triangular cooperation. It enables developing countries to effectively
face their development challenges and harness opportunities to address them.

542. The United Nations Conference on Trade and Development (UNCTAD) helps countries to

1) Achieve beneficial integration into the international trading system


2) Limit their exposure to financial volatility and debt
3) Promote entrepreneurship and innovation
4) Help local firms move up value chains
5) Curb regulations that stifle competition

Select the correct answer using the codes below.


a) 1, 2 and 3 only
b) 2, 4 and 5 only
c) 1, 3 and 4 only
d) 1, 2, 3, 4 and 5

Solution: d)
Justification: Working at the national, regional, and global level, our efforts help countries to (others not mentioned
in the options above):
 Comprehend options to address macro-level development challenges
 Diversify economies to make them less dependent on commodities
 Attract investment and make it more development friendly
 Increase access to digital technologies
 Speed up the flow of goods across borders
 Protect consumers from abuse
 Adapt to climate change and use natural resources more effectively

543. As per Washington Consensus

1) There should be a greater role of state vis-à-vis the private sector in a growing economy.
2) Developed countries should stop deregulation measures which, in the past, have led to increased economic
inequality.

Which of the above is/are correct?

a) 1 only
b) 2 only
c) Both 1 and 2
d) None

Solution: d)
Justification: It is considered to constitute the "standard" reform package promoted for crisis- wracked developing
countries by Washington, D.C.–based institutions such as the International Monetary Fund (IMF), World Bank, and
the US Treasury Department. It basically refers to a more general orientation towards a strongly market-based
approach such as macroeconomic stabilization, economic opening with respect to both trade and investment, and
the expansion of market forces within the domestic economy.

544. Government of India has voluntarily contributed to various UN Funds and Programmes at the UN Pledging
Conference for the year 2017. Consider the following funds and the Ministry/Department that made contribution to
that fund.

1) UNDP: Ministry of Social Justice and Empowerment


2) UNICEF: Ministry of Women & Child Development
3) United Nations Population Fund (UNFP): Ministry of Health & Family Welfare
4) UNEP: Ministry of Environment, Forest and Climate Change

Select the correct answer using the codes below.

a) 1 and 4 only
b) 2, 3 and 4 only
c) 2 and 3 only
d) 1 and 2 only

Solution: b)
Justification: For UNDP, the ministry involved is department of economic affairs, ministry of Finance

545. The World Bank aims to

1) Improve health care, nutrition, childhood development and education


2) Strengthen social security, pension and legal systems
3) Protect and empower the most vulnerable groups of society, including women and the poor in rural areas

Select the correct answer using the codes below.

a) 1 only
b) 2 and 3 only
c) 1 and 3 only
d) 1, 2 and 3

Solution: d)
Justification: The World Bank is the single largest investor in the social sectors worldwide with a portfolio of
approximately USD 40 billion invested in over 600 projects for social development. Apart from the objectives
mentioned above they also aim at reducing poverty, improve living standards and raise incomes and productivity.
The World Bank is the single largest financier of environmental projects worldwide with a portfolio of USD 12 Billion
in projects with clear environmental objectives

546. The Financial Sector Assessment Program (FSAP) is a joint program of the

a) BRICS Development Bank and Asian Development Bank (ADB)


b) Asian Infrastructure Investment Bank (AIIB) and ADB
c) International Monetary Fund and the World Bank
d) World Economic Forum and AIIB

Solution: c)
Justification: The two main Reports of the 2017 India Financial Sector Assessment Programme (FSAP) – the Financial
System Stability Assessment (FSSA) and Financial Sector Assessment (FSA), were recently released by the
International Monetary Fund (IMF) and the World Bank. The FSAP report acknowledges many efforts by Indian
authorities like tackling Non- Performing Assets (NPAs), recent recapitalization measures for banks and
introduction of special resolution regime, formalization of National Pension System (NPS) and making the pension
sector regulator statutory, passing of Insolvency and Bankruptcy Code and setting up of Insolvency and Bankruptcy
Board of India (IBBI), to name a few. Based on its findings, FSAPs produce recommendations of a micro- and macro-
prudential nature, tailored to country-specific circumstances. It appreciates initiatives such as ‘no frills’ account
(under Jan Dhan Yojana), promoting digitization, introduction of unique biometric identification number (AADHAR),
currency exchange initiative etc. Established in 1999, is a comprehensive and in-depth assessment of a country’s
financial sector. It is a joint program of the International Monetary Fund and the World Bank. FSAPs analyze the
resilience of the financial sector, the quality of the regulatory and supervisory framework, and the capacity to
manage and resolve financial crises

547. India has not ratified some of the core or fundamental International Labour Organisation (ILO) Conventions,
namely Freedom of Association and Protection of the Right to Organise Convention, 1948 and Right to Organise and
Collective Bargaining Convention, 1949. This is because

1) Accepting this convention would allow government servants to work for foreign organizations freely.
2) The conventions would give government servants a right to openly criticize Government policies.

Which of the above is/are correct?

a) 1 only
b) 2 only
c) Both 1 and 2
d) None

Solution: c)
Justification: In India, we ratify an ILO Convention only when the national laws are brought fully into conformity
with the provisions of the Convention in question. The main reason for non-ratification of ILO Conventions is due to
certain restrictions imposed on the Government servants. As communicated by Department of Personnel & Training
(DOPT), the ratification of these conventions would involve granting of certain rights that are prohibited under the
statutory rules, for the Government employees. These are: to strike work, to openly criticize Government policies,
to freely accept financial contribution, to freely join foreign organizations etc. India has ratified six out of the eight
core ILO Conventions. These are the

 Forced Labour Convention, 1930


 Abolition of Forced Labour Convention, 1957
 Equal Remuneration Convention, 1951
 Discrimination (Employment and Occupation) Convention, 1958
 Minimum Age Convention, 1973
 Worst Forms of Child Labour Convention, 1999

548. Multilateral Carbon Credit Fund (MCCF) is maintained by the

a) European Investment Bank (EIB) with the European Bank for Reconstruction and Development (EBRD)
b) Multilateral Investment Guarantee Agency (MIGA) with the World Bank (WB)
c) Global Carbon Facility (GCF) with UNFCCC
d) None of the above

Solution: a)
Justification: The European Bank for Reconstruction and Development (EBRD) and the European Investment Bank
(EIB) established the Multilateral Carbon Credit Fund (MCCF) as a key instrument in their strategy for combating
climate change. Fully subscribed, with over €200 million in commitments, the MCCF is one of the few carbon funds
dedicated specifically to countries from Central Europe to Central Asia. By joining the MCCF, private and public
companies as well as EBRD and EIB shareholder countries can purchase carbon credits from emission reduction
projects financed by EIB or EBRD to meet their mandatory or voluntary greenhouse gas (GHG) emission reduction
targets.

549. Consider the following statement.

1) World Bank provides low-interest loans and grants to developing countries.


2) International Monetary Fund (IMF) has the objective, inter alia, of promoting high employment, sustainable
economic growth and reducing poverty around the world.

Which of the above is/are correct?

a) 1 only
b) 2 only
c) Both 1 and 2
d) None

Solution: c)
Justification: Statement 1: These support a wide array of investments in such areas as education, health, public
administration, infrastructure, financial and private sector development, agriculture, and environmental and natural
resource management.
Statement 2: The IMF is an organization of 188 countries, working to foster global monetary cooperation, secure
financial stability, facilitate international trade, promote high employment and sustainable economic growth, and
reduce poverty around the world.

550. Which of the following organizations is known as the private arm of the World Bank?

a) International Bank for Reconstruction and Development (IBRD)


b) Multilateral Investment Guarantee Agency (MIGA)
c) International Finance Corporation (IFC)
d) Bilateral Investment Protection agency (BIPA)

Solution: c)
Justification: The International Finance Corporation (IFC) was set up in 1956 which is also known as the private arm
of the WB. It lends money to private sector companies of its member nations. 'The interest rate charged is
commercial but comparatively low. There are many attractive features of IFC's lending. It finances and provides
advice for private-public ventures and projects in partnership with private investors and, through its advisory work,
helps governments of the member nations to create conditions that stimulate the flow of both domestic and foreign
private savings and investment. It focuses on promoting economic development by encouraging the growth of
productive enterprises and efficient capital markets in its member countries. It participates in an investment only
when it can make a special contribution that complements the role of market investors (as a foreign financial
investor (FFI). It also plays a catalytic role, stimulating and mobilising private investment in the developing world by
demonstrating that investments there too can be profitable.

551. Consider the following about the World Economic Forum (WEF).

1) It is an intergovernmental organization committed to improving economic policy making.


2) All its meetings are conducted in Geneva.
3) It publishes the annual report “Global Economic Prospects”.
4) The Global Health Initiative was launched at one of its meetings.

Select the correct answer using the codes below.

a) 1 only
b) 4 only
c) 1, 2 and 3 only
d) None of the above

Solution: b)
Justification: Statement 1: It is a Swiss non-profit foundation, based in Cologny, Geneva. Recognized by the Swiss
authorities as the international institution for public-private cooperation, its mission is cited as "committed to
improving the state of the world by engaging business, political, academic, and other leaders of society to shape
global, regional, and industry agendas".
Statement 2: The Forum is best known for its annual winter meeting in Davos, a mountain resort in Graubünden, in
the eastern Alps region of Switzerland. The organization also convenes some six to eight regional meetings each
year in locations such as Latin America and East Asia, as well as undertaking two further annual meetings in China
and the United Arab Emirates.
Statement 3: It is published by the World Bank.
Statement 4: The Global Health Initiative was launched by Kofi Annan at the annual meeting in 2002. The GHI's
mission was to engage businesses in public-private partnerships to tackle HIV/AIDS, tuberculosis, malaria, and
health systems.

552. The main functions of the International Monetary Fund (IMF) include

1) To promote exchange rate stability and orderly exchange arrangements


2) To assist member countries by temporarily providing financial resources to correct mal-adjustment in their balance
of payments (BoPs)
3) To serve as the regulator of international lending and borrowing from major international financial institutions

Select the correct answer using the codes below.

a) 1 and 2 only
b) 2 and 3 only
c) 1 and 3 only
d) 1, 2 and 3

Solution: a)
Justification: Statement 3: There is no international institution that regulates such lending and borrowing. World
Bank is one of the largest lenders but it is only a lender, does not regulate the process.
Statement 1 and 2: IMF was set up in 1944 with the main functions as exchange rate regulation, purchasing short-
term foreign currency liabilities of the member nations from around the world, allotting special drawing rights
(SDRs) to the member nations and the most important one as the bailer to the member economies in the situation
of any BoP crisis. Other major functions of the IMF are to facilitate international monetary cooperation and to assist
in the establishment of a multilateral system of payments and the elimination of foreign exchange restrictions.

553. Consider the following about the International Organization for Migration (IOM).

1) It is an inter-governmental organization of countries that are signatories to the Vienna convention.


2) It provides services and advice to governments and humanitarian assistance to migrants.
3) It does not deal with cases of ‘Forced migration’.

Select the correct answer using the codes below.

a) 1 and 3 only
b) 2 only
c) 3 only
d) 1 and 2 only
Solution: b)
Justification: Statement 1: Established in 1951, IOM is the leading inter-governmental organization in the field of
migration and works closely with governmental, intergovernmental and non-governmental partners. It is not related
to Vienna convention.
Statement 2: IOM works to help ensure the orderly and humane management of migration, to promote
international cooperation on migration issues, to assist in the search for practical solutions to migration problems
and to provide humanitarian assistance to migrants in need, including refugees and internally displaced people.
Statement 3: IOM works in the four broad areas of migration management: Migration and development, facilitating
migration, regulating migration and forced migration.

554. India is a member of which of the following organizations?

1) International Energy Agency (IEA)


2) Asian Development Bank (ADB)
3) CERN
4) Asian-Pacific Economic Cooperation organization (APEC)

Select the correct answer using the codes below.

a) 2 and 4 only
b) 1 and 2 only
c) 1, 2 and 3 only
d) 1, 3 and 4 only

Solution: b)
Justification: Statement 1: IEA is an inter-governmental organization established in 1974 as per framework of the
Organisation for Economic Co-operation and Development (OECD). It was established in the wake of the 1973 oil
crisis after the OPEC cartel had shocked the world with a steep increase in oil prices. Presently it has 30 member
countries including India. It is headquartered in Paris, France.
Statement 2: India is an important member and beneficiary of ADB funding.
Statement 3: India is a non-member associate country assisting in CERN research.
Statement 4: India has sought for over twenty years to become a member of the Asian-Pacific Economic
Cooperation organization (APEC), a request that has been allegedly blocked by China.

555. Which of the following forums is often called as the “Asian Davos” and considered to be China's equivalent of
the World Economic Forum?

a) Asia-Pacific Forum on Sustainable Development (AFSD)


b) East Asia Summit
c) Asia-Europe Meeting (ASEM)
d) Boao Forum for Asia (BFA)

Solution: d)
Justification: BFA is non-profit organisation that hosts high-level forums for leaders from government, business and
academia in Asia and other continents to share their vision on most pressing issues in this dynamic region and
world. It is modelled on annual World Economic Forum (WEF) held annually in Davos, Switzerland. The Forum is
committed to promoting regional economic integration and bringing Asian countries even closer to their
development goals. It was established in 2001. Its first meeting was held in April 2002 and since then it is held
annually. Its fixed address is held annually in Bo’ao, Hainan province, China although Secretariat is based in Beijing.
The forum sometimes is known as the “Asian Davos”.

556. With reference to the World Monuments Fund (WMF), consider the following:

1) It is a dedicated fund under UNESCO managed by the affiliates of UNESCO.


2) Every member country of UNESCO has agreed to contribute to this fund.
3) Every year, WMF presents the Hadrian Award to international leaders who have advanced the preservation of
world architecture.

Select the correct answer using the codes below.

a) 1 only
b) 3 only
c) 2 and 3 only
d) 2 only

Solution: b)
Justification: Statement 1: WMF is a private, international, non-profit organization dedicated to the preservation of
historic architecture and cultural heritage sites around the world.
Statement 2: Founded in 1965, through donations and matching funds, WMF has worked with local community and
government partners worldwide to safeguard and conserve places of historic value for future generations. To date,
WMF has worked in more than 91 countries, including many UNESCO World Heritage Sites.
Statement 3: This applies both for preservation of world art and architecture.

557. A new representative body — US- India Strategic Partnership Forum (USISPF) — is being set up to further enhance

a) Business relations between the two countries


b) Defense related strategic relations
c) Education and technology related exchange
d) All of the above

Solution: a)
Justification: The non-profit corporation aims to promote bilateral trade and work on creating meaningful
opportunities that have the power to change the lives of citizens. The body will work closely together with
businesses and government leaders to achieve its goals of driving economic growth, job creation, innovation,
inclusion and entrepreneurship.

558. The Common Reporting Standard (CRS) is an information standard of Automatic Exchange of Information (AEOI)
as developed by the OECD. Its purpose is to

a) Compare National income across developing countries


b) Monitor progress of ecologically sustainable projects in affiliate nations
c) Boost the implementation of the Trade Facilitation Agreement (TFA) of the WTO
d) Combat tax evasion

Solution: d)
Justification: CRS is an information standard for the automatic exchange of tax and financial information on a global
level, which the Organization for Economic Co-operation and Development (OECD) developed in 2014. Its purpose is
to combat tax evasion. It provides for the exchange of non-resident financial account information with the tax
authorities in the account holders’ country of residence. Participating jurisdictions that implement AEOI send and
receive pre-agreed information each year, without having to send a specific request. As of 2016, 83 countries had
signed an agreement to implement it. The CRS has been criticised for leaving many loopholes open, for how
developing countries were not considered and involved and that non-reciprocity agreements were catering to tax
havens. India joined the Multilateral Competent Authority Agreement on Automatic Exchange of Financial Account
Information in June, 2015,

559. Consider the following about Special Drawing Right (SDR).

1) The SDR is an international reserve asset created and administered by the IMF.
2) SDRs can be exchanged for freely usable currencies.
3) A Gold backing is mandatory for a nation to increase SDR deposits.
4) The SDR is a financial claim on the IMF as it is accepted by most international organizations.

Select the correct answer using the codes below.

a) 1 and 2 only
b) 2, 3 and 4 only
c) 1 and 3 only
d) 1, 2 and 4 only

Solution: a)
Justification: Statement 1: The SDR was created by the IMF in 1969 as a supplementary international reserve asset,
in the context of the Bretton Woods fixed exchange rate system. A country participating in this system needed
official reserves—government or central bank holdings of gold and widely accepted foreign currencies—that could
be used to purchase its domestic currency in foreign exchange markets, as required to maintain its exchange rate.

Statement 3: The value of the SDR is based on a basket of five major currencies—the US dollar, the euro, the
Chinese renminbi (RMB), the Japanese yen, and the British pound sterling. No gold backing is needed.

Statement 2 and 4: The SDR is neither a currency, nor a claim on the IMF. Rather, it is a potential claim on the freely
usable currencies of IMF members. Holders of SDRs can obtain these currencies in exchange for their SDRs in two
ways: first, through the arrangement of voluntary exchanges between members; and second, by the IMF
designating members with strong external positions to purchase SDRs from members with weak external positions.

IMF members often need to buy SDRs to discharge obligations to the IMF, or they may wish to sell SDRs in order to
adjust the composition of their reserves. The IMF may act as an intermediary between members and prescribed
holders to ensure that SDRs can be exchanged for freely usable currencies.

560. The United Nations Children's Fund (UNICEF) is a United Nations (UN) programme that provides humanitarian and
developmental assistance to children and mothers in developing countries. Consider the following with reference to
UNICEF’s association with India.

1) UNICEF provided equipment and technical assistance for the first Penicillin Plant established in India.
2) Within a decade of independence, UNICEF signed an agreement with the Government of India to fund some
cooperative milk processing plants to support white revolution.
3) UNICEF helped India in developing the India Mark II which is now the world’s most widely used handpump.

Select the correct answer using the codes below.

a) 1 only
b) 2 only
c) 3 only
d) 1, 2 and 3

Solution: d)
Justification: Statement 3: During the 1970s, UNICEF became a key partner with the Government of India in the
world’s largest rural water supply programme. UNICEF brought drilling rigs to India which could drill boreholes in
hard rock. The Government supplied handpumps. But, due to some problems, India mark II was later adopted.
Gender issues were mainstreamed into the training and communication strategy for the 2011 Census with the help
of UNESCO. The Government, in partnership with UNICEF, WHO, the Bill & Melinda Gates Foundation, Rotary
International and the Centers for Disease Control and Prevention contributed to almost universal awareness of the
need to vaccinate all children under five against polio. As a result of these efforts, India was removed from the list of
endemic countries in 2014.

561. SARTTAC, the newest addition to the IMF’s global network of fourteen regional centers is being supported
financially by its six-member countries, that include

1) India
2) Maldives
3) Nepal
4) Sri Lanka

Select the correct answer using the codes below.

a) 1 and 4 only
b) 2 only
c) 1, 3 and 4 only
d) 1, 2, 3 and 4

Solution: d)
Justification: South Asia Regional Training and Technical Assistance Centre (SARTTAC) is a collaborative venture
between the IMF, the member countries (Bangladesh, Bhutan, India, Maldives, Nepal, Sri Lanka), and development
partners. The Centre’s strategic goal is to help its member countries strengthen their institutional and human
capacity to design and implement macroeconomic and financial policies that promote growth and reduce poverty. It
is a new kind of capacity development institution, fully integrating customized hands-on training with targeted
technical advice in a range of macroeconomic and financial areas, and generating synergies between the two. It was
inaugurated at Delhi in February 2017. SARTTAC is financed mainly by its six member countries with additional
support from Australia, the Republic of Korea, the European Union and the United Kingdom.

562. Sustainable Development Knowledge Platform (SDKP) is an initiative of

a) United Nations
b) International Union for Conservation of Nature (IUCN)
c) WWF (World Wildlife Fund)
d) Conservation International

Solution: a)
Justification: The UN Department of Economic and Social Affairs (UNDESA) launched the UN web- based knowledge
platform for sustainable development (Sustainable Development Knowledge Platform SDKP). The site describes
different sustainable development activities within the UN system, and it provides resources and links to nearly
6,000 related documents. The platform presents analysis on the key topics including in the Rio 20 outcome
document and contains additional links related to Agenda 21 and the Commission for Sustainable Development

563. With reference to the Asia/Pacific Group on Money Laundering (APG), of which India is a member, consider the
following statements.

1) The APG is a voluntary and co-operative international body established by agreement among its members and does
not derive from an international treaty.
2) Purpose of the APG is to facilitate the enforcement of anti-money laundering and anti-terrorist financing standards
set out in the recommendations of the Financial Action Task Force (FATF).

Which of the above is/are correct?

a) 1 only
b) 2 only
c) Both 1 and 2
d) None

Solution: c)
Justification: It was officially established as an autonomous regional anti-money laundering body in February 1997
at the Fourth (and last) Asia/Pacific Money Laundering Symposium in Bangkok. The APG's role includes assisting
jurisdictions in the region to enact laws dealing with the proceeds of crime, mutual legal assistance, confiscation,
forfeiture and extradition. It also includes the provision of guidance in setting up systems for reporting and
investigating suspicious transactions and helping in the establishment of financial intelligence units. The purpose of
the APG is to facilitate the adoption, implementation and enforcement of internationally accepted anti-money
laundering and anti-terrorist financing standards set out in the recommendations of the FATF, which is a globally
body focussed on money laundering. The APG undertakes studies of methods and trends of money laundering and
the financing of terrorism in the Asia/Pacific region. The APG allows for regional factors to be taken into account in
the implementation of anti-money laundering and anti-terrorist financing measures and provides for peer review by
means of a mutual evaluation process.
S2: The APG is voluntary and autonomous. It does not derive from an international treaty nor is it part of any
international organization. India became a member of the APG in 1998.

564. India is a founding member of the International Labour Organization (ILO). Which of these international labour
conventions have been ratified by India?

1) Right to Organise and Collective Bargaining Convention, 1949


2) Freedom of Association and Protection of the Right to Organise Convention, 1948
3) Migration for Employment Convention (Revised), 1949
4) Forced Labour Convention, 1930
5) Equal Remuneration Convention, 1951

Select the correct answer using the codes below.

a) 1, 3, 4 and 5
b) 4 and 5 only
c) 1, 2 and 3 only
d) 1, 2, 3, 4 and 5

Solution: b)
Justification: Out of 47 Conventions and 1 Protocol ratified by India, of which 38 are in force, 5 Conventions and 0
Protocol have been denounced; 3 instruments abrogated; 2 have been ratified in the past 12 months.

565. With reference to the United Nations Department of Economic and Social Affairs (UN DESA), consider the
following statements.

1) It is a part of the United Nations Secretariat.


2) It is responsible for the follow-up to major United Nations Summits and Conferences.
3) It is a member of the United Nations Development Group.
4) It supports international cooperation to achieve Sustainable Development Goals (SDGs).

Select the correct answer using the codes below.

a) 2 only
b) 1 and 4 only
c) 2 and 3 only
d) 1, 2, 3 and 4

Solution: d)
Justification: UN DESA is a vital interface between global policies in the economic, social and environmental spheres
and national action. Its work is guided by the 2030 Agenda for Sustainable Development, along with SDGs. UN DESA
provides services to the United Nations Economic and Social Council and the Second and Third Committees of the
United Nations General Assembly. UN DESA assists countries around the world in agenda-setting and decision-
making with the goal of meeting their economic, social and environmental challenges. It supports international
cooperation to promote sustainable development for all, having as a foundation the 2030 Agenda for Sustainable
Development and the 17 Sustainable Development Goals (SDGs) as adopted by the UN General Assembly on 25
September 2015. It continues to play a key role in monitoring progress towards internationally agreed-upon
development goals.

566. Consider the following statements. Grameen Bank is

1) A microfinance organization owned by the Government of Bangladesh


2) A community development bank which provides loans to the poor without demanding collateral

Which of the above is/are correct?


a) 1 only
b) 2 only
c) Both 1 and 2
d) None
Solution: b)
Justification: Grameen Bank (GB) has reversed conventional banking practice by removing the need for collateral
(security against which a loan is made, e.g. house) and created a banking system based on mutual trust,
accountability, participation and creativity. Professor Muhammad Yunus, the founder of "Grameen Bank" and its
Managing Director, reasoned that if financial resources can be made available to the poor people on terms and
conditions that are appropriate and reasonable, "these millions of small people with their millions of small pursuits
can add up to create the biggest development wonder.”

567. The Next Eleven countries (N-11), a conception of Goldman Sachs investment banker and economist Jim O'Neill,
includes

1) India
2) Mexico
3) Egypt
4) Pakistan
5) South Korea

Select the correct answer using the codes below.

a) 2, 3 and 4 only
b) 2, 3, 4 and 5 only
c) 1, 2, 3 and 5 only
d) 1, 4 and 5 only
Solution: b)
Justification: N-11 are – Bangladesh, Egypt, Indonesia, Iran, Mexico, Nigeria, Pakistan, the Philippines, Turkey, South
Korea and Vietnam – identified by Jim O'Neill in a research paper as having a high potential of becoming, along with
the BRICS countries, among the world's largest economies in the 21st century. The bank chose these states, all with
promising outlooks for investment and future growth, in 2005. Next Eleven countries have an area of 10 million sq.
kilometers while having a population of 1460 million. Combined nominal GDP of these countries is 6.5 trillion
dollars. In terms of purchasing power parity (PPP), their GDP reached 15.5 trillion dollars

568. Consider the following about the World Bank’s Green Bond program.

1) It supports the transition to low-carbon and climate resilient development and growth in client countries.
2) Eligible projects supported by the Green Bond program are nominated by individual member countries to the World
Bank.

Which of the above is/are correct?

a) 1 only
b) 2 only
c) Both 1 and 2
d) None

Solution: a)
Justification: World Bank Green Bonds are an opportunity to invest in climate solutions through a high quality credit
fixed income product.
Statement 1: This includes both mitigation of and adaptation to climate change—all while observing the World
Bank's safeguard policies for environmental and social issues. The World Bank Green Bond raises funds from fixed
income investors to support World Bank lending for eligible projects that seek to mitigate climate change or help
affected people adapt to it. The product was designed in partnership with Skandinaviska Enskilda Banken (SEB) to
respond to specific investor demand for a triple-A rated fixed income product that supports projects that address
the climate challenge.Since 2008, the World Bank has now issued over USD 10.2 billion equivalent in Green Bonds
through more than 135 transactions in 18 currencies.
Statement 2: Eligible projects supported by the Green Bond program are selected by World Bank environment
specialists and meet defined World Bank eligibility criteria for low-carbon and climate resilient development.

569. With reference to Convention on International Civil Aviation, also known as the Chicago Convention, consider
the following statements.

1) It establishes rules for the safety of airlines.


2) The convention exempts air fuels in transit from double taxation.
3) It provides that “Every state has complete and exclusive sovereignty over airspace above its territory”.
4) International Civil Aviation Organization (ICAO) was established by this convention.

Select the correct answer using the codes below.

a) 1 and 2 only
b) 3 and 4 only
c) 2 and 3 only
d) 1, 2, 3 and 4 only

Solution: d)
Justification: Statement 1: The Convention establishes rules of airspace, aircraft registration and safety, and details
the rights of the signatories in relation to air travel.
Statement 2: Air transits would tend to impose tax twice on the aviation fuel if not checked for double taxation. This
is similar to how MNCs would be double taxed if a DTAA hasn’t been signed.
Statement 3: Every other State must refrain from resorting to the use of weapons against civil aircraft in flight.
Statement 4: ICAO is a UN specialized agency, established by States in 1944 to manage the administration and
governance of the Convention on International Civil Aviation (Chicago Convention).

570. With reference to the International Covenant on Economic, Social and Cultural Rights, consider the
following:

1) It was adopted by an informal assembly of European Union and later approved by the UN General Assembly.
2) The Covenant is monitored by the UN Committee on Economic, Social and Cultural Rights.

Which of the above is/are correct?


a) 1 only
b) 2 only
c) Both 1 and 2
d) None

Solution: b)
Justification: ICESCR is a multilateral treaty adopted by the UN General Assembly in 1966, coming into force since
1976. It commits its parties to work toward the granting of economic, social, and cultural rights (ESCR) to the Non-
Self-Governing and Trust Territories and individuals, including labour rights and the right to health, the right to
education, and the right to an adequate standard of living. As of 2015, the Covenant has 164 parties. Six countries,
including the United States, have signed but not ratified the Covenant

UNIT-XV
REPORTS AND INDICES
REPORTS AND INDICES

571. With reference to Voluntary National Review (VNR) on implementation of SDG, consider the following
statements.

1) UN member countries are expected to present their VNR in the High-Level Political Forum (HLPF) of the UN.
2) NITI Aayog is the nodal agency for the implementation of SDGs in India, and is responsible for presenting the VNRs
to the designated body.

Which of the above is/are correct?

a) 1 only
b) 2 only
c) Both 1 and 2
d) None

Solution: c)
Justification: Statement 1: As a signatory to the 2030 Agenda for Sustainable Development, India is committed to
participate in the international review of progress of Sustainable development Goals (SDGs) on a regular basis. The
central platform for international follow-up and review of the 2030 Agenda is the High-Level Political Forum (HLPF),
which has started meeting annually since 2016 under the auspices of the UN Economic and Social Council (ECOSOC).
In the HLPF, UN member countries are expected to present their Voluntary National Review (VNR) on
implementation of SDGs. The VNRs thus serve as a basis for international review of progress of SDGs.
Statement 2: As a part of its role, NITI Aayog has presented the 1st Voluntary National Review on implementation of SDGs in
the country to the 2017 HLPF in July 2017.

572. The Global Human Capital Index, compiled by World Economic Forum (WEF), measures countries against
four key areas of human capital development, which includes?

1) Investment in formal education


2) Accumulation of skills through work
3) Population and per capita income
4) Health and hygiene indicators

Select the correct answer using the codes below.


a) 1 and 2 only
b) 2 and 3 only
c) 2, 3 and 4 only
d) 1 and 3 only

Solution: a)
Justification: It takes into account “the knowledge and skills people possess that enable them to create value in the
global economic system” to measure the ‘human capital’ rank of a country. The report measures 130 countries
against four key areas of human capital development; Capacity (determined by past investment in formal
education), Deployment (accumulation of skills through work), Development (continued upskilling and reskilling of
existing workers) and Know-how (specialized skills-use at work). India has been placed at a low 103 rank, the lowest
among BRICS economies.

573. Consider the following about NHB RESIDEX published by National Housing Board (NHB).

1) It is a composite all India housing price index that takes property price and CPI inflation into account.
2) It depicts housing price indices based on government approved property prices.

Which of the above is/are correct?


a) 1 only
b) 2 only
c) Both 1 and 2
d) None

Solution: d)
Justification: Justification: The Housing Price Indices (HPIs) are a broad measure of movement of residential
property prices observed within a geographic boundary. NHB RESIDEX, India’s first official housing price index, was
an initiative of the National Housing Bank (NHB) undertaken at the behest of the Ministry of Finance, Government
of India.

Statement 1: NHB is not computing the composite all India housing price index as of now. Currently, National
Housing Bank is publishing NHB RESIDEX for 50 cities on quarterly basis with FY 2012-13 as base year. Among 50
cities covered are 18 State/UT capitals and 37 Smart Cities.

Statement 2: Initially, NHB RESIDEX was computed using market data, which 2010 onwards, was shifted to valuation
data received from banks and housing finance companies (HFCs). Thereafter, data was sourced from Central
Registry of Securitization Asset Reconstruction and Security Interest of India (CERSAI) from 2013 to 2015. Overtime,
the base year has been revised to FY 2012-13 to ensure capturing the latest information and accurately reflect the
current economic situation in the country.

574. Energy Transition Index (ETI) was recently published by


a) World Bank
b) World Energy Institute
c) International Energy Agency (IEA)
d) World Economic Forum (WEF)

Solution: d)
Justification: Learning: India was ranked at 78th among 114 countries on World Economic Forum’s (WEF) Energy
Transition Index (ETI) released as part of report titled Fostering Effective Energy Transition. The index ranks
countries on how well they are able to balance energy security and access with environmental sustainability and
affordability. According to report, India has taken bold measures to improve energy access, energy efficiency and to
improve deployment of renewable sources of energy. However, energy transition in India will require large
investments and enabling environment along with robust regulatory frameworks to support transition. Top 10
countries in 2018 ETI: Sweden (1st), Norway (2nd), Switzerland (3rd), Finland (4th), Denmark (5th), Netherlands
(6th), the UK (7th), Austria (8th), France (9th) and Iceland (10th). BRICS Countries in ETI: Brazil (38th), Russia (70th)
and China (76th).

575. The World Development Report is annually published by


a) World Bank
b) UNECOSOC
c) Oxfam India
d) G-20

Solution: a)
Justification: The WB recently published its annual World Development Report (WDR). The World Development
Report puts out the indicators of global growth, major hindrances in achieving it and lays down concrete steps in
resolving those issues. Other such developmental report is UNHDR published by UNDP. UNECOSOC and G-20 do not
publish reports of general nature that consider the overarching theme of development.

576. Human Development Report and World Development Indicators are published by which of these
organizations respectively?
a) Both by the United Nations Development Programme (UNDP)
b) UNDP and World Bank
c) World Bank and UNICEF
d) Oxfam and UNECOSOC

Solution: b)

Justification: HDR is an annual milestone published by the Human Development Report Office of the UNDP. India
ranked 131 in 2016 Human Development Index. World Development Indicators is the primary World Bank collection
of development indicators, compiled from officially-recognized international sources. It presents the most current
and accurate global development data available, and includes national, regional and global estimates. We will cover
such and other reports in detail in upcoming tests.

577. Corruption Perception Index 2017 was published by


a) Oxfam International
b) Human Rights Watch
c) Association for Democratic Reforms (ADR)
d) Transparency International

Solution: d)
Justification: The index ranks 180 countries and territories by their perceived levels of public sector corruption. The
index uses a scale of 0 to 100, where 0 is highly corrupt and 100 is very clean. Performance of India: India has been
ranked 81st. It maintained its score at 40 on a 100-point scale where anything below 30 is considered seriously
corrupt. The index also characterized India as “among the worst regional offenders” in the Asia Pacific region on
grounds of journalists, activists, opposition leaders and even staff of law enforcement or watchdog agencies being
threatened or even murdered. In this, it was clubbed with the Philippines and Maldives.

578. The World Economic Forum (WEF) has released its Global Manufacturing Index. Consider the following
about it.

1) India does not come under top 10 in the list.


2) India has performed worse than all other BRICS countries in this index.

Which of the above is/are correct?


a) 1 only
b) 2 only
c) Both 1 and 2
d) None

Solution: a)
Justification: This is WEF’s first ‘Readiness for the future of production report’. The report has categorised 100
countries in four major groups for its ranking:
 Leading (strong current base, high level of readiness for future).

 High Potential (limited current base, high potential for future).

 Legacy (strong current base, at risk for future).

 Nascent (limited current base, low level of readiness for future). Performance of various countries:

Japan has topped the list. Japan is followed by South Korea, Germany, Switzerland, China, Czech Republic, the US,
Sweden, Austria and Ireland in the top 10. India is at the 30th position, five places below China, but higher than the
other BRICS members Brazil, Russia and South Africa. India has been placed in the “Legacy” group. In terms of scale
of production, India has been ranked 9th.

579. Multidimensional Poverty Index (MPI) does NOT include which of the following indicators in its calculation?
a) Child school attendance
b) Wages received per day
c) Access to safe drinking water
d) Access to Electricity

Solution: b)
Justification: The index uses the same three dimensions as the Human Development Index: health, education, and
standard of living. These are measured using ten indicators. Each dimension and each indicator within a dimension
is equally weighted.

580. The SDG Index and Dashboard collect available data for countries to assess where each country stands in a
particular year with regard to achieving the Sustainable Development Goals (SDGs). It is published by
a) UN Sustainable Development Solutions Network (SDSN)
b) World Bank
c) United Nations Development Programme (UNDP)
d) A consortium of NGOs chosen by party countries for independent assessment

Solution: a)
Justification: It has been operating since 2012 under the auspices of the UN Secretary-General. SDSN mobilizes
global scientific and technological expertise to promote practical solutions for implementation of the SDGs and the
Paris Climate Agreement. SDSN works closely with United Nations agencies, multilateral financing institutions, the
private sector, and civil society. India is ranked 116th on the index for 2017.

581. In the calculation of Human Development Index (HDI), why is the maximum GNI per capita fixed at $75,000?

1) No nation has ever recorded a GNI per capita higher than this.
2) A very high income will distort HDI figures and may not add to further development of population.

Which of the above is/are correct?


a) 1 only
b) 2 only
c) Both 1 and 2
d) None

Solution: b)
Justification: Statement 1: Currently we have only 4 countries with GNI pc above the cap – Liechtenstein, Kuwait,
Qatar and Singapore.

Statement 2: Income is instrumental to human development, but the contribution diminishes as incomes rise. Also,
a high income without being translated into other human development outcomes is of less relevance for human
development. Fixing the maximum income means that for countries with GNI per capita greater than $75,000, only
the first $75,000 contributes to human development. In this way, the higher income is prevented from dominating
the HDI value.

582. With reference to the Purchasing Managers’ Index (PMI), recently seen in news, consider the following
statements.

1) PMI is an indicator of business activity-both in the manufacturing and services sectors.


2) It is a survey-based measure that asks respondents about changes in their perception of some key business
variables from month before.
3) PMI excludes import-export data to firm to reflect domestic preferences of manager and industrialists.

Select the correct answer using the codes below.


a) 1 only
b) 1 and 2 only
c) 1 and 3 only
d) 2 and 3 only

Solution: b)
Justification: Statement 1: It is calculated separately for manufacturing and services sectors and then composite
index is constructed. But the manufacturing sector holds more importance. It is different from IIP.

Statement 2: PMI is usually released at start of month, much before most of official data on industrial output,
manufacturing and GDP growth is made available. It is, therefore, considered a good leading indicator of economic
activity. Manufacturing growth measured by PMI is considered good indicator of industrial output.

Statement 3: The variables used to construct India’s PMI are: Output, New Orders, Employment, Input Costs,
Output Prices, Backlogs of Work, Export Orders, Quantity of Purchases, Suppliers Delivery Times, Stocks of
Purchases and Stocks of Finished Goods. Similar variables (but less in number) are used for the construction of
services PMI. The PMI is very closely watched, as it shows the investor sentiment in an economy’s manufacturing
sector.

583. The Global Gender Gap Index, as part of World Economic Forum's (WEF) Global Gender Gap Report 2017,
measures progress towards parity between men and women in which of these indicators?

1) Educational attainment
2) Health and survival
3) Inter-generational distribution
4) Political empowerment
5) Geographical evenness
6) Economic opportunity

Select the correct answer using the codes below.


a) 1, 2 and 6 only
b) 1, 2, 4 and 6 only
c) 3, 4 and 5 only
d) 1, 2, 3, 4, 5 and 6

Solution: b)
Justification: India was ranked low at 108th position out of 144 in Global Gender Gap Index 2017 released by WEF.
India slipped by 21 places compared to 87th rank last year. Countries are ranked based scores on scale ranging from
0 (lowest i.e. imparity) to 1 (highest i.e. parity) based on 4 factors as mentioned in the solution above. Iceland is
most gender-equal country with score of 0.878. It is followed by Norway (2 rank), Finland (3), Rwanda (4) and
Sweden (5), Nicaragua (6) and Slovenia (7), Ireland (8), New Zealand (9) and the Philippines (10).

584. Consider the following about the Logistics Ease across Different States (LEADS) index that was published
recently.

1) It was published by the World Bank.


2) It is the regional version of the Logistics Performance Index (LPI) which is a global indicator.
3) Gujarat topped the index this year.
4) Regulatory process related to logistics are included in this index.

Select the correct answer using the codes below.


a) 1 and 2 only
b) 3 and 4 only
c) 3 only
d) 1, 3 and 4 only

Solution: b)
Justification: LEADS index is a perception-based index of mobility of goods and efficiency of logistics chain. It is a
composite indicator to assess international trade logistics across states and Union territories. It is based on a
stakeholders’ survey conducted by Deloitte for the ministry of commerce and industry. LEADS is loosely based on
the World Bank’s biannual Logistics Performance Index (LPI), on which India was ranked 35 among 160 countries in
2016, up from 54 in 2014. LEADS is based on eight parameters such as infrastructure, services, timeliness, track and
trace, competitiveness of pricing, safety of cargo, operating environment and regulatory process.

Statement 3: While Gujarat topped the first-of-its-kind index, Punjab and Andhra Pradesh took the second and third
positions, respectively.

585. Which of these is NOT a key pillar of Gross National Happiness (GNH) index of Bhutan?
a) Preservation and promotion of cultural values
b) Sustainable development
c) Inequality between various racial groups
d) Establishment of good governance

Solution: c)
Justification: The GNH Index includes nine domains: Psychological wellbeing; Health; Education; Time use; Cultural
diversity and resilience; Good governance; Community vitality; Ecological diversity and resilience; and Living
standards. It is a holistic reflection of the general wellbeing of the Bhutanese population rather than a subjective
psychological ranking of ‘happiness’ alone.
586. The State of the World’s Children report is published by
a) Child Rights and You (CRY)
b) United Nations Children’s Fund (UNICEF)
c) Oxfam International
d) Child Rights Information Network (CRIN)

Solution: b)
Justification: This is a lateral explanation, i.e. we try to cover related topics, and not deal with the subject at hand
because it’s obvious. International Society for Prevention of Child Abuse and Neglect (ISPCAN). A non-governmental
organization seeking to support individuals and organizations working to protect children from abuse and neglect
worldwide.

SAFE KIDS Worldwide: It is a global network of organizations dedicated to the prevention of unintentional childhood
injury. The Child Accident Prevention Foundation of Southern Africa (CAPFSA) promotes optimal health and
development of all children, via its Childsafe programme. CAPFSA aims to reduce intentional and unintentional
injuries of all severity through research, education, environmental change and recommendations for legislation.

587. With reference to the Logistic Performance Index (LPI), consider the following statements.

1) It is published by the United Nations Conference on Trade and Development (UNCTAD).


2) It indicates the relative ease and efficiency with which products can be moved into and inside a country.
3) India was placed at fifth position on the LPI rankings 2017.

Select the correct answer using the codes below.


a) 1 and 2 only
b) 2 only
c) 1 and 3 only
d) 2 and 3 only

Solution: b)
Justification: Statement 1: Logistics Performance Index is reported by World Bank in every 2 years.
Statement 2: It is the weighted average of the country scores on the following six key dimensions: efficiency of the
clearance process (i.e. speed, simplicity and predictability of formalities) by border control agencies, including
Customs; Quality of trade and transport related infrastructure (e.g. ports, railroads, roads, information technology);
Ease of arranging competitively priced shipments; Competence and quality of logistics services (e.g., transport
operators, customs brokers); Ability to track and trace consignments; Timeliness of shipments in reaching
destination within the scheduled or expected delivery time.
Statement 3: Germany and Singapore are the most efficient and highest ranked LPI countries. India is ranked in the
30s in the rank list.

588. Why do analysts believe that the Global Hunger Index (GHI) as a “Hunger Index” is misleading?

1) Child stunting and child wasting are not considered in the index.
2) Weight and height of children are not solely determined by food intake but also depends on genetics and
sanitation.

Which of the above is/are correct?

a) 1 only
b) 2 only
c) Both 1 and 2
d) None

Solution: b)
Justification: The GHI combines four component indicators:

 the proportion of the undernourished as a percentage of the population;

 the proportion of children under the age of five suffering from wasting (low weight-for- height);

 the proportion of children under the age of five suffering from stunting (low height-for-age);

 the mortality rate of children under the age of five

Statement 1: Undernourishment and child mortality each make up a third of the GHI score, while child stunting and
child wasting make up a sixth of the score, and together make up a third of the score. Three of the four indicators,
refer only to children below five who constitute only 11.5% of India’s population. Further, the percentage of the
undernourished population is inclusive of under nutrition among children. This way, the GHI assigns 70.5%
weightage to children below five who constitute only a minor population share and 29.5% weightage to the
population above five, which constitutes 81.5% of the total population.

Statement 2: Evidence shows that weight and height of children are not solely determined by food intake but are an
outcome of a complex interaction of factors related to genetics, the environment, sanitation and utilization of food
intake. The IFPRI acknowledges that only 45% of child mortality is due to hunger or under nutrition. Therefore, the
term “Hunger Index” is highly biased towards under nutrition of children rather than representing the status of
hunger in the overall population.

589. World Migration Report 2018 has been published by


a) United Nations Industrial Organization (UNIDO)
b) International Organization for Migration
c) International Labour Organization (ILO)
d) World Economic Forum (WEF)

Solution: b)
Justification: The definition of international migrants used in the report is broad, taking into account anyone living
in a country other their own and includes refugees and economic migrants, both those immigrating officially and
those who do so “irregularly”.

Highlights of the Report:

 Indian tops the world in the number of migrants sent abroad. About 16.59 million Indian live abroad. Mexico
sent out 13 million migrants, the second highest number.

 The United Arab Emirates has the largest number of Indian migrants, who number 3.31 million, followed by the
US with 2.3 million, up from 1.04 million.

 The number of migrant from other countries living in India is 5.2 million, a fall of 1.22 million from 2000.

 Most of the international migration takes place among developing countries with 60% of the migrants from Asia
going to other Asian countries. About $400 billion is sent to developing countries by migrants and the
remittances are used to finance education, housing and other activities that promote development.

 Also, according to the United Nations’ 2017 International Migration Report, India has the maximum number of
people living outside its borders, followed by Mexico and the Russian Federation
590. The Baltic Dry Index (BDI) is a/an
a) Crude Oil quality indicator
b) Arctic ice cover indicator
c) Precipitation index
d) Shipping and trade index

Solution: d)
Justification: It was created by the London-based Baltic Exchange that measures change in the cost of transporting
various raw materials. The exchange directly contacts shipping brokers to assess price levels for a given route, a
product to transport and time to delivery, or speed. The Baltic Dry Index is a composite of three sub-indices that
measure different sizes of dry bulk carriers or merchant ships: Capesize, Panamax and Supramax. The Baltic
Exchange, which calculates the index by assessing multiple shipping rates across multiple routes for each of the BDI
component ships, issues the BDI daily. Analyzing multiple geographic routes for each index gives depth to the
index's composite measurement

591. NITI Aayog has released a comprehensive Health Index report titled, “Healthy States, Progressive India”.
Consider the following about the report.

1) Geographically larger states and UTs have performed better than smaller states in terms of health index.
2) States and UTs that start at higher levels of development are generally at an advantage in notching up incremental
progress over States with low Health Index scores.

Which of the above is/are correct?


a) 1 only
b) 2 only
c) Both 1 and 2
d) None

Solution: d)
Justification: States and UTs have been ranked in three categories namely, Larger States, Smaller States, and Union
Territories (UTs), to ensure comparison among similar entities. The Health Index is a weighted composite Index,
which for the larger States, is based on indicators in three domains: (a) Health Outcomes (70%); (b) Governance and
Information (12%); and (c) Key Inputs and Processes (18%), with each domain assigned a weight based on its
importance. It is the first attempt to establish an annual systematic tool to measure and understand the
heterogeneity and complexity of the nation’s performance in Health.

Statement 1: No such observation has been made. Among the Larger States, Kerala, Punjab, and Tamil Nadu ranked
on top in terms of overall performance, while Jharkhand, Jammu & Kashmir, and Uttar Pradesh are the top three
ranking States in terms of annual incremental performance. Among Smaller States, Mizoram ranked first followed
by Manipur on overall performance, while Manipur followed by Goa were the top ranked States in terms of annual
incremental performance.

Statement 2: The Health Index report notes that while States and UTs that start at lower levels of development are
generally at an advantage in notching up incremental progress over States with high Health Index scores, it is a
challenge for States with high Index scores to even maintain their performance levels.

592. Consider the following statements.

1) The Gender Inequality Index (GII) is the ratio of the HDIs calculated separately for females and males using the same
methodology as in the HDI.
2) The Gender Development Index (GDI) measures gender inequalities in three important aspects of human
development including Reproductive health and political representation.

Which of the above is/are correct?


a) 1 only
b) 2 only
c) Both 1 and 2
d) None

Solution: d)
Justification: Statement 1: The GDI measures gender gaps in human development achievements by accounting for
disparities between women and men in three basic dimensions of human development—health, knowledge and
living standards using the same component indicators as in the HDI. It is a direct measure of gender gap showing
the female HDI as a percentage of the male HDI

Statement 2: The GII is an inequality index. It measures gender inequalities in three important aspects of human
development—reproductive health, measured by maternal mortality ratio and adolescent birth rates;
empowerment, measured by proportion of parliamentary seats occupied by females and proportion of adult
females and males aged 25 years and older with at least some secondary education; and economic status,
expressed as labour market participation and measured by labour force participation rate of female and male
populations aged 15 years and older.
The GII is built on the same framework as the IHDI—to better expose differences in the distribution of
achievements between women and men. It measures the human development costs of gender inequality. Thus the
higher the GII value the more disparities between females and males and the more loss to human development

593. Consider the following about the World Economic Outlook (WEO) report.

1) It is a survey conducted and published by the IMF.


2) It is published once in two years.
3) It only covers major economies of OECD and G20.

Select the correct answer using the codes below.


a) 1 only
b) 2 only
c) 3 only
d) 1, 2 and 3

Solution: a)
Justification: Statement 1 and 3: WEO forecasts include key macroeconomic indicators, such as GDP, inflation, fiscal
balance and current account of more than 180 countries around the globe. It also deals with major economic policy
issues.
Statement 2: It is published biannually and partly updated two times a year. It portrays the world economy in the
near and medium context, with growth projections for up to four years into the future.

594. The Human Development Index (HDI) does NOT take account of
a) Per capita income
b) Health
c) Ecological degradation
d) Literacy levels

Solution: c)
Justification: The Human Development Index has been criticized on a number of grounds, including exclusion of
ecological concerns, alleged lack of consideration of technological development or contributions to the human
civilization, focusing exclusively on national performance and ranking. The set of criticisms also include a lack of
attention to development from a global perspective, measurement error of the underlying statistics, and on the
UNDP's changes in formula which can lead to severe misclassification in the categorization of 'low', 'medium', 'high'
or 'very high' human development countries.

595. Consider the following committees in news and their terms of reference.

1) Malegam Committee: Bank Frauds


2) Madhukar Gupta Committee: Sustainable Housing
3) Subramanian Committee: National Policy on Education

Select the correct answer using the codes below.

a) 1 and 2 only
b) 1 and 3 only
c) 2 and 3 only
d) 1, 2 and 3

Solution: b)

596. The World Happiness Report is an annual publication of the


a) William J. Clinton Foundation
b) United Nations Human Rights Commission (UNHRC)
c) World Bank
d) United Nations Sustainable Development Solutions Network (UNSDSN)

Solution: d)
Justification: It contains rankings of national happiness and analysis of the data from various perspectives. In the
reports, experts in fields including economics, psychology, survey analysis, and national statistics, describe how
measurements of well-being can be used effectively to assess the progress of nations, and other topics. Each report
is organized by chapters that delve deeper into issues relating to happiness, including mental illness, the objective
benefits of happiness, the importance of ethics, policy implications, and links with the Organisation for Economic
Co-operation and Development's (OECD) approach to measuring subjective well-being and other international and
national efforts. As of March 2018, Finland was ranked the happiest country in the world.

597. The Gender Parity Index (GPI) is a socioeconomic index usually designed to measure the relative access to
education of males and females. This index is released by
a) UNDP
b) UNESCO
c) World Economic Forum (WEF)
d) UN Commission on Women

Solution: b)
Justification: In its simplest form, it is calculated as the quotient of the number of females by the number of males
enrolled in a given stage of education (primary, secondary, etc.). It is used by international organizations,
particularly in measuring the progress of developing countries. The Institute for Statistics of UNESCO also uses a
more general definition of GPI: for any development indicator one can define the GPI relative to this indicator by
dividing its value for females by its value for males. For example, some UNESCO documents consider gender parity
in literacy by dividing the female value of an indicator by the male value of the same indicator.

Elaboration: GPI equal to 1 indicates parity between females and males. In general, a value less than 1 indicates a
disparity in favour of boys and a value greater than 1 indicates a disparity in favour of girls. However, the
interpretation should be in the other way round for indicators that should ideally approach 0% (e.g. repetition,
dropout, illiteracy rates, etc.). In these cases, a GPI of less than 1 indicates a disparity in favour of girls and a value
greater than 1 indicates a disparity in favour of boys.

Limitations: This index does not show whether improvement or regression is due to the performance of one of the
gender groups (boys or girls). Interpretation of the GPI requires trend analysis of the underlying indicators. The
Union Finance Minister released Gender Parity Index specific to India sometime back. The main aim of developing
Gender Parity Index commissioned jointly by FLO and FICCI is to evaluate gender diversity and empowerment of
women in the formal sector and the progress made over the years.

598. Earlier the “Global Employment Trends” Report and now the revamped version the “World Employment
and Social Outlook (WESO): Trends” is published by
a) World Economic Forum (WEF)
b) International Labour Organization (ILO)
c) World Bank (WB)
d) International Monetary Fund (IMF)

Solution: b)
Justification: Formerly entitled Global Employment Trends, the World Employment and Social Outlook Trends (for
e.g. 2015) includes a forecast of global unemployment levels and explains the factors behind this trend, including
continuing inequality and falling wage shares. It looks at the drivers of the rising middle class in the developing
world as well as the risk of social unrest, especially in areas of elevated youth unemployment. The report addresses
structural factors shaping the world of work, including an aging population and shifts in the skills sought by
employers.

599. World Employment and Social Outlook 2018 was published by


a) World Bank
b) World Economic Forum (WEF)
c) International Labour Organization (ILO)
d) International Monetary Fund (IMF)

Solution: c)
Justification: The International Labour Organisation (ILO) on May 14, 2018 released its annual flagship report titled
‘World Employment and Social Outlook 2018: Greening with Jobs’. As per the report, 24 million new jobs are
expected to be created globally by 2030, if the right policies to promote a greener economy are put in place. The
creation of the ecosystem services could employ 1.2 billion workers. These services include air and water
purification, soil renewal and fertilization, pest control, pollination, protection against extreme weather conditions,
farming, fishing, forestry and tourism activities. The green economy will also enable people to overcome poverty
and deliver improved livelihoods for future generations. Other major reports by ILO are Global Wage Report, Global
Employment Trends for Youth and World Social Protection Report.

600. Consider the following about the Start-up States and UTs Ranking Framework 2018 released by the
Department of Industrial Policy and Promotion (DIPP).

1) It measures the impact of steps initiated at the local community level for building a strong Startup ecosystem.
2) It is largely based on the independent assessment done by the local units of the DIPP with assistance from the
District Chambers of Commerce (DCC).

Which of the above is/are correct?

a) 1 only
b) 2 only
c) Both 1 and 2
d) None

Solution: a)
Justification: The key objective of the Startup States and UTs Ranking Framework is to encourage States and UTs to
take proactive steps towards strengthening the Startup ecosystems at the local level. The Ranking Framework will
measure the impact of each step initiated at the local level for building a strong Startup ecosystem. The Ranking
Framework will also enable continuous learning through the dissemination of good practices.

Statement 2: The Ranking Framework is based on the feedback collected from Startup ecosystem stakeholders,
which include startups, mentors, investors, accelerators, incubators and the government bodies. Areas which
should be given greater thrust like seed funding support, women entrepreneurship are given more score.

The participating States and UTs shall now be evaluated on a score of 100, against 38 action points categorized into
7 areas of intervention such as Startup Policy and implementation, Incubation support, Seed Funding, Angel and
Venture Funding, Simplification of Regulations, Easing Public Procurement and Awareness & Outreach. In order to
evaluate States/UTs in a balanced and transparent manner, the assessment will be based on the aggregate score of
the documentary proofs substantiating the implementation of action points by the State/ UT Government and also
feedback from Startup ecosystem beneficiaries/components. States and Union Territories have participated in the
State Startup Ranking Framework 2018 launched by DIPP.

601. According to a recently released Trade and Development Report (TDR) 2017, India and China at their
current levels of growth will not serve as "growth polls" for global economy in near future. The TDR is a flagship
report of the
a) United Nations Conference on Trade and Development (UNCTAD)
b) World Trade Organization (WTO)
c) International Monetary Fund (IMF)
d) World Economic Forum (WEF)

Solution: a)
Justification: As per the report, world economy in 2017 is picking up but not lifting off. The global growth is
expected to reach 2.6%, well below pre 2008 financial crisis average of 3.2%. Unregulated finance remains at heart
of present hyper-globalized world and failure to tame it and address deep-seated inequalities, generated by it
threatens efforts to build inclusive economies. The report calls for serious examination of market power, rent-
seeking behaviour and winner-take- most rules of the economic game, which have generated exclusionary
outcomes.

602. World Press Freedom Index is published annually by Reporters without Borders (RSF). It is essentially an
indicator of the
a) Quality of journalism
b) Level of freedom available to journalists
c) Impact of public policies for promoting free speech
d) Penetration of mass media among people

Solution: b)
Justification: Published every year since 2002 by Reporters without Borders (RSF), the World Press Freedom Index is
an important advocacy tool based on the principle of emulation between states. The Index ranks 180 countries
according to the level of freedom available to journalists. It is a snapshot of the media freedom situation based on
an evaluation of pluralism, independence of the media, quality of legislative framework and safety of journalists in
each country. It does not rank public policies even if governments obviously have a major impact on their country’s
ranking. Nor is it an indicator of the quality of journalism in each country. In this year’s index, Norway is first for the
second year running, followed — as it was last year — by Sweden. India has dropped from rank 136 last year to rank
138 this year. India fared poorly on indicators such as hate speeches, attacks on journalists on social media, trolling
them and targeting their reputation. SAARC nations: Afghanistan (118), Bhutan (94), Nepal (106), the Maldives
(120), and Sri Lanka (131), all performed better than India; with Pakistan (139) and Bangladesh (146) performing
worse. North Korea continues to rank last.

UNIT-XVI
ECONOMIC SURVEY 2017-18
ECONOMIC SURVEY 2017-18

603. The Economic Survey 2016-17 has proposed Transparency of Rules Act (TORA), a legislation to
a) End any asymmetry of information regarding rules and regulations faced by an average citizen
b) Improve law making by involving lay citizens in the process
c) Instituting a Grievance Redressal Authority for each government service
d) None of the above

Solution: a)
Justification: The ‘opaque mesh’ of regulations prevalent in India not only make life difficult for citizens who cannot
feign ignorance of the rules as a valid defence, but also act as a magnet for corruption and endless litigation. The
TORA is an attempt to change in some ways the relationship between the average normal citizen and the State.
TORA will require all departments to mandatorily place all citizen-facing rules on their website. Officials will not be
able to impose any rule not mentioned beforehand. All laws will have to be updated by the department while
providing access to history of the same webpage. Once a department has shifted to the platform, it can be deemed
“TORA compliant” and citizens can be sure that the information is authentic and updated.

604. The first World Happiness Report was published on


a) The inaugural session of the United Nations General Assembly
b) The founding day of the Organization of Economic Cooperation and Development (OECD)
c) The Earth Summit
d) None of the above

Solution: d)
Justification: It is published by the Sustainable Development Solutions Network, and now supported by a generous
three-year grant from the Ernesto Lily Foundation. It was first launched in 2012, in support of the UN High Level
Meeting on happiness and well- being.

605. The ‘free-rider’ problem in Economics is usually associated with

1) Public goods
2) All private goods
3) Foreign Investment
Select the correct answer using the codes below.

a) 1 and 2 only
b) 3 only
c) 2 and 3 only
d) 1 only

Solution: d)
Justification: The ‘free-rider’ problem means the Consumers will not voluntarily pay for what they can get for free
and for which there is no exclusive title to the property being enjoyed. For e.g. if we consider a public park or
measures to reduce air pollution, the benefits will be available to all irrespective of them paying for it or not. The
consumption of such products by several individuals is not ‘rivalrous’ in the sense that a person can enjoy the
benefits without reducing their availability to others, which results in this free-rider problem. In case of private
goods anyone who does not pay for the good can be excluded from enjoying its benefits. If you do not buy a ticket,
you are excluded from watching a film at a local theatre. However, in case of public goods, there is no feasible way
of excluding anyone from enjoying the benefits of the good (they are non-excludable). Since non-paying users
usually cannot be excluded, it becomes difficult or impossible to collect fees for the public good. So, clearly 1 is
correct and 2 is incorrect. Statement 3 is irrelevant in this context.

606. As per Economic Survey 2017-18, the state with largest potential GST base is
a) Tamil Nadu
b) Kerala
c) Maharashtra
d) Punjab

Solution: c)
Justification: However, the share of Maharashtra’s and Gujarat’s tax base under the GST is lower than their share of
manufacturing. But because these two states also have a significant presence in services, their tax base share
remains in line with their share of GSDP.
607. If we analyses the various economic outcomes for selected Asian countries, we find that many of them
witnessed initial ‘take offs’ from low economic growth into periods of high growth. If this year for China was 1967,
what could be the possible take off year for India as sometimes suggested by the Economic Surveys?
a) 1975
b) 1980
c) 1991
d) 2007

Solution: c)
Justification: Based on IMF data (2006) the dates are 1979, 1973, and 1967 for China, Indonesia, and Korea
respectively. For India, taking the year of takeoff as 1991, when major economic reforms began, the following
narrative is clear as given below:

 India was growing at similar rates as other Asian economies before takeoff. After takeoff, it kept pace with
Indonesia, but China and Korea grew faster.

 After 1991, industrial growth which had slumbered earlier revived significantly, the after effects of which could
be seen even in the 2000s.

 The reforms liberated the services and external sector which drove India’s growth story in the coming decades.

You might also like